You are on page 1of 588

P

ICAP
Financial accounting and reporting I

* The student must refer original handbook of IFRS.

© Emile Woolf International i The Institute of Chartered Accountants of Pakistan


Seventh edition published by
Emile Woolf International
Bracknell Enterprise & Innovation Hub
Ocean House, 12th Floor, The Ring
Bracknell, Berkshire, RG12 1AX United Kingdom
Email: info@ewiglobal.com
www.emilewoolf.com

© Emile Woolf International, February 2021

All rights reserved. No part of this publication may be reproduced, stored in a retrieval system, or transmitted,
in any form or by any means, electronic, mechanical, photocopying, recording, scanning or otherwise, without
the prior permission in writing of Emile Woolf International, or as expressly permitted by law, or under the
terms agreed with the appropriate reprographics rights organisation.

You must not circulate this book in any other binding or cover and you must impose the same condition on
any acquirer.

Notice
Emile Woolf International has made every effort to ensure that at the time of writing the contents of this study
text are accurate, but neither Emile Woolf International nor its directors or employees shall be under any
liability whatsoever for any inaccurate or misleading information this work could contain.

© Emile Woolf International ii The Institute of Chartered Accountants of Pakistan


Certificate in Accounting and Finance

C
Financial accounting and reporting I

Contents
Page

Chapter

1 Accounting and Reporting Concepts 1

2 IAS 1: Preparation of Financial Statements 21

3 IAS 7: Statement of cash flows 47

4 Income and expenditure account 145

5 Preparation of accounts from incomplete records 207

6 Introduction to Cost of production 297

7 IAS 16: Property, plant and equipment 357

8 Non-Current Assets: Sundry standards 409

9 IAS 36: Impairment of assets 451

10 IFRS 15: Revenue from contracts with customers 481

11 Interpretation of financial statements 533

Index 581

© Emile Woolf International iii The Institute of Chartered Accountants of Pakistan


Financial accounting and reporting I

© Emile Woolf International iv The Institute of Chartered Accountants of Pakistan


Certificate in Accounting and Finance

1
Financial accounting and reporting I

CHAPTER
Accounting and reporting concepts

Contents
1 The conceptual framework of IASB

2 Objective based questions and answers

* The student must refer original handbook of IFRS.

© Emile Woolf International 1 The Institute of Chartered Accountants of Pakistan


Financial accounting and reporting I

1 THE CONCEPTUAL FRAMEWORK OF IASB


Section overview

◼ Overview of the Conceptual Framework of IASB


◼ Elements of financial statements
◼ Recognition of elements of the financial statements
◼ Measurements of elements of financial statements
◼ Concept of capital and capital maintenance

1.1 Overview of the Conceptual Framework of IASB


The Conceptual Framework describes the objectives of and concepts for the general purpose
financial statements. Its purposes are:
❑ to assist the International Accounting Standards Board (IASB) to develop IFRS Standards
that are based on consistent concepts;
❑ to assist the IASB in promoting harmonization of regulations, accounting standards and
procedures relating to the presentation of financial statements by providing a basis for
reducing the number of alternative accounting treatments permitted by IFRSs;
❑ to assist national standard-setting bodies in developing national standards;
❑ to assist preparers of financial statements in applying IFRSs and in dealing with topics that
have yet to form the subject of an IFRS;
❑ to assist users of financial statements in interpreting the information contained in financial
statements prepared in compliance with IFRSs; and
The Conceptual Framework is divided into eight chapters, namely:
❑ the objective of general purpose financial reporting;
❑ the qualitative characteristics of useful financial information;
❑ financial statements and the reporting entity;
❑ The elements of financial statements;
❑ Recognition process and criteria and derecognition;
❑ \measurement of the elements from which financial statements are constructed;
❑ Presentation and disclosure; and
❑ Concepts of capital and capital maintenance.
This Conceptual Framework is not an IFRS and nothing in the Conceptual Framework overrides
any specific IFRS.
On very rare occasions there may be a conflict between the Conceptual Framework and an IFRS.
In those cases, the requirements of the IFRS prevail over those of the Conceptual Framework.

1.2 Elements of financial statements


The elements of financial statements defined in the conceptual framework are:
❑ assets, liabilities and equity, which relate to a reporting entity’s financial position; and
❑ income and expenses, which relate to a reporting entity’s financial performance
These elements are linked to the economic resources, claims and changes in economic
resources and claims and are explained as under:

© Emile Woolf International 2 The Institute of Chartered Accountants of Pakistan


Chapter 1: Accounting and reporting concepts

Item discussed Elements Definition or description


Economic Asset A present economic resource controlled by the entity as a
resource result of past events.
An economic resource is a right that has the potential to
produce economic benefits.
Claim Liability A present obligation of the entity to transfer an economic
resource as a result of past events.
Equity The residual interest in the assets of the entity after
deducting all its liabilities.
Changes in Income Increases in assets, or decreases in liabilities, that result in
economic increase in equity, other than those relating to contributions
resources and from holders of equity claims.
claims, reflecting
Expenses Decreases in assets, or increases in liabilities, that result in
financial
decreases in equity, other than those relating to distributions
performance
to holder of equity claims.
Other changes in - Contributions from holders of equity claims, and distributions
economic to them.
resources and
- Exchanges of assets or liabilities that do not result in
claims
increases or decreases in equity.

1.3 Recognition of elements of the financial statements


The IASB Framework states that an element (asset, liability, equity, income or expense) should
be recognized in the statement of financial position or the statement of financial performance
when it:
❑ meets the definition of an element, and also
❑ Satisfies certain criteria for recognition.
Items that fail to meet the criteria for recognition should not be included in the financial
statements. However, some of these items may have to be disclosed as additional details in a
note to the financial statements.
The criteria for recognition are as follows:

1.3.1 Recognition criteria


❑ Only items that meet the definition of asset, liability or equity are recognised in the statement
of financial position. Similarly, only items that meet the definition of income or expenses are
recognised in the statement of financial performance. However, not all items that meet the
definition of one of those elements are recognised.
❑ Not recognising an item that meets the definition of one of the elements makes the statement
of financial position and the statement of financial performance less complete and can
exclude useful information from financial statements. On the other hand, in some
circumstances, recognising some items that meet the definition of one of the elements would
not provide useful information.
❑ An asset or liability is recognised only if recognition of that asset or liability and of any
resulting income, expenses or changes in equity provides users of financial statements:

© Emile Woolf International 3 The Institute of Chartered Accountants of Pakistan


Financial accounting and reporting I

a) Relevant information about the asset or liability and about any resulting income, expenses or
change in equity

b) A faithful representation of the asset or liability and of any resulting income, expenses or
changes in equity
Relevance
Financial information is useful for making a decision only if it is capable of making a difference in
that decision. ‘Relevance’ is the term used in the Conceptual Framework to describe that capability.
It is a fundamental qualitative characteristic of useful financial information.
Faithful representation
❑ Recognition of a particular asset or liability is appropriate if it provides not only relevant
information, but also a faithful representation of that asset or liability and of any resulting
income, expenses or change in equity. Whether a faithful representation can be provided
may be affected by the level of uncertainty in measurement of the asset or liability.
❑ In some cases, the level of uncertainty involved in estimating a measure of an asset or
liability may be so high that it may be questionable whether the estimate would provide
faithful representation of that asset or liability and of any resulting income, expenses or
changes in equity. Whether or not an asset or liability is recognised, a faithful representation
of the asset or liability may need to include explanatory information about the uncertainties
associated with the asset or liability existence or measurement, or with its outcomes—the
amount or timing of any inflow or outflow of economic benefit that will ultimately result from
it.
❑ Faithful representation of a recognised asset, liability, equity, income or expenses involves
not only recognition of that item, but also its measurements as well as presentation and
disclosure of information about it. A complete depiction includes all information necessary
for a user of financial statement to understand the economic phenomenon depicted,
including all necessary description and explanations.

1.4 Measurements of elements of financial statements


The Conceptual Framework allows that several measurement bases are used for the elements in
financial statements. These include:

1.4.1 Historical cost


❑ Historical cost measure provides monetary information about assets, liabilities and related
income and expenses, using information derived, at least in part, from the price of the
transaction or other event gave rise to them. Unlike current value, historical cost does not
reflect changes in values, except to the extent that those changes relate to impairment of an
asset or liability becoming nervous.
❑ The historical cost of an asset, when it is acquired or created is the value of the cost incurred
in acquiring or creating the asset, comprising the consideration paid to acquire or create the
asset plus transaction cost. The historical cost of a liability when it is incurred or taken on is
the value of consideration received to incur or take on the liability minus transaction costs.
❑ When an asset is acquired or created, or a liability is incurred or taken on, as a result of an
event that is not a transaction on market terms, it may not be possible to identify a cost, or
the cost may not provide relevant information about the asset or liability. In such cases, a
current value of the asset or the liability is used as a deemed cost on initial recognition and
that deemed cost is then used as a starting point for subsequent measurement at historical
cost.

© Emile Woolf International 4 The Institute of Chartered Accountants of Pakistan


Chapter 1: Accounting and reporting concepts

❑ The historical cost of an asset is updated overtime to depict, if applicable:


a) The consumption of part of the economic resource that constitute the asset depreciation
or amortization;
b) Payments received that extinguish part or all of the asset;
c) The effect of events that cause part or all the historical cost of the asset to be no longer
recoverable(impairment);
d) Accrual of interest to reflect any financing component of the asset.
❑ One way to apply a historical cost measurement basis to financial assets and financial
liabilities is to measure them at amortized cost. The amortized cost of a financial asset or
financial liability reflects estimates of future cash flows, discounted at a rate determined at
initial recognition. For variable rate instruments, the discount rate is updated to reflect
changes in the variable rate. The amortized cost of a financial asset or financial liability is
updated over time to depict subsequent changes, such as the accrual of interest, the
impairment of a financial asset and receipt or payments.

1.4.2 Current value


❑ Current value measures provide monetary information about assets, liabilities and related
income and expenses, using information updated to reflect conditions at the measurement
date. Because of the updating, current values of assets and liabilities reflect changes, since
the previous measurement date, in estimate of cash flows and other factors reflected in
those current values. Unlike historical cost, the current value of an asset or liability is not
derived, even in part, from the price of the transaction or other event that gave rise to the
asset or liability.
❑ Current value measurement bases include:
a) Fair value
b) Value in use for assets and fulfillment value for liabilities
c) Current cost
a) Fair value
❑ Fair value is the price that would be received to sell an asset, or paid to transfer a liability, in
an orderly transaction between market participants at the measurement date.
❑ Fair value reflects the perspective of market participants—participants in a market to which
the entity has access. The asset or liability is measured using the same assumptions that
market participants would use when pricing the asset or liability if those market participants
act in their economic best interest.
❑ In some cases, fair value can be determined directly by observing prices in an active market.
In other cases, it is determined indirectly using measurement techniques, for example, cash-
flows-based measurement techniques reflecting all the following factors:
a) estimates of future cash flows
b) possible variations in the estimated amount or timing of future cash flows for the asset
or liability being measured, caused by the uncertainty inherent in the cash flows.
c) The time value of money
d) The price for bearing the uncertainty inherent in the cash (a risk premium or risk
discount). The price for bearing that uncertainty depends on the extent of that
uncertainty. It also reflects the fact that investors would generally pay less for an asset
(and generally require more for taking on a liability) that has uncertain cash flows then
for an asset (or liability) whose cash flows are certain.
e) Other factors for example; Liquidity, if market participant would take those factors into
account in the circumstances

© Emile Woolf International 5 The Institute of Chartered Accountants of Pakistan


Financial accounting and reporting I

❑ The factors mentioned earlier include the possibility that a counterparty may fail to fulfill its
liability to the entity (credit risk), or that the entity may fail to fulfill its liability (own credit risk).
❑ Because fair value is not derived, even in part, from the price of the transaction or other
event that gave rise to the asset or liability fair value is not increased by the transaction cost
incurred when acquiring the asset and is not decreased by the transaction cost incurred
when the liability is incurred are taken on. In addition, fair value does not reflect the
transaction cost that would be incurred on the ultimate disposal of the asset or not
transferring or settling the liability.
b) Value in use and fulfillment value
❑ Value in use is the present value of the cash flows or other economic benefit that an entity
expects to derive from the use of an asset and from its ultimate disposal. Fulfillment value is
the present value of the cash or other economic resources that an entity expects to be
obliged to transfer as it fulfills a liability. Those amounts of cash or other economic resources
include not only the amounts to be transferred to the liability counter party, but also the
amounts that the entity expects to be obliged to transfer to other parties to enable it to fulfill
the liability.
❑ Because value in use and fulfillment value are based on future cash flows they don’t include
transaction cost incurred on acquiring an asset or taking on a liability. However, value in use
and fulfillment value include the present value of any transaction cost that entity expects to
incur on the ultimate disposal of the asset or on fulfilling the liability.
❑ Value in use and fulfillment value reflect entity specific assumptions rather than assumptions
by market participants. In practice there may sometimes be little difference between the
assumptions that market participants would use and those that an entity itself would use.
❑ Value in use and fulfillment value cannot be observed directly and or determined using cash
flow based measurement techniques. Value in use and fulfillment value reflect the same
factors described for fair value earlier, but from an entity specific perspective rather than
from a market participant perspective.
c) Current cost
❑ The current cost of an asset is the cost of an equivalent asset at the measurement date
comprising the consideration that would be paid at the measurement date plus the
transaction cost that would be incurred at that date. The current cost of a liability is the
consideration that would be received for an equivalent liability at the measurement date
minus the transaction cost that would be incurred at that date. Current cost, like historical
cost is an entry value; it reflects prices in the market in which the entity would acquire the
asset or would incur the liability. Hence it is different from fair value in use and fulfillment
value, which are exit value. However, unlike historical cost, current cost reflects conditions
at the measurement date.
❑ In some cases, current cost cannot be determined directly by observing prices and in an
active market and must be determined directly by other means. For example, if prices are
available only for new asset the current cost of a used asset might need to be estimated by
adjusting the current price of a new asset to reflect the current age and condition of the asset
held by the entity.

1.5 Concepts of Capital and Capital maintenance


The Conceptual Framework states that there are two concepts of capital:
❑ A financial concept of capital;
❑ A physical concept of capital.
Different systems of accounts used different capital maintenance concepts. The choice of capital
maintenance has a profound effect on the measurement of profit.

© Emile Woolf International 6 The Institute of Chartered Accountants of Pakistan


Chapter 1: Accounting and reporting concepts

Consider the basic accounting equation.

Formula: Accounting equation


Assets = Liabilities + Equity Or Assets − Liabilities = Equity
A = L + E A − L = E
Net assets

Like any other equation, changes on one side of the accounting equation are matched by changes
in the other side. Therefore, Profit or loss for a period can be calculated from the difference between
the opening and closing net assets after adjusting for any distributions during the period.

Formula: Profit

Change in equity = Closing equity − Opening equity

Increase in equity = Profit + capital introduced − distributions

Profit = Increase in equity − capital introduced + distributions

This shows that the value ascribed to opening equity is crucial in the measurement of profit.
Financial capital maintenance
With the financial concept of capital maintenance, a profit is not earned during a period unless
the financial value of equity at the end of the period exceeds the financial value of equity at the
beginning of the period (after adjusting for equity capital raised or distributed).
Historical cost accounting is based on the concept of money financial capital maintenance.
Under this concept, an entity makes a profit when its closing equity exceeds its opening equity
measured as the number of units of currency at the start of the period. Note that this is a separate
issue from asset valuation. Assets could be revalued during the period but this would have no
effect on the opening capital position.
An alternative view of financial capital maintenance is used in constant purchasing power
accounting. This system is based on the concept of real financial capital maintenance. Under
this concept, an entity makes a profit when its closing equity exceeds opening equity premeasured
to maintain its purchasing power.
This requires the opening equity to be uplifted by the general inflation rate. This is achieved by a
simple double entry.

Illustration 01: Adjustment to maintain opening equity

Debit Credit

Statement of profit or loss X

Inflation reserve X

Physical capital maintenance


A physical concept of capital is that the capital of an entity is represented by its productive
capacity or operating capability. Where a physical concept of capital is used, the main concern of
users of the financial statements is with the maintenance of the operating capability of the entity.
With a physical concept of capital maintenance, a profit is not earned during a period unless
(excluding new equity capital raised during the period and adding back any distribution of dividends
to shareholders) the operating capability of the business is greater at the end of the period than at
the beginning of the period.

© Emile Woolf International 7 The Institute of Chartered Accountants of Pakistan


Financial accounting and reporting I

This requires the opening equity to be uplifted by the specific rates of inflation that apply to the
individual components of the net assets of the company. Again, this is achieved by the same simple
double entry.
The following example should help you to understand this.
Illustration 02: Capital maintenance concepts
X Limited commenced business on 1 January with a single item of inventory which costs Rs.
10,000.
During the year it sold the item for Rs. 14,000 (cash).
During the year general inflation was 5% but the inflation specific to the item was 10%.
Profit is calculated under each concept in the following ways.
Capital maintenance concept
Financial Financial (real
(money terms) terms) Physical
Statement of profit or loss Rs. Rs. Rs.
Revenue 14,000 14,000 14,000
Cost of sale (10,000) (10,000) (10,000)
Inflation adjustment (inflation rate applied to opening equity):
5% Rs.10,000 (500)
10% Rs.10,000 (1,000)
4,000 3,500 3,000

Statement of financial position Rs. Rs. Rs.


Net assets 14,000 14,000 14,000
Equity:
Opening equity
Before adjustment 10,000 10,000 10,000
Inflation reserve (see above) 500 1,000
After adjustment 10,000 10,500 11,000
Retained profit (profit for the year) 4,000 3,500 3,000
14,000 14,000 14,000

Commentary on the above illustration


Under historical cost accounting, the profit is Rs. 4,000. If the business paid this out as a dividend
it would have Rs. 10,000 left.
Rs. 10,000 is the opening equity expressed as a number of units of currency. This means that the
company would have maintained its equity expressed as a number of units of currency. However,
inflation in the period has caused the purchasing power of the currency to decline. This means that
Rs. 10,000 no longer has the same purchasing power that it had a year ago. The company has not
maintained its capital in real terms.
To maintain its opening equity in real terms the company would have to ensure that it had the same
purchasing power at the year-end as it had at the start. Inflation was 5% so the company would
need Rs. 10,500 at the year-end in order to have the same purchasing power as it had at the start
of the year. The company can achieve this by transferring Rs.500 from profit and loss into an
inflation reserve. Profit would then be reported as Rs. 3,500.

© Emile Woolf International 8 The Institute of Chartered Accountants of Pakistan


Chapter 1: Accounting and reporting concepts

If the business paid out Rs. 3,500 as a dividend it would have Rs. 10,500 left. This is not enough
to buy the same asset that it had at the start of the year. The asset has been subject to specific
inflation of 10% therefore the company would need Rs. 11,000 at the year-end in order to buy the
same asset.
This means that the company would not have the same capacity to operate as it had a year ago.
To maintain its opening equity in physical terms the company would have to ensure that it had the
same ability to operate at the year-end as it had at the start. In other words, it would need to have
Rs. 11,000. The company can achieve this by transferring Rs. 1,000 from profit and loss into an
inflation reserve. Profit would then be reported as Rs. 3,000.
Comparing the two concepts
Neither the IASB Conceptual Framework nor accounting standards require the use of a specific
capital maintenance concept. In practice, almost all entities use money financial capital
maintenance, but both concepts can provide useful information.
Financial capital maintenance is likely to be the most relevant to investors as they are interested
in maximizing the return on their investment and therefore its purchasing power.
Physical capital maintenance is likely to be most relevant to management and employees as they
are interested in assessing an entity’s ability to maintain its operating capacity. This is particularly
true for manufacturing businesses, where management may need information about the ability of
the business to continue to produce the same or a greater volume of goods.

Example 01: CRITERIA FOR RECOGNITION


❑ A manufacturing unit valuing Rs. 5 million, owned and controlled by the Company
❑ A fleet of trucks valuing Rs 100 million, controlled by another company
❑ A highly skilled workforce, getting an annual compensation of Rs. 12.5 million
Required: Which of the above assets will be recognized in the financial statements of a company
in accordance with the recognition criteria?
Answer:
The assets will be recognized in the financial statements of company in accordance with
conceptual framework as under:
❑ A manufacturing unit valuing Rs. 5 million, owned and controlled by the Company
❑ The fleet of truck will not be recognized because it is not controlled by the entity.
❑ Similarly, workforce will not be recognized by the entity because there is no certainty about the
probability of future economic benefits from workers as they can quit the entity at any time.

Example 02: HABIB FACTORY


Question: A factory building was purchased from Habib Factory by a company for Rs. 100 million
in 2009. Its useful life was estimated to be 25 years. The building was revalued in 2019 at Rs. 75
million.
It was decided to sell the factory building at Rs. 90 million after incurring repairs and painting work
done of Rs. 10 million.
Required: What will be the cost of the asset under each of the following measurement basis?
❑ Historical cost
❑ Current cost
❑ Realizable(settlement) value

© Emile Woolf International 9 The Institute of Chartered Accountants of Pakistan


Financial accounting and reporting I

Answer:
Historical cost
The historical cost of the building is Rs. 100 million.

Current cost
The current cost of the building shall be its revalued amount which is Rs. 75 million.
Realizable(settlement) value
The realizable value of the building will be its selling cost less any costs incurred to sell the asset,
which is 90 million less 10 million, i.e, Rs. 80 million.

Example 03: CARRIE


Question: Carrie starts in business on 1 January Year 1. Carrie’s sole shareholder contributed
capital of Rs. 1,000. Carrie purchased one item of inventory for Rs. 1,000 and sold that inventory
for cash of Rs. 1,400. At the end of Year 1 the replacement cost of the same item of inventory is
Rs. 1,100. General inflation during the year was 7%.
Required
Calculate the profit for the year and set out a summary statement of financial position as of 31
December Year 1 under the following capital maintenance concepts.
(a) Physical capital maintenance
(b) Financial capital maintenance
i. Historical cost accounting
ii. Constant purchasing power accounting
Answer:

(a) Physical (b) Financial Capital


Capital Maintenance
Maintenance
(ii) Constant
(i) Historical
purchasing
cost
power
accounting
Profit for the year accounting
Rs. Rs. Rs.
Sales 1,400 1,400 1,400
Cost of sales (1,000) (1,000) (1,000)
Inflation adjustment
- Specific (1,100 – 1,000) (100) - -
- General (1,000  7%) - - (70)
------------- ------------- -------------
Profit 300 400 330
------------- ------------- -------------
Balance sheet as at 31 December Year 1
Cash at bank 1,400 1,400 1,400
------------- ------------- -------------
Share capital (1,000 + 100) (1,000 + 70) 1,100* 1,000 1,070*
Reserves 300 400 330
------------- ------------- -------------
1,400 1,400 1,400
------------- ------------- -------------

© Emile Woolf International 10 The Institute of Chartered Accountants of Pakistan


Chapter 1: Accounting and reporting concepts

Tutorial note
Share capital at the year end is restated under the physical capital maintenance concept for an
increase in specific price changes and under Constant Purchasing Power accounting for general
price changes. This is the other side of the entry to the inflation adjustments in the statement of
profit or loss

Example 04:
Question: Read the following statements:
A. In case of conflict between requirements of conceptual framework and IFRS, the requirements
of conceptual framework shall prevail.
B. Conceptual framework is not an International financial reporting standard (IFRS)
C. HR related cost is recognized as an asset in the financial statements since economic benefit
is probable from human resource
D. Internally generated goodwill is recognized as asset and measured at fair value in the financial
statements
E. When economic benefits arise over several accounting periods, and the association with
income can only be decided in broad terms, expenses should be recognized in profit and loss
of each accounting period on the basis of systematic and rational allocation procedure
F. When an item of expenditure is not expected to provide any future economic benefit, it is
recognized as an asset in the financial statements
G. In fair value method, assets are measured at the amount that would be paid to purchase the
same or a similar asset currently.
Required:
Analyse the above statements as true or false along with reasons for the selected answer.

Answer:
A. False. In case of conflict between requirements of conceptual framework and IFRS, the
requirements of IFRS shall prevail being an established principle that specific law
requirements prevail over general law requirements.
B. True. Conceptual framework provides foundation for the IFRSs
C. False.HR related cost can never be capitalized as it does not meet the definition criteria of
asset “controlled by the entity”
D. False. Internally generated goodwill can never be recognized as it does not meet one of the
basic recognition criteria i.e. “The item should have a cost or value that can be measured
reliably”
E. True, because of matching principle
F. False. For any item to be recognized as an asset, it must be probable that an item shall
provide future economic benefits to the entity.
G. False. In current cost method assets are measured at the amount that would be paid to
purchase the same or a similar asset currently

Example 05:
Question: Read the following scenarios:
A. The method of measurement is relevant when an entity is not a going concern, and is faced
with liquidation
B. ABC has to incur dismantling cost amounting Rs. 100,000 after 3 years, however the cost is
recorded at Rs. 75,131
C. ABC has purchased PPE costing Rs. 10,000 to be depreciated over a period of 10 years. At the
end of year 3 the Book Value amounted to Rs. 7,000, however ABC considers measuring the
PPE at its current market value amounting Rs. 15,000.
Required: Identify the accounting concepts in each of the above scenarios:

© Emile Woolf International 11 The Institute of Chartered Accountants of Pakistan


Financial accounting and reporting I

Answer:
The accounting concepts are as follows:
A. Realizable value/ settlement value
B. Present value
C. Current cost

Example 06:
Question: ABC received Rs.160 000 in cash on 20 December 2004 from RM in return for having
provided financial advice during the 2004 financial year.
Required:
(a) Explain, with reference to the relevant definitions, which elements should possibly be
recognized in the 2004 financial year.
(b) Briefly identify whether and/ or how your answer would change if the cash received had
been received for financial advice to be provided in the 2005 financial year.
Answer: Part (a)
Income definition:
Income is defined as:
• An increase in economic benefits during the accounting period in the form of increases in
assets or decreases in liabilities resulting in increases in equity other than contributions from
equity participants.
Asset definition:
The essence of the definition of income requires that there is either an increase in assets or a
decrease in liabilities that results in an increase in equity during the year. The transaction involves
the receipt of cash (asset). Thus we will assess whether the transaction has increased our assets.
The definition of an asset is:
• a resource that is a right controlled by the entity as a result of a past event from which future
economic benefits are expected to flow to the entity.
Discussion of the asset definition:
The cash is a resource controlled by the entity (it is secured in ABC’s business account and thus
controllable). The cash was received in exchange for financial advice and since this advice was
provided during the 2004 year, this event represents a past event at reporting date. The cash is
able to be used to generate future income and thus future economic benefits are expected to flow
to the entity from the use of this asset. The cash thus meets the definition of an asset.
Discussion of the income definition:
The receipt of the cash during 2004 meets the definition of an asset and thus represents an
increase in assets. Since there was neither a concomitant increase in liabilities (ABC has not
obligations as a result of this receipt of cash) nor decrease in assets, the receipt of this asset will
have increased equity (using the equation: A = E + L). This increase in equity is not a contribution
from equity participants (e.g. it is not the receipt of cash in exchange for an issue of equity shares).
Thus the receipt of cash meets the definition of income.
Conclusion:
The receipt of cash in return for services rendered leads to an asset and income in 2004.
With reference to the relevant definitions, the elements that should possibly be recognized in the
2004 financial year are discussed as under.
Part (b)
If cash was received in 2004 for services to be provided in 2005, the asset definition would be met
but the income definition would not be. This is because although there is an increase in assets, it
would not lead to an increase in equity: the cash receipt would lead to an obligation to provide
services, which would meet the liability definition. An increase in both assets and liabilities results
in a nil effect on equity.

© Emile Woolf International 12 The Institute of Chartered Accountants of Pakistan


Chapter 1: Accounting and reporting concepts

Example 07:
Question: Read the following scenarios:
1. An amount paid to landlord totaling Rs.120,000 on 1st January 2012 against the rent for the
year ended 31st December 2012. Year end of the entity is 30 June 2012.
2. An expenditure incurred on repairs and maintenance of plant amounting Rs.300, 000.
3. There has been legal dispute between the entity and its customer and company expects the
outflow of Rs. 200,000 in order to settle the dispute.
4. Entity purchased goods costing Rs. 20,000 for trading purposes and the same was sold for
Rs. 25,000.
Required:
Which of the above, would be recognized as expense&/or asset in the financial statements of a
company in accordance with the criteria given in conceptual framework.
Answer:
1. Rent (60,000) – Expense and advance rent (60,000) - Asset
2. Asset, if the cost of the asset is material
3. Legal Charges- Expense
4. Goods(inventory) amounting to Rs. 20,000-Asset

Example 08:
Question: Read the following scenarios
1. Advance received from customer amounting Rs. 50,000 against the goods to be delivered
after 6 months
2. Services provided to ABC and Co. on credit amounting Rs.30, 000.
3. Account Receivables already written off in previous years amounting Rs. 30,000 were received
during the year.
Required:
Which of the above, would be recognized as income &/or liability in the financial statements of a
company in accordance with the criteria given in conceptual framework.

Answer:
1. Advance from customer amounting to Rs. 50,000 – Liability
2. Accounts receivable amounting to Rs. 30,000- Income
3. Reversal of bad debts amounting to Rs. 30,000-Income

© Emile Woolf International 13 The Institute of Chartered Accountants of Pakistan


Financial accounting and reporting I

Example 09: Measurement Bases


Question: Briefly describe the measurement bases that may be used to measure the value of
assets in the financial statements.
Answer:
(i) Historical cost
The historical cost of an asset, when it is acquired or created is the value of the cost incurred
in acquiring or creating the asset, comprising the consideration paid to acquire or create the
asset plus transaction cost.

(ii) Current value


▪ Current value measures provide monetary information about assets using information
updated to reflect conditions at the measurement date.
▪ Current value measurement bases include:
– Fair value
– Value in use for assets
– Current cost

Fair value

Fair value is the price that would be received to sell an asset in an orderly transaction
between market participants at the measurement date. Fair value reflects the perspective
of market participants.

Value in use

Value in use is the present value of the cash flows or other economic benefit that an entity
expects to derive from the use of an asset and from its ultimate disposal. Value in use reflect
entity specific assumptions rather than assumptions by market participants.

Current cost

The current cost of an asset is the cost of an equivalent asset at the measurement date
comprising the consideration that would be paid at the measurement date plus the
transaction cost that would be incurred at that date.

Current cost, like historical cost is an entry value; while fair value is an exit value. However,
unlike historical cost, current cost reflects conditions at the measurement date.

© Emile Woolf International 14 The Institute of Chartered Accountants of Pakistan


Chapter 1: Accounting and reporting concepts

2 OBJECTIVE BASED QUESTIONS


01. Which of the following measurement bases are referred to in the Board's Conceptual
Framework?
(a) Current Cost, Residual Value, Fair Value, Present Value
(b) Current Cost, Historical Cost, Fair Value, Present Value
(c) Current Cost, Fair Value, Present Value, Future Value
(d) Fair Value, Present Value, Future value, Residual Value

02. Financial capital maintenance (money terms) is also referred to as:


(a) Historical cost accounting
(b) Current cost accounting
(c) Constant purchasing power accounting
(d) Fair value accounting

03. Which of the following concepts measures profit in terms of an increase in the productive
capacity of an entity?
(a) Physical capital maintenance
(b) Historical cost accounting
(c) Financial capital maintenance
(d) Going concern concept

04. Which of the following statements is true about historical cost accounts in times of rising
prices?
(a) Profits will be overstated, and assets will be understated
(b) Asset values will be overstated
(c) Unrecognized gains will be recorded incorrectly
(d) Depreciation will be overstated

05. Which of the following criteria need to be satisfied in order for an element to be recognized
within the financial statements?
(i) It meets the definition of an element of the financial statements.
(ii) It is probable that future economic benefits will flow to or from the entity.
(iii) It is certain that future economic benefits will flow to or from the entity.
(iv) The item has a cost or value.
(v) The item has a cost or value that can be reliably measured.
(a) (i), (ii) and (v)
(b) (i), (iii) and (v)
(c) (i), (ii) and (iv)
(d) (i), (iii) and (iv)

© Emile Woolf International 15 The Institute of Chartered Accountants of Pakistan


Financial accounting and reporting I

06. Which of the following is NOT a purpose of the International Accounting Standards Board’s
Conceptual Framework?
(a) To assist the Board in the preparation and review of IFRS Standards.
(b) To assist auditors in forming an opinion on whether financial statements comply with
IFRS Standards.
(c) To assist in determining the treatment of items not covered by an existing IFRS
Standards.
(d) To be authoritative where a specific IFRS Standard conflicts with the Conceptual
Framework.

07. Which of the following items should be recognized as an asset in the statement of financial
position of an entity?
(a) A skilled and efficient workforce which has been very expensive to train. Some of
these staff is still employed by the entity.
(b) A highly lucrative contract signed during the year which is due to commence shortly
after the year-end.
(c) A government grant relating to the purchase of an item of plant several years ago
which has a remaining life of four years.
(d) A receivable from a customer which has been sold (factored) to a finance company.
The finance company has full recourse to the entity for any losses.

08. Which of the following criticisms does NOT apply to historical cost financial statements during
a period of rising prices?
(a) They contain mixed values, some items are at current values, some at out-of-date
values
(b) They are difficult to verify as transactions could have happened many years ago
(c) They understate assets and overstate profit
(d) They overstate gearing in the statement of financial position

09. Financial capital maintenance (real terms) is also referred to as:


(a) Historical cost accounting
(b) Current cost accounting
(c) Constant purchasing power accounting
(d) Fair value accounting

10. Physical capital maintenance is also referred to as:


(a) Historical cost accounting
(b) Current cost accounting
(c) Constant purchasing power accounting
(d) Fair value accounting

© Emile Woolf International 16 The Institute of Chartered Accountants of Pakistan


Chapter 1: Accounting and reporting concepts

11. In which of the following, no adjustment for inflation is considered?


(a) Financial capital maintenance (money terms)
(b) Financial capital maintenance (real terms)
(c) Physical capital maintenance
(d) Fair value accounting

12. In which of the following, inflation adjustment is made on general rate of inflation?
(a) Financial capital maintenance (money terms)
(b) Financial capital maintenance (real terms)
(c) Physical capital maintenance
(d) Fair value accounting

13. In which of the following, inflation adjustment is made on specific rate of inflation?
(a) Financial capital maintenance (money terms)
(b) Financial capital maintenance (real terms)
(c) Physical capital maintenance
(d) Fair value accounting

14. Financial capital maintenance is likely to be most relevant to:


(a) Investors
(b) Management and employees
(c) Neither (a) nor (b)
(d) Capital maintenance is always irrelevant to decision making

15. Physical capital maintenance is likely to be most relevant to:


(a) Investors
(b) Management and employees
(c) Neither (a) nor (b)
(d) Capital maintenance is always irrelevant to decision making

16. An entity made a profit of Rs. 350,000 for the year 2019 based on historical cost accounting
principles. It had opening capital of Rs. 1,000,000.
Specific price indices increase during the year by 20% and general price indices by 5%.
How much profit should be recorded for 2019 under money financial capital maintenance
concept?
Rs. ___________

© Emile Woolf International 17 The Institute of Chartered Accountants of Pakistan


Financial accounting and reporting I

17. An entity made a profit of Rs. 350,000 for the year 2019 based on historical cost accounting
principles. It had opening capital of Rs. 1,000,000.
Specific price indices increase during the year by 20% and general price indices by 5%.
How much profit should be recorded for 2019 under real financial capital maintenance
concept?
Rs. ___________

18. An entity made a profit of Rs. 350,000 for the year 2019 based on historical cost accounting
principles. It had opening capital of Rs. 1,000,000.
Specific price indices increase during the year by 20% and general price indices by 5%.
How much profit should be recorded for 2019 under physical capital maintenance concept?
Rs. ___________

19. An entity acquired an item of plant on 1 October 2012 at a cost of Rs. 500,000. It is being
depreciated over five years, using straight-line depreciation and an estimated residual value of
10% of its historical cost or current cost as appropriate. As at 30 September 2014, the
manufacturer of the plant still makes the same item of plant and its current price is Rs. 600,000.
What is the correct carrying amount to be shown in the statement of financial position as at 30
September 2014 under historical cost accounting?
Rs. ___________

20. An entity acquired an item of plant on 1 October 2012 at a cost of Rs. 500,000. It is being
depreciated over five years, using straight-line depreciation and an estimated residual value of
10% of its historical cost or current cost as appropriate. As at 30 September 2014, the
manufacturer of the plant still makes the same item of plant and its current price is Rs.
600,000.
What is the correct carrying amount to be shown in the statement of financial position as at 30
September 2014 under current cost accounting?
Rs. ___________

21. An entity made a profit of Rs. 480,000 for the year 2018 based on historical cost accounting
principles. It had opening capital of Rs. 1,100,000. During 2018, specific price indices increased
by 15% while general price indices increased by 12%. How much profit should be recorded for
2018 under real financial capital maintenance concept?
(a) Rs. 480,000
(b) Rs. 315,000
(c) Rs. 348,000
(d) Rs. 645,000

© Emile Woolf International 18 The Institute of Chartered Accountants of Pakistan


Chapter 1: Accounting and reporting concepts

22. Which of the following statements is correct about financial statements based on historical cost
in times of rising prices?
(a) Profits will be overstated and assets will be understated
(b) Assets will be overstated
(c) Profits as well as assets will be understated
(d) Depreciation will be overstated

© Emile Woolf International 19 The Institute of Chartered Accountants of Pakistan


Financial accounting and reporting I

2 OBJECTIVE BASED ANSWERS


01. (b)
02. (a)
03. (a) Physical capital maintenance looks at profit in terms of the physical productive
capacity of the business, taking into account specific price changes relevant to
the entity.
04. (a) In times of rising prices, asset values will be understated, as historical cost will
not be a true representation of the asset values. Additionally, the real purchase
cost of replacement items will not be
Incorporated, meaning that profits are overstated.
05. (a) There only has to be probable flow of economic benefits, rather than a certain
flow. Also, the cost or value must be capable of reliable measurement, or no
amount can be put into the financial statements.
06. (d) Where there is conflict between the conceptual framework and an IFRS
Standard, the IFRS Standard will prevail. An example of this is IAS 20
Government grants, where deferred grant income is held as a liability, despite
not satisfying the definition of a liability.
07. (d) As the receivable is ‘sold’ with recourse it must remain as an asset on the
statement of financial position and is not derecognized.
08. (b) Historical cost is the easiest to verify as the cost can be proved back to the
original transaction. Fair value is often more difficult to verify as it may involve
elements of estimation.
09. (c)
10. (b)
11. (a)
12. (b)
13. (c)
14. (a)
15. (b)
16. Rs. 350,000 Money financial capital maintenance looks at the actual physical cash. No
inflation adjustment is required.
17. Rs. 300,000 Rs. 350,000 – (1,000,000 x 5%) = Rs. 300,000
18. Rs. 150,000 Rs. 350,000 – (1,000,000 x 20%) = Rs. 150,000
19. Rs. 320,000 Historical cost annual depreciation = Rs. 90,000 ((500,000 × 90%)/5 years).
After two years carrying amount would be Rs. 320,000 = (500,000 -
(2×90,000)).
20. Rs. 384,000 Current cost annual depreciation = Rs. 108,000 ((600,000 × 90%)/5 years).
After two years carrying amount would be Rs. 384,000 = (600,000 -
(2×108,000)).
21. (c) Rs. 348,000
22. (a) Profits will be overstated and assets will be understated.

© Emile Woolf International 20 The Institute of Chartered Accountants of Pakistan


Certificate in Accounting and Finance

CHAPTER
Financial accounting and reporting I

IAS 1: Preparation of financial


statements

Contents
1 Statement of changes in Equity
2 Objective based questions and answers

* The student must refer original handbook of IFRS.

© Emile Woolf International 21 The Institute of Chartered Accountants of Pakistan


Financial accounting and reporting I

1 STATEMENT OF CHANGES IN EQUITY (SOCIE)


Section overview

◼ IAS 1 Preparation of Financial Statements


◼ The Statement of Changes in Equity (SOCIE)
◼ Purpose and Importance
◼ Elements of Statement of Changes in Equity
◼ Journal Entries
◼ Presentation of Statement of Changes in Equity

1.1 IAS 1 Preparation of Financial Statements


IAS 1 prescribes the basis for presentation of financial statements for the comparability of
company’s financial position to last year’s performance or with other entities. It sets the
requirement, guidelines for structure and presentation for the preparation of following statements.
A complete set of financial statements consists of:
❑ A statement of financial position (balance sheet) as at the end of the period;
❑ A statement of comprehensive income for the period;
❑ A statement of changes in equity for the period;
❑ A statement of cash flows for the period
Additionally, notes to these statements, consisting of a summary of significant accounting policies
used by the entity and other explanatory information; and comparative information also form part
of the financial statements.
IAS 1 - Presentation of Financial Statements specifies what published ‘general-purpose’ financial
statements should include, and provides a format for a statement of financial position, statement
of comprehensive income, and statement of changes in equity.
In your earlier studies you have studied about statement of financial position and statement of
comprehensive income. This chapter focus on the preparation of a statement of changes in equity.

1.2 The Statement of Changes in Equity (SOCIE)


A change in equity is simply the increase or decrease in the net assets of the entity.
The statement of changes in equity separates owner and non-owner changes in equity in the
following manner:
❑ Transactions with owners; and
❑ Non-owner changes in equity, referred to as total comprehensive income.

1.3 Purpose and Importance


The statement of changes in equity actually tells the users about the status of owner’s equity at the
beginning of the financial period, how it has changed during the year and the status of equity at
the end of the period.
It is important to note that the Statement of financial position provides information about the
financial position of the business by presenting total assets, liabilities and equity. Income
statement on the other hand provides information about how business has performed during the
year and how much income has been generated against the expenses and liabilities incurred.
But shareholders are interested in knowing how the business’ financial position and financial
performance has impacted their interest in the business. And this is not particularly addressed by
Statement of Financial Position or Income Statement.

© Emile Woolf International 22 The Institute of Chartered Accountants of Pakistan


Chapter 2: IAS 1: Preparation of financial statements

Thus the statement of changes in equity helps users of financial statement to identify the factors
that cause a change in the owners' equity over the accounting periods.
Therefore, through Statement of Changes in Equity users, especially shareholders can get great
insights about the effects of business operations and related factors on the wealth of the owners
invested in the business.
Examples of the information provided in the statement of changes in equity include share capital
issue and redemption during the period, gains and losses recognised outside profit or loss,
dividends, right shares and bonus shares issued during the period.

1.4 Elements of Statement of Changes in Equity


A statement of changes in equity shows for each component of equity the amount at the beginning
of the period, changes during the period, and its amount at the end of the period that includes:

A). Transactions with owners


The detail of transactions with owners is as under:
❑ Share capital
❑ Share premium
❑ Redemption
❑ Dividend
❑ Bonus shares
❑ Right issue
Share capital
Share capital is the sum of all funds raised by the company in the form of ordinary shares and
(irredeemable) preference shares. Preference shares confer preferential rights for their holders
such entitlement to a dividend out of profits before ordinary shareholders. Once the preference
dividend has been paid, the remaining profit 'belongs' to the ordinary shareholders at the discretion
of the board of directors of the company. The directors may decide to retain some profits (retained
earnings) within the company or declare the dividend.
Preference Shares
Preference shares are of two types: redeemable and irredeemable. Only irredeemable preference
shares are treated as share capital.
Redeemable Preference shares which the company is entitled to redeem in future are treated as
non-current liabilities
Types of Share Capital
Following are the different types of share capital:
Authorized Share Capital
It is the maximum amount of share capital that a company is authorized to raise. A company
does not usually issue the full amount of its authorized share capital. Instead, some of it is held in
reserve by the company for possible future use.
Issued Share Capital
A company may elect to only issue a portion of the total share capital with the plan of issuing more
shares at a later date. Issued share capital is the total value of the shares a company sells. The
issued share capital of a company is the par value of the shares that have actually been issued to
shareholders.
Subscribed Share Capital
When a company issues its authorised share capital, some part of it may not be subscribed by the
investors. Subscribed share capital is the monetary value of all the shares that the investors have
committed to buy.

© Emile Woolf International 23 The Institute of Chartered Accountants of Pakistan


Financial accounting and reporting I

Paid up Share Capital


Paid-up share capital is the amount of money a company has received from shareholders in
exchange for its shares.

Share premium
The difference between the par value of a company’s shares and the total amount a company
received for shares is called Share premium. It is a type of capital reserve.
Example: if a Rs. 10 share is sold for Rs, 12 then Rs. 2 is the share premium.

Redemption
It is the reacquisition of the entity’s own equity instruments. A company may redeem its shares for
a number of reasons such as to buy out certain shareholders or to provide an exit strategy to a
third party investor.

Dividend
It is the distribution of profits to shareholders.
Many companies pay dividends in two stages during the course of their accounting year.
(a) In mid-year, after the half-year financial results are published, the company might pay an
interim dividend.
(b) At the end of the year, the company might propose a further final dividend.
According to financial reporting framework the dividends are incorporated as under
(a) In case of interim dividend, it must be incorporated in the statement of changes in equity of
same year.
(b) In case of final dividend, the final dividend of current year shall be incorporated in the statement
of changes of the next year provided that it has been declared by Directors and approved by
shareholders in annual general meeting. In the current year only disclosure note is given for
final dividend.

Bonus shares
These shares are distributed by a company to its current shareholders free of charge. A bonus
issue does not involve any cash inflow. The company converts some of its reserves (share
premium or retained earnings or both) into new fully-paid share capital issued at its par value.

Right issue
It is an invitation to existing shareholders to purchase additional new shares in proportion to their
shareholding in the company at a discount to the market price on a stated future date.
Say a company with a paid up capital of 10 million shares raises funds by issuing 2 million new
shares. It can offer the new shares to existing shareholder in a '1 for 5' rights issue: each existing
shareholder is offered one new share for every five shares currently held (10 million/2 million = 5).

B). Total comprehensive income


The detail of total comprehensive income is as under:
❑ Profit for the year & Retained Earnings
❑ Revaluation Surplus
Profit for the Year & Retained Earnings
Retained earnings comprise the income (profits and gains less losses) that the company retains
within the business, i.e., income that has not been paid out as dividends or transferred to any other
reserve. A company might hold retained earnings that it has no intention of distributing to owners
as a dividend at any time in the future in a general reserve rather than in retained earnings.

© Emile Woolf International 24 The Institute of Chartered Accountants of Pakistan


Chapter 2: IAS 1: Preparation of financial statements

A debit balance on the retained earnings account indicates that the company has accumulated
losses.
Revaluation surplus
The result of an upward revaluation of a non-current asset is a 'revaluation surplus'. The amount
accumulated in revaluation surplus is non-distributable, as it represents un-realised profits on the
revalued assets. It is another capital reserve. The revaluation surplus may diminish if an asset
which had previously been revalued upwards is devalued later. Revaluation surplus is transferred
to retain earnings in case the asset is sold. If due to upward revaluation the incremental
depreciation occurred, then that incremental balance shell be transferred to retained earnings from
the revaluation surplus.
General Reserves
General Reserves are non-statutory reserves consisting of profits which are distributable as
dividends at the discretion of the company. These are also called revenue reserves.
Profits are transferred to these reserves by making an appropriation out of profits, usually profits
for the year.

Illustration 01:
Profit after taxation 300,000
Appropriations of profit:
Dividend (100,000)
Transfer to general reserve (50,000)
(150,000)
Retained earnings for the year 150,000
Retained earnings b/f 500,000
Retained earnings c/f 650,000

1.5 Journal Entries


Following journal entries are made in order to record the effects of changes in elements of equity
during the period.
Ref Account / Description Debit Credit
1 Bank X
Share Capital X
Issue of new shares/ right shares at par value
2 Bank X
Share capital X
Share Premium X
Issue of new shares/ right shares at premium
3 Retained earnings X
Share capital X
Issue of bonus shares
4 Share premium X
Share capital X
Issue of bonus share
(If share premium balance is available)
5 Retained earnings X
Dividend payable X
On approval of interim / final dividend
6 Dividend payable X
Bank X
Payment of interim/final dividend

© Emile Woolf International 25 The Institute of Chartered Accountants of Pakistan


Financial accounting and reporting I

Ref Account / Description Debit Credit


7 Retained earnings X
General reserves X
Transfer of balance to general reserves
8 Revaluation surplus X
Retained earnings X
Transfer of balance in case of incremental depreciation of
revalued assets
9 Revaluation surplus X
Retained earnings X
Transfer of balance in case of disposal of revalued asset
10 Other comprehensive income X
Retained earnings X
Transfer of balance to retained earnings
11 Share capital X
Bank X
Payment of cash for redemption of shares (at par)
12 Retained earnings X
Capital repurchase reserve account X
Transfer from distributable reserves to capital repurchase reserve account on
redemption.

1.6 Presentation of Statement of Changes in Equity


In accordance with the requirements of IAS 1, an entity is required to present a statement of
changes in equity which includes the following information:
(a) total comprehensive income for the period, showing separately the total amounts attributable
to owners of the parent and to non-controlling interests;
(b) for each component of equity, a reconciliation between the carrying amount at the beginning
and the end of period separately disclosing changes resulting from:
• profit or loss;
• other comprehensive income; and
• transactions with owners in their capacity as owners showing separately
contributions by and distributions to owners

Format of Statement of Changes in Equity


The format is explained through following illustrations

Illustration 02:
Statement of changes in equity for the year ended 31 December 2019
Share Share Retained Revaluation
Total
Capital Premium earnings Surplus
------------------------- Rs. in million ------------------------
Balance as at 1 January 2019 100 10 60 5 175
Profit after tax for the year - - 20 - 20
Revaluation loss - - - (3) (3)
Balance as at 31 December 2019 100 10 80 2 192

© Emile Woolf International 26 The Institute of Chartered Accountants of Pakistan


Chapter 2: IAS 1: Preparation of financial statements

Illustration 03:
Statement of changes in equity for the year ended 31 December 2019 considering there is
redemption, bonus and right issues along with dividends paid during the year
Capital
Share Share Retained
repurchase Total
Capital Premium earnings
reserve
---------- Rs. in million -------
Balance as at 1 January 2019 100 10 - 60 170
Profit after tax for the year - - - 20 20
Bonus issue 10 - - (10) -
Right issue 5 - - - 5
Redemption (10) - 10 (10) (10)
Dividends paid during the year - - - (20) (20)
Balance as at 31 December 2019 105 10 10 40 165

Example 01: SMS Limited


Question: The equity of SMS Ltd as on December 31, 2018 is as follows:
Rs. in million
Total equity at the beginning of the year:
- Share Capital (@ Rs. 10 fully paid ordinary shares) 3,000
- Share premium 1,900
- Retained earnings 4,500
Profit for the year 400
Dividends declared and paid (300)
Total equity at the end of the year 9,500

The company made a bonus issue of 2 to 1.


Required:
The Statement of Changes in Equity of SMS Ltd for the year ended on December 31, 2019.

Answer:

Statement of Changes in equity for the year ended on December 31, 2019
Share Share Retained
Total
Capital Premium earnings
------------------------- Rs. in million ------------------------
Balance as at 1 January 2019 3,000 1,900 4,500 9,400
Profit after tax for the year - - 400 400
Bonus shares 6,000 (1,900) (4,100) -
Dividends - - (300) (300)
Balance as at 31 December 2019 9,000 - 500 9,500

© Emile Woolf International 27 The Institute of Chartered Accountants of Pakistan


Financial accounting and reporting I

Example 02: Pak Ocean Limited (POL)


Question: The following information pertains to draft financial statements of Pak Ocean Limited
(POL) for the year ended 31 December 2018.

2018
Shareholders' equity as at 1 January 2018 was as follows: Rs in million
Share capital (Rs. 100 each) 200
Retained earnings 45
Profit after tax 78
Other comprehensive income (gain on revaluation) 12
Incremental depreciation on revaluation of property, plant and 1.5
equipment

On 30 November 2018, POL issued 25% right shares to its ordinary shareholders at Rs. 120
per share.
Cash dividend @ 18% for the year ending 2017 was declared in January 2018
Bonus issue declared during the year ended 31 December 2018:
❑ Interim(declared with half yearly accounts) 10%
❑ Final 25%

Required:
Prepare the statement of changes in equity for the year ended December 31, 2018 for Pak Ocean
Limited.

Answer:

Statement of Changes in Equity for the year ended 31 December 2018


Share Share Retained Revaluation
Total
capital premium earnings Surplus
------------- Rupees in million -------------
Balance as at 1 January 2018 200 - 45 - 245
Total comprehensive income 78 12 90
Transfer from surplus on
revaluation of incremental
depreciation for the period 1.5 (1.5) -
Final cash dividend at 18% for
the year ended 31 December
2017 (200 × 18%) (36) (36)
Interim bonus issue at 10% for
the year ended 31 December
2018 (200 × 10%) 20 - (20) -
25% Right issue at a premium
of Rs. 20 per share (200 + 20)
× 25% 55 11 - 66
Balance as at 31 December
2018 275 11 68.5 10.5 365

© Emile Woolf International 28 The Institute of Chartered Accountants of Pakistan


Chapter 2: IAS 1: Preparation of financial statements

Example 03: Daffodil Limited (DL)


Question: For the purpose of preparation of statement of changes in equity for the year ended 31
December 2017, Daffodil Limited (DL) has extracted the following information:

2017 2016 2015

Draft Audited Audited

-------------- Rs. in million -------------

Net profit 650 318 214

Transfer to general reserves 112 - 141

Transfer of incremental depreciation - 49 55

Final cash dividend - - 7.5%

(i) Details of share issues:


❑ 25% right shares were issued on 1 May 2016 at Rs. 18 per share. The market price per
share immediately before the entitlement date was also Rs. 18 per share.
❑ A bonus issue of 10% was made on 1 April 2017 as final dividend for 2016.
❑ 50 million right shares were issued on 1 July 2017 at Rs. 15 per share. The market price
per share immediately before the entitlement date was Rs. 25 per share.
❑ A bonus issue of 15% was made on 1 September 2017 as interim dividend.
❑ The asset which was revalued upward in previous years has been sold on 15 October, 2017.

(ii) Share capital and reserves as at 31 December:

2015 2014

------ Rs. in million ------

Ordinary share capital (Rs. 10 each) 1,600 1,600

General reserves 1,850 1,709

Retained earnings 1,430 1,302

Revaluation surplus 75 -

Required:
Prepare DL’s statement of changes in equity for the year ended 31 December 2017 along with
comparative figures.

© Emile Woolf International 29 The Institute of Chartered Accountants of Pakistan


Financial accounting and reporting I

Answer:
Statement of changes in equity for the year ended 31 December 2017

Ordinary Share General Retained Revaluation


Total
share capital premium reserves earnings Surplus

-------------------- Rs. in million --------------------

Balance as at 31 December 2015 1,600 1,850 1,430 75 4,955

Final cash dividend @ 7.5% - 2015 (120) (120)


(1,600×7.5%)

Right issue @ 25% 400 320 720

(1,600×25%) (160×25%×8)

Net profit – 2016 318 318

Transfer of Incremental (49)


49 0
depreciation

Balance as at 31 December 2016 2,000 320 1,850 1,677 26 5,873

Final bonus dividend @ 10% - 2016 200 (200) -


(2,000×10%)

Right issue 500 250.00 750.00


(50×10) (50×5)

Interim bonus dividend @ 15% - 400 (405) -


2017 (2,700×15%)

Net profit – 2017 650 650

Transfer of revaluation surplus due 26 (26) 0


to disposal of asset

Transfer to general reserves 112 (112) -

Balance as at 31 December 2017 3,105 570.00 1,962 1,636 0 7,273

© Emile Woolf International 30 The Institute of Chartered Accountants of Pakistan


Chapter 2: IAS 1: Preparation of financial statements

Example 04: MK Corporation Limited


Question: MK Corporation Limited, an entity listed in Pakistan Stock Exchange is in the business of
manufacturing and sale of yarn products. Company year-end is December. Below is the relevant
information given:

Opening balances as at January 01, 2018 Rs.

Share Capital (at par value of Rs. 10 per share) 25,000,000

Share Premium 7,500,000

General Reserves 750,000

Retained Earnings 18,250,000

Revaluation Surplus 1,500,000

Following events have taken place in year 2018 and 2019:


1. On March 31, 2018, Company issued Right shares for Rs. 20 per share. Right shares were
issued in the proportion of 1 right share against 5 ordinary shares held.
2. Board of Directors of the Company approved Interim dividend of Rs. 2.25 per share for the half
year ended June 30, 2018.
3. Annual profit for the year ended December 31, 2018 is Rs. 10,250,000.
4. The Board of Directors of the Company recommended annual dividend of Rs. 4.25 per share
on February 15, 2019, which was duly approved by the Shareholders on March 21, 2019.
5. The Board of Directors approved Bonus Shares of 20% of the outstanding shares on June 30,
2019 which were duly credited in Shareholders account on August 31, 2019.
6. Board of Directors of the Company approved Interim dividend of Rs. 1.25 per share for the
third quarter ended September 30, 2019.
7. Annual profit for the year ended December 31, 2019 is Rs. 12,500,000.
8. The Board of Directors of the Company recommended annual dividend of Rs. 5 per share on
February 15, 2020, which was duly approved by the Shareholders on March 21, 2020.
9. The Board of Director approved a transfer 5% of the Annual Profit to General Reserves.
10. Company revalued Fixed assets on December 31, 2017 resulting in Revaluation Surplus of Rs.
1,500,000. Remaining useful life of the Asset is 10 years and Company has a straight line
method for Depreciation.
Required:
Prepare statement of changes in equity for the year ended December 31, 2018 and 2019.

© Emile Woolf International 31 The Institute of Chartered Accountants of Pakistan


Financial accounting and reporting I

Answer:
Statement of changes in equity
Share Share General Retained Revaluation
Description Total
Capital Premium Reserves Earnings Surplus
Rupees
Balance as at Jan
01, 2018 25,000,000 7,500,000 750,000 18,250,000 1,500,000 53,000,000
Issuance of Right
Shares 5,000,000 5,000,000 - - - 10,000,000
Interim Dividend (6,750,000) (6,750,000)
Profit for the year 10,250,000 10,250,000
Transfer 512,500 (512,500) -
Transfer of Revaluation
Surplus to Retained
Earnings 150,000 (150,000) -
Balance at the end of
the year 30,000,000 12,500,000 1,262,500 21,387,500 1,350,000 66,500,000
Annual Dividend (12,750,000) (12,750,000)
Bonus Shares 6,000,000 (6,000,000) - -
Interim Dividend (4,500,000) (4,500,000)
Profit for the year 12,500,000 12,500,000
Transfer 625,000 (625,000) -
Transfer of Revaluation
Surplus to Retained
Earnings 150,000 (150,000) -
Balance at the end of
the year 36,000,000 6,500,000 1,887,500 16,162,500 1,200,000 61,750,000

Example 05: ABC Corporation Limited 1


Question: ABC Corporation Limited, an entity listed in Pakistan Stock Exchange is in the business of
manufacturing and sale of Cars. Company year-end is December. Below is the relevant information
given:

Opening balances as at January 01, 2018: Rs.


Share Capital (at par value of Rs. 10 per share) 100,000,000
Share Premium 50,000,000
General Reserves 5,000,000
Retained Earnings 55,000,000

Following events have taken place in year 2018 and 2019:


1) On February 28, 2018, the Company issued Initial Public Offering, thereby offering 5 million
shares for Rs. 25 each. All shares were subscribed and company received the subscription
money by March 31, 2018.

© Emile Woolf International 32 The Institute of Chartered Accountants of Pakistan


Chapter 2: IAS 1: Preparation of financial statements

2) The Board of Directors of the Company recommended annual dividend for the year ended
December 31, 2017 of Rs. 5 per share on February 15, 2018, whereas Shareholders only
approved Rs. 4 per share on March 31, 2018.
3) Board of Directors of the Company approved Interim dividend of Rs. 1 per share for the third
quarter ended September 30, 2018.
4) Annual profit for the year ended December 31, 2018 is Rs. 130,250,000.
5) The Board of Directors of the Company recommended annual dividend of Rs. 4.25 per share
on February 15, 2019, whereas Shareholders approved Rs. 6 per share on March 31, 2019.
6) The Board of Directors approved Bonus Shares of 10% of the Outstanding shares on June 30,
2019.
7) Board of Directors of the Company approved Interim dividend of Rs. 1.5 per share for the third
quarter ended September 30, 2019.
8) Annual profit for the year ended December 31, 2019 is Rs. 175,000,000.
9) The Board of Directors of the Company recommended annual dividend of Rs. 5.5 per share
on February 15, 2020, which was duly approved by the Shareholders on March 31, 2020.
10) The Board of Directors approved a transfer 5% of the Annual Profit to General Reserves.
Required:
Prepare statement of changes in equity for the year ended December 31, 2018 and 2019.

Answer:
The statement of changes in equity as on December 31, 2018-2019 is as under:

Share Share General Retained


Description Total
Capital Premium Reserves Earnings
Rupees
Balance as at Jan 01,
2018 100,000,000 50,000,000 5,000,000 55,000,000 210,000,000
Issuance of Shares under
IPO 50,000,000 75,000,000 - - 125,000,000
Annual Dividend 17 (40,000,000) (40,000,000)
Interim Dividend (15,000,000) (15,000,000)
Profit for the year 130,250,000 130,250,000
Transfer 6,512,500 (6,512,500) -
Balance at the end of the
year 150,000,000 125,000,000 11,512,500 123,737,500 410,250,000
Annual Dividend (63,750,000) (63,750,000)
Bonus Shares 15,000,000 (15,000,000) -
Interim Dividend (24,750,000) (24,750,000)
Profit for the year 175,000,000 175,000,000
Transfer 8,750,000 (8,750,000) -
Balance at the end of the
year 165,000,000 110,000,000 20,262,500 201,487,500 496,750,000

© Emile Woolf International 33 The Institute of Chartered Accountants of Pakistan


Financial accounting and reporting I

Example 06: ABC Corporation Limited 2


Question: ABC Corporation Limited, an entity listed in Pakistan Stock Exchange is in the business of
manufacturing and sale of Cars. Company year-end is December. Below is the relevant information
given:

Opening balances as at January 01, 2018 Rs.


Share Capital (at par value of Rs. 10 per share) 100,000,000
Share Premium 50,000,000
General Reserves 5,000,000
Retained Earnings 55,000,000

Following events taken place in year 2018 and 2019:


1) On February 28, 2018, the Company issued Initial Public Offering, thereby offering 5 million
shares for Rs. 25 each. Only 75% shares were subscribed and company received the
subscription money by March 31, 2018.
2) The Board of Directors of the Company recommended annual dividend for the year ended
December 31, 2017 of Rs. 5 per share on February 15, 2018, whereas Shareholders only
approved Rs. 4 per share on March 31, 2018.
3) Board of Directors of the Company approved Interim dividend of Rs. 1 per share for the third
quarter ended September 30, 2018.
4) Annual profit for the year ended December 31, 2018 is Rs. 175,000,000.
5) The Board of directors has declared a final dividend for the year ended December, 31,2018
at the rate of 1.5 per share on February,15,2019, which was duly approved by the
shareholders in annual general meeting on March 21, 2019
6) The Board of Directors approved Bonus Shares of 10% of the Outstanding shares on June 30,
2019 which were duly credited in Shareholders account on August 31, 2019.
7) Board of Directors of the Company approved Interim dividend of Rs. 1.5 per share for the third
quarter ended September 30, 2019.
8) Annual profit for the year ended December 31, 2019 is Rs. 185,250,000.
9) The Board of Directors of the Company has approved a transfer 5% of the Annual Profit to
General Reserves.

Required:
Prepare statement of changes in equity for the year ended December 31, 2018 and 2019.

© Emile Woolf International 34 The Institute of Chartered Accountants of Pakistan


Chapter 2: IAS 1: Preparation of financial statements

Answer:

The statement of changes in equity as on December 31, 2018-2019 is as under:

Share Share General Retained


Description Total
Capital Premium Reserves Earnings
Rupees
Opening Balance 100,000,000 50,000,000 5,000,000 55,000,000 210,000,000
Issuance of Shares under
IPO 37,500,000 56,250,000 - - 93,750,000
Annual Dividend 17 (40,000,000) (40,000,000)
Interim Dividend (13,750,000) (13,750,000)
Profit for the year 175,000,000 175,000,000
Transfer 8,750,000 (8,750,000) -
Balance at the end of the
year 137,500,000 106,250,000 13,750,000 167,500,000 425,000,000
Annual Dividend (22,687,500) (22,687,500)
Bonus Shares 13,750,000 (13,750,000) -
Interim Dividend (22,687,500) (22,687,500)
Profit for the year 185,250,000 185,250,000
Transfer 9,262,500 (9,262,500) -
Balance at the end of the
year 151,250,000 92,500,000 23,012,500 298,112,500 564,875,500
.

© Emile Woolf International 35 The Institute of Chartered Accountants of Pakistan


Financial accounting and reporting I

Example 07: Wednesday Limited


Question: The following information pertains to Wednesday Limited (WL) for the year ended 30 June
2019:
(i) Shareholders' equity as at 1 July 2018:

Rs. in million
Share capital (Rs. 100 each) 200
Share premium 85
Retained earnings 124
Revaluation surplus 65
(ii) On 30 November 2018, WL issued 30% right shares at a premium of Rs. 120 per share.
(iii) Cash dividend and bonus shares for the last two years:

Final dividend *Interim dividend


For the year ended
Cash Bonus Cash Bonus
30 June 2018 18% - 20% -
30 June 2019 - 25% - 10%
*Declared with half yearly accounts
(iv) Profit for the year amounted to Rs. 95 million.
(iv) Revaluation surplus arising during the year amounted to Rs. 35 million whereas transfer of
incremental depreciation for the year was Rs. 9 million.
Required:
Prepare WL’s Statement of Changes in Equity for the year ended 30 June 2019. (Column for total
and comparative figures are not required)

Answer:
Wednesday Limited
Statement of changes in equity
For the year ended 30 June 2019

Share Share Retained Revaluation


capital premium earnings surplus

--------------- Rs. in million ---------------


Balance as at 1 July 2018 (As given) 200 85 124 65
Final cash dividend 200 x 18% (36)
Right issue @ 30% 60 72
Interim bonus dividend 260 x 10% 26 (26)
Total comprehensive income 95 35
Transfer of incremental depreciation 9 (9)
Balance as at 30 June 2019 286 157 166 91
.

© Emile Woolf International 36 The Institute of Chartered Accountants of Pakistan


Chapter 2: IAS 1: Preparation of financial statements

2 OBJECTIVE BASED QUESTIONS


01. Which TWO of the following are separately identified in statement of changes in equity?
(a) Profit for the year
(b) Transactions with owners
(c) Other comprehensive income
(d) Non-owner changes in equity

02. Which of the following is not considered transaction with owners with reference to statement of
changes in equity?
(a) Share capital
(b) Redemption of equity shares
(c) Profit for the year
(d) Bonus issue of shares (no cash received from owners)

03. The maximum amount of share capital that a company is authorized to raise is called:
(a) Authorized share capital
(b) Issued share capital
(c) Subscribed share capital
(d) Paid up share capital

04. The total value of shares a company offers to subscribe is called:


(a) Authorized share capital
(b) Issued share capital
(c) Subscribed share capital
(d) Paid up share capital

05. The monetary value of all the shares that the investors have committed to buy is called:
(a) Authorized share capital
(b) Issued share capital
(c) Subscribed share capital
(d) Paid up share capital

06. The amount of money a company has received from shareholders in exchange for its shares is
called:
(a) Authorized share capital
(b) Issued share capital
(c) Subscribed share capital
(d) Paid up share capital

© Emile Woolf International 37 The Institute of Chartered Accountants of Pakistan


Financial accounting and reporting I

07. Which of the following issued by an entity is treated as liability?


(a) Ordinary share capital
(b) Redeemable preference share capital
(c) Irredeemable preference share capital
(d) None of above

08. A debit balance on the retained earnings account indicates that:


(a) The company has made more dividend payments than the profit earned.
(b) the company has accumulated losses
(c) the company has redeemed some of its share capital
(d) the company has issued bonus shares

09. A company has profit after tax of Rs. 80 million for the financial year ended on 30 June 2019. It
has share capital of Rs. 500 million. During the year company has declared interim dividend of
10%.
How this dividend shall be presented in financial statements for the year ended 30 June 2019?
(a) Rs. 8 million deducted from retained earnings in statement of changes in equity
(b) Rs. 50 million deducted from retained earnings in statement of changes in equity
(c) Rs. 50 million deducted from profit or loss as finance cost
(d) It shall not be recorded, only disclosure shall be made.

10. A company has profit after tax of Rs. 80 million for the financial year ended on 30 June 2019. It
has share capital of Rs. 500 million. The board of directors proposed a final dividend of 10% just
after the year end, for the year ended 30 June 2019
How this dividend shall be presented in financial statements for the year ended 30 June 2019?
(a) Rs. 8 million deducted from retained earnings in statement of changes in equity
(b) Rs. 50 million deducted from retained earnings in statement of changes in equity
(c) Rs. 50 million deducted from profit or loss as finance cost
(d) It shall not be recorded, only disclosure shall be made.

11. Which TWO of the following are usually shown in statement of changes in equity when right issue
of shares is made?
(a) Increase in share capital
(b) Decrease in share premium
(c) Increase in share premium
(d) Increase in retained earnings

12. Which TWO of the following are usually shown in statement of changes in equity when bonus
issue of shares is made?
(a) Increase in share capital
(b) Decrease in share premium
(c) Increase in share premium
(d) Increase in retained earnings

© Emile Woolf International 38 The Institute of Chartered Accountants of Pakistan


Chapter 2: IAS 1: Preparation of financial statements

13. Transaction costs relating to issue of shares are usually debited to:
(a) Profit or loss
(b) Share capital
(c) Share premium
(d) Revaluation surplus

14. If there is no balance in share premium account, transaction costs relating to issue of shares are
usually debited to:
(a) Profit or loss
(b) Share capital
(c) Retained earnings
(d) Revaluation surplus

15. Incremental depreciation has following effects on statement of changes in equity:


(a) Increase in revaluation surplus and decrease in retained earnings
(b) Decrease in revaluation surplus and increase in retained earnings
(c) Decrease in revaluation surplus and decrease in retained earnings
(d) No effect

16. A company has following balances on 1 January 2019:


Rs. m
Share capital (Rs. 100 each) 100
Share premium 30
Revaluation surplus 20
Retained earnings 35
The company made a right issue of 1 for 5 shares already held at Rs. 145 per share.
What amount of share capital shall be presented in statement of changes in equity as at 31
December 2019?

Rs. ___________

17. A company has following balances on 1 January 2019:


Rs. m
Share capital (Rs. 100 each) 100
Share premium 30
Revaluation surplus 20
Retained earnings 35
The company made a right issue of 1 for 5 shares already held at Rs. 145 per share.
What amount of share premium shall be presented in statement of changes in equity as at 31
December 2019?

Rs. ___________

© Emile Woolf International 39 The Institute of Chartered Accountants of Pakistan


Financial accounting and reporting I

18. A company has following balances on 1 January 2019:

Rs. m
Share capital (Rs. 100 each) 100
Share premium 30
Revaluation surplus 20
Retained earnings 35

The company made a bonus issue of 2 for 5 shares already held.


What amount of share capital shall be presented in statement of changes in equity as at 31
December 2019?

Rs. ___________

19. A company has following balances on 1 January 2019:


Rs. m
Share capital (Rs. 100 each) 100
Share premium 30
Revaluation surplus 20
Retained earnings 35

The company made a bonus issue of 2 for 5 shares already held.


What amount of share premium shall be presented in statement of changes in equity as at 31
December 2019?

Rs. ___________

20. A company has following balances on 1 January 2019:


Rs. m
Share capital (Rs. 100 each) 100
Share premium 30
Revaluation surplus 20
Retained earnings 35

On 2 January 2019, all the revalued assets were disposed of for Rs. 90 million.
Profit for the year ended was Rs. 32 million.
Interim dividend of 5% was paid in July 2019 and final dividend of 8% has been proposed by
directors.
What amount of retained earnings shall be presented in statement of changes in equity as at 31
December 2019?

Rs. ___________

21. Which of the following does not appear in statement of changes in equity?
(a) Share premium
(b) Retained earning
(c) Goodwill
(d) Revaluation surplus

© Emile Woolf International 40 The Institute of Chartered Accountants of Pakistan


Chapter 2: IAS 1: Preparation of financial statements

22. Which of the following statements is likely to be true, for a company making profits?
(a) The operating profit will be less than the profit for the year.
(b) The profit for the year will be greater than the gross profit.
(c) Retained profits at the year-end will be greater than shareholders' equity.
(d) Retained profits at the year-end will be greater than retained profits at the beginning of
the year.

23. Which of the following is NOT a component of the statement of changes in equity?
(a) Total comprehensive income for the period
(b) The revaluation gain
(c) The amount of cash that the company has on hand
(d) Dividends paid to shareholders during the period

24. Which of the following statements is not true about preferred stock?
(a) The rate of dividend is usually fixed
(b) Shareholders always have a voting right
(c) Shareholders' usually have a preference as to assets upon liquidation of the corporation
(d) Shareholders' usually have a preference as to dividends

25. Redeemable preferred shares is required to be reported as:


(a) Liability
(b) Equity
(c) Asset
(d) None of the above

26. Any unpaid dividend is carried forward to the future periods for which type of stock?
(a) Ordinary shares
(b) Cumulative preferred shares
(c) Non-cumulative preferred shares
(d) All of the above

27. What is the impact of dividend payments to shareholders on the statement of changes in equity?
(a) It increases the retained earnings balance
(b) It decreases the retained earnings balance
(c) It increases the share capital balance
(d) It decreases the share capital balance

28. What is the impact of an additional share issue on the statement of changes in equity?
(a) It increases the share capital balance
(b) It increases the retained earnings balance
(c) It decreases the share capital balance
(d) It decreases the retained earnings balance

© Emile Woolf International 41 The Institute of Chartered Accountants of Pakistan


Financial accounting and reporting I

29. Xavier Limited issued 5,000 shares of its Rs.10 par value to its shareholder. These shares were
issued at a premium at a price of Rs.25 per share.
The correct journal entry to record this transaction is:
(a) Cash Rs.125,000 (Debit); Share capital Rs.125,000 (Credit)
(b) Cash Rs.50,000 (Debit); Share capital Rs.50,000 (Credit)
(c) Share capital Rs.50,000 (Debit); Share premium Rs.75,000 (Debit); Cash Rs.125,000
(Credit)
(d) Cash Rs.125,000 (Debit); Share capital Rs.50,000 (Credit); Share premium Rs.75,000
(Credit)

30. Dynasty Limited issues 1 million, Rs.10 shares at Rs.50 for each share. Which of the following
statements is true?
(a) Ordinary share capital will increase by Rs.10 million and share premium will increase by
Rs.50 million.
(b) Ordinary share capital will increase by Rs.10 million and share premium will increase by
Rs.40 million.
(c) Ordinary share capital will increase by Rs.20 million and share premium will increase by
Rs.50 million.
(d) Ordinary share capital will increase by Rs.10 million and share premium will increase by
Rs.30 million.

31. Handsome Limited statement of financial position shows ordinary share capital of Rs.150 million
and share premium of Rs.50 million at the beginning of a financial year. If the ordinary share
capital is Rs.250 million and share premium is Rs.120 million at the end of the financial year, how
much did the ordinary share with share premium issue raise?
(a) Rs.100 million
(b) Rs.150 million
(c) Rs.160 million
(d) Rs.170 million

32. Gigantic Limited opening retained earning balance was Rs.150 million. It made a net profit for the
year ended 31 March 2020 of Rs.30 million. During that year, an ordinary dividend of Rs.50 paisa
per share was paid on 40 million ordinary shares. What was the retained profit for the year ended
31 March 2020?
(a) Rs.150 million
(b) Rs.160 million
(c) Rs.165 million
(d) Rs.170 million

© Emile Woolf International 42 The Institute of Chartered Accountants of Pakistan


Chapter 2: IAS 1: Preparation of financial statements

33. SK Limited paid Rs.10 million in debenture interest and an ordinary dividend of 10 paisa per share
on Rs.50 million ordinary shares. The retained profit was Rs.120 million. What was SK Limited
profit for the year?
(a) Rs. 125 million
(b) Rs.135 million
(c) Rs. 130 million
(d) Rs.140 million

34. Which of the following would be an entry in the statement of changes in equity?
(a) Taxation
(b) Long term loans
(c) Revaluation gain
(d) Revaluation reserve

35. During the year ended 30 June 2021, a company's revaluation reserve increased from Rs.
300,000 to Rs. 380,000 as a result of a property (land) revaluation. At the start of that financial
year, the company's property had been valued at Rs. 810,000. Assuming that no property was
disposed of during the year, which of the following statements is true?
(a) The property's revalued amount was Rs.890, 000.
(b) The property's revalued amount was Rs.1, 190,000.
(c) The property's revalued amount was Rs.380, 000.
(d) The property's revalued amount was Rs.1,310,000

36. A debit balance on the retained earnings account indicates that the company has:
(a) made more dividend payments than the profit earned
(b) redeemed some of its share capital
(c) accumulated losses
(d) issued bonus shares

© Emile Woolf International 43 The Institute of Chartered Accountants of Pakistan


Financial accounting and reporting I

2 OBJECTIVE BASED ANSWERS


01. (b) & (d)

02. (c)

03. (a)

04. (b)

05. (c)

06. (d)

07. (b)

08. (b)

09. (b) Rs. 500 million x 10% = Rs. 50 million to be recognized in statement of
changes in equity.

10. (d) This dividend shall be recognized next year. This year the proposed dividend
shall be disclosed only.

11. (a) & (c)

12. (a) & (b)

13. (c)

14. (c)

15. (b)

16. Rs. 120 million Rs. 100 million +


Rs. 100 million / Rs. 100 x 1/5 x Rs. 100] = Rs. 120 million

17. Rs. 39 million Rs. 30 million +


Rs. 100 million / Rs. 100 x 1/5 x Rs. 45] = Rs. 39 million

18. Rs. 140 million Rs. 100 million +


Rs. 100 million / Rs. 100 x 2/5 x Rs. 100] = Rs. 140 million

19. Rs. Nil Rs. 100 million / Rs. 100 x 2/5 x Rs. 100] = Rs. 40 million shares issued
Rs. 30 million from share premium and remaining Rs. 10 million from
retained earnings.

20. Rs. 82 million Rs. 35 million + Rs. 20 million from revaluation surplus + Profit of Rs. 32
million – Rs. 5 million dividends = Rs. 82 million
Proposed dividend shall be disclosed only.

21. (c)

22. (d)

© Emile Woolf International 44 The Institute of Chartered Accountants of Pakistan


Chapter 2: IAS 1: Preparation of financial statements

23. (c)

24. (b)

25. (a)

26. (b)

27. (b)

28. (a)

29. (d)

30. (b)

31. (d)

32. (b)

33. (a)

34. (c)

35. (a)

36. (c) Accumulated losses

© Emile Woolf International 45 The Institute of Chartered Accountants of Pakistan


Financial accounting and reporting I

© Emile Woolf International 46 The Institute of Chartered Accountants of Pakistan


Certificate in Accounting and Finance

CHAPTER
Financial accounting and reporting I

IAS 7: Statement of cash flows

Contents
1 Introduction
2 Cash flows from operating activities: The indirect method
3 Cash flows from operating activities: The direct method
4 Cash flows from investing activities
5 Cash flows from financing activities
6 Objective based questions and answers

* The student must refer original handbook of IFRS.

© Emile Woolf International 47 The Institute of Chartered Accountants of Pakistan


Financial accounting and reporting I

1 INTRODUCTION
Section overview

◼ Cash Flows
◼ Purpose & Importance of cash flow for business
◼ Cash Flows & Financial Statements
◼ Statement of cash flows
◼ The sections of a statement of cash flows

1.1 Cash Flows


Generating positive, sustainable cash flow is critical for an organisation's long-term success.
Keeping track of cash flows is particularly important for management to project the financial health
of the organisation to potential investors. Analysing the cash flow statement is extremely valuable
because it provides a reconciliation of the beginning and ending cash on the balance sheet.
Cash flows are inflows and outflows of cash and cash equivalents. Cash comprises cash on hand
and demand deposits. Cash equivalents are short-term, highly liquid investments that are readily
convertible to known amounts of cash and which are subject to an insignificant risk of changes in
value.
Cash is one of the major lubricants of business activity, but there are certain things that are not
reflected in cash flows. For example, profit earned after tax during a period because profitability is
composed also of things that are not cash based. Therefore, the overall financial well-being of the
company is not indicated in cash flows. Furthermore, accounts receivable and accounts payable
are also not reflected in the cash flow statement.

1.2 Purpose & Importance of cash flow for business


Monitoring the cash flows is one of the most pressing management tasks for any business. The
most common outflow of cash includes payments of salaries and to suppliers. The inflow includes
the receipt from customers and investors.
Businesses must have sufficient cash; otherwise they cannot survive.
❑ A business can make a loss but still survive if it has sufficient cash or access to liquidity
(cash, assets that can be quickly turned into cash and new sources of borrowing).
❑ On the other hand, a business that is profitable cannot survive if it cannot pay its obligations
when they fall due, because it does not have enough cash or access to other sources of
liquidity.
Cash flow is therefore extremely important, and it is appropriate that entities should present a
statement of cash flows as a financial statement.
The purpose of a statement of cash flows is to show what the cash flows of the entity have been.
It can also be used to make assessments of what the cash flows of the entity might be in the future.
In other words, the cash flow statement is a compressed version of the company's check book that
includes a few other items that affect cash, like the financing section, which shows how much the
company spent or collected from the repurchase or sale of shares, the amount of issuance or
retirement of debt and the amount the company paid out in dividends.
IAS 1 states that a statement of cash flows is a part of a complete set of the financial statements
of an entity. It provides information about:
❑ the cash flows of the entity during the reporting period, and
❑ The changes in cash and cash equivalents during the period.

© Emile Woolf International 48 The Institute of Chartered Accountants of Pakistan


Chapter 3: IAS 7: Statement of cash flows

IAS 7: Statements of cash flows sets out the benefits of cash flow information to users of financial
statements.
❑ A statement of cash flows provides information that helps users to evaluate changes in the
net assets of an entity and in its financial structure (including its liquidity and solvency).
❑ It provides information that helps users to assess the ability of the entity to affect the amount
and timing of its cash flows in order to adapt to changing circumstances and unexpected
opportunities.
❑ It is useful in assessing the ability of the entity to generate cash and cash equivalents.
❑ It helps users of accounts to compare the performance of different entities because unlike
profits, comparisons of cash flows are not affected by the different accounting policies used
by different entities.
❑ Historical cash flows are often a fairly reliable indicator of the amount, timing and certainty
of future cash flows.

1.3 Cash Flows & Financial Statements


When a business makes a profit of Rs. 1,000, this does not mean that it receives Rs. 1,000 more
in cash than it has spent. Profit and cash flow are different, for several reasons:
There are items of cost in the statement of comprehensive income that do not represent a cash
flow. Examples are:
❑ depreciation and amortisation charges; and
❑ The gain or loss on the disposal of non-current assets.
There are items of cash flow that do not appear in the statement of comprehensive income.
Examples are:
❑ Cash flows relating to the acquisition or disposal of investments, such as the purchase of
new non-current assets, and cash from the sale of non-current assets. (The statement of
comprehensive income includes gains or losses on the disposal of non-current assets, but
this is not the same as the cash proceeds from the sale.)
❑ Cash flows relating to financial transactions, such as obtaining cash by issuing shares or
obtaining loans, the repayment of loans and the payment of dividends to ordinary
shareholders.
❑ Theoretically this could be done by analysing every entry in and out of the cash account(s)
over the course of a period. However, the cash account is often the busiest account in the
general ledger with potentially many thousands of entries. Documents that summarise the
transactions are needed.
❑ These documents already exist. They are the other financial statements (statement of
financial position and statement of profit or loss and other comprehensive income).

Illustration 01:
A business might buy 100 new non-current assets over the year. There would be 100 different
entries for these in the cash account.
However, it should be easy to estimate the additions figure from comparing the opening and closing
balances for non-current assets and isolating any other causes of movement.
For example, if we know that property plant and equipment has increased by Rs. 100,000 and that
the only other cause of movement was depreciation of Rs. 15,000 then additions must have been
Rs.115,000.

❑ A lot of the numbers in cash flow statements are derived from comparing opening and
closing positions of line items in the statement of financial position. Other causes of
movement can then be identified leaving the cash double entry as a balancing figure.

© Emile Woolf International 49 The Institute of Chartered Accountants of Pakistan


Financial accounting and reporting I

1.4 Statement of cash flows


A statement of cash flows provides information about where a business obtained its cash during
the financial period, and how it made use of its cash.
A statement of cash flows groups inflows and outflows of cash fewer than three broad headings:
❑ cash generated from or (used in) operating activities
❑ cash obtained from or (used in) investing activities
❑ Cash received from or paid in financing activities.
A statement of cash flows reports the change in the amount of cash and cash equivalents held by
the entity during the financial period.

Definition: Cash, cash equivalents and cash flows


Cash comprises cash on hand and demand deposits.
Cash equivalents are short-term, highly liquid investments that are readily convertible to known
amounts of cash and which are subject to an insignificant risk of changes in value.
Cash flows are inflows and outflows of cash and cash equivalents.

For the purpose of a statement of cash flows, cash and cash equivalents are treated as being the
same thing. This means that cash flows between cash and cash equivalent balances are not shown
in the statement of cash flows. These components are part of the cash management of an entity
rather than part of its operating, investing and financing activities.
Cash and cash equivalents are held in order to meet short-term cash commitments, rather than for
investment purposes or other purposes.
Examples of cash equivalents are:
❑ A bank deposit where some notice of withdrawal is required
❑ Short-term investments with a maturity of three months or less from the date of acquisition
(e.g. Government bills).
Bank borrowings are generally considered to be financing activities. In that case they would be
held outside cash and cash equivalents and movements on the bank borrowings would be shown
under financing activities as a cash inflow if borrowing increase or as a cash outflow if borrowings
fell.
Sometimes, bank overdrafts which are repayable on demand form an integral part of an entity's
cash management. In these circumstances, bank overdrafts are included as a component of cash
and cash equivalents.
It also shows whether there was an increase or a decrease in the amount of cash held by the entity
between the beginning and the end of the period

Illustration 02:
Cash generated from or (used in) operating activities X/(X)
Cash obtained from or (used in) investing activities X/(X)
Cash received from or (paid in) financing activities. X/(X)
Net cash inflow (or outflow) during the period X/(X)
Cash and cash equivalents at the beginning of the period X/(X)
Cash and cash equivalents at the end of the period X/(X)

© Emile Woolf International 50 The Institute of Chartered Accountants of Pakistan


Chapter 3: IAS 7: Statement of cash flows

1.5 The sections of a statement of cash flows


The content and format of statements of cash flows are specified by IAS 7 Statements of cash
flows. IAS 7 does not specify what the exact format of a statement of cash flows should be, but it
provides suggested layouts in an appendix.
Entities are required by IAS 7 to report cash flows for the period under three headings:
❑ Cash flows from operating activities
❑ Cash flows from investing activities
❑ Cash flows from financing activities.
All cash flows (except for changes from cash to cash equivalents or from cash equivalents to cash)
can be included in one of these three categories.
Together, the cash flows arising from these three categories of activity explain the increase or
decrease in cash and cash equivalents during the financial period.
The cash flows for each category might be positive or negative. The total of the cash flows for all
three categories together explains the overall increase or decrease in cash and cash equivalents
during the period.
A single transaction might include more than one type of cash flow. For example, a cash repayment
of a loan might include both interest and capital. In this case the interest element might be classified
as an operating activity and the capital element as a financing activity.

1.5.1 Cash flows from operating activities


Operating activities are the normal trading activities of the entity. Cash flows from operating
activities are the cash inflows or cash outflows arising in normal trading activities.
Operating activities normally provide an operating profit before tax. However, profit is not the same
as cash flow, and the cash flows from operating activities are different from profit.
A statement of cash flows normally makes a distinction between:
❑ Cash generated from operations, which is the cash from sales less the cash payments for
operating costs, and
❑ Net cash from operating activities, which is the cash generated from operations, less
interest payments and tax paid on profits.
Cash flows from operating activities are primarily derived from the principal revenue-producing
activities of the entity. Therefore, they generally result from the transactions and other events that
enter into the determination of profit or loss.
Examples of cash flows from operating activities are:
❑ Cash receipts from the sale of goods and the rendering of services;
❑ Cash receipts from royalties, fees, commissions and other revenue;
❑ Cash payments to suppliers for goods and services;
❑ Cash payments to and on behalf of employees;
❑ Cash receipts and cash payments of an insurance entity for premiums and claims, annuities
and other policy benefits;
❑ Cash payments or refunds of income taxes unless they can be specifically identified with
financing and investing activities; and
❑ Cash receipts and payments from contracts held for dealing or trading purposes.
Some transactions result in the recognition of a gain or loss in profit or loss (e.g. sale of an item of
plant). However, the cash flows relating to such transactions are cash flows from investing
activities.
Cash payments to manufacture or acquire assets held for rental to others and subsequently held
for sale are cash flows from operating activities. The cash receipts from rents and subsequent
sales of such assets are also cash flows from operating activities.

© Emile Woolf International 51 The Institute of Chartered Accountants of Pakistan


Financial accounting and reporting I

The amount of cash flows arising from operating activities is a key indicator of the extent to which
the operations of the entity have generated sufficient cash flows to function without recourse to
external sources of financing. In addition, it forms a basis for forecasting future operating cash
flows.

1.5.2 Cash flows from investing activities


The second section of a statement of cash flows shows cash flows from investing activities.
Investing activities are defined by IAS 7 as ‘the acquisition and disposal of long-term assets and
other investments not included in cash equivalents’. It generally refers to money made or spent
on long-term assets the company has purchased or sold. Investing transactions generate cash
outflows, such as capital expenditures for plant, property and equipment, business acquisitions
and the purchase of investment securities. Inflows come from the sale of assets, businesses and
investment securities. For investors, the most important item in this category is capital
expenditures, made to ensure the proper maintenance of, and additions to, a company's physical
assets to support its efficient operation and competitiveness.
Cash flows from investing activities might also include cash received from investments, such as
interest or dividends received.
The separate disclosure of cash flows arising from investing activities is important because the
cash flows represent the extent to which expenditures have been made for resources intended to
generate future income and cash flows.
Examples of cash flows arising from investing activities are:
❑ cash payments to acquire property, plant and equipment, intangibles and other long-term
assets (including those relating to capitalised development costs and self-constructed
tangible assets);
❑ cash receipts from sales of property, plant and equipment, intangibles and other long-term
assets;
❑ cash payments to acquire equity or debt instruments;
❑ cash receipts from sales of equity or debt instruments of other entities;
❑ cash advances and loans made to other parties (other than advances and loans made by a
financial institution which would be an operating activity);
❑ cash receipts from the repayment of advances and loans made to other parties (other than
advances and loans of a financial institution);

1.5.3 Cash flow from financing activities


The third section of the statement of cash flows shows the cash flows from financing activities.
These activities are defined by IAS 7 as ‘activities that result in changes in the size and composition
of the contributed equity and borrowings of the entity.’ It measures the flow of cash between a firm
and its owners and creditors. Companies often borrow money to fund their operations, acquire
another company or make other major purchases. Here again for investors, the most important
item is cash dividends paid.
Examples of cash flows arising from financing activities are:
❑ cash proceeds from issuing shares or other equity instruments;
❑ cash payments to owners to acquire or redeem the entity's shares;
❑ cash proceeds from issuing debentures, loans, notes, bonds, mortgages and other short-
term or long-term borrowings;
❑ cash repayments of amounts borrowed; and
❑ Cash payments by a lessee for the reduction of the outstanding liability relating to a finance
lease.
The separate disclosure of cash flows arising from financing activities is important because it is
useful in predicting claims on future cash flows by providers of capital to the entity.

© Emile Woolf International 52 The Institute of Chartered Accountants of Pakistan


Chapter 3: IAS 7: Statement of cash flows

2 CASH FLOWS FROM OPERATING ACTIVITIES: THE INDIRECT METHOD


Section overview

◼ Operating Cash flows


◼ Profit before taxation
◼ Non-cash items
◼ Accruals based figures
◼ Taxation
◼ Dividends
◼ Presentation of interest, taxation and dividends cash flows
◼ Working capital
◼ Changes in trade and other receivables
◼ Possible complication: Allowances for doubtful debts
◼ Changes in inventory
◼ Changes in trade payables
◼ Lack of detail

2.1 Operating cash flows


The operations of the business are probably the most significant source of cash.
IAS 7 allows two approaches to identifying the cash flows from operating activities:
❑ Indirect method; and
❑ Direct method
The indirect method identifies the cash flows from operating activities by adjusting the profit before
tax figure. It arrives at the cash from operating activities figure indirectly by reconciling a profit figure
to a cash figure. The adjustments remove the impact of accruals and non-cash items and also
relocate some figures to other positions in the statement of cash flows.
The following illustration shows the net cash flow from operating activities arrived.
Illustration 03:
Cash flows from operating activities
Profit before taxation 80,000
Adjustments for:
Depreciation and amortization charges 20,000
Interest charge in the statement of comprehensive income 2,300
Gains on disposal of non-current assets (6,000)
Losses on disposal of non-current assets 4,500
100,800
Increase/decrease in:
Increase in trade and other receivables (7,000)
Decrease in inventories 2,000
Increase in trade payables 3,000
Cash generated from operations 98,800
Taxation paid (tax on profits) (21,000)
Interest charges paid (2,500)
Net cash flow from operating activities 75,300

© Emile Woolf International 53 The Institute of Chartered Accountants of Pakistan


Financial accounting and reporting I

2.2 Profit before taxation


The starting point for the statement of cash flows for a company is the operating profit after
deducting interest but before taxation.
This profit figure is adjusted to calculate the amount of cash received by the business or the amount
of cash paid out as a consequence of its trading operations.
The adjustments are to remove the effect of:
❑ Non-cash items, for example:
• Depreciation and amortisation;
• Gain or loss on disposal of non-current assets;
• Doubtful debts;
• Provision for obsolete inventory; and
❑ Accruals based figures, for example:
• Interest expense or income;
• Movement on working capital items (receivables, payables and inventory).

2.3 Non-cash items


Depreciation and amortisation
Depreciation and amortisation charges are not cash flows. They are expenses in the statement of
comprehensive income, but do not represent payments of cash.
In order to obtain a figure for cash flow from the figure for profit, charges for depreciation and
amortisation must therefore be added back to the profit figure.
Gains or losses on disposal of non-current assets
Gains or losses on the disposal of non-current assets are not cash flows. The gain or loss is
calculated as the difference between:
❑ the net cash received from the disposal; and
❑ The carrying amount (net book value) of the asset at the date of disposal.
The effect of the gain/loss on disposal (a non-cash item) from the operating profit is removed by:
❑ deducting gain on disposal; and
❑ adding back losses on disposal.
The relevant cash flow is the net cash received from the sale. This is included in cash flows from
investing activities as the net cash flows received from the disposal of non-current assets.

Illustration 04:
A company disposed of an item of equipment for Rs. 40,000. The equipment had originally cost
Rs. 60,000 and the accumulated depreciation charged up to the date of disposal was Rs. 32,000.
Rs.
Cost 60,000
Accumulated depreciation (32,000)
Carrying value at date of disposal 28,000
Cash proceeds from sale (40,000)
Gain on disposal 12,000

In the statement of cash flows, the gain on disposal of Rs. 12,000 is deducted as an adjustment to
the operating profit. The cash proceeds of Rs. 40,000 are included as a cash inflow under the
heading: ‘Cash flows from investing activities’.

© Emile Woolf International 54 The Institute of Chartered Accountants of Pakistan


Chapter 3: IAS 7: Statement of cash flows

2.4 Accruals based figures


Interest
The accruals concept is applied in accounting.
Interest charge in the income statement is an accrual based figure. It is added back to profit and
the actual cash interest paid is deducted further down the cash flow statements.
The final items in the operating cash flows part of a statement of cash flows are the amount of
interest paid and the amount of tax paid (see later).
This figure must be calculated as follows:

Illustration 05:
Rs.
Interest liability at the beginning of the year X
Interest charge for the year (income statement figure) X
Total amount of interest payable in the year X
Interest liability at the end of the year (X)
Interest paid in the year (cash) X

The same approach is used to calculate other figures.


The interest liability at the start of the year and the interest charge during the year is the most the
business would pay. If the business had paid nothing it would owe this figure. The difference
between this amount and the liability at the end of the year must be the amount that the business
has paid.

Example 01:
Question: A company had liabilities in its statement of financial position at the beginning and at the
end of 2017, as follows:
Interest payable (Rs.)
Beginning of 2017 4,000
End of 2017 22,000

During the year, interest charges in the income statement were Rs. 22,000.
Required: Calculate the amount of interest paid during 2017.

Answer: The interest payment for inclusion in the statement of cash flows can be calculated as
follows:
Rs.
Liability at the start of the year 4,000
Charge for the year 22,000
Total amount payable in the year 26,000
Liability at the end of the year (22,000)
Cash paid 4,000

Note that this approach would work to find the cash paid in respect of any liability for which expense
was recognised in the statement of profit or loss.
It would not matter if you did not know anything about the type of liability as long as you are told
that there is a movement and you are given the amount recognised in the statement of profit or
loss. For example, instead of the above example being about interest it could be about warranty
provision, gratuity, retirement benefit, health insurance, bonus, and so on.

© Emile Woolf International 55 The Institute of Chartered Accountants of Pakistan


Financial accounting and reporting I

2.5 Taxation
The tax paid is the last figure in the operating cash flow calculation.
There is no adjustment to profit in respect of tax. This is because the profit figure that we start with
is profit before tax; therefore, tax is not included in it to be adjusted!
However, there is a tax payment and this must be recognised as a cash flow. It is calculated in the
same way as shown above.

Example 02:
Question: A company had liabilities in its statement of financial position at the beginning and at
the end of 2017, as follows:
Taxation (Rs.)
Beginning of 2017 53,000
End of 2017 61,000

During the year, taxation on profits was Rs. 77,000.

Required: Calculate the amount of tax paid during 2017.

Answer: The tax payment (cash flows) for inclusion in the statement of cash flows can be calculated
as follows:
Rs.
Taxation liability at the start of the year 53,000
Charge for the year 77,000
Total amount payable 130,000
Taxation liability at the end of the year (61,000)
Cash paid 69,000

2.6 Dividends
A question might require the calculation of cash paid out as dividends in the year.
This is calculated in the usual way remembering that the dividend charge is a debit in the statement
of changes in equity.

Illustration 06:
Rs.
Dividend liability at the beginning of the year X
Dividend charge for the year X
Total amount of dividend payable in the year X
Dividend liability at the end of the year (X)
Dividend paid in the year (cash) X

Pakistan
Typically, in Pakistan a company will pay a dividend once a year. Dividend payments in Pakistan
must be approved by the members in a general meeting and this usually takes place after the year
end. This means that the dividend expensed in any one year is the previous year’s dividend (which
could not be recognised last year as it had not yet been approved in the general meeting).

© Emile Woolf International 56 The Institute of Chartered Accountants of Pakistan


Chapter 3: IAS 7: Statement of cash flows

Listed companies often pay an interim dividend part way through a year and a final dividend after
the year end. The actual dividend payment recognised in any one year would then be that year’s
interim dividend and the previous year’s final dividend (which could not be recognised last year as
it had not yet been approved in the general meeting).
A question may tell you that a dividend was declared at just before or just after the year end but
the company is not allowed to recognise that dividend until it is approved. Last year’s figure is
needed.

Example 03:
Question: A company had liabilities in its statement of financial position at the beginning and at
the end of 2017, as follows:
Dividends (Rs.)
Beginning of 2017 65,000
End of 2017 71,000
The company had share capital of Rs. 1,000,000. The directors recommended a dividend of
20% (2016: 18%) on 25th December 2017. The company AGM is held in March each year.

Required: Calculate the amount of dividend paid during 2017.

Answer: The dividend payment (cash flows) for inclusion in the statement of cash flows can be
calculated as follows:
Rs.
Dividend liability at the start of the year 65,000
Dividend in the year (18% of 1,000,000) 180,000
Total amount payable 245,000
Dividend liability at the end of the year (71,000)
Cash paid 171,000

2.7 Presentation of interest, taxation and dividends cash flows


IAS 7 allows some variations in the way that cash flows for interest and dividends are presented
in a statement of cash flows, although the following should be shown separately:
❑ interest received
❑ dividends received
❑ interest paid
❑ Dividends paid.

Interest payments
IAS 7 states that there is no consensus about how to treat interest payments by an entity, other
than a financial institution such as a bank. Interest payments may be classified as either:
❑ an operating cash flow, because they are deducted when calculating operating profit before
taxation, or
❑ A financing cash flow, because they are costs of obtaining finance.
In examples of statements of cash flows in the appendix to IAS 7, interest paid is shown as a
separate line item within cash flows from operating activities. This approach is therefore used here.

© Emile Woolf International 57 The Institute of Chartered Accountants of Pakistan


Financial accounting and reporting I

Interest and dividends received


Interest and dividends received may be classified as either:
❑ an operating cash flow, because they are added when calculating operating profit before
taxation, or
❑ An investing cash flow, because they represent returns on investment.
In examples of statements of cash flows in the appendix to IAS 7, interest received and dividend
received are shown as separate items within cash flows from investing activities. This approach is
therefore used here.

Dividends paid
IAS 7 allows dividend payments to be treated as either:
❑ a financing cash flow because they are a cost of obtaining financial resources, or
❑ A component of the cash flows from operating activities, in order to assist users to determine
the ability of the entity to pay dividends out of its operating cash flows.
In examples of statements of cash flows in the appendix to IAS 7, dividends paid are shown as a
line item within cash flows from financing activities. This approach is therefore used here.

Taxes on profits
Cash flows arising from taxation on income should normally be classified as a cash flow from
operating activities (unless the tax payments or refunds can be specifically associated with an
investing or financing activity).
The examples of statements of cash flows in this chapter therefore show both interest paid and tax
paid as cash flow items, to get from the figure for cash generated from operations to the figure for
‘net cash from operating activities’.

2.8 Working capital

Definition
Working capital is current assets less current liabilities.

The previous section showed that taxation and interest cash flows can be calculated by using a
figure from the statement of comprehensive income and adjusting it by the movement on the
equivalent balances in the statement of financial position.
This section shows how this approach is extended to identify the cash generated from operations
by making adjustments for the movements between the start and end of the year for:
❑ trade receivables and prepayments;
❑ inventories; and
❑ Trade payables and accruals.
Assuming that the calculation of the cash flow from operating activities starts with a profit (rather
than a loss) the adjustments are as follows:

Increase in balance from start to Decrease in balance from start to the


Balance
the end of the year end of the year
Receivables Subtract from profit before tax Add back to profit before tax
Inventory Subtract from profit before tax Add back to profit before tax
Payables Add back to profit before tax Subtract from profit before tax

These are known as the working capital adjustments and are explained in more detail in the rest
of this section.

© Emile Woolf International 58 The Institute of Chartered Accountants of Pakistan


Chapter 3: IAS 7: Statement of cash flows

Working capital is made up of the following balances:

Illustration 07:
Rs.
Inventory X
Trade and other receivables X
Cash X
Trade payables (X)
Working capital X

Trade and other receivables include any prepayments.


Trade payables include accrued expenses, provided the accrued expenses do not relate to other
items dealt with separately in the statement of cash flows, in particular:
❑ accrued interest charges; and
❑ Taxation payable.
Interest charges and payments for interest are presented separately in the statement of cash flows,
and so accrued interest charges should be excluded from the calculation of changes in trade
payables and accruals.
Similarly, taxation payable is dealt with separately; therefore, taxation payable is excluded from the
calculation of working capital changes.
Accrued interest and accrued tax payable must therefore be deducted from the total amount for
accruals, and the net accruals (after making these deductions) should be included with trade
payables.

Changes in working capital and the effect on cash flow


When working capital increases, the cash flows from operations are less than the operating profit,
by the amount of the increase.
Similarly, when working capital is reduced, the cash flows from operations are more than the
operating profit, by the amount of the reduction.
This important point will be explained with several simple illustrations and examples.

2.9 Changes in trade and other receivables


Sales revenue in a period differs from the amount of cash received from sales by the amount of
the increase or decrease in receivables during the period.
When trade and other receivables go up during the year, cash flows from operations are less than
operating profit by the amount of the increase.
When trade and other receivables go down during the year, cash flows from operations are more
than operating profit by the amount of the reduction.
In a statement of cash flows presented using the indirect method, the adjustment for receivables is
therefore:
❑ subtract the increase in receivables during the period (the amount by which closing
receivables exceed opening receivables); or
❑ Add the reduction in receivables during the period (the amount by which opening receivables
exceed closing receivables).
Prepayments in the opening and closing statement of financial position should be included in the
total amount of receivables and therefore does not show separately.

© Emile Woolf International 59 The Institute of Chartered Accountants of Pakistan


Financial accounting and reporting I

Illustration 08: A company had receivables at the beginning of the year of Rs. 6,000 and at the end
of the year receivables were Rs. 9,000.
During the year, sales were Rs. 50,000 in total. Purchases were Rs. 30,000, all paid in cash.
The company holds no inventories. The profit before tax for the year was Rs. 20,000 (Rs. 50,000 –
Rs. 30,000).
The cash flow from operations is calculated as follows:
Rs.
Profit before tax 20,000
Adjustments for:
Increase in receivables (9,000 – 6,000) (3,000)
17,000

Illustration 09:
Proof
Cash flow from operations can be calculated as follows:
Rs.
Receivables at the beginning of the year 6,000
Sales in the year 50,000
56,000
Receivables at end of the year (9,000)
Cash received 47,000
Cash paid (purchases) (30,000)
Cash flow from operations 17,000

2.10 Possible complication: Allowances for doubtful debts


A question might provide information on the allowance for doubtful debts at the start and end of
the year.
There are two ways of dealing with this:
❑ Adjust the profit for the movement on the allowance as a non-cash item and adjust the profit
figure for the movement in receivables using the gross amounts (i.e. the balances before
any deduction of the allowance for doubtful debts); or
❑ Make no adjustments for the movement on receivables as a non-cash item adjust the profit
figure for the movement in receivables using the net amounts (i.e. the balances after the
deduction of the allowance for doubtful debts).

Illustration 10: The following information is available:

2016 2017
(Rs. m) (Rs. m)
Receivables 5,000 7,100
Allowance for doubtful debts (500) (600)
Net-amount 4,500 6,500

© Emile Woolf International 60 The Institute of Chartered Accountants of Pakistan


Chapter 3: IAS 7: Statement of cash flows

Rs. m or Rs. m
Profit before taxation 10,000 10,000
Adjustments for non- cash items:
Increase in allowance for doubtful debts 100 −
10,100 10,000
Increase in receivables:
Gross amounts: (7,100 − 5,000) (2,100)
Net amounts: (6,500 − 4,500) (2,000)
8,000 8,000

2.11 Changes in inventory


Purchases in a period differ from the cost of sales by the amount of the increase or decrease in
inventories during the period.
If all purchases were paid for in cash, this means that cash payments and the cost of sales (and
profit) would differ by the amount of the increase or decrease in inventories.
When the value of inventory goes up between the beginning and end of the year, cash flows from
operations are less than operating profit by the amount of the increase.
When the value of inventory goes down between the beginning and end of the year, cash flows
from operations are more than operating profit by the amount of the reduction.
In a statement of cash flows presented using the indirect method, the adjustment for inventories
is therefore:
❑ subtract the increase in inventories during the period (the amount by which closing inventory
exceeds opening inventory); or
❑ Add the reduction in inventories during the period (the amount by which opening inventory
exceeds closing inventory).

Illustration 11:
A company had inventory at the beginning of the year of Rs. 5,000 and at the end of the year the
inventory was valued at Rs. 3,000.
During the year, sales were Rs. 50,000 and there were no receivables at the beginning or end of
the year.
Purchases were Rs. 28,000, all paid in cash.
The operating profit for the year was Rs. 20,000, calculated as follows:
Rs.
Sales 50,000
Opening inventory 5,000
Purchases in the year (all paid in cash) 28,000
33,000
Closing inventory (3,000)
Cost of sales (30,000)
Profit before tax 20,000

© Emile Woolf International 61 The Institute of Chartered Accountants of Pakistan


Financial accounting and reporting I

Rs.
Profit before tax 20,000
Adjustments for:
decrease in inventory (5,000 – 3,000) 2,000
22,000

Proof: The cash flow from operations is calculated as follows:


Rs.
Cash from sales in the year 50,000
Purchases paid in cash (28,000)
Cash flow from operations 22,000

2.12 Changes in trade payables


Payments for purchases in a period differ from purchases by the amount of increase or decrease
in trade payables during the period.
When trade payables go up between the beginning and end of the year, cash flows from
operations are more than operating profit by the amount of the increase.
When trade payables go down between the beginning and end of the year, cash flows from
operations are less than operating profit by the amount of the reduction.
In a statement of cash flows presented using the indirect method, the adjustment for trade
payables is therefore:
❑ add the increase in trade payables during the period (the amount by which closing trade
payables exceed opening trade payables); or
❑ Subtract the reduction in trade payables during the period (the amount by which opening
trade payables exceed closing trade payables).
Accruals in the opening and closing statement of financial position should be included in the total
amount of trade payables.
However, deduct interest payable and tax payable from opening and closing payables, if the total
for payables includes these items.

Illustration 12:
A company had no inventory and no receivables at the beginning and end of the year. All its sales
are for cash, and sales in the year were Rs. 50,000.
Its purchases are all on credit. During the year, its purchases were Rs. 30,000.
Trade payables at the beginning of the year were Rs. 4,000 and trade payables at the end of the
year were Rs. 6,500.
The operating profit for the year was Rs. 20,000 (Rs. 50,000 – Rs. 30,000)

Rs.
Profit before tax 20,000
Adjustments for:
Increase in payables (6,500 – 4,000) 2,500
22,500

© Emile Woolf International 62 The Institute of Chartered Accountants of Pakistan


Chapter 3: IAS 7: Statement of cash flows

Proof: The cash flow from operations is calculated as follows:


Rs.
Trade payables at the beginning of the year 4,000
Purchases in the year 30,000
34,000
Trade payables at the end of the year (6,500)
Cash paid to suppliers 27,500
Cash from sales (50,000)
Cash flow from operations 22,500

The cash flow is Rs. 2,500 more than the operating profit, because trade payables were increased
during the year by Rs. 2,500.

Illustration 13:
A company made an operating profit before tax of Rs. 16,000 in the year just ended.
Depreciation charges were Rs. 15,000.
There was a gain of Rs. 5,000 on disposals of non-current assets and there were no interest charges.
Values of working capital items at the beginning and end of the year were:

Receivables Inventory Trade payables

Beginning of the year Rs. 9,000 Rs. 3,000 Rs. 4,000

End of the year Rs. 6,000 Rs. 5,000 Rs. 6,500

Taxation paid was Rs. 4,800.


The calculation is as under:

Rs. Rs.

Cash flows from operating activities

Profit before taxation 16,000

Adjustments for:

Depreciation and amortisation charges 15,000

Gains on disposal of non-current assets (5,000)

26,000

Decrease in trade and other receivables 3,000

Increase in inventories (2,000)

Increase in trade payables 2,500

Cash generated from operations 29,500

Taxation paid (tax on profits) (4,800)

Net cash flow from operating activities 24,700

© Emile Woolf International 63 The Institute of Chartered Accountants of Pakistan


Financial accounting and reporting I

2.13 Lack of detail


A question might not provide all the detail needed to split out working capital into all of its
component parts. If this is the case the adjustment must be made using whatever totals are
available in the question.

Illustration 14:
A company made an operating profit before tax of Rs. 16,000 in the year just ended.
Depreciation charges were Rs. 15,000.
There was a gain of Rs. 5,000 on disposals of non-current assets and there were no interest charges.
Values of working capital items at the beginning and end of the year were:
Current assets Trade payables
Beginning of the year Rs. 12,000 Rs. 4,000
End of the year Rs. 11,000 Rs. 6,500
Taxation paid was Rs. 4,800.
The calculation is as under:
Rs. Rs.
Cash flows from operating activities
Profit before taxation 16,000
Adjustments for:
Depreciation and amortisation charges 15,000
Gains on disposal of non-current assets (5,000)
26,000
Decrease in current assets 1,000
Increase in trade payables 2,500
Cash generated from operations 29,500
Taxation paid (tax on profits) (4,800)
Net cash flow from operating activities 24,700

Example 04:
Question: The following information has been extracted from the financial statements of Hopper
Company for the year ended 31 December 2017.
Rs.
Sales 1,280,000
Cost of sales (400,000)
Gross profit 880,000
Wages and salaries (290,000)
Other expenses (including depreciation Rs. 25,000) (350,000)
240,000
Interest charges (50,000)
Profit before tax 190,000
Taxation (40,000)
Profit after tax 150,000

© Emile Woolf International 64 The Institute of Chartered Accountants of Pakistan


Chapter 3: IAS 7: Statement of cash flows

Extracts from the statement of financial position:

At 1 At 31
January December
2017 2017
Rs. Rs.
Trade receivables 233,000 219,000
Inventory 118,000 124,000
Trade payables 102,000 125,000
Accrued wages and salaries 8,000 5,000
Accrued interest charges 30,000 45,000
Tax payable 52,000 43,000

Required: Using the above information, prepare cash flows from operating activities section of
statement of cash flows for Hopper Limited using indirect method.
Answer: The cash flows from operating activities using the indirect method is as under:
Statement of cash flows Rs.
Cash flows from operating activities
Profit before taxation 190,000
Adjustments for:
Depreciation charges 25,000
Interest expense 50,000
265,000
Decrease in trade receivables (233,000 – 219,000) 14,000
Increase in inventories (124,000 – 118,000) (6,000)
Increase in trade and other payables 20,000
(125,000 + 5,000) – (102,000 + 8,000)
Cash generated from operations 293,000
Taxation paid (W1) (49,000)
Interest paid (W1) (35,000)
Net cash flow from operating activities 209,000
Workings
(W1) Interest and tax payments Tax Interest
Rs. Rs.
Liability at the beginning of the year 52,000 30,000
Taxation charge/interest charge for the year 40,000 50,000
92,000 80,000
Liability at the end of the year (43,000) (45,000)
Tax paid/interest paid during the year 49,000 35,000

© Emile Woolf International 65 The Institute of Chartered Accountants of Pakistan


Financial accounting and reporting I

3 CASH FLOWS FROM OPERATING ACTIVITIES: THE DIRECT METHOD


Section overview

◼ Cash from sales


◼ Cash paid for materials
◼ Cash paid for wages and salaries
◼ Cash paid for other expenses

3.1 Cash from sales


The format for the direct method of presenting a statement of cash flows is as follows:

Illustration 15:
Statement of cash flows: direct method Rs.
Cash flows from operating activities
Cash receipts from customers 348,800
Cash payments to suppliers (70,000)
Cash payments to employees (150,000)
Cash paid for other operating expenses (30,000)
Cash generated from operations 98,800
Taxation paid (tax on profits) (21,000)
Interest charges paid (2,500)
Net cash flow from operating activities 75,300

The task is therefore to establish the amounts for cash receipts and cash payments. In an
examination, you might be expected to calculate any of these cash flows from figures in the opening
and closing statements of financial position, and the statement of profit or loss.
The cash receipts from sales during a financial period can be calculated as follows:

Illustration 16:

Rs.
Trade receivables at the beginning of the year X
Sales in the year X
X
Trade receivables at the end of the year (X)
Cash from sales during the year X

A T account could also be used to calculate the cash receipt


Receivables
Balance b/f X
Sales X Cash (balancing figure) X
Balance c/f X
X X

© Emile Woolf International 66 The Institute of Chartered Accountants of Pakistan


Chapter 3: IAS 7: Statement of cash flows

3.2 Cash paid for materials


To calculate the amount of cash paid to suppliers, you might need to calculate first the amount of
material purchases during the period.

Illustration 17: Calculation of purchases in the year


Rs.
Closing inventory at the end of the year X
Cost of sales X
X
Opening inventory at the beginning of the year (X)
Purchases in the year X

Having calculated purchases from the cost of sales, the amount of cash payments for purchases
may be calculated from purchases and opening and closing trade payables.

Illustration 18:
Rs.
Trade payables at the beginning of the year X
Purchases in the year (as above) X
X
Trade payables at the end of the year (X)
Cash paid for materials X

A T account could also be used to calculate the cash paid


Payables
Balance b/f X
Cash (balancing figure) X Purchases X
Balance c/f X
X X

Note that if the business had paid for goods in advance at the start or end of the year they would
have an opening or closing receivable but this situation would be quite unusual.

3.3 Cash paid for wages and salaries


Cash payments for wages and salaries can be calculated in a similar way.
Illustration 19:
Rs.
Accrued wages and salaries at the beginning of the year X
Wages and salaries expenses in the year X
X
Accrued wages and salaries at the end of the year (X)
Cash paid for wages and salaries X
A T account could also be used to calculate the cash paid
Payables
Balance b/f X
Cash (balancing figure) X Purchases X
Balance c/f X
X X

© Emile Woolf International 67 The Institute of Chartered Accountants of Pakistan


Financial accounting and reporting I

If wages and salaries had been paid in advance the business would have a receivable and the
workings would change to the following.

Illustration 20:

Rs.

Wages and salaries paid in advance at the beginning of the year (X)

Wages and salaries expenses in the year X

Wages and salaries paid in advance at the end of the year X

Cash paid for wages and salaries X

A T account could also be used to calculate the cash paid

Payables

Balance b/f X

Cash (balancing figure) X Purchases X

Balance c/f X

X X

3.4 Cash paid for other expenses


Other expenses in the statement of profit or loss usually include depreciation charges, which are
not cash flows. Depreciation charges should therefore be excluded from other expenses when
calculating cash payments.
Cash payments for other expenses can be calculated as follows.

Illustration 21:

Rs.

Payables for other expenses at the beginning of the year X

Other expenses in the year, excluding depreciation and amortisation X

Payables for other expenses at the end of the year (X)

Cash paid for other expenses X

Payables for other expenses should exclude accrued wages and salaries, accrued interest
charges and taxation payable.

© Emile Woolf International 68 The Institute of Chartered Accountants of Pakistan


Chapter 3: IAS 7: Statement of cash flows

Example 05:
Question: The following information has been extracted from the financial statements of Hopper
Company for the year ended 31 December 2015.
Rs.
Sales 1,280,000
Cost of sales (400,000)
Gross profit 880,000
Wages and salaries (290,000)
Other expenses (including depreciation Rs. 25,000) (350,000)
240,000
Interest charges (50,000)
Profit before tax 190,000
Tax on profit (40,000)
Profit after tax 150,000
Extracts from the statement of financial position:
At 1 January At 31 December
2015 2015
Rs. Rs.
Trade receivables 233,000 219,000
Inventory 118,000 124,000
Trade payables 102,000 125,000
Accrued wages and salaries 8,000 5,000
Accrued interest charges 30,000 45,000
Tax payable 52,000 43,000

Required: Using the above information, prepare cash flows from operating activities section of
statement of cash flows for Hopper Limited using direct method.

Answer: The calculation is as under:

Direct method
Statement of cash flows: direct method Rs.
Cash flows from operating activities
Cash receipts from customers(W1) 1,294,000
Cash payments to suppliers(W3) (383,000)
Cash payments to employees(W4) (293,000)
Cash paid for other operating expenses (325,000)
Cash generated from operations 293,000
Taxation paid (tax on profits)(W5) (49,000)
Interest charges paid(W5) (35,000)
Net cash flow from operating activities 209,000
.

© Emile Woolf International 69 The Institute of Chartered Accountants of Pakistan


Financial accounting and reporting I

Workings
(W1) Cash from sales Rs.
Trade receivables at 1 January 2015 233,000
Sales in the year 1,280,000
1,513,000
Trade receivables at 31 December 2015 (219,000)
Cash from sales during the year 1,294,000

(W2) Purchases Rs.


Closing inventory at 31 December 2015 124,000
Cost of sales 400,000
524,000
Opening inventory at 1 January 2015 (118,000)
Purchases in the year 406,000

(W3) Cash paid for materials supplies Rs.


Trade payables at 1 January 2015 102,000
Purchases in the year (W2) 406,000
508,000
Trade payables at 31 December 2015 (125,000)
Cash paid for materials 383,000

(W4) Cash paid for wages and salaries Rs.


Accrued wages and salaries at 1 January 2015 8,000
Wages and salaries expenses in the year 290,000
298,000
Accrued wages and salaries at 31 December 2015 (5,000)
Cash paid for wages and salaries 293,000

(W5) Interest and tax payments Tax Interest


Rs. Rs.
Liability at the beginning of the year 52,000 30,000
Taxation charge/interest charge for the year 40,000 50,000
92,000 80,000
Liability at the end of the year (43,000) (45,000)
Tax paid/interest paid during the year 49,000 35,000

© Emile Woolf International 70 The Institute of Chartered Accountants of Pakistan


Chapter 3: IAS 7: Statement of cash flows

4 CASH FLOWS FROM INVESTING ACTIVITIES


Section overview

◼ Cash paid for the purchase of property, plant and equipment


◼ Cash from disposals of property, plant and equipment
◼ Cash paid for the purchase of investments and cash received from the sale of investments
◼ Non-cash purchases

4.1 Cash paid for the purchase of property, plant and equipment
This is the second part of a statement of cash flows, after cash flows from operating activities.
The most important items in this part of the statement are cash paid to purchase non-current assets
and cash received from the sale or disposal of non-current assets but it also includes interest
received and dividends received on investments.
It is useful to remember the following relationship:

Illustration 22: Movement on non-current assets


Rs.
Carrying amount at the start of the year X
Depreciation (X)
Disposals (X)
Additions X
Revaluation X/(X)
Carrying amount at the end of the year X

When there are no disposals or revaluations during the year


When there are no disposals or revaluations of non-current assets during the year, purchases of
non-current assets (normally assumed to be the amount of cash paid for these purchases) may be
calculated as follows:

Illustration 23:
Using cost: Rs.
Non-current assets at the end of the year at cost X
Non-current assets at the beginning of the year at cost (X)
Additions to non-current assets X

Alternatively carrying amount (NBV) can be used Rs.


Non-current assets at the end of the year at NBV X
Non-current assets at the beginning of the year at NBV (X)
X
Depreciation X
Additions X

© Emile Woolf International 71 The Institute of Chartered Accountants of Pakistan


Financial accounting and reporting I

Illustration 24: The plant and equipment of PM Company at the beginning and the end of its
financial year were as follows:
At cost Accumulated depreciation Net book value
Rs. Rs. Rs.
Beginning of the year 180,000 (30,000) 150,000
End of the year 240,000 (50,000) 190,000
There were no disposals of plant and equipment during the year.
The cash paid for plant and equipment in the year (additions) may be calculated in either of the
following ways.
Rs. Rs.
At cost at the end of the year Carrying amount (NBV) at the
240,000 end of the year 190,000
At cost at the beginning of the Carrying amount (NBV at the
year 180,000 beginning of the year 150,000

Additions 60,000 Increase in NBV 40,000

Depreciation charge for the year


(50,000 – 30,000) 20,000

Additions 60,000

Note that in the above illustration it is assumed that the purchases have been made for cash.
This might not be the case. If the purchases are on credit the figure must be adjusted for any
amounts outstanding at the year end.

Illustration 25: PM company has purchased various items of property, plant and equipment on
credit during the year. The total purchased was Rs. 60,000.
The statements of financial position of PM company at the beginning and end of 2017 include the
following information:

2016 2017
(Rs. m) (Rs. m)

Payables:

Suppliers of non-current assets 4,000 12,000

The cash paid to buy property, plant and equipment in the year can be calculated as follows:

Rs. m

Additions 60,000

Less: increase in payables that relate to these items (8,000)

Cash paid in the year 52,000

This can be thought of as the payment of the Rs. 4,000 owed at the start and a payment of Rs.
48,000 towards this year’s purchases.

If the payables had decreased the movement would be added to the additions figure to find the
cash outflow.

© Emile Woolf International 72 The Institute of Chartered Accountants of Pakistan


Chapter 3: IAS 7: Statement of cash flows

Illustration 26: PM company has purchased various items of property, plant and equipment on
credit during the year. The total purchased was Rs. 60,000.
The statements of financial position of PM company at the beginning and end of 2017 include the
following information:

2016 2017
(Rs. m) (Rs. m)
Payables:
Suppliers of non-current assets 14,000 4,000

The cash paid to buy property, plant and equipment in the year can be calculated as follows:
Rs. m
Additions 60,000
Less: increase in payables that relate to
these items 10,000
Cash paid in the year 70,000

This can be thought of as the payment of the Rs. 14,000 owed at the start and a payment of
Rs. 56,000 towards this year’s purchases.

When there are disposals during the year


When there are disposals of non-current assets during the year, the purchases of non-current
assets may be calculated as follows:

Illustration 27: Movement on non-current assets

Rs.
Assets at cost at the beginning of the year X
Disposals during the year (cost) (X)
X
Additions to non-current assets (balancing figure) X

Assets at cost at the end of the year X

Alternatively carrying amount (NBV) can be used Rs.


Non-current assets at the beginning of the year at NBV X
Depreciation (X)
Disposals during the year (NBV) (X)

X
Additions to non-current assets (balancing figure) X

Non-current assets at the end of the year at NBV (X)

© Emile Woolf International 73 The Institute of Chartered Accountants of Pakistan


Financial accounting and reporting I

Example 06:
Question: The motor vehicles of PM Company at the beginning and the end of its financial year
were as follows:
Accumulated Carrying
At cost depreciation amount
Rs. Rs. Rs.
Beginning of the year 150,000 (105,000) 45,000
End of the year 180,000 (88,000) 92,000
During the year a vehicle was disposed of for a gain of Rs. 3,000. The original cost of this asset was
Rs. 60,000. Accumulated depreciation on the asset was Rs. 45,000.

Required: Calculate the cash paid for acquisition of motor vehicles.


Answer:
Cost NBV
Balance at the start of the year 150,000 45,000
Disposals during the year:
At cost (60,000)
At carrying amount: (60,000 – 45,000) (15,000)
Depreciation (88,000 – (105,000 – 45,000) (28,000)
90,000 2,000
Additions (balancing figure) 90,000 90,000
Balance at the end of the year 180,000 92,000

Rs.
Assets at cost at the end of the year 180,000
Assets at cost at the beginning of the year 150,000
30,000
Disposals during the year: original asset cost 60,000
Purchases 90,000

Alternatively using carrying amount (NBV): Rs.


Assets at carrying amount (NBV) at the end of the year 92,000
Assets at carrying amount (NBV) at the beginning of the year 45,000
47,000
Disposals during the year (carrying amount): (60,000 – 45,000) 15,000
Depreciation (88,000 – (105,000 – 45,000) 28,000
Purchases 90,000

When there are revaluations during the year


When there are revaluations of non-current assets during the year, the purchases of non-current
assets should be calculated as follows.

© Emile Woolf International 74 The Institute of Chartered Accountants of Pakistan


Chapter 3: IAS 7: Statement of cash flows

Illustration 28: Movement on non-current assets


Rs.
At cost or valuation, at the beginning of the year X
Disposals during the year (cost) (X)
Upward/(downward) revaluation during the year X/(X)
X
Additions to non-current assets (balancing figure) X
At cost or valuation, at the end of the year X

Alternatively carrying amount (NBV) can be used Rs.


Non-current assets at the beginning of the year at NBV X
Depreciation (X)
Disposals during the year (NBV) (X)
Upward/(downward) revaluation during the year X/(X)
X
Additions to non-current assets (balancing figure) X
Non-current assets at the end of the year at NBV (X)

Example 07:
Question: The statements of financial position of Grand Company at the beginning and end of 2017
include the following information:

Property, plant and equipment 2016 2017


Rs. Rs.
At cost/re-valued amount 1,400,000 1,900,000
Accumulated depreciation 350,000 375,000
Carrying value 1,050,000 1,525,000

During the year, some property was re-valued upwards by Rs. 200,000. An item of equipment was
disposed of during the year at a profit of Rs. 25,000. This equipment had an original cost of Rs.
260,000 and accumulated depreciation of Rs. 240,000 at the date of disposal.
Depreciation charged in the year was Rs. 265,000.
Required: Calculate the cash paid for acquisition of property, plant and equipment.
Answer:

Rs.
At cost/re-valued amount, at the end of the year 1,900,000
At cost/re-valued amount, at the beginning of the year 1,400,000
500,000
Add: Cost of assets disposed of in the year 260,000
Subtract: Asset revaluation during the year (200,000)
Purchases during the year 560,000

© Emile Woolf International 75 The Institute of Chartered Accountants of Pakistan


Financial accounting and reporting I

Alternatively using carrying amount (NBV): Rs.


Assets at carrying amount (NBV) at the end of the year 1,525,000
Assets at carrying amount (NBV) at the beginning of the year 1,050,000
475,000
Revaluation during the year (200,000)
Carrying amount of assets disposed of in the year
(260,000 – 240,000) 20,000
Depreciation charged during the year 265,000
Purchases during the year 560,000

When there are other additions during the year


The above example showed the need to take revaluation into account when reconciling the opening
and closing balances on non-current assets so as to find the additions figure as a balancing
amount.
This applies to other additions too:
❑ Transfers from capital work in progress
• These are assets constructed by a company for its own use.
• During the course of construction costs are accumulated in a capital work in progress
account and these are transferred into the relevant category of non-current asset on
completion.
• The cash consequence of capital work in progress is estimated as a separate
exercise.
• Transfers into the relevant category of non-current asset on completion show as an
addition and so must be taken into account when trying to estimate the cash additions.
❑ Assets acquired under finance leases.
• A finance lease is capitalised on the statement of financial position as an assets and
as a liability.
• The asset side of the entry will show as an addition into non-current assets and so
must be taken into account when trying to estimate the cash additions.
• The liability is a form of loan. Movements on the liability represent new amounts
borrowed (additions to non-current assets) and repayments of capital.

Example 08:
Question: The statements of financial position of Grand Company at the beginning and end of 2017
include the following information:
Property, plant and equipment 2016 2017
Rs. Rs.
At cost/re-valued amount 1,400,000 1,900,000
Accumulated depreciation 350,000 375,000
Carrying value 1,050,000 1,525,000

Capital work in progress 600,000 620,000

Lease liability 300,000 410,000

© Emile Woolf International 76 The Institute of Chartered Accountants of Pakistan


Chapter 3: IAS 7: Statement of cash flows

During the year:


Property was revalued upwards by Rs. 200,000.
An item of equipment was disposed of at a profit of Rs. 25,000. This equipment had an original
cost of Rs. 260,000 and accumulated depreciation of Rs. 240,000 at the date of disposal.
Depreciation charged in the year was Rs. 265,000.
The company capitalised Rs. 200,000 as capital work in progress and repaid Rs. 50,000 of the
finance lease loan.
Required: Calculate the cash paid for acquisition (additions) of property, plant and equipment.
Answer:

Cost NBV
Balance at the start of the year 1,400,000 1,050,000
Disposals during the year:
At cost (260,000)
At carrying amount: (260,000 – 240,000) (20,000)
Depreciation (265,000)
Revaluation 200,000 200,000
Additions – new assets under finance leases (W) 160,000 160,000
Additions – Transfer from capital WIP (W) 180,000 180,000

1,680,000 1,305,000
Additions (balancing figure) 220,000 220,000

Balance at the end of the year 1,900,000 1,525,000

4.2 Cash from disposals of property, plant and equipment


A statement of cash flows should include the net cash received from any disposals of non-current
assets during the period.
This might have to be calculated from the gain or loss on disposal and the carrying amount of the
asset at the time of its disposal.

Illustration 29: Disposal of property, plant and equipment

Rs.

At cost (or revalued amount at the time of disposal) X

Accumulated depreciation, at the time of disposal (X)

Net book value/carrying amount at the time of disposal X

Gain or (loss) on disposal X

Net disposal value (= assumed cash flow) X

If there is a gain on disposal, the net cash from the disposal is more than the net book value.
If there is a loss on disposal the net cash from the disposal is less than the net book value.

© Emile Woolf International 77 The Institute of Chartered Accountants of Pakistan


Financial accounting and reporting I

Illustration 30:
During an accounting period, an entity disposed of some equipment and made a gain on disposal
of Rs. 6,000.
The equipment originally cost Rs. 70,000 and at the time of its disposal, the accumulated
depreciation on the equipment was Rs. 56,000.
What was the amount of cash obtained from the disposal of the asset?
Disposal of equipment Rs.

At cost 70,000

Accumulated depreciation, at the time of disposal (56,000)

Net book value/carrying amount at the time of disposal 14,000

Gain on disposal 6,000

Net disposal value (assumed cash flow) 20,000

This cash flow would be included in the cash flows from investing activities.

Note that in the above illustration it is assumed that the cash received for the disposal has been
received. This might not be the case. If the disposal was on credit the figure must be adjusted for
any amounts outstanding at the year end.

4.3 Cash paid for the purchase of investments and cash received from the sale of
investments
A statement of cash flows should include the net cash paid to buy investments in the period and
the cash received from the sale of investment in the period.
It is useful to remember the following relationship:

Illustration 31: Movement on investments

Rs.

Carrying amount at the beginning of the year X

Disposals (X)

Additions X

Revaluation X/(X)

Carrying amount at the end of the year X

The issues to be considered in calculating cash paid for investments or cash received on the sale
of investments are very similar to those for the purchase and sale of property, plant and equipment
except for the absence of depreciation.

Illustration 32:
The statements of financial position of Grand Company at the beginning and end of 2017 include
the following information:
2016 2017
(Rs. m) (Rs. m)
Non-current asset investments 1,000 1,500

© Emile Woolf International 78 The Institute of Chartered Accountants of Pakistan


Chapter 3: IAS 7: Statement of cash flows

Additional information:
The investments were revalued upwards during the year. A revaluation gain of Rs. 150m has been
recognised.
Investments sold for Rs. 250m resulted in a profit on the sale (measured as the difference
between sale proceeds and carrying amount at the date of sale) of Rs. 50m
The cash paid to buy investments in the period can be calculated as a balancing figure as follows:

Rs. m
Investments at the start of the year (given) 1,000
Disposal (carrying amount of investments sold = Rs. 250m – Rs. 50m) (200)
Revaluation gains (given) 150
950
Additions (as balancing figure): 550
Investments at the end of the year (given) 1,500

4.4 Non-cash purchases


IAS 7 states that investing and financing transactions that do not require the use of cash must be
excluded from the statement of cash flows, but that details of these transactions should be
disclosed somewhere in the financial statements, possibly as a note to the financial statements.
An example of a non-cash transaction is the acquisition of non-current assets under a finance lease
arrangement. The assets are included in the financial statements at cost, but the lessee has not
paid the purchase price.
IAS 7 therefore suggests that there should be a disclosure, in a note to the financial statements, of
the total amount of property, plant and equipment acquired during the period, and the cash
payments that were made to acquire them. These two amounts are different, because some of the
non-current assets might have been acquired under finance lease arrangements.

Illustration 33:
A note to the financial statements is as follows.
During the period, the company acquired property, plant and equipment with an aggregate cost of
Rs. 250,000, of which Rs. 60,000 was acquired by means of leases. Cash payments of Rs. 190,000
were made to purchase property, plant and equipment.
In this illustration, Rs. 190,000 would appear as a cash outflow in the statement of cash flows in
the section for cash flows from investing activities for the period.
❑ The Rs. 190,000 is the amount of cash actually paid for purchases of property, plant and
equipment in the period.
❑ The cash payments under the terms of the leases are not included in this part of the statement
of cash flows.

© Emile Woolf International 79 The Institute of Chartered Accountants of Pakistan


Financial accounting and reporting I

5 CASH FLOWS FROM FINANCING ACTIVITIES


Section overview

◼ Examples of cash flows from financing activities


◼ Cash from new share issues
◼ Cash from new loans/cash used to repay loans
◼ Dividend payments to equity shareholders
◼ Financing of a sole proprietor or a partnership

5.1 Examples of cash flows from financing activities


Examples of cash flows from financing activities are listed below:

Cash payments Cash receipts


Cash payments to redeem/buy back shares Cash proceeds from issuing shares
Cash payments to repay a loan or redeem Cash proceeds from a loan or issue of
bonds bonds

As explained earlier, payments of dividends are also usually included within cash flows from
financing activities, in this part of the statement of cash flows. (Some entities may also include
interest payments in this section, instead of including them in the section for cash flows from
operating activities.)

5.2 Cash from new share issues


The cash raised from new share issues can be established by comparing the equity share capital
and the share premium in the statements of financial position at the beginning and the end of the
year.

Illustration 34:
Rs.
Share capital + Share premium at the end of the year X
Share capital + Share premium at the beginning of the year X
Cash obtained from issuing new shares in the year X

Illustration 35: The statements of financial position of Company P at 1 January and 31 December
included the following items:
1 January 31 December
2017 2017
Rs. Rs.
Equity shares of Rs. 1 each 600,000 750,000
Share premium 800,000 1,100,000
The cash obtained from issuing shares during the year is calculated as follows.
Rs.
Share capital + Share premium at the end of 2017 1,850,000
Share capital + Share premium at the beginning of 2017 1,400,000
= Cash obtained from issuing new shares in 2017 450,000

© Emile Woolf International 80 The Institute of Chartered Accountants of Pakistan


Chapter 3: IAS 7: Statement of cash flows

5.3 Cash from new loans/cash used to repay loans


Cash from new loans or cash paid to redeem loans in the year can be calculated simply by looking
at the difference between the liabilities for loans and bonds at the beginning and the end of the
year.
❑ An increase in loans or bonds means there has been an inflow of cash during the year.
❑ A reduction in loans or bonds means there has been a payment (outflow) of cash.
Remember to add any loans, loan notes or bonds repayable within one year (current liability) to
the loans, loan notes or bonds repayable after more than one year (non-current liability) to get the
total figure for loans, loan notes or bonds.
Illustration 36:
Rs.
Loans at end of year (current and non-current liabilities) X
Loans at beginning of year (current and non-current liabilities) (X)
Cash inflow or outflow X/(X)

Note: The same calculation can be applied to bonds or loan notes that the company might have
issued. Bonds and loan notes are long-term debt.

Illustration 37: The statements of financial position of Company Q at 1 January and 31 December
included the following items:
1 January 31 December
2017 2017
Rs. Rs.
Loans repayable within 12 months 760,000 400,000
Loans repayable after 12 months 1,400,000 1,650,000
The cash flows relating to loans during the year are calculated as follows.
Rs.
Loans outstanding at the end of 2017 2,050,000
Loans outstanding at the beginning of 2017 2,160,000
= Net loan repayments during the year (= cash outflow) 110,000

5.4 Dividend payments to equity shareholders


These should be the final dividend payment from the previous year and the interim dividend
payment for the current year. The dividend payments during the year are shown in the Statement
of Changes in Equity (SOCIE).
You might be expected to calculate dividend payments from figures for retained earnings and the
profit after tax for the year.
The equity dividend payments can be calculated as follows:
Illustration 38:
Rs.
Retained earnings at the beginning of the year X
Profit after tax X
Any other transfer into the account X
Increase in the retained earnings reserve X
Retained earnings at the end of the year (X)
Equity dividend payments X

© Emile Woolf International 81 The Institute of Chartered Accountants of Pakistan


Financial accounting and reporting I

Example 09:
Question: From the following information, calculate the cash flows from financing activities for
Company X in 2017.
Beginning End
of 2017 of 2017
Rs. Rs.
Share capital (ordinary shares) 400,000 500,000
Share premium 275,000 615,000
Retained earnings 390,000 570,000
1,065,000 1,685,000
Loans repayable after more than 12 months 600,000 520,000
Loans repayable within 12 months or less 80,000 55,000
The company made a profit of Rs. 420,000 for the year after taxation.
Required
Calculate for 2017, for inclusion in the statement of cash flows:
(a) the cash from issuing new shares
(b) the cash flows received or paid for loans
(c) The payment of dividend to ordinary shareholders.
Answer:
Proceeds from new issue of shares Rs.
Share capital and share premium:
At the end of the year (500,000 + 615,000) 1,115,000
At the beginning of the year (400,000 + 275,000) (675,000)
Proceeds from new issue of shares during the year 440,000

Repayment of loans Rs.


Loans repayable:
At the end of the year (520,000 + 55,000) 575,000
At the beginning of the year (600,000 + 80,000) (680,000)
Repayment of loans during the year 105,000

Payment of dividends Rs.


Retained earnings at the beginning of the year 390,000
Profit after taxation for the year 420,000
810,000
Retained earnings at the end of the year (570,000)
Dividends paid during the year 240,000
Cash flows from financing activities can now be presented as follows.
Cash flows from financing activities Rs. Rs.
Proceeds from issue of shares 440,000
Repayment of loans (105,000)
Dividends paid to shareholders (240,000)
Net cash from financing activities 95,000

© Emile Woolf International 82 The Institute of Chartered Accountants of Pakistan


Chapter 3: IAS 7: Statement of cash flows

5.5 Financing of a sole proprietor or a partnership


You may face a question asking for the preparation of a statement of cash flows for a sole proprietor
or partnership. Such a question might require the calculation of cash flows between the owners
and the business. These cash flows would be capital introduced and drawings.
It is useful to remember the following relationship:

Illustration 39:

Rs.

Capital at the beginning of the year X

Profit (loss) after tax X/(X)

Capital introduced X

Drawings (X)

Capital at the end of the year X

The drawings and capital introduced figures might be provided in the question in which case you
simply have to slot the figures into the cash flow statement.
Other questions might need you to identify one or other of these as balancing figure.

EXAMPLE 10: Universal Limited (UL)


Question: Following are the extracts from the financial statements of Universal Limited (UL) for the
year ended 30 June 2017:

Statement of financial position as on 30 June 2017

Assets 2017 2016 Equity & liabilities 2017 2016

Rs. in ‘000 Rs. in ‘000

Property, plant and 158,500 120,000 Share capital (Rs. 10 175,000 150,000
equipment each)

Retained earnings 54,434 21,500

Deferred tax asset 8,500 - Revaluation surplus 10,000 -

Stock in trade 58,000 45,000 Debentures (Rs. 100 18,000 20,000


each)

Trade receivables 68,000 56,000 Deferred tax liability - 6,000

Cash 39,434 48,000 Interest payable 1,000 2,500

Trade payables 42,000 39,000

Accrued liabilities 20,000 18,000

Unearned maintenance 2,000 4,000

Provision for taxation 10,000 8,000

332,434 269,000 332,434 269,000

© Emile Woolf International 83 The Institute of Chartered Accountants of Pakistan


Financial accounting and reporting I

Statement of profit or loss for the year ended 30 June 2017


Rs. in '000
Sales 273,000
Cost of sales (187,500)
Gross profit 85,500
Operating expenses (46,766)
Other income 11,200
Profit before interest and tax 49,934
Interest expense (2,000)
Profit before tax 47,934
Tax expense (15,000)
Profit after tax 32,934

Additional information:
i. 60% of sales were made on credit.
ii. UL maintains a provision for doubtful receivables at 6%. During the year, trade receivables of
Rs. 7 million were written off.
iii. Depreciation expense for the year was Rs. 22.5 million. 70% of the depreciation was charged
to cost of sales.
iv. Other income comprises of:
• gain of Rs. 3 million on disposal of vehicles for Rs. 12 million;
• maintenance income of Rs. 8 million; and
• discount of Rs. 10 per debenture which were redeemed during the year.
Required: Based on the above information, prepare UL’s statement of cash flows for the year ended
30 June 2017 using direct method.
Answer:
Statement of Cash Flows
For the year ended 30 June 2017
Cash flows from operating activities Rs. in ‘000

Cash receipts from customers

(Cash sales: 109,200; Credit sales: 144,034) (W-1) 253,234

Cash receipts from customers - maintenance services (W-2) 6,000

Cash paid to suppliers (W-3) (181,750)

Cash paid to other vendors (W-4) (30,250)

Income taxes paid (8,000+6,000+15,000–10,000+8,500) (27,500)

Interest paid (2,500+2,000–1,000) (3,500)

Net cash inflow from operating activities 16,234

© Emile Woolf International 84 The Institute of Chartered Accountants of Pakistan


Chapter 3: IAS 7: Statement of cash flows

Rs. in ‘000
Cash flows from investing activities

Purchase of property, plant and equipment [120,000–158,500–9,000


(i.e. 12,000–3,000)–22,500+10,000] (60,000)
Proceeds from disposal of vehicles 12,000
Net cash outflow from investing activities (48,000)

Cash flows from financing activities


Redemption of debentures [(20,000–18,000)–(20×10)] (1,800)
Proceeds from issue of shares (175,000–150,000) 25,000
Net cash inflow from financing activities 23,200
Net decrease in cash and cash equivalents (8,566)
Cash and cash equivalent at the beginning of the year 48,000
Cash and cash equivalent at the end of the year 39,434
Workings:
W-1: Cash receipts from customers - sales
Trade receivables – opening (56,000÷0.94) 59,574
Sales for the year 273,000
Bad debts written off (7,000)
Trade receivables – closing (68,000÷0.94) (72,340)
Cash received from customers 253,234

W-2: Cash receipts from customers - maintenance service


Unearned maintenance – opening (4,000)
Maintenance income for the year 8,000
Unearned maintenance – closing 2,000
6,000

W-3: Cash paid to suppliers --------- Rs. in ‘000 ---------


Trade payables – opening 39,000
Add: Purchases / Manufacturing cost
Stock in trade – closing 58,000
Cost of goods sold less dep. [187,500–(22,500×70%)] 171,750

Stock in trade – opening (45,000) 184,750


Less: Trade payables – closing (42,000)
Cash paid to suppliers 181,750

© Emile Woolf International 85 The Institute of Chartered Accountants of Pakistan


Financial accounting and reporting I

W-4: Cash paid to other vendors Rs. in ‘000


Accrued liabilities – opening 18,000
Operating expense for the year 46,766
Depreciation (22,500×30%) (6,750)
Bad debt expense (W-4.1) (7,766)
Accrued liabilities – closing (20,000)
30,250
W-4.1: Bad debts expense for the year
Provision for doubtful receivables – opening (56,000÷0.94×0.06) (3,574)
Bad debts written off 7,000
Provision for doubtful receivable – closing (68,000÷0.94×0.06) 4,340
7,766

EXAMPLE 11: TRANGO LIMITED


Question: The following information has been extracted from the financial statements of Trango
Limited for the year ended 31 December 2015.
Statement of comprehensive income for the year ended 31 December 2015
Sales 905,000
Cost of sales (311,000)
Gross profit 594,000
Loss on disposal of non-current asset (9,000)
Wages and salaries (266,000)
Other expenses (including depreciation Rs.46,000) (193,000)
126,000
Interest charges (24,000)
Profit before tax 102,000
Tax on profit (38,000)
Profit after tax 64,000
The asset disposed of had a carrying amount of Rs. 31,000 at the time of the sale.

Extracts from the statement of financial At 1 Jan At 31 Dec


position: 2015 2015
Rs.
Trade receivables 157,000 173,000
Inventory 42,000 38,000
Trade payables 43,600 35,700
Accrued wages and salaries 4,000 4,600
Accrued interest charges 11,200 10,000
Tax payable 45,000 41,000

Required: Present the cash flows from operating activities as they would be presented in a statement
of cash flows using direct method.

© Emile Woolf International 86 The Institute of Chartered Accountants of Pakistan


Chapter 3: IAS 7: Statement of cash flows

Answer:

Statement of cash flows Rs.


Cash receipts from customers (W1) 889,000
Cash payments to suppliers (W2) (314,900)
Cash payments to employees (W3) (265,400)
Cash paid for other expense (193,000-46,000) (147,000)
Cash generated from operations 161,700
Taxation paid (W4) (42,000)
Interest charges paid(W4) (25,200)
Net cash flow from operating activities 94,500

Workings:
W1 Cash receipts from customers
Trade receivables at the beginning of the year 157,000
Sales in the year 905,000
Trade receivables at the end of the year (173,000)
Cash from sales during the year 889,000

W2 Cash paid to suppliers


Calculation of Purchases
Closing inventory at the end of the year 38,000
Cost of sales 311,000
Opening inventory at the beginning of the year (42,000)
Purchases in the year 307,000

Trade payables at the beginning of the year 43,600


Purchases in the year (as above) 307,000
350,600
Trade payables at the end of the year (35,700)
Cash paid for materials 314,900

W3 Cash paid to employees


Accrued wages and salaries at the beginning of the year 4,000
Wages and salaries expenses in the year 266,000
Accrued wages and salaries at the end of the year (4,600)
Cash paid for wages and salaries 265,400

W4: Interest and tax payments Tax Interest


Rs. Rs.
Liability at 1 January 2015 45,000 11,200
Taxation charge/interest charge for the year 38,000 24,000
Liability at 31 December 2015 (41,000) (10,000)
Tax paid/interest paid during the year 42,000 25,200

© Emile Woolf International 87 The Institute of Chartered Accountants of Pakistan


Financial accounting and reporting I

EXAMPLE 12: NARDONE LIMITED


Question: The following information has been extracted from the draft financial information of
Nardone Limited.
Statement of comprehensive income for the year ended 31 December 2015
Rs.000 Rs.000
Sales revenue 490
Administration costs (86)
Distribution costs (78)
(164)
Operating profit 326
Interest expense (23)
Profit before tax 303
Taxation (87)
Profit after tax 216
Dividends paid (52)
Retained profit for the year 164

Statement of financial position


31 December 2015 31 December 2014
Rs.000 Rs.000 Rs.000 Rs.000
Non-current assets (see below) 1,145 957
Current assets:
Inventory 19 16
Receivables 38 29
Bank 19 32
76 77
Total assets 1,221 1,034

Share capital 323 232


Revaluation reserve 170 0
Retained earnings 553 389
1,046 621
Non-current liabilities:
Long-term loans 70 320
Current liabilities:
Trade payables 12 17
Tax payable 93 76
105 93
Total equity and liabilities 1,221 1,034

© Emile Woolf International 88 The Institute of Chartered Accountants of Pakistan


Chapter 3: IAS 7: Statement of cash flows

Note on non-current assets

Land Fixtures
and Machinery & Total
buildings fittings

Rs.000 Rs.000 Rs.000 Rs.000

Cost or valuation

At 31 December 2014 830 470 197 1,497

Additions - 43 55 98

Disposals - (18) - (18)

Adjustment on revaluation 70 - - 70

At 31 December 2015 900 495 252 1,647

Depreciation

At 31 December 2014 (90) (270) (180) (540)

Charge for the year (10) (56) (8) (74)

Disposals - 12 - 12

Adjustment on revaluation 100 - - 100

At 31 December 2015 0 (314) (188) (502)

Carrying amount:

At 31 December 2014 740 200 17 957

At 31 December 2015 900 181 64 1,145

You have been informed that included within distribution costs is Rs. 4,000 relating to the loss on a
disposal of a non-current asset.
Required:
Prepare a statement of cash flows for Nardone Limited for the year ended 31 December 2015.

© Emile Woolf International 89 The Institute of Chartered Accountants of Pakistan


Financial accounting and reporting I

Answer:
Nardone Limited
Statement of cash flows for the year ended 31 December 2015
Rs.000 Rs.000
Cash flows from operating activities
Profit before taxation 303
Adjustments for:
Depreciation 74
Interest charges in the statement of comprehensive income 23
Losses on disposal of non-current assets 4
404
Increase in receivables (38 – 29) (9)
Increase in inventories (19 – 16) (3)
Decrease in trade payables (17 – 12) (5)
Cash generated from operations 387
Taxation paid (W1) (70)
Interest charges paid (23)
Net cash flow from operating activities 294
Cash flows from investing activities
Purchase of non-current assets (98)
Proceeds from sale of non-current assets (W2) 2
Net cash used in (or received from) investing activities (96)
Cash flows from financing activities
Proceeds from issue of shares (323 – 232) 91
Repayment of loans (320 – 70) (250)
Dividends paid to shareholders (52)
Net cash used in (or received from) financing activities (211)
Net increase/(decrease) in cash and cash equivalents (13)
Cash and cash equivalents at beginning of the year 32
Cash and cash equivalents at the end of the year 19

Workings
W1: Taxation paid Rs.000
Taxation payable at the beginning of the year 76
Tax charge for the year (statement of comprehensive income) 87
163
Taxation payable at the end of the year (93)
Therefore tax paid during the year 70

W2: Disposal of machinery Rs.000


Cost of machinery disposed of 18
Accumulated depreciation on machinery disposed of (12)
Net book value at disposal 6
Loss on disposal 4
Therefore cash received from the disposal 2

© Emile Woolf International 90 The Institute of Chartered Accountants of Pakistan


Chapter 3: IAS 7: Statement of cash flows

EXAMPLE 13: HOT SAUCE LIMITED


Question: Hot Sauce Limited summarised final accounts are as follows:
Statement of financial position
31 December 2014 31 December 2015
Rs.000 Rs.000 Rs.000 Rs.000
Non-current assets:
Plant and machinery at cost 2,700 3,831
Accumulated depreciation (748) (1,125)
Carrying amount 1,952 2,706
Current assets:
Inventory 203 843
Receivables 147 184
Bank 51 -
401 1,027
Total assets 2,353 3,733
Ordinary share capital (Rs1 shares) 740 940
Share premium account 0 100
Retained earnings 671 1,034
1,411 2,074
Non-current liabilities:
Loans 320 150
Current liabilities:
Bank overdraft 0 766
Trade payables and accruals 152 141
Current taxation 470 602
622 1,509
Total equity and liabilities 2,353 3,733

Statement of comprehensive income for year ended 31 December 2015


R.s 000
Profit before tax 1,195
Taxation (602)
Profit after tax 593

Dividend payments during the year were Rs. 230,000.


The following information is also available:
(1) The only new loan raised during the year was a five-year bank loan amounting to Rs.
65,000.
(2) Interest charged during the year was Rs. 156,000. Interest accrued was Rs. 24,000 last
years and Rs. 54,000 this year.
(3) Depreciation charged during the year amounted to Rs. 401,000. This does not include any
profit or loss on disposal of non-current assets.
(4) During the year plant which originally cost Rs. 69,000 was disposed of for Rs. 41,000.
(5) During the year the company issued 200,000 new shares.
Required: Prepare a statement of cash flows for Hot Sauce Limited for the year ended 31 December
2015.

© Emile Woolf International 91 The Institute of Chartered Accountants of Pakistan


Financial accounting and reporting I

Answer:
Hot Sauce Limited

Statement of cash flows for the year ended 31 December 2015


Rs.000 Rs.000
Cash flows from operating activities
Profit before taxation 1,195
Adjustments for:
Depreciation 401
Loss on sale of plant (W1) 4
Interest charges in the statement of comprehensive income 156
1,756
Increase in receivables (184 – 147) (37)
Increase in inventories (843 – 203) (640)
Decrease in trade payables (W2) (41)
Cash generated from operations 1,038
Taxation paid (W3) (470)
Interest charges paid (W4) (126)
Net cash flow from operating activities 442

Cash flows from investing activities


Purchase of non-current assets (1,200)
Proceeds from sale of non-current assets (see W1) 41
Net cash used in (or received from) investing activities (1,159)
Cash flows from financing activities
Proceeds from issue of shares (W5) 300
Bank loan raised 65
Repayment of loans (W6) (235)
Dividends paid to shareholders (230)
Net cash used in (or received from) financing activities (100)
Net increase/(decrease) in cash and cash equivalents (817)
Cash and cash equivalents at beginning of the year 51
Cash and cash equivalents at the end of the year (766)
Cost of asset disposed of 69
Accumulated depreciation on asset disposed of (24)
Carrying amount at date of disposal 45
Disposal proceeds (41)
Therefore loss on disposal 4

© Emile Woolf International 92 The Institute of Chartered Accountants of Pakistan


Chapter 3: IAS 7: Statement of cash flows

W2: Increase or decrease in trade payables Rs.000 Rs.000


Trade payables and accruals at 31 December 2015 141
Less accrued interest (54)
87
Trade payables and accruals at 31 December 2014 152
Less accrued interest (24)
(128)
Decrease in trade payables and accruals (41)
Tutorial note
The accrued interest is removed from the figures because accrued interest is relevant to the amount
of interest paid in the year. This is a separate item in the statement of cash flows.
W3: Taxation paid
Current taxation liability at 31 December 2014 470
Taxation charge in the year 602
1,072
Current taxation liability at 31 December 2015 (602)
Therefore taxation paid in the year 470

W4: Interest paid


Accrued interest liability at 31 December 2014 24
Interest charge in the year 156
180
Accrued interest liability at 31 December 2015 (54)
Therefore interest paid in the year 126

W5: Proceeds from the issue of shares


Ordinary share capital at 31 December 2015 940
Share premium at 31 December 2015 100
1,040
Ordinary share capital at 31 December 2014 740
Share premium at 31 December 2014 -
(740)
Proceeds from the issue of shares 300

W6: Loans repaid


Loans at 31 December 2014 320
New loan during the year 65
385
Loans at 31 December 2015 (150)
Therefore loans repaid during the year 235

© Emile Woolf International 93 The Institute of Chartered Accountants of Pakistan


Financial accounting and reporting I

EXAMPLE 14: QUETTA TRACK LIMITED


Question: The statement of financial position of Quetta Track Limited as at 30 June was as follows:
2015 2014
Rs. 000 Rs. 000 Rs. 000 Rs. 000
ASSETS
Non-current assets
Property cost 22,000 12,000
Depreciation (4,000) (1,000)
————— 18,000 ————— 11,000
Plant and equipment
Cost 5,000 5,000
Depreciation (2,250) (2,000)
————— 2,750 ————— 3,000
————— —————
20,750 14,000

Current assets
Inventories 16,000 11,000
Trade receivables 9,950 2,700
Cash and cash equivalents – 1,300
————— 25,950 ————— 15,000
————— —————
Total assets 46,700 29,000
————— —————
EQUITY AND LIABILITIES
Capital and reserves
Equity capital 3,000 3,000
Accumulated profits 16,200 3,800
————— —————
19,200 6,800
Non-current liabilities
Loan 6,000 10,000
Current liabilities
Operating overdraft 11,000 –
Trade payables 8,000 11,000
Income tax payable 1,800 1,000
Accrued interest 700 200
————— 21,500 ————— 12,200
————— —————
Total equity and liabilities 46,700 29,000
————— —————

© Emile Woolf International 94 The Institute of Chartered Accountants of Pakistan


Chapter 3: IAS 7: Statement of cash flows

Statement of comprehensive income (extracts)

2015 2014

Rs.000 Rs.000

Operating profit 15,400 5,900

Financing cost (Interest) (1,000) (1,400)

————— —————

Profit before tax 14,400 4,500

Income tax expense (2,000) (1,500)

————— —————

Net profit for the year 12,400 3,000

————— —————
Equipment of carrying amount Rs.250,000 was sold at the beginning of 2015 for Rs.350,000. This
equipment had originally cost Rs.1,000,000.

In recent years, no dividends have been paid.

Required:
Prepare a statement of cash flows, under the indirect method, for the year ended 30 June 2015
Answer:
Quetta Track Limited

Statement of cash flow for the year ended 30 June, 2015.

Rs.000 Rs.000
Cash flows from operating activities
Net profit before tax 14,400
Adjustments for
Depreciation (Rs. 3,000 + 1,000) 4,000
Profit on sale of non-current assets (W3) (100)
Interest expense 1,000
—————
Operating profit before working capital adjustments 19,300
Increase in inventories (5,000)
Increase in trade receivables (7,250)
Decrease in trade payables (3,000)
—————
Cash generated from operations 4,050
Interest paid (W5) (500)
Income taxes paid (W4) (1,200)
—————
Net cash from operating activities 2,350

© Emile Woolf International 95 The Institute of Chartered Accountants of Pakistan


Financial accounting and reporting I

Cash flows from investing activities

Purchase of property (10,000)

Purchase of plant and equipment (W1) (1,000)

Proceeds from sale of plant and equipment (W3) 350

—————

Net cash used in investing activities (10,650)

Cash flows from financing activities

Part repayment of loan (4,000)


—————
—————
Net cash used in financing activities (4,000)
—————
Net decrease in cash and cash equivalents (12,300)

Cash and cash equivalents at beginning of year 1,300


—————
Cash and cash equivalents at end of period (11,000)
—————

Workings
(1) Plant and machinery – Cost

Rs.000 Rs.000

Bal b/d 5,000 Disposal 1,000


Additions () 1,000 Bal c/d 5,000
——– ——–
6,000 6,000
——– ——–

(2) Plant and machinery – Accumulated depreciation

Rs.000 Rs.000
Disposal 750 Bal b/d 2,000
Bal c/d 2,250 Depreciation charge for year () 1,000
——– ——–
3,000 3,000
——– ——–

© Emile Woolf International 96 The Institute of Chartered Accountants of Pakistan


Chapter 3: IAS 7: Statement of cash flows

(3) Plant and machinery – Disposals

Rs.000 Rs.000
Cost 1,000 Accumulated depreciation 750
Profit on sale 100 Proceeds 350
——– ——–
1,100 1,100
——– ——–
(4) Tax payable

Rs.000 Rs.000

Cash paid () 1,200 Bal b/d 1,000


Bal c/d 1,800 Tax charge to P&L 2,000
——– ——–
3,000 3,000
——– ——–
(5) Interest payable

Rs.000 Rs.000

Cash paid () 500 Bal b/d 200


Bal c/d 700 Charge to P&L 1,000
——– ——–
1,200 1,200
——– ——–

Example 15: MARDAN SOFTWARE LIMITED


Question: The following are the summarized accounts of Mardan Software Limited
Statement of financial position at 31 December
2014 2015
Rs.(000) Rs.(000) Rs.(000) Rs.(000)
ASSETS
Non-current assets
Plant and equipment 2,086 2,103
Fixtures and fittings 1,381 1,296
———— ————
3,467 3,399
Current assets
Inventory 1,292 1,952
Trade receivables 713 1,486
Short term investment 1,050 600
Cash 197 512
———— 3,252 ———— 4,550
———— ————
Total assets 6,719 7,949
———— ————

© Emile Woolf International 97 The Institute of Chartered Accountants of Pakistan


Financial accounting and reporting I

EQUITY AND LIABILITIES


Capital and reserves
Equity capital 4,200 4,500
Share premium reserve 800 900
Accumulated profits (Note 1) 431 1,180
———— ————
5,431 6,580
Current liabilities
Dividend payable 132 154
Income tax payable 257 312
Trade payables 899 903
———— 1,288 ———— 1,369
———— ————
Total equity and liabilities 6,719 7,949
———— ————

Statement of comprehensive income (extracts) for the year ended 31 December 2015

Rs.(000)

Profit before taxation 1,381

Income tax expense (310)

Net profit for the period 1,071

Note 1 Accumulated profits

Rs.(000)

Balance at 1 January 431

Net profit for the year 1,071

Dividend for the year (322)

Balance at 31 December 1,180

Further information:
(1) Plant and equipment with a carrying amount of Rs184,000 was disposed of for Rs.203,000,
whilst a new item of plant was purchased for Rs312,000
(2) Fixtures and fittings with a carrying amount of Rs100,000 were disposed of for Rs95,000;
(3) Depreciation recognised on fixtures and fittings amounted to Rs 351,000.
(4) Dividend for the year was declared during the year. Dividend payable in the statements of
financial position at each year end relate to dividends declared in that year but not paid over
to shareholders by the reporting date.
Required: Prepare a statement of cash flows for the year ended 31 December 2015 in accordance
with IAS 7: Statement of cash flows.

© Emile Woolf International 98 The Institute of Chartered Accountants of Pakistan


Chapter 3: IAS 7: Statement of cash flows

Answer:
Mardan Software Limited
Statement of cash flows for the year ended 31 December 2015

Rs.000 Rs.000
Cash flow from operating activities
Net profit before tax 1,381
Adjustments for
Depreciation charges (111 + 351) (W1, W2) 462
Profit on sale of machinery (W1) (19)
Loss on sale of fixtures (W2) 5
—————
Operating profit before working capital adjustments 1,829
Increase in inventories (660)
Increase in trade receivables (773)
Increase in trade payables 4
—————
Cash generated from operations 400
Income tax paid (W3) (255)
—————
Net cash from operating activities 145

Cash flows from investing activities


Purchase of plant and equipment (312 + 366) (W1, W2) (678)
Proceeds from sale of plant and equipment (203 + 95) (W1, W2) 298
—————
Net cash used in investing activities (380)

Cash flows from financing activities


Equity dividends paid (W4) (300)
Proceeds from issuance of ordinary share capital 400
—————
100
—————
Net decrease in cash and cash equivalents (135)

Cash and cash equivalents at beginning of year (1,050 + 197) 1,247


—————

Cash and cash equivalents at end of year (600 + 512) 1,112


—————

© Emile Woolf International 99 The Institute of Chartered Accountants of Pakistan


Financial accounting and reporting I

Workings
(1) Plant and equipment (carrying amt.)

Rs.000 Rs.000

Balance b/f 2,086 P & E – disposal 184


Bank – purchase 312 Depreciation (al fig) 111
Balance c/f 2,103
——– ——–
2,398 2,398
——– ——–

Plant and equipment– Disposal

Rs.000 Rs.000

P & E – Carrying amt. 184 Cash – proceeds 203


Gain on disposal 19
—— ——
203 203
—— ——

(2) Fixtures and fittings (Carrying amount)

Rs.000 Rs.000

Balance b/f 1,381 F & F – disposal 100


Bank – purchase (Bal. figure) 366 Depreciation 351
Balance c/f 1,296
——– ——–
1,747 1,747
——– ——–

Fixtures and fittings – Disposal

Rs.000 Rs.000

F & F – Carrying amt. 100 Cash – proceeds 95


Loss on disposal 5
—— ——
100 100
—— ——

(3) Tax payable

Rs.000 Rs.000

Bank – tax paid (al fig) 255 Balance b/f 257


Balance c/f 312 P&L a/c 310
—— ——
567 567
—— ——

© Emile Woolf International 100 The Institute of Chartered Accountants of Pakistan


Chapter 3: IAS 7: Statement of cash flows

(4) Dividends paid

Rs.000 Rs.000

Bank – dividends paid (al fig) 300 Balance b/f 132


Balance c/f 154 2015 dividend 322
—— ——
454 454
—— ——

Example 16: TARBELA TRADERS LIMITED


Question: The statement of financial position of Tarbela Traders Limited is presented below:
Statement of financial position

31 December 2015 31 December 2014


Rs.000 Rs.000 Rs.000 Rs.000
Non-current assets (at WDV)
Premises 37,000 38,000
Equipment 45,800 17,600
Motor vehicles 18,930 4,080
101,730 59,680
Investments 25,000 17,000
126,730 76,680
Current assets
Inventories 19,670 27,500
Trade receivables and prepayments 11,960 14,410
Cash and bank balances 5,500 5,400
37,130 47,310
Total assets 163,860 123,990

Capital and reserves


Share capital 67,940 67,940
Retained earnings 10,670 7,100
78,610 75,040
Non-current liabilities
Interest-bearing borrowings 25,000 28,000
Current liabilities
Trade payables and accrued expenses 32,050 20,950
Bank overdraft 28,200 –
60,250 20,950
163,860 123,990

© Emile Woolf International 101 The Institute of Chartered Accountants of Pakistan


Financial accounting and reporting I

Profit for the year ended 31 December 2015 is Rs. 3,570,000 (after accounting for);
Rs.000
Depreciation
Premises 1,000
Equipment 3,000
Motor vehicles 3,000
Profit on disposal of equipment 430
Loss on disposal of motor vehicle 740
Interest expense 3,000
The written down value of the assets at date of disposal was:
Equipment 5,200
Motor vehicles 2,010
Interest accrued at 31 December 2015 is Rs.400,000.
Required: Prepare a statement of cash flows for the year ended 31 December 2015.

Answer:
Tarbela Trader Limited
Statement of cash flows for the year ended 31 December 2015
Rs.000 Rs.000
Cash flows from operating activities
Net profit 3,570
Adjustments for
Depreciation 7,000
Net loss on disposals 310
Interest expense 3,000

Operating profit before working capital changes 13,880


Decrease in trade receivables (11,960 – 14,410) 2,450
Decrease in inventories (19,670 – 27,500) 7,830
Increase in trade payables ((32,050 – 400) – 20,950) 10,700
Cash generated from operations 34,860
Interest paid (3,000 – 400) (2,600)
Net cash from operating activities 32,260

Cash flows from investing activities


Purchase of long-term investments (25,000 – 17,000) (8,000)
Purchase of equipment and cars
(36,400 (W1)+ 19,860 (W2)) (56,260)
Proceeds from sale of equipment and cars (W3) 6,900
Net cash used in investing activities (57,360)
Cash flows from financing activity
Borrowing repayment (3,000)

Net decrease in cash and cash equivalents (28,100)

Cash and cash equivalents at beginning of period (3,600 + 1,800) 5,400


Cash and cash equivalents at end of period
(4,800 + 700 – 28,200) (22,700)

© Emile Woolf International 102 The Institute of Chartered Accountants of Pakistan


Chapter 3: IAS 7: Statement of cash flows

Workings
(1) Equipment (WDV)

Rs.000 Rs.000
Bal b/d 17,600 Disposal 5,200
Depreciation 3,000
Additions () 36,400 Bal c/d 45,800
——– ——–
54,000 54,000
——– ——–

(2) Motor vehicles (WDV)

Rs.000 Rs.000
Bal b/d 4,080 Disposal 2,010
Depreciation 3,000
Additions () 19,860 Bal c/d 18,930
——– ——–
23,940 23,940
——– ——–

(3) Disposals

Rs.000 Rs.000
Equipment 5,200
Motor vehicle 2,010 Loss on disposal (vehicles) 740
Profit on disposal (equipment) 430 Proceeds () 6,900
——– ——–
7,640 7,640
——– ——–

Example 17: THE SINDH ROBOTICS COMPANY


Question: The statements of financial position and statement of comprehensive incomes of The
Sindh Robotics Company for two consecutive financial years are shown below.
Statements of financial position
31 December 2013 31 December 2014
Cost Dep’n Net Cost Dep’n Net
Rs.000 Rs.000 Rs.000 Rs.000 Rs.000 Rs.000
Non-current assets
Land 43,000 – 43,000 63,000 – 63,000
Buildings 50,000 10,000 40,000 90,000 11,000 79,000
Plant 10,000 4,000 6,000 11,000 5,000 6,000
—————— —————— —————— —————— —————— ——————
103,000 14,000 89,000 164,000 16,000 148,000
—————— —————— —————— ——————

Investments 50,000 80,000

© Emile Woolf International 103 The Institute of Chartered Accountants of Pakistan


Financial accounting and reporting I

31 December 2013 31 December 2014


Cost Dep’n Net Cost Dep’n Net
Rs.000 Rs.000 Rs.000 Rs.000 Rs.000 Rs.000
Current assets
Inventories 55,000 65,000
Trade receivables 40,000 50,000
Bank 3,000 –
—————— 98,000 —————— 115,000
—————— ——————
237,000 343,000
—————— ——————
Capital
Issued shares of
Rs. 1 each 40,000 50,000
Share premium 12,000 14,000
Revaluation – 20,000
surplus
Accumulated profit 25,000 25,000
—————— ——————
77,000 109,000
Non-current
liabilities
10% loan 100,000 150,000
borrowings

Current liabilities
Trade payables 40,000 60,000
Dividend payable 20,000 20,000
Bank overdraft – 4,000
—————— ——————
60,000 84,000
—————— ——————
237,000 343,000
—————— ——————

Statements of comprehensive incomes 2013 2014


Rs.000 Rs.000
Revenue 200,000 200,000
Cost of sales (100,000) (120,000)
———— ————
Gross profit 100,000 80,000
Distribution and administration expenses (50,000) (47,000)
———— ————
50,000 33,000
Interest (10,000) (13,000)
———— ————
Net profit for year 40,000 20,000
———— ————
Only one dividend is declared each year which is paid in the following year. No sales of non-
current assets have occurred during the relevant period. Ignore taxation.
Required: Prepare a statement of cash flows for the year ended 31 December 2014 using the
direct method.

© Emile Woolf International 104 The Institute of Chartered Accountants of Pakistan


Chapter 3: IAS 7: Statement of cash flows

Answer:
The Sindh Robotics Company
Statement of cash flows for the year ended 31 December 2014

Rs.000 Rs.000
Cash flows from operating activities
Cash receipts from customers (W1) 190,000
Cash paid to suppliers and employees (W2) (155,000)
Cash generated from operations 35,000
Interest paid (13,000)
Dividends paid* (20,000)
Net cash from operating activities 2,000
Cash flows from investing activities
Purchase of property and plant (40,000 + 1,000) (41,000)
Purchase of investments (30,000)
Net cash used in investing activities (71,000)
Cash flows from financing activities
Proceeds from issued shares (10,000 + 2,000) 12,000
Proceeds from long-term borrowings 50,000
Net cash from financing activities 62,000
Net decrease in cash and cash equivalents (7,000)
Cash and cash equivalents at 1 January 2014 3,000
Cash and cash equivalents at 31 December 2014 (4,000)

* Could be shown as a financing cash flow.


Workings
(1) Receipts from sales
Receivables control

Rs.000 Rs.000
Balance b/d 40,000 Cash receipts (al fig) 190,000
Sales 200,000 Balance c/d 50,000
_______ ________
240,000 240,000
———— ————
(2) Payments
Payables and wage control

Rs.000 Rs.000
Cash paid (al fig) 155,000 Balance b/d 40,000
Depreciation * 2,000 Purchases (W3) 130,000
Balance c/d 60,000 Expenses 47,000
________
________ 217,000
217,000 ————
————

© Emile Woolf International 105 The Institute of Chartered Accountants of Pakistan


Financial accounting and reporting I

(3) Cost of sales

Rs.000 Rs.000
Opening inventory 55,000 Cost of sales 120,000
Purchases and wages 130,000 Closing inventory 65,000
________ ________
185,000 185,000
———— ————
* Alternatively, depreciation could be adjusted against cost of sales.

EXAMPLE 18: ABIDA LIMITED


Question: Abida Ltd. made a net profit of Rs. 256,800 for the year ended June 30, 2015 after
charging depreciation of Rs. 17,500 and loss on disposal of furniture of Rs. 6,800. The sale proceeds
of the furniture were Rs. 12,000.
During the year, the net book value of non-current assets decreased by Rs. 7,400; receivables
increased by Rs. 11,700; inventories decreased by Rs. 21,600 and creditors increased by Rs. 8,900.
A long-term loan of Rs. 75,000 was repaid during the year and Abida withdrew Rs. 120,000 for her
own use.
Required:
Prepare the statement of cash flows for the year ended June 30, 2015.
Answer:

Abida Limited

Cash flow for year ended June 30, 2015


Profit for the year 256,800
Depreciation 17,500
Loss on sale of furniture 6,800
Increase/decrease in working capital
Decrease in inventories 21,600
Increase in payables 8,900
Increase in receivables (11,700)
18,800
299,900
Add: Proceeds from sale of non-current assets 12,000
311,900
Less: Purchase of non-current assets (W) 28,900
Payment of long term loan 75,000
Drawings 120,000
223,900
Net increase in bank balance 88,000

© Emile Woolf International 106 The Institute of Chartered Accountants of Pakistan


Chapter 3: IAS 7: Statement of cash flows

W Non-current assets
Decrease in assets 7,400 Depreciation 17,500
Purchase of assets – balancing figure 28,900 Sale of furniture 12,000
Loss on above sale 6,800
36,300 36,300

EXAMPLE 19: MOOSANI LIMITED


Question: The comparative statements of financial position of Moosani Ltd. show the following
information:

December 31
2015 2014
Rs. Rs.
Cash 5,200 41,400
Accounts receivable 31,700 21,500
Inventory 25,000 19,400
Investments - 16,900
Furniture 80,000 64,000
Equipment 86,000 43,000
Total 227,900 206,200

Allowance for doubtful accounts 6,500 9,700


Accumulated depreciation on equipment 24,000 18,000
Accumulated depreciation on furniture 8,000 15,000
Trade creditors 10,800 6,500
Accrued expenses 4,300 10,800
Bills payable 6,500 8,600
Long-term loans 31,800 53,800
Capital 136,000 83,800
Total 227,900 206,200
Additional data related to 2015 is as follows:
(i) Equipment that had cost Rs. 23,000 and was 40% depreciated at the time of
disposal was sold for Rs. 6,500.
(ii) Payments against long-term loans amounted to Rs. 22,000 of which Rs. 12,000
was paid by Mr. Moosani out of his personal account.
(iii) On January 1, 2015, the furniture was completely destroyed by a fire.
Proceeds received from the insurance company amounted to Rs. 60,000.
(iv) Investments were sold at Rs. 7,500 above their cost.
(v) Mr. Moosani withdraws Rs. 15,000 each month for his personal use.
Required:
Prepare a statement of cash flows for the year ended 31 December 2015.

© Emile Woolf International 107 The Institute of Chartered Accountants of Pakistan


Financial accounting and reporting I

Answer:
Moosani Limited
Statement of cash flows for the year ended 31 December 2015
Rs.000 Rs.000
Cash flows from operating activities
Net profit for the year (W1) 220,200
Adjustments for
Depreciation – equipment (24,000 + 9,200 – 18,000) 15,200
– furniture 8,000
Loss on sale of equipment (23,000 – 9,200 – 6,500) 7,300
Gain on sale of investments (7,500)
Insurance claim over book value (60,000 – [64,000 – 15,000]) (11,000)

Operating profit before working capital adjustments 232,200


Increase in payables 4,300
Decrease in bills payable (2,100)
Decrease in accrued expenses (6,500)
Increases in receivables (13,400)
Increase in inventory (5,600)
Cash generated from operations 208,900

Net cash from operating activities 208,900

Cash flows from investing activities


Insurance claim against furniture 60,000
Sale of investments (16,900 + 7,500) 24,400
Sale of equipment 6,500
Capital Expenditure – purchase of equipment
(86,000 + 23,000 – 43,000) (66,000)
Capital Expenditure – purchase of furniture
(80,000 + 64,000 – 64,000) (80,000)
Net cash used in investing activities (55,100)

Cash flows from financing activities


Capital invested (payment of long-term loan) 12,000
Payment of long-term loan (22,000)
Drawings / withdrawals of capital (15,000 x 12) (180,000)

Net cash used in financing activities (190,000)


Net decrease in cash and cash equivalents (36,200)
Cash and cash equivalents at beginning of year 41,400
Cash and cash equivalents at end of period 5,200

© Emile Woolf International 108 The Institute of Chartered Accountants of Pakistan


Chapter 3: IAS 7: Statement of cash flows

Working
W1: Profit for the year Rs.
Capital b/f 83,800
Capital introduced (loan repayment) 12,000
Less: drawings (180,000)
Profit for the year (balancing figure) 220,200
Capital c/f 136,000

EXAMPLE 20: SAKHAWAT HUSSAIN LIMITED


Question: The statements of financial position of Sakha at Hussain Ltd. as at December 31, 2015
and 2014 are as follows:

2015 2014
Rs. Rs.
Current assets 4,750,000 2,850,000
Investments 2,600,000 2,500,000
Non-current assets 9,750,000 9,600,000
Accumulated depreciation (2,950,000) (2,450,000)
14,150,000 12,500,000
Non-current liability (loan) 2,000,000 2,000,000
Current liabilities 1,850,000 1,450,000
Interest liability 200,000 150,000
Capital 9,000,000 8,000,000
Profit and loss account 1,100,000 900,000
14,150,000 12,500,000

Other information for the year 2015 is as follows:


(i) Investments costing Rs. 250,000 were sold for Rs. 320,000.
(ii) Fully depreciated furniture costing Rs. 200,000 was written-off.
(iii) Non-current assets costing Rs. 960,000 with a net book value of Rs. 160,000 were sold for
Rs. 250,000.
(iv) Interest amounting to Rs. 180,000 was paid during the year.
(v) Sakhawat Hussain withdrew Rs. 1,200,000 from the profits of 2014 and 2015.
(vi) 20% of the opening and closing balances of current assets are represented by cash.

Required:
Prepare a statement of cash flows for the year ended December 31, 2015.

© Emile Woolf International 109 The Institute of Chartered Accountants of Pakistan


Financial accounting and reporting I

Answer:
Sakhawat Hussain Limited
Statement of cash flows for the year ended December 31, 2015
Rs.000 Rs.000
Cash flows from operating activities
Net profit before tax 1,400,000
Adjustments for
Depreciation on non-current assets
(2,950,000 – 2,450,000)+200,000+(960,000 – 160,000) 1,500,000
Profit on sale of investment (70,000)
Profit on sale of non-current assets (90,000)
Interest expense (180 + 200 – 150) 230,000

Operating profit before working capital adjustments 2,970,000


Increase in payables 400,000
Increase in current assets 80% of (4,750,000 – 2,850,000) (1,520,000)

Cash generated from operations 1,850,000


Interest paid (180,000)
Net cash from operating activities 1,670,000

Cash flows from investing activities


Purchase of non-current assets
(9,750,000 + 200,000 + 960,000 – 9,600,000) (1,310,000)
Purchase of investment
(2,600,000+250,000 – 2,500,000) (350,000)
Proceeds from sale of investment 320,000
Proceeds from sale of non-current assets 250,000 (1,090,000)
Net cash used in investing activities

Cash flows from financing activities


Capital introduced 1,000,000
Withdrawal by owner against profits (1,200,000)
Net cash used in financing activities (200,000)
Net increase in cash and cash equivalents 380,000
Cash and cash equivalents at beginning of year 570,000
Cash and cash equivalents at end of period 950,000
Working: Profit for the year

Closing balance 1,100,000


Drawings 1,200,000
2,300,000
Less: opening balance 900,000
Net profit for the year 1,400,000

© Emile Woolf International 110 The Institute of Chartered Accountants of Pakistan


Chapter 3: IAS 7: Statement of cash flows

EXAMPLE 21: JUNAID JANJUA LIMITED


Question: Junaid Janjua Ltd. has provided you the following statements of financial position and
statement of comprehensive income.
Statements of financial position as on December 31, 2015

2015 2014

Rupees

Cash 145,000 32,000

Accounts receivable 280,000 104,000

Long-term investments 220,000 170,000

Inventory 424,000 200,000

Prepaid insurance 24,000 36,000

Office supplies 14,000 7,000

Land 1,810,000 2,500,000

Building 2,800,000 2,300,000

Accumulated depreciation (890,000) (720,000)

Equipment 1,200,000 1,150,000

Accumulated depreciation (380,000) (350,000)

Total assets 5,647,000 5,429,000

Accounts payable 158,000 263,000

Wages payable 40,000 24,000

Short-term loans 580,000 580,000

Long-term loans 985,000 1,160,000

Capital 3,884,000 3,402,000

Total liabilities and equity 5,647,000 5,429,000

Statement of comprehensive income for the year ended December 31, 2015

2015
Rupees
Sales revenue 9,280,000
Cost of goods sold (6,199,000)
Gross profit 3,081,000

© Emile Woolf International 111 The Institute of Chartered Accountants of Pakistan


Financial accounting and reporting I

2015
Operating expenses Rupees
Selling expenses 634,000
Administrative expenses 1,348,000
Depreciation expenses 230,000
(2,212,000)
Income from operations 869,000
Other revenues/expenses
Gain on sale of land 64,000
Gain on sale of long term investment 32,000
Loss on sale of equipment (15,000)
81,000
Net income 950,000
Drawings (568,000)
Retained earnings 382,000
Notes:
(a) Part of the long term loan amounting to Rs. 100,000 was paid by Mr. Junaid from his personal
account.
(b) Long term investments costing Rs. 100,000 were sold during the year.
(c) Depreciation charged during the year on equipment amounted to Rs. 60,000. Equipment
having a book value of Rs. 75,000 was sold during the year.
Required: Prepare a statement of cash flows for the year ended December 31, 2015.

Answer:
Junaid Janjua Limited
Statement of cash flows for the year ended 31 December 2015
1

Cash flow from operating activities


Net income 950,000
Adjustments for:
Depreciation (170,000 + 60,000) 230,000
Gain on sale of land (64,000)
Gain on sale of long term investment (32,000)
Loss on sale of equipment 15,000
Operating profit before working capital adjustment 1,099,000

Increase in Accounts receivable (176,000)


Increase in Inventory (224,000)
Decrease in Prepaid insurance 12,000
Increase in Office supplies (7,000)
Decrease in accounts payable (105,000)
Increase in wages payable 16,000
Net cash from operating activities 615,000

© Emile Woolf International 112 The Institute of Chartered Accountants of Pakistan


Chapter 3: IAS 7: Statement of cash flows

Cash flows from investing activities


Proceeds from investment 100,000
Proceeds from sale of:
- Land (2,500,000 – 1,810,000 + 64,000) 754,000
- Equipment (75,000 – 15,000) 60,000
- Long term investments (100,000 + 32,000) 132,000 946,000
Fixed capital expenditure
– building (2,800,000 – 2,300,000) (500,000)
*
– equipment (1,200,000+105,000 –1,150,000) (155,000)
Long term investments (220,000 +100,000-170,000) (150,000)
Cash generated from investing activities 241,000

Cash flows from financing activities


Payment of long term loan (1,160,000– 985,000) (175,000)
Drawings (568,000)
Cash generated from financing activities (743,000)

Increase in cash and cash equivalent 113,000


Cash - opening 32,000
Cash - closing 145,000
*Book value 75,000 + accumulated depreciation 30,000 = Cost Rs. 105,000

EXAMPLE 22: AMIN INDUSTRIES LIMITED


Question: The statements of financial position of Amin Industries Ltd. as at 31 August 2014 and
2015 are as follows:

2015 2014 2015 2014


Rs. Rs. Rs. Rs.
Capital 33,433,000 27,942,000 Non-current assets –
book value 15,172,000 12,346,000

Current liabilities Current assets


Short term Investments
finance 2,545,000 1,616,000 4,911,000 -
Creditors 3,457,000 2,850,000 Inventory 12,178,000 14,950,000
6,002,000 4,466,000 Trade debts – net of
provision for bad debts 6,732,000 4,887,000
Bank 442,000 225,000
24,263,000 20,062,000
39,435,000 32,408,000 39,435,000 32,408,000

© Emile Woolf International 113 The Institute of Chartered Accountants of Pakistan


Financial accounting and reporting I

The following information is also available:


Rupees
Profit during the year ended 31 August 2015 161,000
Mr. Amin’s withdrawals during the year 120,000
Accumulated depreciation on non-current assets – 31 August 2014, 605,000
Accumulated depreciation on non-current assets – 31 August 2015 470,000
Provision for bad debts – 31 August 2014 385,000
Provision for bad debts – 31 August 2015 484,000
During the year non-current assets costing Rs. 1,500,000 with a carrying amount of Rs. 867,000
were sold for Rs. 1,284,000.
Required: Prepare a statement of cash flows for the year ended 31 August 2015. Show necessary
workings.
Answer:
Amin Industries Limited
Statement of cash flows for the year ended 31 August 2015
Cash flows from operating activities Rs. Rs.
Profit for the year 3,161,000
Adjustments for:
Depreciation charge 2,498,000
Profit on sale of non-current assets
(1,284,000 – 867,000) (417,000)
Provision for doubtful debts (484,000 – 385,000) 99,000
Operating profit before working capital adjustments 5,341,000

Decrease in inventory 2,772,000


(Increase) in trade debts (1,944,000)
Increase in payables 607,000
Increase in short term finance 929,000
Net cash from operating activities 7,705,000

Cash flows from investing activities


Purchase of non-current assets (6,191,000)
Sale proceeds of non-current assets 1,284,000
Purchase of investment (4,911,000)
Net cash from investing activities (9,818,000)

Cash flows from financing activities


Capital input 5,450,000
Withdrawals by Mr. Amin (3,120,000)
Net cash from financing activities 2,330,000
Increase in cash and cash equivalents 217,000
Opening bank balance 225,000
Closing bank balance 442,000

© Emile Woolf International 114 The Institute of Chartered Accountants of Pakistan


Chapter 3: IAS 7: Statement of cash flows

Workings Non-current assets – cost

Opening (12,346+5605) 17,951,000 Sale 1,500,000

Additions – balancing figure 6,191,000 Closing (15,172+7,470) 22,642,000

24,142,000 24,142,000

Accumulated depreciation
On assets sold (1,500-867) 633,000 Opening 5,605,000
Closing balance 7,470,000 Charge for the year 2,498,000
8,103,000 8,103,000

Trade debts
Opening (4,887+385) 5,272,000
Increase in balance 1,944,000 Closing (6,732+484) 7,216,000
7,216,000 7,216,000

Capital account – Mr. Amin


Withdrawals 3,120,000 Opening 27,942,000
Closing balance 33,433,000 Profit for the year 3,161,000
Capital introduced 5,450,000
36,553,000 36,553,000

EXAMPLE 23: NADIR LIMITED


Question: Following information pertains to Nadir Limited:
Extract from statement of profit or loss for the year ended 31 December 2017

Rs. in ‘000
Profit before taxation 8,955
Taxation (2,945)
Profit after taxation 6,010
Extract from statement of financial position as on 31 December 2017

2017 2016 2017 2016


Equity and liabilities Assets
---- Rs. in ‘000 ---- ---- Rs. in ‘000 ----
Share capital 12,400 10,000 Property plant &
Share premium 1,400 - equipment – net 21,400 15,800
Retained earnings 13,450 12,440 Current assets:
Surplus on revaluation 4,000 - Stock-in-trade 5,600 5,750
Non-current liabilities: Trade receivables – net 6,840 4,446
Long-term loans 4,100 5,000 Other receivables 2,385 800

© Emile Woolf International 115 The Institute of Chartered Accountants of Pakistan


Financial accounting and reporting I

Current liabilities: Cash & bank 2,355 3,204


Trade payables 1,900 1,400
Accruals & other payables 680 660
Tax liability 650 500
38,580 30,000 38,580 30,000
Other information:
(i) Shares issued during the year were as follows:
a. 10% bonus shares in March 2017.
b. Right shares in July 2017.
(ii) During the year, a plant costing Rs. 9,500,000 and having a book value of Rs. 5,200,000
was disposed of for Rs. 4,800,000 of which Rs. 1,800,000 are still outstanding.
(iii) Depreciation for the year amounted to Rs. 7,350,000.
(iv) Financial charges for the year amounted to Rs. 1,100,000. Accrued financial charges as on
31 December 2017 amounted to Rs. 112,000 (2016: Rs. 48,000).
(v) Provision for doubtful trade receivables is maintained at 5%.
Required: Prepare statement of cash flows for the year ended 31 December 2017, in accordance
with IAS 7 ‘Statement of Cash Flows’ using indirect method.
Answer:
Nadir Limited
Statement of cash flows for the year ended 31 December 2017

Cash flows from operating activities:


Profit before tax 8,955
Adjustment for:
Depreciation 7,350
Loss on disposal 5,200–4,800 400
Financial charges 1,100
Increase in provision for doubtful receivables (6,840×5÷95) – (4,446×5÷95) 126
17,931
Working capital changes:
31 December 31 December
2017 2016
Stock-in-trade 5,600 5,750 150
Other receivables 585 800 215
2,385–1,800
Trade receivables-gross 7,200 4,680 (2,520)
(6,840÷0.95);(4,446÷0.95)
Trade payables (1,900) (1,400) 500
Accrued exp./Other payables (568) (612) (44)
(680–112);(660–48)
Increase in working capital (1,699)
Cash generated from operations 16,232

© Emile Woolf International 116 The Institute of Chartered Accountants of Pakistan


Chapter 3: IAS 7: Statement of cash flows

Payment of interest 1,100 –112+48 (1,036)


Payment of taxes 2,945–650+500 (2,795)
12,401
Cash flows from investing activities
Additions to PP&E 15,800–7,350–5,200+4,000–21,400 (14,150)
Disposals of PP&E 4,800–1,800 3,000
(11,150)
Cash flows from financing activities
Issue of right shares (12,400–10,000)–(10,000×10%)+1,400 2,800
Loan repaid 5,000–4,100 (900)
Cash dividend paid [(12,440+6,010)–(10,000×10%)]–13,450 (4,000)
(2,100)
Net cash outflows (849)
Cash and cash equivalent at beginning of the year 3,204
Cash and cash equivalent at year-end 2,355

EXAMPLE 24: LIAQUAT INDUSTRIES


Question: The statement of financial position of Liaquat Industries as at 31 December 2016 is as
follows:

Equity and 2016 2015 2016 2015


Assets
liabilities -------- Rupees -------- -------- Rupees --------
Owner’s capital 13,938,060 13,665,280 Freehold land 4,778,400 6,600,000
Long-term loan 1,000,000 1,000,000 Building – WDV 5,057,600 4,171,200
Short term loan 1,331,200 1,531,200 Vehicle – WDV 600,000 800,000
Accounts payable 417,120 694,320 Equipment – WDV 1,643,100 2,112,000
Accrued interest 105,600 63,360 Capital work in 1,478,400 1,821,600
progress
Long-term deposits 580,800 448,800
Inventory 685,608 320,628
Accounts receivable 1,273,272 595,452
Cash 694,800 84,480
16,791,980 16,954,160 16,791,980 16,954,160
The following information has been extracted from income statement:

Rupees
Depreciation expenses 932,500
Finance cost 141,872
Gain on sale of fixed assets (net) 98,960
Net profit before tax 1,525,948

© Emile Woolf International 117 The Institute of Chartered Accountants of Pakistan


Financial accounting and reporting I

Additional information:
a Details of gain on sale of fixed assets are as follows:
Rupees
Gain on sale of freehold land 168,960
Loss on disposal of equipment due to fire (70,000)
98,960
The loss on disposal of equipment represents the WDV of the equipment. The amount of
insurance claim received, amounting to Rs. 30,000 was erroneously credited to accumulated
depreciation.
b Repairs to building amounting to Rs. 50,000 were erroneously debited to building account
on 31 December 2016.
c Transfers from capital work in progress to building amounted to Rs. 1,200,000.
d The owner withdrew Rs. 150,000 per month.
Required: Prepare statement of cash flows for the year ended 31 December 2016, in accordance
with IAS – 7 using indirect method.

Answer:
Liaquat Industries Limited
Statement of cash flows for the year ended 31 December 2016

Cash flows from operating activities Rupees


Net profit before tax (1,525,948 – 50,000+ 30,000) 1,505,948

Adjustments for:
Depreciation expenses 932,500
Gain on disposal (70,000 – 30,000 – 168,960) (128,960)
Finance cost 141,872
Adjusted profit before working capital changes 2,451,360

Working capital changes:


Accounts receivable (595,452 – 1,273,272) (677,820)
Inventory (320,628 – 685,608) (364,980)
Accounts payable (417,120 – 694,320) (277,200)
Net cash from operating activities 1,131,360

Cash flows from investing activities


Proceed from sale of fixed assets (W-2) 2,020,560
Capital expenditure (1,821,600 – 1,200,000 – 1,478,400) (856,800)
Long term deposits (448,800 – 580,800) (132,000)
Net cash from investing activities 1,031,760

© Emile Woolf International 118 The Institute of Chartered Accountants of Pakistan


Chapter 3: IAS 7: Statement of cash flows

Rupees
Cash flow from financing activities
Interest paid (105,600 – 63,360 – 141,872) (99,632)
Drawing made by the owner (150,000×12) (1,800,000)
Amount injected by the owner (W-1) 546,832
Repayment of short term loan (1,331,200 – 1,531,200) (200,000)
Net cash used in financing activities (1,552,500)
Net increase in cash and cash equivalents 610,320
Cash at the beginning of year 84,480
Cash at the end of year 694,800

W-1: Movement in capital account


Opening capital 13,665,280
Less: Drawings (150,000×12) (1,800,000)
Add: Profit (1,525,948–50,000+30,000) 1,505,948
13,371,228
Less: Closing capital (13,938,060+30,000–50,000) 13,918,060
Capital injected 546,832

W-2: Disposal proceeds from sale of fixed assets


Freehold land – Opening 6,600,000
Lees: Freehold land – Closing (4,778,400)
Disposal cost 1,821,600
Add: Gain on disposal of freehold land 168,960
Sale proceeds from disposal of freehold land 1,990,560
Insurance claim received against fixed assets 30,000
2,020,560

EXAMPLE 25: QUALITY ENTERPRISES


Question: Following are the extracts from income statement of Quality Enterprises (QE) for the year
ended 31 December 2015 and its statement of financial position as at that date, together with
some additional information:
Income statement for the year ended 31 December 2015
Rs. in ‘000
Profit from operations 6,402
Other income 1,357
Interest expense (100)
Profit before tax 7,659
Income tax expense (1,376)
Profit for the year 6,283

© Emile Woolf International 119 The Institute of Chartered Accountants of Pakistan


Financial accounting and reporting I

Statement of financial position as at 31 December 2015


2015 2014 2015 2014
Equity and liabilities Assets
--- Rs. in ‘000 --- --- Rs. in ‘000 ---
Non-current assets
Owner’s capital 14,219 10,703 Property, plant and 19,628 11,845
equipment
Unappropriated profit 10,652 6,697 Investments 7,645 6,498
27,273 18,343
Revaluation surplus 2,676 1,911
10% bank loan 6,000 -
Current liabilities Current assets
Trade and other 3,337 4,953 Inventories 4,642 3,073
payables
Income tax payable 1,300 994 Trade and other receivables 2,273 3,865
Bank overdraft - 27 Cash and bank 3,996 4
4,637 5,974 10,911 6,942
38,184 25,285 38,184 25,285
Additional information:
(i) During the year, movements in property, plant and equipment include:
• Depreciation amounting to Rs. 5,280,000.
• Machinery having a carrying amount of Rs. 2,481,000 was sold for Rs. 3,440,000.
• Factory building was revalued from a carrying amount of Rs. 5,963,000 to Rs. 8,000,000.
• An office building which had previously been revalued was sold at its carrying Amount of
Rs. 2,599,000.
(ii) The owner of QE withdrew Rs. 300,000 per month. The amounts were debited to
unappropriated profit.
(iii) Trade debts written off during the year amounted to Rs. 200,000. The provision for bad debts
as at 31 December 2015 was Rs. 400,000 (2014: Rs. 550,000)
(iv) The interest on bank loan is payable on 30th June every year. The bank loan was received on
1 November 2015. Interest for two months has been accrued and included in trade and other
payables.
(v) Other income includes investment income of Rs. 398,000. As at 31 December 2015, trade
and other receivables included investment income receivable amounting to Rs. 96,000 (2014:
Rs. 80,000).
Required: Prepare a statement of cash flows for Quality Enterprises for the year ended 31 December
2015, using the indirect method.

Answer:
Quality Enterprises Statement of cash flows for the year ended 31 December 2015

Rupees
Cash flow from operating activities
Profit before tax 7,659,000
Non-cash adjustments
Investment income (398,000)
Interest expense 100,000
Depreciation charge 5,280,000
Bad debt expense [(200,000 +(400,000 – 550,000)] 50,000
Profit on disposal of property, plant and equipment (959,000)
(3,440,000–2,481,000)

© Emile Woolf International 120 The Institute of Chartered Accountants of Pakistan


Chapter 3: IAS 7: Statement of cash flows

Changes in working capital


Increase in inventories (3,073,000–4,642,000) (1,569,000)
Decrease in trade and other receivables 1,558,000
[(3,865,000–80,000–2,273,000+96,000
+(550,000– 400,000– 200,000))]
Decrease in trade and other payables (1,716,000)
[(4,953,000–(3,337,000–100,000))]
Net changes in working capital (cash generated from operations) 10,005,000
Income tax paid (W-2) (1,070,000)
Net cash from operating activities 8,935,000

Cash flow from investing activities


Purchase of property, plant and equipment (W-3) (16,106,000)
Proceeds from sale of property, plant and equipment 6,039,000
(3,440,000+2,599,000)
Investment income received (W-1) 382,000
Purchase of investments (7,645,000 – 6,498,000) (1,147,000)
Net cash used in investing activities (10,832,000)

Rupees
Cash flow from financing activities
Obtain bank loan 6,000,000
Additional capital (14,219,000 – 10,703,000) 3,516,000
Drawings (3,600,000)
Net cash from financing activities 5,916,000
Net increase in cash and cash equivalents 4,019,000
Cash and cash equivalents at beginning of period (4,000–27,000) (23,000)
Cash and cash equivalents at end of period 3,996,000

W-1: Investment income received Amount in Rs.

Balance b/d 80,000 Cash (balancing) 382,000


Income Statement 398,000 Balance c/d 96,000

478,000 478,000

W-2: Tax paid Amount in Rs.

Taxes paid (balancing) 1,070,000 Balance b/d 994,000


Balance c/d 1,300,000 Income Statement 1,376,000

2,370,000 2,370,000

© Emile Woolf International 121 The Institute of Chartered Accountants of Pakistan


Financial accounting and reporting I

W-3: Property, plant and Equipment Amount in Rs.

Balance b/d 11,845,000 Disposals (2,481,000+2,599,000) 5,080,000


Revaluation surplus
(8,000,000−5,963,000) 2,037,000 Depreciation 5,280,000
Additions (balancing) 16,106,000 Balance c/d 19,628,000
29,988,000 29,988,000

EXAMPLE 26: KLEA


Question: The statement of financial position and statement of profit or loss for Klea for the year to
31st March 2015 are provided below.
Statement of financial position as at 31st March 2015
2015 2014
Rs. in ‘000
Assets
Non-current assets
Intangible assets 300 200
Property, plant and equipment 3,450 1,600
Financial assets 400 200
4,150 2,000
Current assets
Inventory 3,200 2,000
Trade receivables 2,400 2,000
Cash and cash equivalents 32 580
5,632 4,580
Total assets 9,782 6,580

Equity and liabilities


Equity
Issued share capital 3,000 2,000
Share premium account 838 560
Retained earnings 910 354
Total equity 4,748 2,914
Revaluation surplus 1,000 -

Non-current liabilities
Interest-bearing loans and liabilities 1,600 2,000
Current liabilities
Bank overdraft 414 -
Trade payables 1,600 1,266
Taxation 420 400
2,434 1,666
Total liabilities 4,034 3,666
Total equity and liabilities 9,782 6,580

© Emile Woolf International 122 The Institute of Chartered Accountants of Pakistan


Chapter 3: IAS 7: Statement of cash flows

Statement of profit or loss for the year ended 31 st March 2015


Revenue 10,000
Other income 100
Change in inventory of finished goods and WIP 1,300
Raw materials and consumables used 4,000
Employee benefits costs 3,000
Depreciation and amortisation expense 800
Other expenses 1,724
Total expenses (9,524)
1,876
Finance costs (320)
Finance income 50
Profit before tax 1,606
Income tax expense (650)
Profit for the year 956

Additional information
(i) Non-current assets Rs. in ‘000

2015 2014

Cost Deprec’n Cost Deprec’n

Intangible assets 700 400 400 200

Property, plant and equipment 5,000 1,550 3,000 1,400

(ii) At 1 April 2014 land was revalued from Rs. 1million to Rs. 2 million.
(iii) During the year, plant and machinery costing Rs. 600,000 and depreciated by Rs. 500,000
was sold for Rs. 150,000.
(iv) The interest bearing loans relate to debentures which were issued at their nominal value. Rs.
400,000 of these debentures was redeemed at par during the year.
(v) Ordinary shares were issued for cash during the year.
(vi) Rs. 100,000 of current asset investments held as cash equivalents were sold during the year
for Rs. 94,000.
Dividends paid in the year were Rs. 200,000 relating to the 2014 proposed dividend and Rs.
200,000 interim dividends for 2015.
Required:
Prepare a statement of cash flows for Klea for the year ended 31 March 2015 in accordance with
IAS 7 using the indirect method.

© Emile Woolf International 123 The Institute of Chartered Accountants of Pakistan


Financial accounting and reporting I

Answer:

KLEA’s Statement of cash flows for the year ended 31 st March 2015
Rs. in ‘000
Cash flows from operating activities
Profit before taxation 1,606
Adjustments for:
Depreciation (W4) 800
Finance income (50)
Interest expense 320
2,676

Increase in trade receivables (400)


Increase in inventories (1,200)
Increase in trade payables 334

Cash generated from operations 1,410


Interest paid (320)
Income taxes paid (W1) (630)

Net cash from operating activities 460


Cash flows from investing activities
Purchase of intangible assets (W2) (300)
Purchase of property, plant and equipment (W3) (1,600)
Proceeds from sale of equipment 150
Purchase of long-term investments (200)
Finance income received 50

Net cash used in investing activities (1,900)

Cash flows from financing activities


Proceeds from issue of share capital (1,000 + 278) 1,278
Payments to redeem debentures (400)
Dividends paid (400)

Net cash used in financing activities 478

Net decrease in cash and cash equivalents (962)


Cash and cash equivalents at 1 April 2014 580
Cash and cash equivalents at 31 March 2015 (32 - 414) (382)
(Note: Alternative classifications of the cash flows in accordance with IAS 7 should receive full credit
i.e. interest and dividends received as investing activities or operating cash flows, interest and
dividends paid as financing or operating cash flows.)

© Emile Woolf International 124 The Institute of Chartered Accountants of Pakistan


Chapter 3: IAS 7: Statement of cash flows

Notes
(1) Analysis of cash and cash equivalents
Rs. in ‘000
2015 2014
Cash on hand and balances with bank 32 580
Bank overdraft (414) -
-------------- --------------
Cash and cash equivalents (382) 580
-------------- --------------
(2) Material non-cash transactions
During the year land was re-valued upwards by Rs.1million
Workings
(W1) Taxation paid
Rs. in ‘000
Taxation creditor brought forward 400
Taxation expense for period 650
--------------
1,050
Taxation creditor carried forward (420)
--------------
Taxation paid in the year 630
--------------
(W2) Intangible assets
Net book value brought forward 200
Capitalised in the year (from (i)) 300
--------------
500
Amortisation charged in year (from (i)) (200)
--------------
Intangibles acquired in the year 300
--------------
(W3) Property, plant and equipment
Cost brought forward 3,000
Revaluation in year (from (ii)) 1,000
Disposals (from (iii)) (600)
Additions (balancing figure) 1,600
--------------
Cost carried forward 5,000
--------------
(W4) Depreciation and amortisation
Depreciation (150 movement + 500 on disposal) 650
Amortisation 200
Profit on disposal (W5) (50)
--------------
Charge shown in statement of profit or loss 800
--------------
Hence add back of depreciation and amortisation also takes account of the profit on disposal
of the plant and machinery.

© Emile Woolf International 125 The Institute of Chartered Accountants of Pakistan


Financial accounting and reporting I

(W5) Disposal
Rs. in ‘000
Cost of disposal 600
Accumulated depreciation (500)
--------------
Net book value 100
Proceeds of sale 150
--------------
Profit on sale 50
--------------
.

Example 27: Sunday Traders Limited


Question: Following are the extracts from the financial statements of Sunday Traders Limited (STL)
for the year ended 30 June 2019:
Statement of financial position as on 30 June 2019
2019 2018 2019 2018
Assets Equity & liabilities
Rs. in million Rs. in million
Property, plant & equipment 8,555 7,240 Share capital 4,650 3,450
(Rs. 100 each)
Investment property 1,800 1,120 Share premium 1,600 1,240
Stock in trade 4,800 4,500 Retained earnings 1,652 (655)
Prepayments 184 268 Long term loans 6,024 6,523
Trade receivables 3,800 3,600 Trade payables 3,422 5,390
Cash 194 480 Contract liability 250 40
Accrued liabilities 310 180
Interest payable 135 110
Current maturity:
long-term loans 850 700
Provision for taxation 440 230
19,333 17,208 19,333 17,208

Statement of profit or loss for the year ended 30 June 2019


Rs. in million
Sales 29,700
Cost of sales (15,750)
Gross profit 13,950
Distribution cost (6,185)
Administrative cost (2,302)
Other income 404
Profit before interest and tax 5,867
Interest expense (1,210)
Profit before tax 4,657
Tax expense (1,150)
Profit after tax 3,507

© Emile Woolf International 126 The Institute of Chartered Accountants of Pakistan


Chapter 3: IAS 7: Statement of cash flows

Additional information:
(i) 72% of sales were made on credit.
(ii) Depreciation expense for the year amounted to Rs. 750 million which was charged to
distribution and administrative cost in the ratio of 3:1.
(iii) Distribution cost includes:
▪ Rs. 40 million in respect of loss on disposal of equipment. The written down value at the
time of disposal was Rs. 152 million.
▪ impairment loss on vehicles amounting to Rs. 24 million.
(iv) Loan instalments (including interest) of Rs. 1,984 million were paid during the year.
(v) Other income comprises of:
▪ increase in fair value of investment property amounting to Rs. 220 million.
▪ rent received from investment property amounting to Rs. 184 million.
(vi) During the year, STL issued right shares at premium.

Required:
Prepare STL’s statement of cash flows for the year ended 30 June 2019 using direct method.

Answer:
Sunday Traders Limited
Statement of Cash Flows
For the year ended 30 June 2019
Cash flows from operating activities Rs. in million
Cash receipts from customers (Cash sales: 8,316 ; Credit sales: 21,394) (W-1) 29,710
Cash receipts from tenants 184
Cash paid to suppliers (W-2) (18,018)
Cash paid to other vendors (W-3) (7,459)
Cash generated from operations 4,417
Interest paid 110+1,210–135 (1,185)
Income taxes paid 230+1,150–440 (940)
Net cash inflow from operating activities 2,292

Cash flows from investing activities


Purchase of property, plant and equipment (8,555–7,240)+24+152+750 (2,241)
Proceeds from disposal of property, plant and equipment 152–40 112
Purchase of investment property (1,800–1,120)–220 (460)
Net cash outflow from investing activities (2,589)

Cash flows from financing activities


Proceeds from issue of shares (4,650–3,450)+(1,600–1,240) 1,560
Dividend paid 1,652+655–3,507 (1,200)
Repayment of loans 1,984–1,185 (799)
New loans acquired (6,024–6,523)+799+(850–700) 450
Net cash inflow from financing activities 11

© Emile Woolf International 127 The Institute of Chartered Accountants of Pakistan


Financial accounting and reporting I

Net decrease in cash and cash equivalents (286)


Cash and cash equivalent at the beginning of the year 480
Cash and cash equivalent at the end of the year 194

Workings:
W-1: Cash receipts from customers – sales Rs. in million
Sales for the year 29,700
Increase in trade receivables balances 3,600–3,800 (200)
Increase in contract liability balances 250–40 210
Cash received from customers 29,710

W-2: Cash paid to suppliers Rs. in million


Cost of sales 15,750
Increase in stock balances 4,800–4,500 300
Decrease in trade payable balances 5,390–3,422 1,968
Cash paid to suppliers 18,018

W-3: Cash paid to other vendors Rs. in million


Distribution cost 6,185
Administrative cost 2,302
Depreciation (750)
Loss on disposal (40)
Impairment (24)
Increase in accrued liabilities balances 180–310 (130)
Decrease in prepayment balances 184–268 (84)
7,459
.

Example 28: Broad Peak Limited


Question: You are working as Finance Manager in Broad Peak Limited (BPL). Faraz has recently
joined BPL as an internee for three months. You have asked him to develop an understanding
of the statement of cash flows. After going through few statements, he has raised the following
queries:
(i) Depreciation is not a cash flow but was still appearing as an addition in the statement
of cash flows.
(ii) In the statement of cash flows of a competitor, interest paid was shown as a financing
activity but BPL showed it in operating activities.
(iii) BPL purchased inventories throughout the year but total purchases of inventory were
not shown in the statement. However, only decrease in inventory was added.
(iv) Cash and bank balance in the statement of financial position was not in agreement with
the opening and closing balances at the end of statement of cash flows.
Required:
Briefly answer the queries raised by Faraz.

© Emile Woolf International 128 The Institute of Chartered Accountants of Pakistan


Chapter 3: IAS 7: Statement of cash flows

Answer:
(i) A statement of cash flows begins with net profit which is arrived after deducting depreciation
expense. So to convert the net profit into net cash flow the deduction of depreciation is
reversed (i.e. added).
(ii) As per IAS 7, interest paid can be shown as either cash flow from financing activities or cash
flow from operating activities. Both classifications are correct as long as they are
consistently applied by an entity.
(iii) A statement of cash flows begins with net profit which is arrived after deducting cost of
sales. So to convert the effect of cost of goods sold into outflow for purchases of inventory,
change in inventory is adjusted i.e. increase is deducted and decrease is added.
(iv) Statement of financial position shows cash and bank balances while the statement of cash
flows ends with cash and cash equivalents which may differ from cash and bank balances
due to existence of bank overdraft and short term investments.

Example 29: Taxila Limited (TL)


Question: Statement of financial position of Taxila Limited (TL) as on 30 June 2020 is as follows:
Assets 2020 2019 Equity & liabilities 2020 2019
Rs. in million Rs. in million
Property, plant and equipment Share capital (Rs. 100
1,619 1,200 each) 1,200 800
Investment property 290 120 Share premium 290 150
Inventories 205 180 Retained earnings 260 90
Trade receivables 342 291 Revaluation surplus 215 200
Prepayments and other Long-term loans
receivables 14 20 367 445
Short-term investments Trade and other
60 48 payables 144 120
Cash and bank balances 24 6 Current portion of
long-term loans
78 60
2,554 1,865 2,554 1,865
Additional information:
(i) Equipment having fair value of Rs. 240 million was acquired by issuing 2 million shares.
(ii) As a result of revaluation carried out on 30 June 2020, property, plant and equipment was
increased by Rs. 80 million out of which Rs. 35 million was credited to profit and loss account.
(iii) During the year, fully depreciated items of property, plant and equipment costing Rs. 36
million were sold for Rs. 8 million out of which Rs. 3 million is still outstanding.
(iv) Depreciation on property, plant and equipment for the year amounted to Rs. 290
million.
(v) An investment property was acquired for Rs. 180 million. TL applies cost model for
subsequent measurement of its investment property.
(vi) Financial charges for the year amounted to Rs. 45 million. Trade and other payables include
accrued financial charges of Rs. 12 million (2019: Rs. 17 million).
(vii) Short-term investments amounting to Rs. 35 million are readily convertible to cash (2019: Rs.
20 million). Investment income for the year amounted to Rs. 6 million.
Required:
Prepare TL’s statement of cash flows for the year ended 30 June 2020 in accordance with the
requirements of IFRSs.

© Emile Woolf International 129 The Institute of Chartered Accountants of Pakistan


Financial accounting and reporting I

Answer:
Taxila Limited
Statement of cash flows for the year ended 30 June 2020
Rs. in million
Cash flows from operating activities
Profit (W-1) 140
Adjustments for:
Depreciation on property, plant and equipment 290
Depreciation on investment property 120+180–290 10
Gain on disposal of property, plant and equipment (8)
Revaluation gain (35)
Interest expense 45
Operating profit before working capital changes 442
Changes in working capital:
Increase in inventory 205–180 (25)
Decrease in prepayments and other receivables (14–3)–20 9
Increase in trade receivables 342–291 (51)
Decrease in short-term investments (60–35)–(48–20) 3
Increase in trade and other payables (144–12)–(120–17) 29
(35)
Cash generated from operations 407
Interest paid 17+45–12 (50)
Net cash flows from operating activities 357

Cash flows from investing activities


Purchase of property, plant and equipment (W-2) (389)
Purchase of investment property (180)
Proceeds from disposal of property, plant & equipment 8–3 5
Net cash flows used in investing activities (564)

Cash flows from financing activities


Proceeds from issue of shares at premium (W-3) 300
Repayment of long term loan (367+78)– (445+60) (60)
Net cash flows from financing activities 240
Net increase in cash and cash equivalents 33
Cash and cash equivalents at beginning of the year 6+20 26
Cash and cash equivalents at the end of the year 24+35 59

© Emile Woolf International 130 The Institute of Chartered Accountants of Pakistan


Chapter 3: IAS 7: Statement of cash flows

W-1: Profit for the year Rs. in million


Retained earnings – closing 260
Transfer from revaluation surplus 200+(80–35)–215 (30)
Retained earnings – opening (90)
140

W-2: Purchase of property, plant and equipment Rs. in million


Opening balance 1,200
Equipment acquired against issuance of shares 240
Revaluation surplus 80
Depreciation for the year (290)
Closing balance (1,619)
389

W-3: Issuance of shares at premium Rs. in million


Share capital and share premium – closing 1,200+290 1,490
Issuance of shares for property, plant and equipment (240)
Share capital and share premium – opening800+150 (950)
300

© Emile Woolf International 131 The Institute of Chartered Accountants of Pakistan


Financial accounting and reporting I

6 OBJECTIVE BASED QUESTIONS


01. Faria Limited is involved in the business of furniture. At 1 January 2018 the company’s issued
share capital consists of 50,000 Rs. 1 shares. During the year 2018 company has made a bonus
issue of 1 for 5 shares.
What is impact of bonus issue on cash flows of the business?
(a) Decrease in cash flows from operating activities
(b) Increase in cash flows from financing activities
(c) No impact
(d) Increase in cash generated from operations

02. A company has incurred a loss of Rs. 40,000 during the year 2018; however, the balance in the
bank account at end of the year is more than the balance at start of the year.
What does this mean?
(a) Company has allowed a longer credit period to the credit customers
(b) Company has purchased more stock
(c) Company has made a right issue during the year
(d) Company has purchased fixed assets during the year

03. A company has provided the following information:


2018 2017
Rs. Rs.
Share capital 110,000 100,000
Share premium 30,000 40,000
A bonus issue of 1 for every 10 shares held has been made during the year.
What is the amount to be reported in cash flow from financing activities for the year 2018?
(a) Rs. 10,000 Inflow
(b) 0
(c) Rs. 10,000 outflow
(d) Cannot be determined

04. A company has provided following balances


Rs.
Non–current asset – 31 December 2018 125,000
Accumulated depreciation 1 January 2018 25,000
Accumulated depreciation 31 December 2018 38,000
During the year an asset having cost Rs. 10,000 was sold for Rs. 6,000 and gain on disposal
was Rs. 3,000.
What is the charge for depreciation for the year to be adjusted in statement of cash flows?
(a) Rs. 13,000
(b) Rs. 19,000
(c) Rs. 20,000
(d) Rs. 38,000

© Emile Woolf International 132 The Institute of Chartered Accountants of Pakistan


Chapter 3: IAS 7: Statement of cash flows

05. A company has provided following information as at 31 March 2019:


2019 2018
Rs. Rs.
Retained earnings 50,000 38,000
Following adjustments were made during the year 2019:
Dividends paid Rs. 5,000
Transfer to general reserves Rs. 12,000
Tax charge Rs. 4,000
What is the amount of profit before tax for the year 2019 for the purposes of preparing statement
of cash flows?
(a) Rs. 29,000
(b) Rs. 33,000
(c) Rs. 24,000
(d) Rs. 25,000

06. A company has provided the following data:


Rs.
Receivables at 1 April 2018 12,000
Receivables at 31 March 2019 25,000
Credit sales during the year 75,000
Discount allowed during the year 3,000
What is the amount to be shown as cash received from customers in statement of cash flows
using direct method?
(a) Rs. 62,000
(b) Rs. 75,000
(c) Rs. 59,000
(d) Rs. 65,000

07. Which TWO of the following are considered as inflows in a company’s statement of cash flows?
(a) Bonus shares issued
(b) Decrease in accounts receivables
(c) Increase in inventory
(d) Increase in accounts payables

08. Which of the following item will appear in cash flows from financing activities section of
statement of cash flows?
(a) Cash paid to acquire non-current assets
(b) Dividends paid
(c) Bonus shares issued
(d) Depreciation for the year

© Emile Woolf International 133 The Institute of Chartered Accountants of Pakistan


Financial accounting and reporting I

09. Following data is available for a company for the year ended 31 December 2018:
Rs.
Operating profit before working capital changes 30,000
Increase in accounts receivables 5,000
Increase in inventory 3,000
Increase in accounts payable 2,000
Interest paid 500
What is the net cash generated from cash flows from operating activities for the year ended 31
December 2018?
(a) Rs. 23,500
(b) Rs. 24,500
(c) Rs. 29,500
(d) Rs. 19,500

10. Which of the following is an advantage of statement of cash flows?


(a) It determines the profitability of a business
(b) It helps users to estimate the future expected cash flows of the business
(c) It determines the ratio of business debts and equity
(d) It helps in determining the net assets of a business

11. A company has made following investments during the year:


Rs.
6 months Advance rent paid to landlord 30,000
Short term investments bond (highly liquid) 25,000
Debentures purchased- redeemable after 7 years 50,000
Non–current assets purchased 45,000
What is the amount to be shown in investing activities for the year?
(a) Rs. 45,000
(b) Rs. 150,000
(c) Rs. 95,000
(d) Rs. 100,000

12. A company has provided following data at the end of year 2017:
2017
Rs.
Share capital Rs. 1 each 100,000
Share premium 3,000
The company has made a right issue of 1 for 5 shares during the year 2018 at Rs. 1.2 per share.
What is the amount to be shown in the cash flows from financing activities?
(a) Rs. 24,000 outflow
(b) Rs. 24,000 inflow
(c) Rs. 20,000 inflow
(d) Rs. 4000 inflow

© Emile Woolf International 134 The Institute of Chartered Accountants of Pakistan


Chapter 3: IAS 7: Statement of cash flows

13. How should gain on sale of used equipment be reported in a cash flow statement, using indirect
approach?
(a) In operating activities as deduction from Profit before tax
(b) In investing activities as a reduction in cash inflow
(c) In investing activities as an increase in cash inflows
(d) In operating activities as addition to profit before tax

14. Which TWO of the following are added as non-cash adjustments to the profit before tax in the
cash flow from operating activities section of statement of cash flows?
(a) Interest expense
(b) Interest income
(c) Loss on sale of non–current assets
(d) Tax charge for the year

15. Where, in a company are financial statements complying with international accounting
standards, should you find the proceeds of non-current assets sold during the period?
(a) Statement of cash flows and statement of financial position
(b) Statement of changes in equity and statement of financial position
(c) Statement of profit or loss and statement of cash flows
(d) Statement of cash flows only

16. Zahid & Co. reported a profit Rs. 40,000 for the year, after charging the following:
Rs.

Depreciation 4,000

Loss on sale of assets 3,000


During the year there was a decrease in accounts receivables of Rs. 1,000.
What was the net cash flow generated from operations based on above data?
Rs. ___________

17. Asmat Limited made a profit for the year of Rs. 320,500, after accounting for depreciation Rs.
32,500. During the year following transactions took place:
Rs.

Purchase of machinery 125,000

Increase in accounts receivables 45,000

Increase in inventory 28,000

Increase in accounts payable 12,600


What is the net increase in cash and bank balance during the year?
Rs. ___________

© Emile Woolf International 135 The Institute of Chartered Accountants of Pakistan


Financial accounting and reporting I

18. A company has provided following information:


Rs.
4% Loan notes 1,000,000
Interest payable 1 January 2018 10,000
Interest payable 31 December 2018 20,000
What is the amount to be reported as interest paid during the year 2018 in the Statement of
Cash Flows?
Rs. ___________

19. Furqan Limited has provided following information about non–current assets:
Rs.
Cost as at 1 January 2018 350,000
Cost as at 31 December 2018 450,000
During the year an asset costing Rs. 100,000 and having net book value of Rs. 40,000 was sold
at a profit of Rs. 30,000.
What is the net to be shown as outflow in the “Cash flow from investing activities” section in
Statement of Cash Flows?
Rs. ___________

20. The following amounts have been calculated for inclusion in the statement of cash flow of House
Limited:
Rs.
Net cash inflow from financing activities 145,000
Net cash outflow from investing activities 160,000
Increase in cash and cash equivalents 24,000
Income taxes paid 65,000
Interest paid 12,000
How much cash has been generated from operations?
Rs. ___________

21. A cash flow statement provides information that enables users to evaluate the changes in:
(a) Solvency
(b) Net assets
(c) Its financial structure
(d) Its liquidity

22. Daily sales and purchases and employee costs comprise:


(a) Operating activities
(b) Investing activities
(c) Financing activity
(d) Component of cash and cash equivalent

© Emile Woolf International 136 The Institute of Chartered Accountants of Pakistan


Chapter 3: IAS 7: Statement of cash flows

23. Which of the following involves a movement of cash?


(a) A rights issue
(b) Depreciation of fixed assets
(c) Creation of a provision for doubtful debts
(d) A bonus issue

24. Activities that result in changes in the size and composition of the equity capital and borrowings
of an entity are called:
(a) Operating activities
(b) Investing activities
(c) Financing activity
(d) None of these

25. Which of the following are not the operating activities?


(a) Interest paid
(b) Cash payments of income taxes
(c) Collections from customers
(d) Payment of dividends

26. Amplifier Limited had sales of Rs.120 million during the year. Trade and other receivables
increased from Rs.12 million to Rs.16 million, an increase of Rs. 4 million. What amount of cash
was received from customers during the year?
(a) Rs.124 million
(b) Rs.116 million
(c) Rs.120 million
(d) None of these

27. Cost of sales for Shah Textile Limited during the year was Rs.100 million. Opening inventory
was Rs.20 million and closing inventory was Rs. 28 million. Opening trade payables were Rs.5
million and closing trade payables were Rs.9 million. What amount of cash was paid to
suppliers?
(a) Rs.102 million
(b) Rs.104 million
(c) Rs.108 million
(d) Rs.110 million

28. Zaman Limited extracted general ledger from which it shows salaries and wages expense of
Rs.50 million during the year. Its cash flow statement reported cash paid to employees of Rs.42
million. The opening balance of accrued salaries and wages was Rs.3.6 million. What was the
closing balance for accrued salaries and wages?
(a) Rs.11.6 million
(b) Rs.11.8 million
(c) Rs.4.4 million
(d) Rs.3.8 million

© Emile Woolf International 137 The Institute of Chartered Accountants of Pakistan


Financial accounting and reporting I

29. Sale proceeds from disposal of property, plant and equipment are classified as:
(a) Financing activities
(b) Operating activities
(c) Investing activities
(d) Either financing or operating activities, depending on which method (direct or indirect)
is used to determine cash flows from operating activities

30. Which one of the following events will increase the cash balances of a business?
(a) Loan repayment to banks
(b) Bank granting it an overdraft facility
(c) Debtors paying amounts owed
(d) Sale of stock on credit

31. A company with healthy profits is facing a cash shortage. Which of the following events could
account for this?
(a) Delaying payments to creditors
(b) The shortening of the credit period granted to debtors
(c) The recent acquisition of machinery
(d) An increase in dividend proposed by the directors

32. Which one of the following companies is most likely to run into cash flow problems?
(a) A loss making company making components of vital strategic importance to the
government
(b) A profitable new retailer about to embark on ambitious expansion plans
(c) A company which has recently sold part of its operations so as to concentrate on its
core areas
(d) Reasonably profitable, long established company with no expansion plans

33. What is the immediate effect of making a capital repayment on a loan on cash flow and profits?
(a) On profit - None; On cash – Decrease
(b) On profit - Increase; On cash – Decrease
(c) On profit - Decrease; On cash – Decrease
(d) On profit - Decrease; On cash – None

34. A company has a negative cash flow from operating activities. What could explain this negative
cash flow?
(a) High levels of dividend payments
(b) A substantial investment in new fixed assets
(c) A sudden increase in credit sales
(d) The repayment of a loan

© Emile Woolf International 138 The Institute of Chartered Accountants of Pakistan


Chapter 3: IAS 7: Statement of cash flows

35. Which of the following is NOT a cash outflow for the firm?
(a) Dividends
(b) Interest payments.
(c) Taxes
(d) Bad debts

36. Which of the following would cause negative net cash flow from operating activities?
(a) Decrease in depreciation expense
(b) A substantial investment in fixed assets
(c) A significant increase in credit sales
(d) Repayment of a long-term loan

© Emile Woolf International 139 The Institute of Chartered Accountants of Pakistan


Financial accounting and reporting I

6 OBJECTIVE BASED ANSWERS


01. (c) Bonus issue of shares involves transfer from Reserves to share capital of the
company. There is no cash flow involved.
02. (c) The statement indicates that the company had net cash receipts (inflows)
despite the losses, which is indicative of receipts of cash by issuing right
shares.
03. (b) No cash is paid or received for bonus issue of share capital.
Entry to record bonus issue
Dr CR
Rs. Rs.
Share premium 10,000
Share capital 10,000

Share Capital + Share premium


b/d 100,000+40,000 140,000
c/d
110,000 + 30,000 140,000
140,0000 140,000

04. (c)
Accumulated depreciation
Particulars Rs... Particulars Rs.
Disposal (see below) 7,000 b/f 25,000
c/f 38,000 Depreciation 20,000
45,000 45,000

Disposal
Particulars Rs... Particulars Rs.
Asset 10,000 Provision for dep. 7,000
(bal)
Gain on disposal 3,000 Cash 6,000
13,000 13,000

05. (b)
Retained earnings
Particulars Rs... Particulars Rs.
Dividends paid 5,000 b/f 38,000
Transfer to reserves 12,000 Profit for the year 29,000
c/f 50,000
67,000 67,000
Profit after tax 29,000 + Tax 4,000 = Rs. 33,000 profit before tax

© Emile Woolf International 140 The Institute of Chartered Accountants of Pakistan


Chapter 3: IAS 7: Statement of cash flows

06. (c)
Accounts receivables
Particulars Rs... Particulars Rs.
b/f 12,000 Cash (bal.) 59,000
Sales 75,000 Discount allowed 3,000
c/f 25,000
87,000 87,000

07. (b) & (d) Decrease in accounts receivables indicates that they have paid the debt,
hence, inflow for us.
Increase in accounts payable indicates that we have not paid them, thus
reducing outflows (or increasing cash flows)
Bonus shares issued do not affect cash flows.
Increase in inventory is cash outflows.
08. (b) Dividend is paid to shareholders who provide finance to the business;
therefore, it is treated as financing activity.
Cash paid to acquire non-current assets is shown in investing activities.
Bonus issues have no impact on cash flows of the business.
Depreciation is non-cash item and is adjusted in operating activities.
09. (a)
Rs.
Operating profit before working capital changes 30,000
Increase in accounts receivables (5,000)
Increase in inventory (3,000)
Increase in accounts payable 2,000
Interest paid (500)
23,500

10. (b) Users of financial statements may predict future cash flows from past data of
how the entity generates and uses its cash.
Profitability is reflected in statement of comprehensive income.
Debt/Equity and net assets are reflected in statement of financial position.
11. (c) Only debentures and non – current assets purchased are included in investing
activities; Rs. 50,000+45,000= Rs. 95,000
Investment in short term bonds will be considered cash equivalent and
advance rent would affect operating activities cash flows.
12. (b) Shares issued = 100,000/5 = 20,000
Cash received = 20,000xRs.1.2= Rs. 24,000
13. (a) The gain on disposal in included in profit before tax as other income. This is
deducted back in order to determine the cash figure.
14. (a & c) Interest expense is added back as interest paid is separately reported.
Loss on disposal is added back as this is included in profit before tax as an
expense.
Interest income is deducted back.
Tax charge need not be added back as already the amount taken is profit
before tax.

© Emile Woolf International 141 The Institute of Chartered Accountants of Pakistan


Financial accounting and reporting I

15. (d)
16. Rs. 48,000
Rs.
Profit before tax 40,000
Adjustments for non-cash items
Depreciation 4,000
Loss on sale of fixed assets 3,000
Operating profit before working capital changes 47,000
Decrease in accounts receivables 1,000
Cash generated from operations 48,000

17. Rs. 167,600


Cash flows from operating activities Rs.
Profit before tax 320,500
Depreciation 32,500
Operating profit before working capital changes 353,000
Increase in accounts receivables (45,000)
Increase in inventory (28,000)
Increase in accounts payable 12,600
292,600

Cash flow from investing activities


Purchase of machinery (125,000)
167,600

18. Rs. 30,000


Interest payable
Particulars Rs. Particulars Rs.
Cash 30,000 b/f 10,000
c/f 20,000 Interest expense 40,000
50,000 50,000
Interest expense = Rs. 1,000,000x4%= Rs. 40,000
19. Rs. 130,000
Amounts to be shown in Cash flows from investing activities are;
Cash flows from investing activities Rs.
Cash paid to acquire assets (200,000)
Cash received on disposal 70,000
130,000

© Emile Woolf International 142 The Institute of Chartered Accountants of Pakistan


Chapter 3: IAS 7: Statement of cash flows

Non-current assets
Particulars Rs. Particulars Rs.
b/f 350,000 Disposal 100,000
Cash 200,000 c/d 450,000
550,000 550,000

Disposal
Particulars Rs. Particulars Rs.
Asset 100,000 Acc. Dep [10,000 – 60,000
4,000]
Gain on disposal 30,000 Cash 70,000
130,000 130,000

20. Rs. 116,000


Rs.
Cash generated from operations (β) 116,000
Interest paid (12,000)
Income taxes paid (65,000)
Net cash from operating activities (β) 39,000
Net cash outflow from investing activities (160,000)
Net cash inflow from financing activities 145,000
Increase in cash and cash equivalents 24,000

21. (d)
22. (a)
23. (a)
24. (c)
25. (d)
26. (b)
27. (b)
28. (a)
29. (c)
30. (c)
31. (c)
32. (b)
33. (a)
34. (c)
35. (d)
36. (c) A significant increase in credit sales

© Emile Woolf International 143 The Institute of Chartered Accountants of Pakistan


Financial accounting and reporting I

© Emile Woolf International 144 The Institute of Chartered Accountants of Pakistan


Certificate in Accounting and Finance

CHAPTER
Financial accounting and reporting I

Income and expenditure account

Contents
1 Not for profit organisations
2 Income and expenditure account
3 Statement of financial position
4 Objective based questions and answers

* The student must refer original handbook of IFRS.

© Emile Woolf International 145 The Institute of Chartered Accountants of Pakistan


Financial accounting and reporting I

1 NOT FOR PROFIT ORGANISATIONS


Section overview

◼ Introduction
◼ Receipts and payments account

1.1 Introduction
Many organisations do not exist in order to make a profit. Such organisations include:
❑ Clubs and societies; and
❑ Charities
❑ Trusts
❑ NGOs
❑ Hospitals
Non-profit making organisations (also called not for profit organisations) have revenue which they
raise and costs which must be paid just like other organisations.
Non-profit making organisations prepare an income and expenditure account (I & E account)
instead of a statement of comprehensive income. This is similar to a statement of comprehensive
income in that it is prepared on the accruals basis but there are differences.
Different terminology is used.
❑ What a statement of comprehensive income would describe as profit for the period, an
income and expenditure account describes as a surplus of income over expenditure.
❑ What a statement of comprehensive income would describe as loss for the period, an income
and expenditure account describes this as a deficit of income over expenditure.
❑ In the statement of financial position, a company has equity reserves whereas a not for profit
organisation has equity fund accounts.
❑ In the statement of financial position, a company would add the profit for the year (deduct a
loss) to an equity account called retained profits. A not for profit organisation would add the
surplus (deduct a deficit) to an equity account called an accumulated fund (or accumulated
surplus of income over expenditure).
Also the sort of organisation that prepares income and expenditure accounts might be subject to
much less regulation than entities that exist for a profit.

1.2 Receipts and payments account


A clubs or society may not be required to prepare accruals based financial information. They might
choose to do so but they may prepare a receipt and payments account instead.
This is simply a summary of cash receipts and payments during the accounting period. The
accruals concept is not applied.
All cash receipts are recorded on debit side (receipts side) and all cash payments are recorded on
credit side (payments side) of receipts and payments account.

© Emile Woolf International 146 The Institute of Chartered Accountants of Pakistan


Chapter 4: Income and expenditure account

Illustration 01:

Receipts and payments account


Balance b/d X Donation X
Subscriptions X Repairs X
Functions X Telephone X
Sale of land X Extension of club house X
Bank interest X Furniture X
Bequest X Heat and light X
Sundry income X Salary and wages X
Sundry expenses X
Balance c/d X
X X
Balance b/d X

A receipt and payment account gives far less information than a set of financial statements based
on the accruals concept.
For all practical purposes this is a cash account.

© Emile Woolf International 147 The Institute of Chartered Accountants of Pakistan


Financial accounting and reporting I

2 INCOME AND EXPENDITURE ACCOUNT


Section overview

◼ Introduction
◼ Categories of income
◼ Subscriptions account
◼ Life membership fee
◼ Donations
◼ Surplus from running an operation
◼ Surplus from running an event
◼ Type of expenditures
◼ Format

2.1 Introduction
An income and expenditure account is an accruals based statement listing the different types of
income of a club followed by the different categories of expenditure of the club.

2.2 Categories of income


A club may have several categories of income including:
❑ Membership fees and subscriptions;
❑ Life membership fees;
❑ Donations to the club;
❑ Investment income;
❑ Surplus from running a coffee bar or a shop;
❑ Surplus from running an event;
Note that if a club has a coffee bar or shop or runs an event the “profit” from these is generally
calculated separately (in an account known as a trading account) and presented as a line in the
income and expenditure account.

2.3 Subscriptions account


At each year end there will usually be some members who have paid their subscriptions in advance
and some who are in arrears. These are both included as balances brought down and carried down
on a single subscription account. Cash received is credited to this account and the balance on the
account is transferred to the income and expenditure account (as income for the year).
Illustration 02: Subscription account
Subscription account
Rs. Rs.
Balance b/d Balance b/d
(members in arrears) X (members who have prepaid) X
Income and expenditure X Cash X
Balance c/d Balance c/d
(members who have prepaid) X (members in arrears) X
X
Balance b/d Balance b/d (members who
(members in arrears) X have prepaid) X

© Emile Woolf International 148 The Institute of Chartered Accountants of Pakistan


Chapter 4: Income and expenditure account

Example 01: Subscription account


Question: At 31 March 2016 a cricket club had membership subscriptions in arrears amounting to
Rs. 48,000 and had received Rs. 12,000 subscriptions in advance.
During the year to 31 March 2017 the club received Rs. 624,000 including 26 memberships for
the year to 31 March 2018 at Rs. 1,200 per annum.
At 31 March 2017 16 members owed subscriptions of Rs. 1,200 each.
Required: How the above transactions would be recorded in the subscription’s ledger account for
the year to 31 March 2017?
Answer:

Subscriptions

Rs. Rs.

Balance b/d: Balance b/d:


Members in arrears 48,000 Advance payments 12,000

Cash 624,000

Membership fees for the


year (to I&E) 576,000

Balance c/d: Balance c/d:


Advance payments Members in arrears
(26 × 1,200) 31,200 (16 × 1,200) 19,200

655,200 655,200

Balance b/d: 19,200 Balance b/d: 31,200

Write off of subscriptions


Questions often include the write off of subscriptions from members who have stopped attending
the club.

Illustration 03: Write off of subscriptions


Debit Credit
Income and expenditure account X
Subscription account X

Example 02: Subscription account


Question: At 31 March 2016 a cricket club had membership subscriptions in arrears amounting to
Rs. 48,000 and had received Rs. 12,000 subscriptions in advance.
During the year to 31 March 2017 the club received Rs. 624,000 including 26 memberships for
the year to 31 March 2018 at Rs. 1,200 per annum.
At 31 March 2017 16 members owed subscriptions of Rs. 1,200 each.
Half of the members who were in arrears at the end of the previous period still had not paid by 31
March 2017. It was decided to write these amounts off.
Required: How the above transactions would be recorded in the subscription’s ledger account for
the year to 31 March 2017?

© Emile Woolf International 149 The Institute of Chartered Accountants of Pakistan


Financial accounting and reporting I

Answer:
Subscriptions
Rs. Rs.
Balance b/d: Balance b/d:
Members in arrears 48,000 Advance payments 12,000
Cash 624,000
Membership fees for the
year (to I&E) 600,000 Bad debts (1/2  48,000) 24,000
Balance c/d: Advance Balance c/d: Members in
payments (26 × 1,200) 31,200 arrears (16 × 1,200) 19,200
679,200 679,200
Balance b/d: 19,200 Balance b/d: 31,200

2.4 Life membership fee


A club should have an accounting policy for these. Possible policies include:
❑ Recognition as income when received.
❑ Recognition as income over a specified period.
❑ Recognition in an equity reserve (an accumulated fund).
Recognition as income when received

Illustration 04:

Debit Credit

Bank (cash received) X

Income and expenditure account X

Recognition as income over a specified period

Illustration 05:

On receipt: Debit Credit

Bank (cash received) X

Deferred income ( accredit account on the face of the


statement of financial position) X

Each year over the a specified future period:

Deferred income X

Income and expenditure account X

This treatment recognises the amount received as income over several years.

© Emile Woolf International 150 The Institute of Chartered Accountants of Pakistan


Chapter 4: Income and expenditure account

Recognition in an equity reserve (an accumulated fund)

Illustration 06:
Debit Credit
Bank (cash received) X
Life membership fund (an accumulated fund account in
equity) X

This might then be transferred to the accumulated surplus of income over expenditure over a pre-
defined period or on the death of the member.

2.5 Donations
A club might receive a donation or bequest. If the donation has not been made for a specific
purpose the club might recognise the donation as income in the period in which it is received.
A club might receive a donation for a particular purpose. For example, a member might donate
money for a new cricket square. In this case the money is credited to a fund account set up for the
purpose.

Illustration 07:
Debit Credit
Bank (cash received) X
Cricket square fund (an accumulated fund account in
equity) X

2.6 Surplus from running an operation


If a club has a coffee bar or shop the “profit” from these is generally calculated separately (in an
account known as a trading account).
Any expenses directly related to the operation of a coffee bar or shop would be deducted from the
gross profit of the operation and the net profit would be presented on a separate line in the income
and expenditure account.

Illustration 08: Coffee bar trading account

Rs. Rs.
Income
Sales X
Opening inventory X
Purchases X
X
Closing inventory (X)
Cost of sales (X)
Gross profit X
Coffee shop worker’s salary (X)
Net profit (this figure to the face of the income and
expenditure account) X

© Emile Woolf International 151 The Institute of Chartered Accountants of Pakistan


Financial accounting and reporting I

2.7 Surplus from running an event


If a club runs an event any surplus (or loss) generally calculated separately and presented as a
separate line in the income and expenditure account.

Illustration 09: Event surplus

Rs.
Sports day entry fees X
Cost of prizes (X)
Surplus/deficit (this figure to the face of the income
and expenditure account) X

2.8 Type of expenditures


The expenditures related to non -profit organization are quite similar to profit organizations like:
❑ Salaries
❑ Rent
❑ Electricity
❑ Repair and maintenance
❑ Depreciation of fixed assets etc.

2.9 Formats
There are no mandatory formats for income & expenditure account. It can either be presented in
a statement form or an account form, both illustrated below:

Illustration 10: Income and expenditure account (Statement Form) for the year ended XX/XX/XX

Rs. Rs.
Income
Subscription income X
Donations X
Interest on bank deposit X
Coffee bar/shop profit X
Tournament income X
Less: Prizes (X)
X
X
Expenditure
Club expenses X
Rent X
Electricity X
Depreciation X
Repairs X
X
Surplus (deficit) of income over expenditure X

© Emile Woolf International 152 The Institute of Chartered Accountants of Pakistan


Chapter 4: Income and expenditure account

Illustration 11: Income & expenditure account (Account Form) for the year ended June 30, XXXX

Amount Amount
Expenditure Income
(Rs.) (Rs.)
Salaries X Subscriptions X
Rent X Interest X
Travelling expenses X Gain on sale of furniture X
Printing and stationary X
General charges X
Periodicals X
Depreciation on furniture X
Excess of income over expenditure X
X X

© Emile Woolf International 153 The Institute of Chartered Accountants of Pakistan


Financial accounting and reporting I

3 STATEMENT OF FINANCIAL POSITION


Section overview

◼ Format
◼ Special funds

3.1 Format
A not for profit organisation may or may not prepare a statement of financial position but if it does
so the statement of financial position would be similar to that of a business. The main difference is
in the equity section. The equivalent of the capital section of a business is called the accumulated
fund. The below illustrations shows two formats, statement form and account form:

Illustration 12: Statement of financial position (Statement Form) of a club

Rs.
Assets
Non-current assets
Club house X

Current assets
Subscriptions in arrears X
Investments X
Shop inventory X
Prepayments X
Cash X
Total assets X

Equity and liabilities


Accumulated fund
At start of year X
Surplus / (deficit) for the year X
At end of year X

Current liabilities
Subscriptions in advance X
Accruals X
Total accumulated fund and liabilities X

© Emile Woolf International 154 The Institute of Chartered Accountants of Pakistan


Chapter 4: Income and expenditure account

Illustration 13: Statement of financial position (Account Form) of a club


As at 31 December XXXX

General Fund & Liabilities Rs. in ‘000 Assets Rs. in ‘000

General fund X Non-Current Assets

Excess of income over expenditure X Land/Capital advance X

X Furniture & fixtures X

Long term advance X Van X

Long term deposits X

Long term prepayment X

Current Liabilities

Creditors X Current Assets

Accrued expenses X Stock X

Advance subscription X Accounts receivable X

X Advance & prepayments X

Bank X

Total General Fund & Liabilities X Total Assets X

3.2 Special funds


An organisation might also have other funds in addition to the accumulated fund.
These “special” funds arise in a number of circumstances including:
❑ when an organisation receives cash for a designated purpose; or
❑ when an organisation sets aside resources for a designated purpose.
The organisation might also set aside assets (say cash) to match to the fund so that they can be
used for the specified purpose.

Illustration 14: Receipt of cash for a specified purpose

Debit Credit
Cash X
Special fund X

Allocation of assets to the fund


Special fund cash X
Cash X

The following journals reflect cash being spent on the specified purpose.

© Emile Woolf International 155 The Institute of Chartered Accountants of Pakistan


Financial accounting and reporting I

Illustration 15: Receipt of cash for a specified purpose


Debit Credit
Special fund X
Cash (or “Special fund cash” if so allocated) X

Illustration 16: Special fund


Ali has been very successful in business.
When he was a young man he very much enjoyed playing cricket and has very fond memories of
his days at the village cricket club.
He has donated Rs. 1,000,000 to the club to fund the building of a new club house.
This would be accounted for as follows:
Debit Credit
Cash 1,000,000
Special fund (clubhouse) 1,000,000
Allocation of assets to the fund
Special fund cash 1,000,000
Cash 1,000,000

An organisation itself might set aside funds for a particular purpose.

Illustration 17: Set up fund for a specified purpose


Debit Credit
Accumulated fund X
Special fund X
Allocation of assets to the fund
Special fund cash X
Cash X

Illustration 18: Special fund


A social club in a small town has managed to accumulate a significant balance on its accumulated
fund over the years.
Its members have decided that the club should establish a fund to contribute to the school fees of
children of high promise from the town. Parents of such children would apply to the club for a grant
of Rs. 50,000.
Rs. 1,500,000 is to be set aside for this purpose.
This would be accounted for as follows:
Setting up the fund Debit Credit
Accumulated fund 1,500,000
Special fund (Education fund) 1,500,000
Allocation of assets to the fund
Special fund cash 1,000,000
Cash 1,000,000

© Emile Woolf International 156 The Institute of Chartered Accountants of Pakistan


Chapter 4: Income and expenditure account

On the award of a grant. Debit Credit


Special fund (Education fund) 50,000
Special fund cash 50,000

Example 03: Nawabshar Youth Movement


Question: The following were the assets and liabilities of the Nawabshar Youth Movement at 30
April 2017.

Rs. 000

Fixtures and fittings (net) 16,340

Inventory of refreshment (coffee bar) 4,460

Land 51,600

Subscription received in advance 4,900

Payables for drinks supplied (coffee bar) 6,780

Cash at bank 7,466

The accountant’s receipts and payments account for the year to 30 April 2018 shows the following:

Receipts Rs. 000

Donations received 500

Rent of hall 5,600

Members’ subscription 24,000

Sale of brochure 1,740

Sale of dance tickets 3,400

Sale of refreshments (coffee bar) 10,200

Payments

Repairs and maintenance 3,218

Salaries and wages 6,309

Gifts and donations 600

Dance expenses 950

Refreshment supplies (coffee bar) 19,415

Sundry expenses 10,000

Further information:

(i) Wages of Rs. 556,000 were due but unpaid at the year-end.

(ii) Inventories of drinks at 30 April 2018 were Rs. 14,210,000

(iii) Provide for depreciation on fixtures and fittings at Rs. 1,900,000

(iv) Subscription due but not paid at 30 April 2018 was Rs. 1,900,000
Required: Based on the above information, prepare the club’s income and expenditure account
for the year ended 30 April 2018 and the statement of financial position as at that date.

© Emile Woolf International 157 The Institute of Chartered Accountants of Pakistan


Financial accounting and reporting I

Answer:
Income and expenditure account Rs.000
Income:
Subscriptions (W1) 30,800
Donations 500
Rent of hall 5,600
Sales of brochure 1,740
Sales of dance tickets 3,400
Net income from coffee bar (W4) 7,315
49,355
Less expenses
Repairs and Maintenance 3,218
Salaries and Wages (W2) 6,865
Gifts and Donations 600
Dance expenses 950
Sundry expenses 10,000
Depreciation of fixtures and fittings 1,900
(23,533)
Net surplus 25,822

Statement of financial position as at 30 April 2018 Rs.000


Non-current assets
Land 51,600
Fixtures and Fittings 16,340
Depreciation (1,900)
14,440
66,040
Current Assets:
Inventory of drinks 14,210
Subscriptions unpaid 1,900
Cash and Bank Balance 12,414
94,564
Financed By:
Accumulated Fund (W5) 68,186
Surplus of income over expenditure 25,822
94,008
Current Liabilities : Wages accrued 556
94,564

© Emile Woolf International 158 The Institute of Chartered Accountants of Pakistan


Chapter 4: Income and expenditure account

W1 Subscriptions account
Rs. 000 Rs. 000
Balance b/d 4,900
Subscriptions for the period 30,800 Bank 24,000
Balance c/d 1,900
30,800 30,800

W2 Salaries and wages


Rs. 000 Rs. 000
Bank 6,309 Expenditure 6,865
Balance c/d 556
6,865 6,865

W3 Payables
Rs. 000 Rs. 000
Bank 19,415 Balance b/d 6,780
Expenditure 12,635
19,415 19,415

W4 Coffee bar
Sales 10,200
Opening inventory 4,460
Purchases (W3) 12,635
Closing inventory (14,210)
(2,885)
Profit (gross) 7,315

W5 Accumulated fund at start of the year


Assets:
Fixtures and Fittings 16,340
Inventory of refreshments 4,460
Land 51,600
Cash and Bank Balances 7,466
79,866
Liabilities:
Subscription received in Advance 4,900
Payables for drinks supplied 6,780
(11,680)
Accumulated fund 68,186

© Emile Woolf International 159 The Institute of Chartered Accountants of Pakistan


Financial accounting and reporting I

Example 04: Peshawar Business Club


Question: The statement of financial position of Peshawar Business Club as at 31 December 2017
is shown as follows:

Accumulated Carrying
Cost
depreciation amount
Rs.000 Rs.000 Rs.000
Furniture and Fittings 40,000 10,000 30,000
Games Equipment 20,000 7,200 12,800
Motor van 30,000 10,000 20,000
90,000 27,200 62,800

Current Assets: Cash at bank and at hand 9,200


72,000

Financed by: Accumulated funds 72,000

The following transactions took place during the year 1 January 2018 to 31 December 2018:
Receipts Rs. 000
Subscriptions (10,000 members @ 1,600 each) 16,000
Donations 1,600
Sale of tickets for annual dinner 10,800
Payments
Electricity 4,000
Expenses for annual dinner 6,200
New games equipment 3,200
Cleaners’ wages 2,080
Repairs and renewal 1,660
Motor van repairs 2,520
Further information:
(i) An electricity bill of Rs. 900,000 was owed at 31 December 2018.
(ii) Depreciation should be calculated at 10% of cost of the assets.
Required: Based on the above information, prepare the receipt and payment account and income
and expenditure account of Peshawar Business Club for the year ended 31 December 2018 and
statement of financial position as at that date.
Answer:

Receipts and payments


Rs. 000 Rs. 000
Balance b/d 9,200 Electricity 4,000
Subscription 16,000 Expenses for annual dinner 6,200
Donations 1,600 New games equipment 3,200
Sale of Tickets for annual dinner 10,800 Cleaner’s wages 2,080
Repairs and renewals 1,660
Motor van repairs 2,520
Balance c/d 17,940
37,600 37,600
.

© Emile Woolf International 160 The Institute of Chartered Accountants of Pakistan


Chapter 4: Income and expenditure account

Income and expenditure account


Income: Rs.000
Subscriptions 16,000
Donations 1,600
Sales of dance tickets 10,800
28,400
Less expenses
Electricity (4,000,000 + 900,000) 4,900
Annual expenses 6,200
Cleaner’s wages 2,080
Repairs and renewals 1,660
Motor van repairs 2,520
Depreciation (W) 9,320
(26,680)
Net surplus 1,720

Working: Depreciation (W)


Rs.000
Furniture and Fittings 0.1 @ 40,000,000 4,000
Game Equipment (20,000,000 + 3,200,000) x 0.1) 2,320
Motor Van 30,000,000 @ 0.1 3,000
9,320

Statement of financial position as at 30 April 2018


Non-current assets
Cost Accumulated depreciation Carrying amount
Rs.000 Rs.000 Rs.000
Furniture and Fittings 40,000 14,000 26,000
Games and Equipment 23,200 9,520 13,680
Motor Van 30,000 13,000 17,000
93,200 36,520 56,680

Current Assets: Rs. 000


Cash and Bank Balance 17,940
74,620
Financed By:
Accumulated Fund 72,000
Surplus of income over expenditure 1,720
73,720
Current Liabilities : Electricity accrual 900
74,620

© Emile Woolf International 161 The Institute of Chartered Accountants of Pakistan


Financial accounting and reporting I

Example 05: GILTAN GOLF CLUB


Question: The treasurer of the Giltan Golf Club has prepared the following receipts and payments
account for the year ended 31 March 2016.
Rs.(000) Rs.(000)
Balance at 1 April 2015 682 Functions 305
Subscriptions 2,930 Repairs 146
Functions 367 Telephone 67
Sale of land 1,600 Extension of club house 600
Bank interest 60 Furniture 135
Bequest (legacy) 255 Heat and light 115
Sundry income 46 Salary and wages 2,066
Sundry expenses 104
Balance at 31 March 2016 2,402
5,940 5,940

(a) Subscriptions received included Rs. 65,000 which had been in arrears at 31 March 2015 and
Rs. 35,000 which had been paid for the year commencing 1 April 2016.
(b) Land sold had been valued in the club's books at cost Rs. 500,000.
(c) Accrued expenses
31 March 2015 31 March 2016
Rs. (000) Rs. (000)
Heat and light 32 40
Wages 12 14
Telephone 14 10
58 64
(d) Depreciation is to be charged on the original cost of assets appearing in the books at 31
March 2016 as follows:
Buildings 5%
Fixtures and fittings 10%
Furniture 20%

(e) The following balances are from the club's books at 31 March 2015:
Rs.(000)
Land at cost 4,000
Buildings at cost 3,200
Buildings allowance for depreciation 860
Fixtures and fittings at cost 470
Fixtures allowance for depreciation 82
Furniture at cost 380
Furniture allowance for depreciation 164
Subscriptions in arrears 80
(including Rs. 15,000 irrecoverable - member had emigrated)
Subscriptions in advance 30

Required: Prepare an income and expenditure account for the year ended 31 March 2016 and a
Statement of financial position as at that date.

© Emile Woolf International 162 The Institute of Chartered Accountants of Pakistan


Chapter 4: Income and expenditure account

Answer:
Income and expenditure account for Giltan Golf Club for year ending 31 March 2016
Rs.(000) Rs.(000)
Income
Functions surplus (367 − 305) 62
Sale of land (1,600 − 500) 1,100
Bank interest 60
Bequest 255
Sundry income 46
Subscriptions(W1) 2,860
4,383
Expenditure
Bad debts 15
Repairs 146
Telephone (67 − 14 + 10) 63
Heat and light (115 − 32 + 40) 123
Salaries and wages (2,066 − 12 + 14) 2,068
Sundry expenses 104
Depreciation - building 190
Depreciation - furniture 103
Depreciation - fixtures and fittings 47
(2,859)
Surplus for the year 1,524

Giltan golf club: Statement of financial position as at 31 March 2016


Non-current assets Cost Accumulated Carrying
depreciation amount
Rs.(000) Rs.(000) Rs.(000)
Land (4,000 – 500) 3,500 - 3,500
Buildings(W3) 3,800 (1,050) 2,750
Fixtures and fittings(W4) 470 (129) 341
Furniture(W5) 515 (267) 248
8,285 (1,446) 6,839
Current assets
Bank 2,402
9,241

Accumulated Fund(W2) 7,618


Surplus for the year 1,524
9,142
Current liabilities
Accruals 64
Subscriptions in advance 35
99
9,241

© Emile Woolf International 163 The Institute of Chartered Accountants of Pakistan


Financial accounting and reporting I

Workings
W1 Subscriptions account

Rs.(000) Rs.(000)

Subs in arrears b/d 80 Subs in advance b/d 30


Income and expenditure 2,860 Bank 2,930
Subs in advance c/d 35 Bad debts 15
_____ _____
2,975 2,975
_____ _____

W2 Opening statement of affairs 2015


Assets Rs.(000)
Bank 682
Subscriptions in arrears 80
Land 4,000
Buildings (3,200 − 860) 2,340
Fixtures (470 − 82) 388
Furniture (380 − 164) 216
Liabilities 7,706
Accruals (58 + 30) (88)
7,618

W3 Buildings

Cost Acc. Depreciation


Rs.(000) Rs.(000)
Balance b/d 3,200 860
Extension to clubhouse 600
Depreciation (5%  3,800) 190
3,800 1,050

W4 Fixtures and fittings

Cost Acc. Depreciation


Balance b/d 470 82
Depreciation (10%  470) 47
470 129

W5 Furniture

Cost Acc. Depreciation


Balance b/d 380 164
Additions 135
Depreciation (20%  515) 103
515 267

© Emile Woolf International 164 The Institute of Chartered Accountants of Pakistan


Chapter 4: Income and expenditure account

Example 06: LANGTON HOCKEY CLUB


Question: The Langton Hockey club does not have any formal accounting records but the following
information is available.
(1) The payments that have been made by the club for the year ending 30 June 2016 are as
follows:
Rs.(000)
Purchase of second hand table tennis table 250
Rent 600
Tea stall purchases 900
Annual fair expenses 1,450
Outings expenses 370
Prizes for whist evenings 90
Repairs to snooker table 35
Refreshments at social evenings 240

(2) The club's income, apart from annual subscriptions, is as follows:


Rs.(000)
Contributions to outings 300
Takings at the annual fair 2,150
The club also run a tea stall in the village car park every Sunday in the summer months. This
sells tea and coffee, cakes, biscuits and ice creams etc. The profit margin on the tea stall is
normally 20% of selling price.
(3) All the club's transactions are in cash but if there are any surplus funds they are banked in
a local bank account. The balance on the bank account was Rs. 30,000 at 1 July 2015.
(4) The club has an annual subscription rate of Rs. 20,000 per annum per person or Rs.
50,000 per annum for a family membership. Members are asked to pay their subscription
in the July at the beginning of the club's accounting year.
There are 10 family members of the club. Of these two paid their 2016 subscription in June
2015 and all the rest were received before 30 June 2016.
No individual members had paid their 2016 subscriptions in advance but at 30 June 2016
four members still owed their subscriptions. They had been contacted and all four had
promised to pay at the next evening social event. There are in total 80 individual members.
(5) The club has the following other assets and liabilities:
30 June 30 June
2015 2016
Rs. (000) Rs. (000)
Sports equipment 2,560 Note 6
Inventory for the tea stall 120 60
Payables for the tea purchases 110 190
Prepayment of rent 40 50

(6) The sports equipment is all depreciated at 20% per annum on net book value on the basis
of the equipment held at 30 June each year.
(7) The old table tennis table was sold during the year for Rs. 40,000. Its value as recorded by
the club at 30 June 2015 was Rs. 30,000.
Required:
Prepare an income and expenditure account for the year ended 30 June 2016 and a statement of
financial position at that date.

© Emile Woolf International 165 The Institute of Chartered Accountants of Pakistan


Financial accounting and reporting I

Answer:
Income and Expenditure Account for the year ended 30 June 2016
Rs.(000) Rs.(000)
Income
Profits from tea stall (W1) 260
Profit from annual fair (2,150 - 1,450) 700
Subscriptions (W4) 2,100
Profit on sale of table tennis table (40 - 30) 10
______
3,070
Expenditure
Rent (600 + 40 - 50) 590
Net expense of outings (370 - 300) 70
Prizes for whist evenings 90
Repairs to snooker table 35
Refreshments 240
Depreciation (W2) 556
______
1,581
______
Excess of income over expenditure 1,489
______

Statement of financial position as at 30 June 2016


Rs.(000) Rs.(000)
Assets
Non-current assets
Sports equipment 2,224

Current assets
Inventories for tea stall 60
Subscriptions due (4  20) 80
Prepayments - rent 50
Bank (W3) 1,805
______

1,995
______

Total assets 4,219


______

Equity and liabilities

Accumulated fund b/f (W5) 2,540


Excess of income over expenditure 1,489
______

4,029
Current liabilities
Trade payables (tea stall) 190
______

Total equity and liabilities 4,219


______

© Emile Woolf International 166 The Institute of Chartered Accountants of Pakistan


Chapter 4: Income and expenditure account

Workings
(W1) Tea stall
Rs.(000) Rs.(000)
Opening inventory 120
Purchases (900 - 110 + 190) 980
______

1,100
Less: Closing inventory 60
______

Cost of sales 1,040


______

Sales 1,040  10080 1,300


Cost of sales 1,040
_____

Gross profit 1,300  20% 260


______

(W2)
Opening value of sports equipment 2,560
Less: Table tennis table disposed of (30)
Add: Purchase of new table tennis table 250
______

2,780
Less: Depreciation (20%  2,780) 556
______

Book value at 30 June 2016 2,224


______

(W3)
Cash account

Rs.(000) Rs.(000)

Opening balance 30 Table tennis table 250


Contribution to outings 300 Rent 600
Annual fair takings 2,150 Tea stall purchases 900
Tea stall sales (W1) 1,300 Annual fair 1,450
Subscriptions (1,520 + 400) Outings 370
Sale of table tennis table 1,920 Prizes 90
40 Repairs 35
Refreshments 240
Bal c/f (bal fig) 1,805
_____ _____

5,740 5,740
_____ _____

© Emile Woolf International 167 The Institute of Chartered Accountants of Pakistan


Financial accounting and reporting I

(W4) Subscriptions account

Rs.(000) Rs.(000)
Income and expenditure Bal. b/f - Family (2  50,000) 100
(bal fig) 2,100 Bank - Family (8  Rs.50,000) 400
Bank - Individual (76  20,000) 1,520
_____ 80
_____
Bal. c/f - Individual (4  20,000)
2,100
_____ 2,100
_____

(W5) Opening accumulated fund


Rs.(000)

Sports equipment 2,560


Inventory for tea stall 120
Subscriptions in advance (2  50,000) (100)
Rent prepaid 40
Bank 30
Payables for the tea stall (110)
_____

2,540
_____

Example 07: GULSHAN CRICKET CLUB


Question: The following balances have been obtained from the books of Gulshan Cricket Club:

June 30, 2014 June 30, 2015


Building 6,024,000 6,438,150
Furniture 3,012,000 2,710,800
Books 1,129,500 1,246,950
Sports equipment 1,807,200 1,595,200
Investments - 436,000
Advance subscription 86,000 92,000
Prepaid expenses 122,000 176,000
Expenses payable 186,900 207,600
Subscriptions receivable 326,000 357,000
Cash 1,204,800 1,586,500

The following information is also available in respect of the year ended June 30, 2015:
(i) Depreciation for the year has been credited directly to the asset accounts. The rates of
depreciation are as follows:

Building 5%
Furniture and books 10%
Sports equipment 20%

© Emile Woolf International 168 The Institute of Chartered Accountants of Pakistan


Chapter 4: Income and expenditure account

(ii) The club had 600 members on June 30, 2015. No fresh members were admitted during the
year but 10 members left the club on January 1, 2015. Subscription per member is Rs. 500
per month.
Required:
(a) Summary of receipts and payments made during the year ended June 30, 2015.
(b) Income and Expenditure Account for the year ended June 30, 2015.

Answer:
Receipt & payment account for the year ended June 30, 2015
Receipts Rupees Payments Rupees
Balance b/d 1,204,800 Additions: to:
Building 753,000
Subscriptions received 3,605,000 Sports Equipment 186,800
Books 256,000
Investments made 436,000
Expenses (payments)
Balancing 1,591,500
Balance c/d 1,586,500
4,809,800 4,809,800

Income & expenditure account for the year ended June 30, 2015
Expenditures Rupees Incomes Rupees
Expenses A/c 1,558,200 Subscription
(600 x 6000 + 10 x 3000) 3,630,000
Dep. Exp. -Building 338,850
-Furniture 301,200
-Sports
Equipment 398,800
-Books 138,550

Surplus of Income over Exp. 894,400


3,630,000 3,630,000

Workings
Building Account
Rupees Rupees
6,024,000 Depreciation
Balance b/d (6,438,150×5/95) 338,850
Addition 753,000
Balance c/d 6,438,150
6,777,000 6,777,000

© Emile Woolf International 169 The Institute of Chartered Accountants of Pakistan


Financial accounting and reporting I

Sports Equipment Account


Rupees Rupees
Depreciation
Balance b/d 1,807,200 (1,595,200 20/80) 398,800
Addition 186,800
Balance c/d 1,595,200
1,994,000 1,994,000

Furniture Account
Rupees Rupees
Depreciation
Balance b/d 3,012,000 (2,710,800 10/90) 301,200
Balance c/d 2,710,800
3,012,000 3,012,000

Books Account
Rupees Rupees
Depreciation
Balance b/d 1,129,500 (1,246,950 10/90) 138,550
Addition 256,000
Balance c/d 1,246,950
1,385,500 1,385,500

Subscription Account
Rupees Rupees
Sub. Receivables - Balance
b/d 326,000 Adv. Subscription - b/d 86,000
Income & Exp. Account 3,630,000 Cash Received 3,605,000
Adv. Subscription - Balance Sub. Receivables - Balance
c/d 92,000 c/d 357,000
4,048,000 4,048,000

Expenses Account
Rupees Rupees
Balance b/d 122,000 Balance b/d 186,900
Payment made (Rcpt. & Pay. Income & Exp A/c (Bal.
A/c) 1,591,500 Amount) 1,558,200
Balance c/d 207,600 Balance c/d 176,000
1,921,100 1,921,100

© Emile Woolf International 170 The Institute of Chartered Accountants of Pakistan


Chapter 4: Income and expenditure account

Example 08: SEHAT CLUB


Question: Following is the Receipts and Payments Account of Sehat Club for the year ended 30
June 2015:
Receipts and payments account for the year ended 30 June 2015

Receipts Rupees Payments Rupees


Opening balance 15,000 Salaries 63,500
Subscriptions 201,000 Rent 34,000
Entrance fees 63,000 Travelling expenses 1,500
Donations 38,000 Printing and stationery 1,000
Interest 16,000 General charges 2,500
Receipt on disposal of furniture 500 Periodicals 500
Investments 200,000
Closing balance 30,500
333,500 333,500

The club’s statement of financial position as on 30 June 2014 was as follows:


Statement of financial position as on 30 June 2014

Liabilities Rupees Assets Rupees


General Fund 172,500 Furniture – net 40,000
Liabilities: Rent 11,000 Sports equipment – net 20,000
Salaries 17,500 Investments 100,000
Subscription receivable 15,000
Interest receivables 11,000
Bank balance 15,000
201,000 201,000
Other details for the year ended 30 June 2015 are as follows:
(i) Furniture purchased on 1 July 2013 costing Rs. 4,000 was disposed of on 1 January 2015
at a scrap value of Rs. 500.
(ii) On 1 July 2014, furniture having written down value of Rs. 6,000 was traded-in with new
furniture having fair value of Rs. 6,700.
(iii) Depreciation is charged on diminishing balance basis at 20% on furniture and 15% on
sports equipment.
(iv) Sports equipment worth Rs. 12,000 were received at year end as donation.
(v) Following amounts are receivable /outstanding as at 30 June 2015:
Subscription receivable 8,000
Entrance fee receivable 3,000
Salaries outstanding 4,000
Rent outstanding 2,000

Required:
Prepare an income and expenditure account of Sehat Club for the year ended 30 June 2015 and
its statements of financial position on that date.

© Emile Woolf International 171 The Institute of Chartered Accountants of Pakistan


Financial accounting and reporting I

Answer:
Sehat Club: Income and Expenditure Account for the year ended 30 June 2015
Amount Amount
Expenditure Income
(Rs.) (Rs.)
Salaries (63.5+4-17.5) 50,000 Subscriptions (201+8-15) 194,000
Rent (34+2-11) 25,000 Entrance fees (63+3) 66,000
Travelling expenses 1,500 Donation (38+12) 50,000
Printing and stationary 1,000 Interest (16-11) 5,000
General charges 2,500 Gain on trade-in of furniture 700
Periodicals 500
Depreciation on furniture *7,820
Depreciation on sports 3,000
equipment
Loss on furniture disposed of 2,380
(2880-500)
Excess of income over 222,000
expenditure
315,700 315,700

Sehat Club: Statement of financial position as at 30 June 2015

Assets Rupees

Furniture (see account below) 30,000

Sports equipment (20-3+12) 29,000

Investments (100+200) 300,000

Subscription receivable 8,000

Entrance fee receivable 3,000

Bank balance 30,500

400,500

Equity and liabilities Rupees

General fund

Opening balance 172,500

Add: Excess of income over expenditure 222,000 394,500

Liabilities:

Salaries payable 4,000

Rent payable 2,000

400,500

© Emile Woolf International 172 The Institute of Chartered Accountants of Pakistan


Chapter 4: Income and expenditure account

Furniture Account

Rupees Rupees

Balance b/d 40,000 Asset disposed off (4,000 – 800 – 320) 2,880

New furniture 6,700 Asset exchanged 6,000

Depreciation expense *7,820

Balance c/d 30,000

46,700 46,700

* Depreciation on furniture:
20% of (40,000+6,700–3,200–6,000) = 7,500+320 (i.e. 10% of Rs. 3,200).

Example 09: AB SPORTS AND SOCIAL CLUB


Question: You have agreed to take over the role of bookkeeper for the AB sports and social club.
The summarised statement of financial position on 31 December 2014 as prepared by the previous
bookkeeper contained the following items.
Assets Rs.

Heating oil for clubhouse 1,000


Shop and cafe inventories 7,000
New sportswear, for sale, at cost 3,000
Used sportswear, for hire, at valuation Equipment for grounds man 750
Cost 5,000

Depreciation 3,500 1,500


Subscriptions due Bank 200
Current account 1,000

Deposit account 10,000


Capital and liabilities
Accumulated fund 23,150
Payables:
Shop and cafe inventories 1,000
Sportswear 300

The bank account summary for the year to 31 December 2015 contained the following items.

Receipts Rs.
Subscriptions Bank 11,000
Shop and café 20,000
Sale of sportswear 5,000
Hire of sportswear 3,000
Interest on deposit account 800
39,800

© Emile Woolf International 173 The Institute of Chartered Accountants of Pakistan


Financial accounting and reporting I

Payments Rs.
Rent and repairs of clubhouse 6,000
Heating oil 4,000
Sportswear 4,500
Grounds person 10,000
Shop and cafe purchases 9,000
Transfer to deposit account 6,000
39,500

You discover that the subscriptions due figure as at 31 December 2014 was arrived at as follows.
Subscriptions unpaid for 2013 10
Subscriptions unpaid for 2014 230
Subscriptions paid for 2015 40
Corresponding figures at 31 December 2015 are:
Subscriptions unpaid for 2013 10
Subscriptions unpaid for 2014 20
Subscriptions unpaid for 2015 90
Subscriptions paid for 2016 200
Subscriptions due for more than 12 months should be written off with effect from 1 January 2015.
Asset balances at 31 December 2015 include:

Heating oil for club house 700

Shop and cafe inventories 5,000

New sportswear, for sale, at cost 4,000

Used sportswear, for hire, at valuation 1,000

Closing payables at 31 December 2015 are for:

shop and cafe inventories 800

Sportswear 450

heating oil for clubhouse 200

Two thirds of the sportswear purchases made in 2015 had been added to inventory of new
sportswear in the figures given in the list of assets above, and one third had been added directly to
the inventory of used sportswear for hire.
Half of the resulting new sportswear for sale at cost at 31 December 2015 is actually over two
years old. You decide, with effect from 31 December 2015, to transfer these older items into the
inventory of used sportswear, at a valuation of 25% of their original cost.
No cash balances are held at 31 December 2014 or 31 December 2015. The equipment for the
grounds person is to be depreciated at 10% per annum, on cost.
Required:
Prepare the income and expenditure account and statement of financial position for the AB
sports club for 2015.

© Emile Woolf International 174 The Institute of Chartered Accountants of Pakistan


Chapter 4: Income and expenditure account

Answer:
AB Sports and social club: Income and expenditure account
Rs. Rs.
Subscriptions (W1) 10,720
Shop and cafe profit (W2) 9,200
Sale of sportswear (W3) 1,400
Hire of sportswear (W4) 1,700
Interest on deposit account 800
23,820
Rent of clubhouse 6,000

Heating oil (1,000 + 4,000 + 200 − 700) 4,500

Grounds person 10,000


Bad debts (unpaid subscriptions = 10 +20) 30
Depreciation 500
21,030
Net surplus 2,790

AB Sports and Social Club statement of financial position as at 31 December 2014


Non-current assets
Equipment for grounds person
Cost 5,000
Depreciation 4,000
1,000
Current assets
Heating oil 700
Shop and cafe inventories 5,000
New sportswear 2,000
Hire sportswear 1,500
Subscriptions due 90
Bank
Current account 1,300
Deposit account 16,000
26,590
27,590

© Emile Woolf International 175 The Institute of Chartered Accountants of Pakistan


Financial accounting and reporting I

Rs. Rs.
Capital and liabilities
Accumulated fund b/f 23,150
Surplus for year 2,790
25,940
Current liabilities
Shop and cafe 800
Sportswear 450
Heating oil 200
Subscriptions prepaid 200
1,650
27,590
Workings
(W1) Subscriptions
Summary subscriptions account
Rs. Rs.
Opening balance (10 + 230) 240 Opening balance 40
Income for period 10,720 Bank 11,000
Bad debts (10 + 20) 30
Closing balance 200 Closing balance 90
11,160 11,160

(W2) Shop and cafe results


Rs. Rs.
Sales 20,000
Opening inventory 7,000
Purchases (9,000 + 800 − 1,000) 8,800
15,800
Closing inventory 5,000
10,800
Profit (gross) 9,200

(W3) Sale of sportswear

Sales 5,000
Opening inventory 3,000
Purchases (4,500 + 450 − 300)  2 3 3,100
6,100
Closing inventory 4,000
2,100
Profit (gross) 2,900
Loss on sportswear transferred 1,500
Profit 1,400

© Emile Woolf International 176 The Institute of Chartered Accountants of Pakistan


Chapter 4: Income and expenditure account

(W4) Hire of sportswear


Rs. Rs.
Rentals 3,000
Opening balance 750
Additions of cost (4,500 + 450 − 300)  1 3 1,550
2,300
Closing inventory at valuation 1,000
1,300
Surplus 1,700

Example 10: GD SPORTS CLUB


Question: The GD Sports Club does not keep any accounting records other than notes concerning
the subscriptions of members and the amounts paid for expenses. During discussions with the club
committee you discover the following:
(1) The club does not have a bank account and conducts all its transactions in cash, any surplus
being paid into a building society account. The interest credited to this account for the year to
31 March 2015 was Rs.350.
(2) A summary of the payments for the year is:

Rs.

Deposit to building society account 250

Purchase of dartboards 100

Heat/light 262

Repairs to snooker tables 176

Cafe payables 7,455

Rental of premises 1,000

Club match referees’ fees and expenses 675

Trophies, etc (treated as an expense) 424

Refreshments for visiting teams 235

(3) The club has 100 members who each pay Rs.5 per annum subscription. However, on 31
March 2014 ten members had already paid their subscriptions for 2015.
On 31 March 2015 two members who had not been seen in the club since August 2014 had
not paid their subscriptions for 2015 and it has been decided that the amount due be written
off and that their names be removed from the list of members.
(4) The club has only two sources of income from club members – subscriptions and cafe sales. A
profit margin of 30% of selling price is normally applied to determine cafe selling prices but
during the year Rs.397 of goods were sold at cost.

© Emile Woolf International 177 The Institute of Chartered Accountants of Pakistan


Financial accounting and reporting I

(5) The club has the following other assets/liabilities:

1 April 2014 31 March 2015


Rs. Rs.
Equipment 4,000 ?
Building society account 4,600 5,200
Cafe inventories 840 920
Cafe payables 630 470
Cash in hand Nil nil
Creditor for heat/light 34 41
(6) Equipment is depreciated at 10% of the value of equipment held on 31 March each year.

Required:
(a) Prepare a cafe trading account for the year ended 31 March 2015;
(b) Prepare an income and expenditure account for the year ended 31 March 2015;
(c) Prepare a statement of financial position at 31 March 2015.

Answer:
(a) The GD sports club: Cafe trading account for the year ended 31 March 2015

Rs. Rs.
Sales (W1) 9,740 + 397 10,137
Opening inventory 840
Purchases (W3) 7,295
Closing inventory (920)
7,215

Cafe profit 2,922

(b) Income and expenditure account for the year ended 31 March 2015

Income Rs. Rs.

Subscriptions (W2) 500

Profit

From cafe 2,922

Building society interest 350

3,772

© Emile Woolf International 178 The Institute of Chartered Accountants of Pakistan


Chapter 4: Income and expenditure account

Rs. Rs.
Expenditure
Rent of premises 1,000
Heat and light (262 – 34 + 41) 269
Repairs to snooker tables 176
Referees’ fees and expenses 675
Trophies etc. 424
Refreshments for visitors 235
Bad debts (unpaid subscriptions) 10
Depreciation (10%  (4,000 + 100)) 410
3,199
Surplus for the year 573
(c) Statement of financial position at 31 March 2015
Assets Rs. Rs.
Non-current assets:
Equipment at 1 April 2015 4,000
Additions in year – Dartboards 100
4,100
Less: Depreciation (410)
3,690
Current assets:
Cafe inventory 920
Building society deposit 5,200
6,120
9,810

Capital and liabilities


Accumulated fund:
Surplus at 1 April 2015 (W4) 8,726
Surplus for the year 573 9,299
Current liabilities:
Payables
Cafe 470
Heat and light 41
511
9,810

© Emile Woolf International 179 The Institute of Chartered Accountants of Pakistan


Financial accounting and reporting I

Workings
(W1)
Cash account
Rs. Rs.
Balance b/d Cash Nil Payments as per note 2 10,577
Subscriptions (W2) 440 Balance c/d Cash Nil
Cafe sales
At cost 397
At normal selling price (bal fig) 9,740
10,577 10,577
Tutorial note: Sales have been found as a balancing figure from the cash account. An alternative
approach is to use the profit margin supplied in the question. Total purchases need to be computed
(W3) and then calculate:
Purchases Sales
Rs. Rs.
At cost 397 397
100
At margin 6,818  9,740
70
7,215 10,137
(W2)
Subscriptions account
Rs. Rs.
Income and expenditure account Balance b/d
(bal fig) 500 (Subs in advance (10 Rs.5)) 50
Cash receipts
((100 − 10 − 2)  Rs.5) 440
Bad debt (2 Rs. 5) 10
500 500
(W3)
Cafe purchases account
Rs. Rs.
Cash payments 7,455 Balance b/d 630
Balance c/d 470 Purchases (bal fig) 7,295
7,925 7,925
(W4) Accumulated fund at 31 March 2015
Rs.

Equipment 4,000
Cafe inventory 840
Building society account 4,600
Payables - Cafe (630)
Payables - Heat and light (34)
Subscriptions in advance (50)
8,726

© Emile Woolf International 180 The Institute of Chartered Accountants of Pakistan


Chapter 4: Income and expenditure account

Example 11: HB TENNIS CLUB


Question: The HB Tennis Club was formed on 1 April 2015 and has the following receipts and
payments account for the six months ended 30 September 2015:

Receipts Rs. Payments Rs.


Subscriptions 12,600 Purchase of equipment 4,080
Tournament fees 465 Grounds man’s wages 4,520
Bank interest 43 Rent and business rates 636
Sale of club ties 373 Heating and lighting 674
Life membership fees 4,200 Postage and stationery 41
Court maintenance 1,000
Tournament prizes 132
Purchase of club ties 450
Balance c/d 6,148
17,681 17,681

Notes:
(1) The annual subscription fee is Rs.300. On 30 September there were five members who had
not paid their subscriptions, but this money was received on 4 October 2015.

(2) The equipment is expected to be used by the club for five years, after which time it will need
to be replaced. Its estimated scrap value at that time is Rs.50.

(3) During the six months, the club purchased 100 ties printed with its own design. Forty of these
ties remained unsold at 30 September 2015.

(4) The club has paid business rates in advance on 30 September 2015 of Rs.68.

(5) The club treasurer estimates that the following amounts should be accrued for expenses:

Rs.

Grounds man’s wages 40

Postage and stationery 12

Heating and lighting 53

(6) The life membership fees received relate to payments made by four families. The scheme
allows families to pay Rs.1,050 which entitles them to membership for life without further
payment. It has been agreed that such receipts would be credited to income and expenditure
in equal instalments over 10 years.

Required:
(a) Prepare the club’s income and expenditure account for the six months ended 30
September 2015.
(b) Prepare the club’s statement of financial position at 30 September 2015.

© Emile Woolf International 181 The Institute of Chartered Accountants of Pakistan


Financial accounting and reporting I

Answer:
(a) HB Tennis Club income and expenditure account for the six months ended 30 September
2015
Rs. Rs.
Income
Subscriptions (W1) 7,050
Net income from tournaments (465 − 132) 333
Bank interest
Profit from sale of club ties (W2) 103
Life membership (W3) 210
7,739
Expenditure
Groundsman’s wages (4,520 + 40) 4,560
Rent and rates (636 − 68) 568
Heating and lighting (674 + 53) 727
Postage and stationery (41 + 12) 53
Court maintenance 1,000
Depreciation of equipment (W4) 403
(7,311)
Excess of income over expenditure 428
(b) HB Tennis Club statement of financial position as at 30 September 2015
Rs. Rs.
Non-current assets
Equipment at cost 4,080
Accumulated depreciation (403)
3,677
Current assets
Inventory of ties ( 40100 450) 180
Subscriptions in arrears (5  300  612 ) 750
Rates paid in advance 68
Balance at bank 6,148
7,146
10,823

Accumulated fund Rs. Rs.


Excess of income over expenditure 428
Life membership fund (W3) 3,990
4,418
Current Liabilities
Subscriptions in advance (12,600  612 ) 6,300
Accrued expenses (40 + 12 + 53) 105
6,405
10,823

© Emile Woolf International 182 The Institute of Chartered Accountants of Pakistan


Chapter 4: Income and expenditure account

(W1) The subscriptions received of Rs.12,600 are for a full year and we are also told that 5
subscriptions were paid after 30 September.

Rs.
Subscriptions paid for 6 month period ( 612  12,600) 6,300

Subscriptions in arrears (5  612  300) 750

Subscription income 7,050

Alternatively, this may be presented:

Subscriptions account
Rs. Rs.
Income and expenditure 7,050 Bank 12,600
Bal c/d subscriptions in Bal c/d subscriptions in
advance ( 12  12,600) arrears (5  300  12 )
6 6
6,300 750
13,350 13,350

Note also that the remaining Rs.6,300 that has been paid for subscriptions but which
relates to the six months from 1 October 2015 to 31 March 2016 will be shown as a
creditor, subscriptions in advance, in the statement of financial position.
(W2)
Rs. Rs.
Sale of ties 373
Cost of sales
Purchases 450
Closing inventory ( 40100 450) (180)

270

Profit on sale of ties 103


(W3) The life membership fees paid of Rs.4,200 are to be taken to the income and
expenditure account over 10 years or 120 months. Therefore the amount to be taken to
income and expenditure in this 6 month period is 6/120 Rs.4,200 = Rs.210.
This will leave Rs.4,200 −Rs.210 = Rs.3,990 in the Life membership fund on 30
September 2015.
(W4)
Rs.
Cost of equipment 4,080
Less: Estimated scrap value 450
4,030

This is to be depreciated over 5 years or 60 months.


 4,030 = Rs.403.
6
Depreciation charge 60

© Emile Woolf International 183 The Institute of Chartered Accountants of Pakistan


Financial accounting and reporting I

Example 12: MONARCH SPORTS CLUB


Question: The Monarch Sports Club has the following summary of its cash book for the year ended
30 June 2015:
Rs. Rs.
Opening bank balance 12,500
Receipts:
Subscriptions 18,000
Life membership fees 3,000
Competition receipts Entrance 7,500
fees 2,500

Equipment sold 1,000


32,000
44,500
Payments:
Transport to matches 3,700
Competition prizes 4,300
Coaching fees 2,100
Repairs to equipment 800
Purchase of new equipment 4,000
Purchase of sports pavilion 35,000
(49,900)
Closing balance (overdrawn) (5,400)

The following information is available regarding the position at the beginning and end of the
accounting year:

1 July 2014 30 June 2015


Rs. Rs.
Subscriptions in advance 1,100 900
Subscriptions in arrears 200 300
Coaching fees outstanding 150 450

Of the subscriptions outstanding at the beginning of the year, only half were eventually received.
The equipment sold during the year had a net book value of Rs. 1,200 at 1 July 2014. Equipment
is to be depreciated at 20% per annum straight line. Life membership fees are taken to cover 10
years.
The treasurer insists that no depreciation needs to be charged on the sports pavilion, as buildings
do not decrease in value. He says that the last club of which he was treasurer did charge
depreciation on its buildings but that when the club came to replace them, there was still
insufficient money in the bank to pay for the new building.

Required:
Prepare an income and expenditure account for the Monarch Sports Club for the year ended 30
June 2015.

© Emile Woolf International 184 The Institute of Chartered Accountants of Pakistan


Chapter 4: Income and expenditure account

Answer:
Monarch Sports Club: Income and expenditure account year ended 30 June 2015
Rs. Rs.
Income
Annual subscriptions (W1) 18,400
Life membership (3,000  10%) 300
Entrance fees 2,500
Surplus from competitions (W2) 3,200
24,400
Expenditure
Transport 3,700
Coaching fees (2,100 − 150 + 450) 2,400
Repairs 800
Bad debts 100
Loss on disposal of equipment (W3) 200
Depreciation (W4) 800
(8,000)
Surplus for the year 16,400

Workings
(W1)

Subscriptions account
Rs. Rs.
Balance b/d (in arrears) 200 Balance b/d (in advance) 1,100
I + E a/c 18,400 Cash
Balance c/d (in advance) 900 Bad debts 100
Balance c/d (in arrears) 300
19,500 19,500

(W2) Competitions

Rs.
Receipts 7,500
Prizes (4,300)
Surplus 3,200

(W3) Sale of equipment


Disposals account
Rs. Rs.
NBV 1,200 Cash 1,000
Loss to I & E a/c 200
1,200 1,200
(W4) Depreciation
20%  4,000 = 800.

© Emile Woolf International 185 The Institute of Chartered Accountants of Pakistan


Financial accounting and reporting I

Example 13: LH SPORTS CLUB


Question: The LH Sports Club opened on 1 May 2014 having purchased premises for Rs. 80,000
and furniture for Rs. 18,000, through a contribution from a member. The club secretary has
produced the following income and expenditure account for the year to 30 April 2015.

Income Rs. Rs.


Joining fees (89 members Rs.200 each) 17,800
Annual subscriptions 12,000
Cafe profits 8,450
Dinner Dance surplus 830
Equipment hire receipts 1,750
40,830
Expenditure
Premises costs 10,990
Equipment costs 5,590
Secretary’s expenses 470
Bank charges 125 (17,175)
Surplus for the year 23,655

The income and expenditure account has been prepared after taking into account the following
items at 30 April 2015:
❑ cafe inventories Rs. 1,400
❑ payables for cafe supplies Rs.1,320
❑ rates and insurances prepaid Rs. 2,280

The following items have not been taken into account:


❑ the equipment costs figure includes Rs. 4,000 for the purchase of equipment
❑ depreciation is to be provided as follows:
• at 2% on premises
• at 10% on furniture
• at 20% on equipment
❑ joining fees are to be spread over a five-year period
❑ the annual subscriptions figure includes Rs.960 paid in advance
❑ subscriptions outstanding at the end of the year, and expected to be collected, amount to
Rs.300.

The bank balance at 30 April 2015 was Rs. 21,295.

Required:
(a) Calculate the correct surplus for the year.
(b) Prepare the statement of financial position at 30 April 2015.

© Emile Woolf International 186 The Institute of Chartered Accountants of Pakistan


Chapter 4: Income and expenditure account

Answer:
Surplus for the year Rs. Rs.
Surplus per draft income and expenditure account 23,655
Add capital expenditure 4,000
Deduct depreciation
Premises 1,600
Furniture 1,800
Equipment 800
(4,200)
Less 80% joining fee (14,240)
Less net subscriptions in advance (960 − 300) (660)
New surplus for year 8,555

LH Sports Club: Statement of financial position as at 30 April 2015


Assets Rs. Rs.
Non-current assets
Premises 78,400
Furniture 16,200
Equipment 3,200 97,800

Current assets
Inventory 1,400
Subscriptions in arrears 300
Prepaid rates and insurance 2,280
Bank 21,295 25,275
123,075

Capital and liabilities Rs. Rs.


Accumulated fund at 1 May 2014 98,000
Surplus for year 8,555
Accumulated fund at 30 April 2015 106,555
Joining fees c/f 14,240

Current liabilities
Payables 1,320
Subscriptions in advance 960 2,280
123,075
Working
Non-current assets Cost Depreciation Net
Rs. Rs. Rs.
Premises 80,000 (1,600) 78,400
Furniture 18,000 (1,800) 16,200
Equipment 4,000 (800) 3,200
102,000 4,200 97,800

© Emile Woolf International 187 The Institute of Chartered Accountants of Pakistan


Financial accounting and reporting I

Example 14: LEISURE CLUB


Question: The accountant of Leisure Club was terminated on account of charges of fraud on 31
December 2016 and Mr. Emad has been appointed in his place. Emad has gathered the following
information in respect of the year ended 31 December 2016:
(i) The club has 3,300 members and the membership fee is Rs. 10,000 per annum. The fee
payable by each member becomes due on the first day of the quarter in which he became a
member. The fee received in each quarter was as follows:

Quarter First Second Third Fourth


Subscription received (Rs.) 9,900,000 8,250,000 5,500,000 9,350,000
Last year the fee was Rs. 9,000 per annum. However, the number of members was the same.
(ii) A summary of the bank account for the year is shown below:
Deposits Rupees Withdrawals Rupees
Balance as at 1 Jan. 2016 3,700,500 Insurance 175,000
Cash deposited into bank 37,848,500 Rent and rates 4,200,000
Written off amount recovered 1,860,000 Utilities 4,365,000
Disposal of fixed assets 750,000 Freehold land purchased 17,000,000
Members subscription received Cash withdrawals from bank 6,120,000
directly in bank account 19,800,000 Payment to creditors 18,155,000
Repairs and maintenance 700,000
Exercise equipment 7,350,000
Balance as at 31 Dec. 2016 5,894,000
63,959,000 63,959,000
(iii) Amounts paid from petty cash were as follows:
Rupees
Salaries 2,300,000
Sundry expenses 640,000
(iv) The club has a tuck shop which earns a profit margin of 20% of sales. All sales of tuck shop
are made on cash. During the year, stock costing Rs. 500,000 was destroyed by fire.
(v) The opening WDV of fixed assets was Rs. 28,000,000. Exercise equipment was purchased
on 1 October 2016. Fixed assets having opening WDV of Rs. 800,000 were disposed of on
31 March 2016. Fixed assets are depreciated @ 20% under the reducing balance method.
(vi) The opening and closing balances of cash in hand were Rs. 300,000 and Rs.
25,000 respectively.
(vii) The following balances have been extracted through a scrutiny of the available records:
2016 2015
------- Rupees -------
Creditors 3,330,000 2,500,000
Prepaid rent 175,000 168,000
Stock- tuck shop 2,500,000 2,300,000
Required:
(a) Determine the amount of loss incurred by the club due to fraud committed by the
previous accountant.
(b) An income and expenditure account for the year ended 31 December 2016.
(c) Statement of financial position as at 31 December 2016.

© Emile Woolf International 188 The Institute of Chartered Accountants of Pakistan


Chapter 4: Income and expenditure account

Answer:
Determination of amount of loss incurred due to fraud
Rupees
Opening cash balance 300,000

Cash receipts
Collection from members [(3,300 x 10,000) – 19,800,000] 13,200,000
Bank withdrawals 6,120,000
Tuck shop sales (W-2) 22,856,250
42,176,250
Cash payments
Salaries (2,300,000)
Sundry expenses (640,000)
Cash deposited into bank (37,848,500)
(40,788,500)
Closing cash should have been 1,687,750
Closing cash-actual (25,000)
Loss due to fraud 1,662,750
Income and expenditure account

Income Rupees
Subscription income (W-1) 31,817,500
Income from tuck shop (22,856,250(W-2) – 18,285,000 (W-2)) 4,571,250
Other income – Bad debts recovered 1,860,000
38,248,750
Expenditures
Salaries 2,300,000
Insurance 175,000
Rent expense (168,000 + 4,200,000 – 175,000) 4,193,000
Utilities 4,365,000
Repair and maintenance 700,000
Depreciation (W-3.1) 5,847,500
Sundry expenses 640,000
Loss on disposal [750,000 – (800,000 – 40,000)] 10,000
Loss of inventory due to fire 500,000
Loss due to fraud 1,662,750
20,393,250
Excess of income over expenditures 17,855,500

© Emile Woolf International 189 The Institute of Chartered Accountants of Pakistan


Financial accounting and reporting I

Statement of Financial Position

2016 2016
Fund and Liabilities Assets
Rupees Rupees

Accumulated fund 39,181,500 Fixed asset - WDV (W-3) 45,742,500


(Balancing)

Creditors 3,330,000 Stock 2,500,000

Unearned subscription (W-1) 11,825,000 Prepaid rent 175,000

Cash and bank 5,919,000


(5,894,000+25,000 )

Total fund and liabilities 54,336,500 Total assets 54,336,500

2015
W-1: Determination of subscription income Rupees

Opening unearned subscription income (11,825,000(W-1.1)×9/10) 10,642,500

Add: Receipts for the year (3,300×10,000) 33,000,000

Less: Closing unearned subscription income (W-1.1) (11,825,000)

Subscription income for the year 31,817,500

W-1.1: Closing unearned subscription income

Quarter - 1 -

Quarter – 2 2,062,500

Quarter – 3 2,750,000

Quarter – 4 7,012,500

11,825,000

W-2: Tuck shop sales & cost of sales

Opening stock 2,300,000

Add: Purchases of stock (W-2.1) 18,985,000

Less: Loss due to fire, charged to expenditures (500,000)

Less: Closing stock (2,500,000)

Cost of goods sold 18,285,000

Sales (18,285,000/0.80) 22,856,250

© Emile Woolf International 190 The Institute of Chartered Accountants of Pakistan


Chapter 4: Income and expenditure account

2015
W-2.1: Purchases of tuck shop Rupees
Closing creditors 3,330,000
Add: Payments to creditors 18,155,000
Less: Opening creditors (2,500,000)
Purchases 18,985,000

W-3: Fixed assets and depreciation


Opening WDV before disposal 28,000,000
Add: Addition (7,350,000 + 17,000,000) 24,350,000
Less: WDV of assets disposed off (800,000 – 40,000) (760,000)
Less: Depreciation for the year (W-3.1) (5,847,500)
Closing WDV 45,742,500

W-3.1: Depreciation

Depreciation on opening WDV [(28,000,000–800,000)×20%] 5,440,000


Depreciation on disposed asset (800,000×20%×3/12) 40,000
Depreciation on addition (7,350,000×20%×3/12) 367,500

Depreciation for the year 5,847,500

Example 15: SEAVIEW CLUB


Question: Seaview Club started its operations on 1 February 2015. Sponsor of the club contributed
Rs. 50 million towards general fund for the start of operations and placed the amount in the bank.
Following is the receipts and payments summary for the period from 1 February 2015 to 31
December 2015:

Receipts Rs. in ‘000 Payments Rs. in ‘000


Sponsor's contribution 50,000 Furniture & fixtures 1,200
Joining fees 20,800 Van 1,500
Subscription from members 29,952 Salaries 1,000
Sale of beverages 1,500 Rent 3,600
Utilities 570
Insurance 120
Repairs and maintenance 275
Purchase of beverages 1,367
Advance for plot of land 65,000
Balance 27,620
102,252 102,252

© Emile Woolf International 191 The Institute of Chartered Accountants of Pakistan


Financial accounting and reporting I

Additional information:
(i) The joining fee for award of membership is Rs. 50,000. Annual subscription is Rs. 24,000.
All new members pay three years’ subscription in advance.
The memberships were awarded as follows:

Month March June September December

No. of members 112 98 101 105

(ii) The club sells beverages at a gross profit margin of 20%. All sales are billed in the first week
of the next month and the payment is received in the same month. Sale of beverages during
December 2015 amounted to Rs. 150,000.
(iii) 25% of total purchases of beverages made during the year remained unsold at year-end.
(iv) Salaries are paid on the first day of next month. The amount of salaries includes an advance
amounting to Rs. 10,000 paid to an employee on 1 December 2015. The advance is
repayable on 1 February 2016.
(v) Rent for three years was paid in advance on 1 February 2015.
(vi) Presently the club is operating on rental premises. However, a plot of land has been
purchased on which construction would commence shortly. Title of land would be transferred
after completion of legal formalities.
(vii) Payments for utilities include security deposit paid to utility companies amounting to Rs.
20,000. Utility bills are paid on the 7th day of the next month.
(viii) Insurance premium was paid on 1 February 2015 covering a period of 12 months.
(ix) Repairs and maintenance include an advance of Rs. 100,000 paid to a contractor for
construction of a parking shed. Repair bills amounting to Rs. 7,000 were outstanding at year-
end.
(x) Furniture & fixtures and van were purchased on 1 February 2015. Depreciation on these
assets is to be charged at 10% and 20% respectively.

Required: Prepare statement of financial position as at 31 December 2015 and income &
expenditure account of Seaview Club for the period ended31 December 2015.
Answer:
Seaview Club
Income & Expenditure Account for the period ended 31 December 2015
Expenditure Rs.in ‘000 Income Rs.in ‘000
Salaries and wages (1000-10+99) 1,089 Joining fee 20,800
Rent (3,600/3x11/12) 1,100 Subscription income 4,630
(W-1)
Utilities (570-20+55) 605 Profit on sale of 330
beverages (W-2)
Insurance (120/12x11) 110
Repairs and maintenance (275–100+7) 182
Depreciation expense 385
(1,200x10%x11/12+1,500x20%x11/12)
Excess of income over expenditure 22,289
25,760 25,760

© Emile Woolf International 192 The Institute of Chartered Accountants of Pakistan


Chapter 4: Income and expenditure account

Statement of Financial Position as at 31 December 2015

General Fund & Liabilities Rs. in ‘000 Assets Rs. in ‘000

General fund 50,000 Non-Current Assets

Excess of income over 22,289 Land/Capital advance 65,000


expenditure

72,289 Furniture & fixtures (1,200−110) 1,090

Van (1,500−275) 1,225

Deferred income/long term Advance for parking shed 100


advance (W-1)
15,338 Long term deposits 20

Long term prepayment 1,300

Current Liabilities 68,735

Creditors (1,760 – 1,367) 393 Current Asset

Accrued expenses (7+55+99) 161 Stock (W-2) 440

Advance subscription (W-1) 9,984 Debtors for beverages (credit 150


sale)

10,538 Advance & prepayments (W-3) 1,220

Bank 27,620

29,430

Total General Fund & Liabilities 98,165 Total Assets 98,165

W-1: Subscription income


Subscription for 3 years is Rs. 72,000 so subscription for 1 year is Rs. 24,000 or Rs. 2,000 per
month

No. of No. of Subscription income Deferred subscription


members months for the year income
Month
A×B×2,000 A×(36−B)×2,000
A B
--------------- Rupees ---------------

March 112 10 2,240,000 5,824,000

June 98 7 1,372,000 5,684,000

September 101 4 808,000 6,464,000

December 105 1 210,000 7,350,000

4,630,000 25,322,000

Less: Short term (9,984,000)


[(112+98+101+105)×24,000]

Long term 15,338,000

© Emile Woolf International 193 The Institute of Chartered Accountants of Pakistan


Financial accounting and reporting I

W-2: Beverage sale results Rs. in ‘000


Sales (1,500 + 150) 1,650

Less: Cost of sales 1,760


Purchases (1,320/0.75) (440) 1,320

Closing stock (1,760×25%) 330

W-3: Advance & prepayments Rs. in ‘000

Rent [(3,600–1,100)–1,300(long term)] 1,200


Insurance (120 – 110) 10

Advance salary 10

1,220

Example 16: Chongtar International Hospital


Question: Following is the trial balance of Chongtar International Hospital as on 31 December
2019:
Debit Credit
---- Rs. in million ----
Burns ward - capital work in progress 55.3
Cafeteria sales 24.4
Cash and bank balances 8.4
Donations for burns ward 75.1
Expenses and gifts for ‘walk on diabetes day’ 2.6
Fees from patients 125.0
General donations 82.6
General fund 195.6
Inventory - cafeteria 4.7
Inventory - medicines 19.4
Inventory - hospital supplies 8.5
Medical equipment 185.4 64.2
Miscellaneous expenses 8.5
Other fixed assets 110.7 54.7
Payables 38.9
Purchases - cafeteria 16.4
Purchases - medicines 60.5
Purchases - hospital supplies 18.7
Receivables - panel corporates 31.4
Rent 19.6
Sponsorship for ‘walk on diabetes day’ 2.2
Salaries - administrative staff 24.0
Salaries - doctors and nursing staff 38.2
Short term investments 38.0
Utilities 12.4
662.7 662.7

© Emile Woolf International 194 The Institute of Chartered Accountants of Pakistan


Chapter 4: Income and expenditure account

Additional information:
(i) Cost of closing physical inventory of medicines and hospital supplies was Rs. 25.8
million and Rs. 13.8 million respectively. Medicines costing Rs. 3.1 million were found
expired. Medicines are only used to treat the admitted patients and are not sold
separately.
(ii) Year-end physical count of cafeteria inventory could not take place. Goods are sold in
cafeteria at a gross margin of 25% on sales.
(iii) Rent outstanding at year-end was Rs. 1.4 million.
(iv) 15% of salaries and 10% of rent are related to cafeteria.
(v) Hospital facilities of Rs. 48.6 million were provided free of charge to the patients.
(vi) ‘Walk on diabetes day’ was organised in December 2019. Expenses relating to the
event amounting to Rs. 1.2 million were outstanding and unrecorded at year end.
(vii) Medical equipment having fair value of Rs. 36.8 million were received as donation.
These have been brought into use but have not been recorded in the books.
(viii) Depreciation is charged on reducing balance method at 15% per annum.
Required:
(a) Prepare income and expenditure account for the year ended 31 December 2019
(b) Prepare statement of financial position as on 31 December 2019

Answer:
Chongtar International Hospital
Part (a)
Income and expenditure account for the year ended 31 December 2019

Rs. in million
Income:
Fees from patients 125.0
General donations 82.6+36.8 119.4
Profit from cafeteria 0.4
244.8
Expenditures:
Salaries - Administrative staff 24×0.85 20.4
Salaries - Doctors and nursing staff 38.2
Medicines used (19.4+60.5) – (25.8–3.1) 57.2
Hospital supplies used (8.5+18.7)–13.8 13.4
Rent (19.6 +1.4)×0.9 18.9
Walk on diabetes day -net (2.6+1.2)–2.2 1.6
Depreciation - Medical equipment [(185.4–64.2)+36.8]×15% 23.7
Depreciation - Other fixed assets (110.7–54.7)×15% 8.4
Utilities 12.4
Miscellaneous expenses 8.5
(202.7)
Excess of income over expenditure 42.1

© Emile Woolf International 195 The Institute of Chartered Accountants of Pakistan


Financial accounting and reporting I

Cafeteria trading account for the year ended 31 December 2018


Rs. in million
Sales 24.4
Cost of goods sold:
Opening stock 4.7
Purchases 16.4
Closing stock Balancing (2.8)
(18.3)
Gross profit 24.4×0.25 6.1
Expenses:
Salaries 24×0.15 (3.6)
Rent (19.6+1.4)×0.1 (2.1)
Profit from cafeteria 0.4

Part (b)
Statement of financial position as on 31 December 2019
Rs. in million
Non-current assets:
Medical Equipment (185.4–64.2)+36.8–23.7 134.3
Other fixed assets (110.7–54.7–8.4) 47.6
Burns ward - CWIP 55.3
237.2
Current assets:
Cafeteria stock 2.8
Medicines stock (25.8 -3.1) 22.7
Hospital supplies 13.8
Receivables 31.4
Short term investments 38.0
Cash in hand 8.4
117.1
354.3
General funds:
Opening balance 195.6
Excess of income over expenditure 42.1
237.7

Burns ward fund 75.1

Liabilities:
Creditors 38.9
Accrued expenses 1.4+1.2 2.6
41.5
354.3
.

© Emile Woolf International 196 The Institute of Chartered Accountants of Pakistan


Chapter 4: Income and expenditure account

4 OBJECTIVE BASED QUESTIONS


01. Which of the following is generally considered as a non-profit oriented organization?
(a) Charitable organization
(b) Corporation
(c) Audit firms
(d) Insurance companies

02. Expenditures greater than incomes of a non-profit organization give rise to a:


(a) Loss
(b) Profit
(c) Surplus
(d) Deficit

03. An advance receipt of subscription from a member of the non-profit organization is considered
as a/an:
(a) Expense
(b) Liability
(c) Equity
(d) Asset

04. Income and expenditure account is based on;


(a) Cash accounting
(b) Accrual accounting
(c) Government accounting
(d) Management accounting

05. Life membership fees of not for profit concerns is?


(a) Capital Receipts
(b) Revenue Receipts
(c) Both (a) & (b)
(d) None (a) & (b)

06. When cash is received for life membership, which one of the following double entries is passed?
(a) Cash Debit and capital Credit
(b) Life membership Debit and cash Credit
(c) Investment Debit and cash Credit
(d) Cash Debit and life membership fund Credit

© Emile Woolf International 197 The Institute of Chartered Accountants of Pakistan


Financial accounting and reporting I

07. XYZ club has a bar that maintains a separate trading account for its trading activities. Which of
the following is the treatment of profit or loss on bar trading activities?
(a) Profit or loss is directly shown in the statement of financial position
(b) Profit or loss is to be presented in income and expenditure account
(c) Profit or loss is credited in income statement
(d) Profit or loss is added to accumulated fund

08. Which of the following is the accounting equation for a non-profit organization?
(a) Asset = Capital + Liabilities
(b) Capital + Liabilities = Assets
(c) Accumulated fund + Liabilities = Assets
(d) Liabilities = Asset + Accumulated fund

09. Subscription earned but not yet received is considered as a/an:


(a) Asset
(b) Liability
(c) Income
(d) Expenditure

10. A non-profit organization received Rs. 100,000 as the entrance fee of a new member. If 20% of
the fee has to be capitalized, what is the amount of fee needs to be shown in the income and
expenditure account?
(a) Rs. 20,000
(b) Rs. 80,000
(c) Rs. 90,000
(d) Rs. 10,000

11. Rs. 1,000,000 received as the annual membership subscription. Out of this, Rs. 200,000 is
pertaining to the previous accounting period whereas Rs. 100,000 is receivable at the end of the
current accounting period.
Calculate the amount of subscription that will be shown in the income and expenditure account.
(a) Rs. 100,000
(b) Rs. 900,000
(c) Rs. 1,200,000
(d) Rs. 800,000

12. Income and expenditure accounts show:


(a) Cash available to an organization
(b) Closing capital of an organization
(c) Cash available in the bank account
(d) Surplus or deficit for the current accounting period

© Emile Woolf International 198 The Institute of Chartered Accountants of Pakistan


Chapter 4: Income and expenditure account

13. On what basis the ‘receipts and payments account’ is prepared?


(a) Cash basis
(b) Accrual basis
(c) Both accrual and cash basis
(d) None of the two

14. Payment of Honorarium to secretary is treated as?

(a) Capital Expenditure

(b) Revenue Expenditure

(c) Cash Expenses

(d) None of these

15. Income and Expenditure Account records:


(a) Capital items
(b) Revenue items
(c) A and B both
(d) None of these

16. A club has 500 members. Annual membership fees are Rs. 1,000. Therefore, membership fees
for the year should be Rs. 500,000.
The club’s subscription records for the year ended 31 December 2013 show the following:
At 31 December At 31 December
2012 2013
Subscriptions in advance 10,000 6,000
Subscriptions in arrears 18,000 22,000
Calculate the amount of cash received during the year.
Rs. ___________

17. At 31 March 2012 a cricket club had membership subscriptions in arrears amounting to Rs.
48,000 and had received Rs. 12,000 subscriptions in advance.
During the year to 31 March 2013 the club received Rs. 624,000 including 26 memberships for
the year to 31 March 2014 at Rs. 1,200 per annum in advance.
At 31 March 2013 16 members owed subscriptions of Rs. 1,200 each.
Calculate the amount of subscription income during the year.
Rs. ___________

© Emile Woolf International 199 The Institute of Chartered Accountants of Pakistan


Financial accounting and reporting I

18. At 31 March 2012 a cricket club had membership subscriptions in arrears amounting to Rs.
48,000 and had received Rs. 12,000 subscriptions in advance.
During the year to 31 March 2013 the club received Rs. 624,000 including 26 memberships for
the year to 31 March 2014 at Rs. 1,200 per annum.
At 31 March 2013 16 members owed subscriptions of Rs. 1,200 each.
Half of the members who were in arrears at the end of the previous period still had not paid by
31 March 2013. It was decided to write these amounts off.
Required:
Calculate the amount of subscription income during the year.
Rs. ___________

19. Seaview Club started its operations on 1 February 2015. Total subscription received for the
period ended 31 December 2015 was Rs. 29,952,000
Annual subscription is Rs. 24,000. All new members pay three years’ subscription in advance.
The memberships were awarded as follows:
Month March June September December
No. of member 112 98 101 105
What amount of subscription income should be included in income and expenditure account for
the period ended 31 December 2015?
Rs. ___________

20. Seaview Club started its operations on 1 February 2015. Total subscription received for the
period ended 31 December 2015 was Rs. 29,952,000
Annual subscription is Rs. 24,000. All new members pay three years’ subscription in advance.
The memberships were awarded as follows:
Month March June September December
No. of member 112 98 101 105
What amount of advance subscription should be included in non-current liabilities as at 31
December 2015?
Rs. ___________

21. The main objective of a non-profit organization is;


(a) To earn profits
(b) To create monopoly
(c) Welfare of the society
(d) To provide for owner’s dividends

22. Non-profit organizations prepare all of the following accounts except the;
(a) Receipt and payment account
(b) Income and expenditure account
(c) Statement of financial position
(d) Profit or loss account

© Emile Woolf International 200 The Institute of Chartered Accountants of Pakistan


Chapter 4: Income and expenditure account

23. Examples of non-profit organization is:


(a) Commercial banks
(b) Civil hospital
(c) Private educational institutions
(d) Association of person

24. The main source of income for non-profit organization is:


(a) Subscription
(b) Sales
(c) Dividends
(d) Other income

25. Income and expenditure accounts show;


(a) Cash available to an organization
(b) Closing capital of an organization
(c) Cash available to the bank account
(d) Surplus or deficit for an accounting period

26. The statement of financial position of a non-profit organization does not contain the;
(a) Owner’s equity
(b) Liability
(c) Asset
(d) Income

27. Rent expense of a non-profit organization paid in advance. Which of the following is the
correct classification of rent?
(a) Expense
(b) Liability
(c) Asset
(d) Equity

28. An advance receipt of subscription from a member of the non-profit organization is considered
as a/an?
(a) Expense
(b) Liability
(c) Asset
(d) Income

© Emile Woolf International 201 The Institute of Chartered Accountants of Pakistan


Financial accounting and reporting I

29. The capital of a non-profit organization is generally known as


(a) Equity
(b) Accumulated funds
(c) Retained earning
(d) Cash fund

30. When cash is received for life membership, which one of the following double entries is passed?
(a) Cash (debit) and capital (credit)
(b) Life membership (debit) and cash (credit)
(c) Cash (debit) and investment (credit)
(d) Cash (debit) and Life membership (credit)

31. If debit side of receipt and payment account exceeds credit, it represents:
(a) Cash at bank
(b) Bank overdraft
(c) Surplus
(d) Deficit

32. Receipt and payment account include:


(a) Revenue items
(b) Capital items
(c) Both capital and revenue items
(d) None of above

33. Sale of an old newspaper is classified as:


(a) Expense
(b) Liability
(c) Asset
(d) Income

34. Gift presented to Chief Guest at annual function by a non-profit organization is:
(a) Gift
(b) Reward
(c) Honorarium
(d) Grant

© Emile Woolf International 202 The Institute of Chartered Accountants of Pakistan


Chapter 4: Income and expenditure account

35. Morning Football Club has a monthly subscription fee of Rs. 800 per member. The club has 240
members on 31 December 2018. No fresh members were admitted during 2018 but 30 members
left the club on 1 July 2018. As at 31 December 2018, the club has received subscription in
advance amounting to Rs. 60,000. The club’s subscription income for 2018 would be:
(a) Rs. 2,448,000
(b) Rs. 2,388,000
(c) Rs. 2,160,000
(d) Rs. 2,100,000

36. Alpha Club’s financial year ends on 31 December. Following information pertain to its members’
subscription:
Rupees
Subscription received in 2018 for 2019 180,000
Subscription received in 2019 for 2018 90,000
Subscription received in 2019 for 2019 1,400,000
Subscription received in 2019 for 2020 200,000
Subscription for 2018 outstanding as on 31 December 2018 150,000
Subscription for 2019 outstanding as on 31 December 2019 325,000

Subscription income for the year ended 31 December 2019 is:


(a) Rs. 1,845,000
(b) Rs. 1,705,000
(c) Rs. 1,905,000
(d) Rs. 1,665,000

© Emile Woolf International 203 The Institute of Chartered Accountants of Pakistan


Financial accounting and reporting I

4 OBJECTIVE BASED ANSWERS


01. (a)
02. (d)
03. (b)
04. (b)
05. (a)
06. (d)
07. (b)
08. (c)
09. (a)
10. (b)
11. (b)
Subscription a/c
Particulars Rs. Particulars Rs.
b/d 200,000 Cash received 1,000,000
I&E 900,000 c/d 100,000
1,100,000 1,100,000

12. (d)
13. (a)
14. (b)
15. (b)
16. Rs. 492,000
Subscriptions
Rs. Rs.
Balance b/d 18,000 Balance b/d 10,000
I&E 500,000 Cash 492,000
Balance c/d 6,000 Balance c/d 22,000
524,000 524,000

17. Rs. 576,000


Subscriptions
Rs. Rs.
Balance b/d 48,000 Balance b/d 12,000
I&E 576,000 Cash 624,000
Balance c/d Balance c/d
[26 x Rs. 1,200] 31,200 [16 x Rs. 1,200] 19,200
655,200 655,200

© Emile Woolf International 204 The Institute of Chartered Accountants of Pakistan


Chapter 4: Income and expenditure account

18. Rs. 600,000


Subscriptions
Rs. Rs.
Balance b/d 48,000 Balance b/d 12,000
I&E 600,000 Cash 624,000
Bad debts
[48,000 x ½] 24,000
Balance c/d Balance c/d
[26 x Rs. 1,200] 31,200 [16 x Rs. 1,200] 19,200
679,200 679,200

19. Rs. 4,630,000 Subscription for 3 years is Rs. 72,000 so subscription for 1 year is Rs.
24,000 or Rs. 2,000 per month
Rs.000

Receipt / I&E Current Non-current


Members Months Rs. Months Rs. Months Rs.
Mar / 112 10 2,240 12 2,688 14 3,136
Jun / 98 7 1,372 12 2,352 17 3,332
Sep / 101 4 808 12 2,424 20 4,040
Dec / 105 1 210 12 2,520 23 4,830
4,630 9,984 15,338

20. Rs. 15,338,000 Subscription for 3 years is Rs. 72,000 so subscription for 1 year is Rs.
24,000 or Rs. 2,000 per month
Rs.000

Month / I&E Current Non-current


Members Months Rs. Months Rs. Months Rs.
Mar / 112 10 2,240 12 2,688 14 3,136
Jun / 98 7 1,372 12 2,352 17 3,332
Sep / 101 4 808 12 2,424 20 4,040
Dec / 105 1 210 12 2,520 23 4,830
4,630 9,984 15,338

21. (c)
22. (d)
23. (b)
24. (a)
25. (d)
26. (a)
27. (c)

© Emile Woolf International 205 The Institute of Chartered Accountants of Pakistan


Financial accounting and reporting I

28. (b)
29. (b)
30. (d)
31. (a)
32. (c)
33. (d)
34. (c)
35. (a) Rs. 2,448,000
36. (c) Rs. 1,905,000

© Emile Woolf International 206 The Institute of Chartered Accountants of Pakistan


Certificate in Accounting and Finance

CHAPTER
Financial accounting and reporting I

Preparation of accounts from


incomplete records

Contents
1 The nature of incomplete records
2 Techniques for incomplete records
3 Objective based questions and answers

* The student must refer original handbook of IFRS.

© Emile Woolf International 207 The Institute of Chartered Accountants of Pakistan


Financial accounting and reporting I

1 THE NATURE OF INCOMPLETE RECORDS


Section overview

 The meaning of incomplete records


 Dealing with incomplete records

1.1 The meaning of incomplete records


Incomplete records, as the term suggests, are accounting records where information is missing.
Problems of incomplete records may arise with small businesses where the owner of the business
has not kept up-to-date accounting records or does not have a double entry book-keeping system.
He might simply keep invoices or receipts for expenses and copies of invoices to customers. In
addition, details of bank transactions can be obtained from a bank statement or other banking
records.
The task of the accountant is to use these invoices, receipts and banking records, together with
other information obtained from the business owner, to prepare financial statements for the year
(and in particular a statement of comprehensive income, which provides a basis for calculating the
taxable income of the business owner from his or her business).
Other circumstances that cause problems include loss of records because of some kind of disaster,
for example a fire in the office.
Another scenario is where records have not been maintained because a dishonest employee has
stolen cash or inventory.
Whatever the cause of the problem the accountant’s task involves piecing together information that
is available in order to produce a set of financial statements or to calculate missing figures.

1.2 Dealing with incomplete records


Questions on incomplete records are a good test of knowledge and understanding of book-keeping
and accounting. The task is often to identify the missing figures that the incomplete records do not
provide.
Possible approaches to establishing missing numbers include:
 establishing the value of assets and liabilities to calculate the business capital, particularly
opening capital at the start of the financial period
 using memorandum control accounts, for receivables or payables, to calculate the sales or
purchases for the period
 using a memorandum account for bank and cash transactions, to establish a missing figure
for cash receipts or cash payments, such as a missing figure for cash taken from the
business by the owner as drawings
 Using cost structures (gross profit percentage or mark-up) to establish a cost of sales, or a
missing figure such as the value of inventory stolen or lost in a fire.
 Income & expenses accounts carrying opening or closing accrual or prepayment balances

© Emile Woolf International 208 The Institute of Chartered Accountants of Pakistan


Chapter 5: Preparation of accounts from incomplete records

2 TECHNIQUES FOR INCOMPLETE RECORDS


Section overview

 The accounting equation


 Memorandum control accounts
 Memorandum cash and bank account
 Cost structures
 Missing inventory figure

2.1 The accounting equation


The accounting equation is:

Formula: Accounting equation

Assets = Liabilities + Equity Or Assets  Liabilities = Equity

A = L + E A  L = E

Net assets

Profit or loss for a period can be calculated from the difference between the opening and closing
net assets after adjusting for any drawings during the period.

Formula:

Increase in net assets = Profit + capital introduced – drawings

The profit figure can be calculated as follows:

Illustration 01:

Rs.

Closing assets – liabilities X

Opening assets – liabilities X

Increase/(decrease) in net assets in the period X

Add drawings X

Subtract new capital introduced by the owner(s) (X)

Profit /(loss) for the year X

© Emile Woolf International 209 The Institute of Chartered Accountants of Pakistan


Financial accounting and reporting I

Example 01:
Question: At 1 January 2017, the business of Tom Canute had assets of Rs. 214,000 and liabilities
of Rs. 132,000. At 31 December 2017, the business had assets of Rs. 281,000 and liabilities of
Rs. 166,000.
Tom took Rs. 25,000 in cash and Rs. 3,000 in goods out of the business during the year for his
personal use. He did not introduce any new capital.
Required:
Calculate the profit of the business of Tom Canute in the year to 31 D1ecember 2017.
Answer:

Rs. Rs.
Assets at 31 December 2017 281,000
Liabilities at 31 December 2017 (166,000)
Net assets at 31 December 2017 115,000

Assets at 1 January 2017 214,000


Liabilities at 1 January 2017 (132,000)
Net assets at 1 January 2017 (82,000)
Increase in net assets during the year 33,000
Add: Drawings (25,000 + 3,000) 28,000
Profit for the year 61,000

Alternatively, the profit figure could be calculated using the equation:


Increase in net assets =Profit + Capital introduced – Drawings
Rs. 33,000 = Profit + 0 – Rs. 28,000
Profit = Rs. 61,000

Example 02:
Question: The accountant for a sole trader has established that the total assets of the business at
31 December Year 4 were Rs. 376,000 and total liabilities were Rs. 108,000. Checking the
previous year’s financial statements, he was able to establish that at 31 December Year 3 total
assets were Rs. 314,000 and total liabilities were Rs. 87,000. During Year 4 the owner has taken
out drawings of Rs. 55,000. In December Year 4 the owner had been obliged to input additional
capital of Rs. 25,000.
Required: Calculate the profit of the business for the year to 31 December Year 4.
Answer:

Rs.
Net assets at 31 December (376,000 – 108,000) 268,000
Net assets at 1 January (314,000 – 87,000) (227,000)
Increase in net assets 41,000
Drawings 55,000
New capital introduced in the year (25,000)
Profit for the year 71,000
.

© Emile Woolf International 210 The Institute of Chartered Accountants of Pakistan


Chapter 5: Preparation of accounts from incomplete records

Identifying missing balances


The approach can also be used to identify a missing balance at the end of a period.

Illustration 02: Missing balance


At the start of the year a business had opening capital of Rs. 350,000.
Profit for the year was Rs. 200,000 and the owner had taken Rs. 120,000 as drawings. No capital
was introduced in the period.
At the end of the year the company cashier disappeared with an amount of cash.
The owner was able to identify the following balances at the year-end:
Rs.
Property, plant and equipment 95,000
Inventory 85,000
Receivables 65,000
Liabilities (55,000)

The missing cash balance can be calculated as follows:


Step 1: Work out what closing net assets should be: Rs.
Capital (net assets) at the start 350,000
Profit for the year 200,000
Less: Drawings (120,000)
Capital (net assets) at the end should be: 430,000

Step 2: Work out what closing net assets are: Rs.


Property, plant and equipment 95,000
Inventory 85,000
Receivables 65,000
Liabilities (55,000)
190,000
Step 3: Identify the missing amount: 240,000

The cashier has stolen cash amounting to Rs. 240,000.

Calculation of opening capital


It might be necessary to establish the opening capital of a sole trader. This can be done simply by
obtaining figures for the assets and liabilities of the business at the beginning of the financial period.
Opening capital is the difference between total assets and total liabilities. (Non-current assets for
this purpose are measured at their carrying amount, i.e. net book value.)

Example 03:
Question: A sole trader does not keep any accounting records, and you have been asked to prepare
a statement of comprehensive income and statement of financial position for the financial year
just ended. To do this, you need to establish the opening capital of the business at the beginning
of the year.

© Emile Woolf International 211 The Institute of Chartered Accountants of Pakistan


Financial accounting and reporting I

You obtain the following information about assets and liabilities at the beginning of the year:
Rs.
Motor van (balance sheet valuation) 1,600
Bank overdraft 560
Cash in hand 50
Receivables 850
Trade payables 370
Payables for other expenses 90
Inventory 410

Required:
Calculation for the capital of the business as at the beginning of the year.

Answer:
Rs. Rs.
Assets
Motor van (balance sheet valuation) 1,600
Inventory 410
Receivables 850
Cash in hand 50
Total assets 2,910
Liabilities
Bank overdraft 560
Trade payables 370
Payables for other expenses 90
Total liabilities 1,020
Net assets = Capital 1,890

Example 04:
Question: A sole trader has not maintained full records but is able to supply the following
information for two years ended 31 December:
2017 2016
Rs. 000 Rs. 000
Accrued expenses 10 -
Accounts receivable 196 130
Prepaid expenses - 16
Bank balances (40) 200
Investment 500 -
Cash balance 366 106
Accounts payable 74 90
Property 1,500 1,500
Delivery van 260 260
Inventory 190 74
Loan from bank 300 300

© Emile Woolf International 212 The Institute of Chartered Accountants of Pakistan


Chapter 5: Preparation of accounts from incomplete records

Further information:
(i) An allowance for doubtful debts should be established on 31 December 2017 in the amount
of Rs. 3,000.
(ii) Depreciation is to be provided on the carrying amounts as follows:
Property 5%
Delivery van 10%
(iii) Additional capital of Rs. 250,000 was introduced into the business during the year.
(iv) The owner withdrew a total sum of Rs. 20,000 during the year.
Required:
Calculate the profit of the business for the year ended 31 December 2017, using above data.

Answer:
The capital at the start of the year by preparing a statement of net assets at that date is as
under:
Net assets (capital) at the start of the year
Assets: Rs.000
Property 1,425
Delivery van 234
Inventory 74
Accounts receivable 130
Prepaid expenses 16
Bank balance 200
Cash balance 106
2,185
Liabilities:
Bank loan 300
Accounts payable 90
(390)
Net assets (capital) 1,795
The statement of net assets at the end of the year is as under:
Net assets (capital) at the end of the year
Assets: Rs.000
Property 1,354
Delivery van 211
Investment 500
Inventory 190
Accounts receivable 193
Cash balance 366
2,814
Liabilities:
Bank loan 300
Bank overdraft 40
Accounts payable 74
Accrued expenses 10
(424)
Net assets (capital) 2,390

© Emile Woolf International 213 The Institute of Chartered Accountants of Pakistan


Financial accounting and reporting I

The profit for the year is as under:

Profit for the year

Rs.000

Net assets (capital) at the year-end 2,390

Net assets (capital) at the start of the year (1,795)

Increase in net assets 595

Drawings 20

Capital introduced (250)

Profit for the year 365

2.2 Memorandum control accounts


A memorandum account is not a part of a proper ledger accounting system. When there are
incomplete records, a memorandum account can be used to calculate a ‘missing’ figure, such as
a figure for sales or purchases and expenses in the period.
Calculating a missing figure for sales
The records of a sole trader might be incomplete because the trader does not keep any record of
sales in the period. However, it might be possible to obtain the following figures:
 receivables at the beginning of the year (from last year’s balance sheet);
 receivables at the end of the year, from copies of unpaid sales invoices;
 money banked during the year (assumed to be money from customers for sales);
 Any bad debts written off.
Where a business makes some sales for cash, there might also be a figure for cash sales where
the money has not been banked. The amount of these cash sales might be calculated from the
sum of:
 the increase in cash in hand at the end of the year; plus
 Any expenses paid in cash, for which receipts are available.

Illustration 03:
An accountant is looking through the records of a sole trader who does not have a bookkeeping
system. He has established the following information:

Rs.

Receivables at the beginning of the year 650

Receivables at the end of the year 720

Bad debt written off during the year 800

Money paid into the business bank account 58,600

Cash sales where the money was not banked 300

© Emile Woolf International 214 The Institute of Chartered Accountants of Pakistan


Chapter 5: Preparation of accounts from incomplete records

The sales for the year can be calculated as the balancing figure in a receivables memorandum
account.

Receivables memorandum account


Rs. Rs.
Opening balance 650 Money banked 58,600
Sales 59,770 Cash sales, money not banked 300
(= balancing figure, 60,420 –
650)
Bad debt written off 800

Closing balance 720


60,420 60,420
The same calculation could be presented in a vertical format, as follows:
Rs.
Receivables at the beginning of the year (650)
Receivables at the end of the year 720
Increase/(decrease) in receivables 70
Money paid into the business bank account 58,600
Cash sales where the money was not banked 300
Bad debt written off during the year 800
Sales for the year 59,770

Illustration 04: The sales for the period from the following information can be calculated:

Rs.
Receivables at the start of the period 2,400
Receivables at the end of the period 1,800
Cash banked during the period 12,500
Bad debt written off 200
The sales for the year can be calculated as the balancing figure in a receivable memorandum
account.

Receivables memorandum account

Rs. Rs.

Opening balance 2,400 Money banked 12,500

Sales (bal fig) 12,100 Bad debt written off 200

Closing balance 1,800

14,500 14,500

© Emile Woolf International 215 The Institute of Chartered Accountants of Pakistan


Financial accounting and reporting I

Calculating a missing figure for purchase


A similar approach can be taken using knowledge of the payables control account.

Illustration 05:
An accountant is looking through the records of a sole trader who does not have a book-keeping
system. He has established the following information.

Rs.
Payables at the beginning of the year 1,200
Payables at the end of the year 1,800

Money paid out of the business bank account to suppliers 18,700


The purchases for the year can be calculated as the balancing figure in a payables memorandum
account.
Payables memorandum account
Rs. Rs.
Cash paid 18,700 Opening balance 1,200
Closing balance 1,800 Purchases (balancing figure) 19,300
20,500 20,500
The same calculation could be presented in a vertical format, as follows:
Rs.
Payables at the beginning of the year 1,200
Payables at the end of the year 1,800
Increase in payables 600
Money paid out of the business bank account 18,700
Purchases for the year 19,300

Illustration 06:
The purchases for the period from the following information can be calculated.

Rs.

Payables at the start of the period 1,400

Payables at the end of the period 1,900

Cash paid to suppliers during the period 11,300


The purchases can be calculated as a balancing figure in a payables memorandum account
Payables memorandum account
Rs. Rs.
Cash paid 11,300 Opening balance 1,400
Closing balance 1,900 Purchases (bal fig) 11,800
13,200 13,200

© Emile Woolf International 216 The Institute of Chartered Accountants of Pakistan


Chapter 5: Preparation of accounts from incomplete records

2.3 Memorandum cash and bank account


A memorandum account may also be used to record transactions in cash (notes and coins) and
through the bank account, in order to establish a missing figure for a cash payment or possibly a
cash receipt.
You might be given figures for:
 cash in hand and in the bank account at the beginning of the year;
 cash in hand and in the bank account at the end of the year;
 cash receipts (cash, cheques and other forms of receiving money);
 payments during the period for purchases, salaries and other cash expenses.
If there is a missing figure for a cash payment, this should emerge as a balancing figure.
Note: Cash in hand consists of banknotes and coins. Often, it is just petty cash. However, some
businesses hold a large amount of cash in hand because they sell goods for cash; for example,
retail stores may hold fairly large quantities of cash in hand.

Illustration 07:
An accountant is trying to prepare the financial statements of a sole trader from incomplete
records. A problem is that the owner of the business admits to having taken cash from the business,
but he has not kept a record of how much he has taken.
The accountant has established the following information:

Rs.
Cash in hand at the beginning of the year 200
Bank balance at the beginning of the year 2,300
Cash in hand at the end of the year 500
Bank balance at the end of the year 3,500
Receipts 42,800
Payments to employees 12,800
Payments to suppliers 17,100
Payments of interest/bank charges 400

On the basis of above information, the cash drawings by the owner during the year can be
calculated as the balancing figure in a cash and bank memorandum account as below.

Cash and bank memorandum account


Rs. Rs.
Opening balance, cash in
hand 200 Payments to suppliers 17,100
Opening balance, bank 2,300 Payments to employees 12,800
Receipts 42,800 Payments of interest/bank charges 400
Drawings (= balancing figure) 11,000
Closing balance, cash in hand 500
Closing balance, bank 3,500
45,300 45,300

© Emile Woolf International 217 The Institute of Chartered Accountants of Pakistan


Financial accounting and reporting I

The same calculation could be presented in a vertical format, as follows:


Rs. Rs.
Cash in hand and bank at the beginning of the year 2,500
Receipts during the year 42,800
45,300
Payments to suppliers 17,100
Payments to employees 12,800
Payments for interest/bank changes 400
Total payments recorded (30,300)
15,000
Cash in hand and bank at the end of the year (4,000)
Difference = missing figure = drawings 11,000

Illustration 08:
The drawings for the period from the following information can be calculated.

Rs.
Cash in hand at the beginning of the year 100
Bank balance at the beginning of the year 2,400
Cash in hand at the end of the year 150
Bank balance at the end of the year 5,200
Receipts 51,700
Payments to employees 3,400
Payments to suppliers 38,200
The drawing for the year can be calculated as a balancing figure in a cash and bank
memorandum account as below.

Cash and bank memorandum account


Rs. Rs.
Opening balance, cash in hand 100 Payments to suppliers 38,200
Opening balance, bank 2,400 Payments to employees 3,400
Receipts 51,700
Drawings (= balancing figure) 7,250
Closing balance, cash in hand 150
Closing balance, bank 5,200
54,200 54,200

2.4 Cost structures


Missing figures can sometimes be estimated by using cost structures which describe the
relationship that exists between sales, cost of sales and gross profit.
The relationship between revenue and cost of sales can be expressed as a percentage.

© Emile Woolf International 218 The Institute of Chartered Accountants of Pakistan


Chapter 5: Preparation of accounts from incomplete records

There are two ways of doing this:


 Gross profit is expressed as a percentage of cost of sales – this is known as mark-up; or
 Gross profit is expressed as a percentage of sales – this is known as profit margin.

Illustration 09: Cost structures


Rs. Mark-up Profit margin
Revenue 100,000 125% 100%
Cost of sales (80,000) 100% 80%
Gross profit 20,000 25% 20%

Illustration 10:
A sole trader does not keep a record of sales. However, she does keep a record of purchases. The
accountant has established that the gross profit margin is 20%, and that:
a) opening inventory was Rs. 700 at the beginning of the year
b) closing inventory is Rs. 1,200 at the end of the year
c) purchases during the year were Rs. 23,500.
Sales for the year can be calculated by first calculating the cost of sales figure and then adding the
mark up to it.
20% (= gross profit/sales), the mark-up on cost is 25% of cost (= 20/ (100 – 20)).
Rs.
Opening inventory 700
Purchases 23,500
24,200
Closing inventory (1,200)
Cost of sales 23,000
Gross profit (25% of cost) 5,750
Sales 28,750

Illustration 11:
A business operates on the basis of a mark-up of 40%.
The following information is available for the year.
Rs.
Opening inventory 3,100
Closing inventory 4,000
Purchases 42,100
The sales figure for the year can be calculated as under:
Rs.
Opening inventory 3,100
Purchases 42,100
Less: closing inventory (4,000)
Cost of sales 41,200
Mark-up at 40% 16,480
Sales (41,200  140%) 57,680
.

© Emile Woolf International 219 The Institute of Chartered Accountants of Pakistan


Financial accounting and reporting I

Example 05:
Question: In the following table there are missing values.
Rs. Rs. Rs. Rs.
Opening inventory 1,000 2,000 1,000 ?
Closing inventory (1,200) (1,500) (500) (2,000)
Purchases 5,000 8,700 ? 15,000
Sales 8,000 15,000 ? 20,000
Cost of sales ? ? ? ?
Gross profit ? ? ? 5,000
Profit margin ? ? 20% ?
Mark-up ? ? ? 33.3%
Required: From the information given, calculate the missing figures in the table
Answer:
Rs. Rs. Rs. Rs.
Sales 8,000 15,000 ? 20,000
Opening inventory 1,000 2,000 1,000 2,000
Purchases 5,000 8,700 7,500 15,000
6,000 10,700 8,500 17,000
Closing inventory (1,200) (1,500) (500) (2,000)
Cost of sales (4,800) (9,200) (8,000) (15,000)
Gross profit 3,200 5,800 2,000 5,000
Profit margin 40% 38.7% 20% 25%
Mark-up 66.7% 63% 25% 33.3%

More than one cost structure


A question might explain that a business has more than one cost structure.
You have to work carefully through the information to establish missing numbers.

Illustration 12: Multiple cost structures


A company has sales of Rs. 1,000.
The company sells three types of good.
60% of sales are of type A which is sold at a mark-up of 20%.
Type B goods are sold at a margin of 30%. The cost of type B sold in the year was Rs. 154.
Total gross profit for the year was Rs. 184.
To prepare sales, cost of sales and gross profit workings for each product and in total for the
business and show the margin for type C goods.

Step 1: Set up a table and enter the known facts

Type A % Type B % Type C % Total

Sales 600 100 1,000

Cost of sales 100 154

Gross profit 20 30 184

© Emile Woolf International 220 The Institute of Chartered Accountants of Pakistan


Chapter 5: Preparation of accounts from incomplete records

Step 2: Fill in the easy figures

Type A % Type B % Type C % Total

Sales 600 120 100 1,000

Cost of sales 100 154 70 816

Gross profit 20 30 184

Step 3: Apply the cost structures to calculate cost of sales and gross profits

Type A % Type B % Type C % Total


Sales 600 120 220 100 1,000
Cost of sales 500 100 154 70 816
Gross profit 100 20 66 30 184

Step 4: Complete the table

Type A % Type B % Type C % Total


Sales 600 120 220 100 180 100 1,000
Cost of sales 500 100 154 70 162 90 816
Gross profit 100 20 66 30 18 10 184

2.5 Missing inventory figure


The gross profit margin (or mark-up) can also be used to establish the value of inventory that is
missing or lost, for example due to theft or a fire. In these situations, you might know the value of
sales in the period, purchases during the period and opening and closing inventory.
By calculating the cost of sales from sales and the gross profit margin, it should be possible to
establish the value of missing inventory that is unaccounted for, as a balancing figure.

Illustration 13:
A sole trader operates his business from a warehouse, which has been damaged by a fire, which
occurred at the end of the financial year. After the fire, the remaining inventory that is undamaged
amounts to Rs. 2,000 (cost).
The accountant establishes the following information:
a) Inventory at the beginning of the year was Rs. 16,000
b) Purchases during the year were Rs. 115,000
c) Sales during the year were Rs. 140,000
d) The trader sells his goods at a mark-up of 25% of cost.
The cost of the inventory lost in the fire would be calculated is as under:
Gross profit = 25% of cost.
As a proportion of sales, gross profit = (25/ (25 + 100)) = 0.20 or 20%.
Sales = Rs. 140,000.
Therefore, gross profit = 20% × Rs. 140,000 = Rs. 28,000
Cost of sales = 80% × Rs. 140,000 = Rs. 112,000.

© Emile Woolf International 221 The Institute of Chartered Accountants of Pakistan


Financial accounting and reporting I

Rs.
Opening inventory 16,000
Purchases 115,000
131,000
Cost of sales (112,000)
Closing inventory should be 19,000
Actual closing inventory (2,000)
Balancing figure = inventory lost in the fire 17,000

Illustration 14:
A business operates on the basis of a mark-up on cost of 40%.
Additional information include:
Rs.
Opening inventory 50,100
Purchases 71,200
Sales 98,000
Based on the above information, closing inventory can be calculated as follows:
Rs. %
Sales 98,000 140
Cost of sales
Opening inventory 5,000
Purchases 71,200
Less: closing inventory (balancing figure) (6,200)
70,000 100
Working: Cost of sales = 100/140× Sales 100/140 × 98,000 = 70,000

Illustration 15:
A fire on 31 March destroyed some of the inventory of a company, and its inventory records were
also lost. The following information is available.
The company makes a standard gross profit margin of 30%.
Rs.
Inventory at 1 March 127,000
Purchases for March 253,000
Sales for March 351,000
Inventory in good condition at 31 March 76,000
The cost of the inventory lost in the fire can be calculated as under
Rs.
Inventory at 1 March 127,000
Purchases for March 253,000
380,000
Closing inventory (76,000)
Cost of sales + cost of lost inventory 304,000
Cost of sales (Rs. 351,000  70%) (245,700)
Inventory lost in the fire 58,300

© Emile Woolf International 222 The Institute of Chartered Accountants of Pakistan


Chapter 5: Preparation of accounts from incomplete records

Example 06:
Question: Rashid owns a shop which sells telephone recharge cards, making a mark-up of 25%.
He does not keep a cash book.
On 1 January 2017, the statement of financial position of his business was as follows:

Rs. 000
Net non-current assets 200.0
Current assets:
Inventory 100.0
Cash in bank 30.0
Cash in till 2.0
332.0
Financed by:
Capital 320.0
Trade payables 12.0
332.0

The bank statements for the year show the following:


Receipts Rs. 000
Banking of receipts 417.5
Payments
Trade payables 360.0
Sundry expenses 56.0
Drawings 44.0

Further information:
(i) There were no credit sales.
(ii) The following payments were also made in cash.
Rs. 000
Trade payables 8,000
Sundry expenses 15,000
Drawings 37,000

(iii) At 31 December 2017, the business had cash in the till of Rs. 4,500 and trade payables of
Rs. 14,000. The cash balance in the bank was not known and the value of closing inventory
has not yet been calculated. There were no accruals or prepayments. No further non-current
assets were purchased during the year. The depreciation charged for the year was Rs. 9,000.

Required:
Prepare the statement of profit or loss for the year ended 31 December 2017 and the statement
of financial position as at that date.

© Emile Woolf International 223 The Institute of Chartered Accountants of Pakistan


Financial accounting and reporting I

Answer:
Statement of profit or loss account for the year ended 31 December 2017
Rs.000
Sales 480.0
Cost of sales
Opening inventory 100.0
Purchases 370.0
Closing inventory 470.0
(86.0)

(384.0)
Gross profit 96.0
Less expenses
Sundry expenses (Rs. 15,000 + Rs. 56,000) 71.0
Depreciation 9.0
(80.0)
Net profit 16.0

Statement of financial position as at December 2017


Rs.000
Non-current assets (Rs. 200,000 – Rs. 9,000) 191.0
Current Assets:
Inventory 86.0
Cash(W1) 4.5
90.5
281.5
Financed By:
Capital at start of the year 320.0
Profit for the year 16.0
336.0
Less: Drawings (81.0)
Capital at end of the year 255.0
Current Liabilities :
Payables(W3) 14.0
Bank overdraft(W1) 12.5
26.5
281.5

© Emile Woolf International 224 The Institute of Chartered Accountants of Pakistan


Chapter 5: Preparation of accounts from incomplete records

W1 Cash book
Cash Bank Cash Bank
Rs. 000 Rs. 000 Rs. 000 Rs. 000
Balance b/d 2.0 30.0 Banking 417.5 
Receipts  417.5 Payables 8.0 360.0
Sales (cash) (W2) 480.0 Expenses 15.0 56.0
Balance c/d  12.5 Drawings 37.0 44.0
Balance c/d 4.5 
482.0 460.0 482.0 460.0

W2 Sales (proof)
Rs. 000
Receipts banked 417.5
Add:
Payments out of till 60.0
Closing cash balance 4.5
482.0
Less: Opening cash balance (2.0)
480.0

W3 Payables
Rs. 000 Rs. 000
Bank 360 Balance b/d 12
Balance c/d 8 Purchase (balance) 370
Balance c/d 14
382 382

W4 Mark-up and margin


Mark-up Margin
Sales 125 125
Cost (100) 100
Profit 25 25 25

Mark-up (25/100  100) 25%

Margin (25/125  100) 20%


.

© Emile Woolf International 225 The Institute of Chartered Accountants of Pakistan


Financial accounting and reporting I

Example 07: CLOSING INVENTORY


Question: A business makes all of its sales at a mark-up of 25%. During the year sales totalled Rs.
98,000 and purchases were Rs. 71,000. The inventory at the start of the year was valued at Rs.
10,200.

Required:
What was the value of the closing inventory at the end of the year?

Answer:
Rs.
Sales 98,000
Cost of sales (98,000  100/125) 78,400
Opening inventory 10,200
Purchases 71,000
81,200
Less: closing inventory (bal fig) (2,800)
Cost of sales 78,400

Example 08: SALES


Question: A business has the following assets and liabilities at the start and end of March.

1 March 31 March
Rs. Rs.
Trade receivables 6,100 7,400
Trade payables 3,900 3,500

The summarised bank statements for the year showed the following figures:
 Banking for the year were Rs. 78,500
 Payments to suppliers for the year were Rs. 49,700
 The owner banks her takings from the till each month but before doing so in March she took
Rs. 5,000 for her own use.

Required:
What are the sales for the year?

Answer:
For the calculation of sales, receivable control account would be used. The detailed calculation is
as under:
Receivables account
Rs. Rs.
Opening balance 6,100 Takings (78,500 + 5,000) 83,500
Sales (bal fig) 84,800 Closing balance 7,400
-------------------- --------------------
90,900 90,900
-------------------- --------------------

© Emile Woolf International 226 The Institute of Chartered Accountants of Pakistan


Chapter 5: Preparation of accounts from incomplete records

Example 09: CASH BALANCE


Question: An accountant has prepared the following list of the assets and liabilities of a business,
but has forgotten to enter the cash balance.
Rs.
Trade payables 4,900
Inventory 9,300
Non-current assets 98,900
Capital 97,200
Bank loan 15,700
Receivables 16,800
Bank ?

Required:
What is the missing figure for 'Bank'?

Answer:
The amount of bank overdraft is calculated by preparing balance sheet and the balancing figure is
bank over-draft. The detail calculation is as under:
Assets Rs. Rs.
Non-current assets 98,900
Inventory 9,300
Receivables 16,800
125,000
Liabilities
Capital 97,200
Bank loan 15,700
Trade payables 4,900
117,800
Assets exceed liabilities: therefore bank overdraft 7,200
.

Example 10: IRUM


Question: Irum is a sole trader. She does not keep a full set of accounting records but does keep
some records of transactions and documents. She has asked you to prepare her accounts for the
year ended 31 December 2015.
You have been given a list of the assets and liabilities of the business at the start and end of the
year.
Assets and liabilities

At 1 Jan 2015 At 31 Dec 2015


Rs.000 Rs.000
Trade receivables 5,500 6,100
Trade payables 2,800 3,500
Inventory 10,400 ?
Irum has no idea what her inventory value was at 31 December as that she did not count or value
her inventory at the year end.
She has also been given you a summary of her bank statements for the year.

© Emile Woolf International 227 The Institute of Chartered Accountants of Pakistan


Financial accounting and reporting I

Summary of bank statements

Receipts Payments
Rs.000 Rs.000
1 Jan Balance b/d 1,620 To suppliers 42,800
Bankings 65,400 For expenses 9,300
Living expenses 10,400
31 Dec Balance c/d 4,520
You have also been able to gather the following information from Irum:
i. Irum banks her takings from the till each week but before doing so pays Rs. 50,000 to her
employees and takes Rs. 30,000 herself. The business operates for 50 weeks each year.
ii. The till always has a cash float of Rs. 100,000.
iii. The sales of the business are both cash and credit sales and are all made at a mark-up of
40%.
Required:
(a) Calculate sales for the year.
(b) Calculate the value of the closing inventory at 31 December 2015.

Answer:
a)

Cash account
Rs.000 Rs.000
Opening balance 100 Bankings 65,400
Wages (50 x Rs.50,000) 2,500
Drawings (50 x Rs.30,000) 1,500
Cash takings (bal fig) 69,400 Closing balance 100
69,500 69,500

Total receivables account


Rs.000 Rs.000
Balance b/d 5,500 Cash takings 69,400
Sales (bal fig) 70,000 Closing balance 6,100
75,500 75,500
b)
Sales (part a) = Rs.70,000,000
Cost of sales = Rs.70,000,000 x 100/140 =Rs.50,000,000

Total payables account


Rs.000 Rs.000
Payments to suppliers 42,800 Opening balance 2,800
Closing balance 3,500 Purchases (bal fig) 43,500
46,300 46,300

© Emile Woolf International 228 The Institute of Chartered Accountants of Pakistan


Chapter 5: Preparation of accounts from incomplete records

Cost of sales
Rs.000
Opening inventory 10,400
Purchases 43,500
53,900
Less: closing inventory (balancing figure) (3,900)
Cost of sales (above) 50,000

Example 11: COST STRUCTURES


Question: Respond to following scenarios:
(a) A greengrocer made sales during the month of Rs. 49,200. Opening inventory amounted to
Rs. 3,784 and month-end inventory was Rs. 5,516. During the month he purchased for cash
goods which cost Rs. 38,632.
Required:
Determine the gross profit and calculate the gross profit percentage as a percentage of
sales value.
Answer:
Greengrocer % Rs. Rs.
Sales revenue 100 49,200
Less Cost of goods sold
Opening inventory 3,784
Purchases 38,632
42,416
Less Closing inventory (5,516)
(75) (36,900)
Gross profit 25 12,300

(b) A rival has made sales of Rs. 50,100 at a fixed mark-up of 25%. Closing inventory was
valued at Rs. 5,438 and he purchased goods during the month amounting to Rs. 38,326.
Required:
Determine the value of the opening inventory.
Answer:
Rival % Rs.
Sales revenue 125 50,100
100
Cost of goods sold (  Rs.50,100 = Rs.40,080) (100) (40,080)
125
Gross profit 25 10,020

Opening inventory (βal fig) 7,192


Purchases 38,326
45,518
Less Closing inventory (5,438)
Cost of goods sold 40,080

© Emile Woolf International 229 The Institute of Chartered Accountants of Pakistan


Financial accounting and reporting I

(c) A local store makes sales at a fixed gross profit of 10% on sales value. Sales during the
month amounted to Rs. 186,460; closing inventory was Rs. 16,800 and represents an
increase of 25% over the value of the opening inventory.
Required:
Determine the cost of purchases during the month.

Answer:
The cost of purchases during the month is based on cost of goods sold statements and balancing
figure would be purchases. The detailed calculation is as under;
Local store
% Rs.
Sales revenue 100 186,460
Cost of goods sold (90) (167,814)
Gross profit (10%  Rs.186,460) 10 18,646

100 13,440
Opening inventory (  Rs.16,800)
125
Purchases (βal fig) 171,174
184,614
Closing inventory (16,800)
Cost of goods sold 167,814

Example 12: TAHIR


Question: Tahir retired from his employment abroad and returned to this country, where he
purchased a small kiosk.
He took over the business on 1 July 2014, acquiring the existing inventory at a valuation of Rs.
1,142,000. The rest of the purchase price was apportioned as to Rs. 1,500,000 for fixtures and
fittings and the balance for goodwill.
The following day he acquired a second-hand computer and accounts package at a price of Rs.
80,000. Unfortunately, Tahir made an error when printing his year-end accounts causing him to lose
all data except for printed a summary listing of payments from the till. Other than this, the only
records available were his bank statements and a number of vouchers. Surplus cash was banked
during the year.
A summary of his bank account for the year ended 30 June 2015 shows the following.
Rs.000 Rs.000
Cash introduced 5,000 Purchase of business 3,192
Bankings from shop 16,427 Purchase of accounts computer 80
Loan from mother (long-term) Rent (15 months to
(interest at 5% pa) 1,000 30 September 2015) 500
Rates (9 months to
31 March 2015) 84
Electricity 92
Purchases for resale 14,700
Private cheques 1,122
Balance 30 June 2015 2,657
22,427 22,427

© Emile Woolf International 230 The Institute of Chartered Accountants of Pakistan


Chapter 5: Preparation of accounts from incomplete records

The computer print-out was as follows.

Rs.000

Cash purchases for resale 1,606

Staff wages 742

Sundry shop expenses 156

Cash drawings 520

On 30 June 2015 inventory, measured at cost, amounted to Rs. 1,542,000, amounts due from
customers Rs. 74,000, and cash in hand amounted to Rs. 54,000. Depreciation is to be recognised
on fixtures and fittings at a rate of 10%.
Accounts outstanding on 30 June 2015 were purchases of Rs. 470,000 and rates of Rs. 120,000
for the year ended 31 March 2016.
Required:
Prepare Tahir’s statement of comprehensive income for the year ended 30 June 2015 and a
statement of financial position at that date.

Answer:
Statement of comprehensive income for the year ended 30 June 2015

Rs.000 Rs.000
Revenue (74 + 16,427 + 3,024 + 54) 19,579
Opening inventory 1,142
Purchases (14,700 + 1,606 + 470) 16,776
17,918
Closing inventory (1,542)
(16,376)
Gross profit 3,203
Less Expenses
Rent (500 – 100) 400
Rates (84 + 30) 114
Electricity 92
Wages 742
Sundry expenses 156

Depreciation (10%  Rs. 1,580,000) 158

Loan interest (5%  Rs. 1,000,000) 50

(1,712)
Net profit 1,491

© Emile Woolf International 231 The Institute of Chartered Accountants of Pakistan


Financial accounting and reporting I

Statement of financial position at 30 June 2015


Rs.000 Rs.000
Non-current assets
Intangible – Goodwill (3,192 – 1,500 – 1,142) 550
Tangible – Fixtures and fittings (1,500 + 80 – 158) 1,422
1,972
Current assets
Inventory 1,542
Receivables 74
Prepaid rent 100
Bank 2,657
Cash in hand 54
4,427
6,399

Capital account
Capital introduced 5,000
Profit for the year 1,491
6,491
Drawings (1,122 + 520) (1,642)
4,849
Non-current liability
Loan 1,000

Current liabilities
Trade payables 470
Accrued expenses (30 + 50) 80
550
Total capital and liabilities 6,399

Example 13: IJAZ


Question:
Ijaz is in business but does not keep proper books of account. In order to prepare his income and
expenditure account for the year ended 31 December 2015 you are given the following information.

1 Jan 2015 31 Dec 2015


Rs.000 Rs.000
Inventory on hand 1,310 1,623
Receivables 268 412
Payables for goods 712 914
Payables for expenses 116 103
In addition, you are able to prepare the following summary of his cash and bank transactions for the
year.

© Emile Woolf International 232 The Institute of Chartered Accountants of Pakistan


Chapter 5: Preparation of accounts from incomplete records

Cash account
Rs.000 Rs.000
Balance 1 January 62 Payments into bank 3,050
Shop takings 4,317 Purchases 316
Cheques cashed 200 Expenses 584
Drawings 600
Balance 31 December 29
4,579 4,579

Bank account
Rs.000 Rs.000
Balance 1 January 840 Cash withdrawn 200
Cheques from customers 1,416 Purchases 2,715
Cash paid in 3,050 Expenses 519
Drawings 400
Delivery van (purchased
1 September) 900
Balance 31 December 572
5,306 5,306

In addition, Ijaz says that he had taken goods for personal consumption and estimates that those
goods cost Rs. 100,000.
In considering accounts receivable Ijaz suggests that a provision is to be made of 5% of amounts
due after writing off a specific bad debt of Rs. 30,000.
Depreciation on the delivery van is to be recognised at 20% per annum.
Required:
Prepare the statement of comprehensive income and a statement of financial position at 31
December 2015.

Answer:
Statement of comprehensive income for the year ended 31 December 2015
Rs.000 Rs.000
Revenue (W2) 5,877
Opening inventory 1,310
Add Purchases (W3) 3,133
4,443
Less: Closing inventory (1,623)
(2,820)
Gross profit 3,057
Expenses (W4) 1,090
Bad debts (W6) 49
Depreciation (W7) 60
(1,199)
Net profit 1,858

© Emile Woolf International 233 The Institute of Chartered Accountants of Pakistan


Financial accounting and reporting I

Statement of financial position at 31 December 2015


Rs.000 Rs.000 Rs.000
Non-current asset
Delivery van, at cost 900
Less: Depreciation (W7) (60)
840
Current assets
Inventory 1,623
Receivables 382
Less: Provision for doubtful debts (W6) (19)
363
Cash at bank 572
Cash in hand 29 601
2,587
Total assets 3,427

Capital account
At 1 January 2015 (W1) 1,652
Add: Profit for year 1,858
3,510
Less: Drawings (W5) (1,100)
2,410
Current liabilities
Trade payables 914
Accrued expenses 103
1,017
Total capital and liabilities 3,427

Workings
(1) Opening statement of affairs

Rs.000
Inventory 1,310
Receivables 268
Cash 62
Bank 840
————
2,480
Less: Payables (712 + 116) (828)
————
Capital at 1 January 2015 1,652
————

© Emile Woolf International 234 The Institute of Chartered Accountants of Pakistan


Chapter 5: Preparation of accounts from incomplete records

(2) Total sales (receivables) a/c

Rs.000 Rs.000

Balance b/d 268 Cheques receipts


from customers 1,416
Revenue for year (Bal fig) 5,877 Bad debt written off 30
Cash takings 4,317
Receivables c/d (412 – 30) 382
———— ————
6,145 6,145
———— ————

Balance b/d 382

(3) Total purchases (payables) a/c

Rs.000 Rs.000

Cash 316 Balance b/d 712


Bank 2,715 Drawings 100
Balance c/d 914 Purchases for year (Bal fig) 3,133
———— ————
3,945 3,945
———— ————

Balance b/d 914

(4) Expenses

Rs.000 Rs.000

Cash 584 Balance b/d 116


Bank 519 P & L a/c 1,090
Balance c/d 103
———— ————
1,206 1,206
———— ————

Balance b/d 103

© Emile Woolf International 235 The Institute of Chartered Accountants of Pakistan


Financial accounting and reporting I

(5) Drawings

Rs.000 Rs.000
Purchases 100
Cash a/c 600
Bank a/c 400 Balance c/d (or trf capital) 1,100
———— ————
1,100 1,100
———— ————

(6) Bad debts a/c

Rs.000 Rs.000
Bad debt (w/off receivables) 30 P & L a/c 49
Provision for doubtful debts
a/c (5%  382) (increase) 19
——— ———
49 49
——— ———
(7) Depreciation
20%  900,000 4/12 = Rs.60,000

Example 14: RASHID


Question:
Rashid is coming to the end of his first year’s trading. He has not kept proper books and records.
The following information relates to the year ended 30 September 2015.
(1) He set up in business when he won Rs. 200,000,000 on football pools. He invested the
money in the bank and set up in business as a retailer of clothing.
(2) He banks his takings periodically after payment of the following amounts.
Wages Rs. 75,000 per week
Cleaning Rs. 10,000 per week
Sundries Rs. 15,000 per week
Personal Expense Rs. 25,000 per week
Cash in hand at the end of the year was Rs. 250,000.
(3) A summary of his bank statements reveals the following.
Rs.000 Rs.000
Capital introduced 200,000 Purchase of leasehold premises 150,000
Bankings 125,750 Purchase of vans 6,000
Telephone 896
Rent and rates 1,682
Payments to suppliers 86,232
Wages 15,282
Repairs 3,637
Personal expenses 323
Balance c/d 61,698
325,750 325,750

© Emile Woolf International 236 The Institute of Chartered Accountants of Pakistan


Chapter 5: Preparation of accounts from incomplete records

An un-presented cheque of Rs. 385,000 for repairs was still outstanding.


(4) Other assets and liabilities at 30 September 2015 were as follows.
Rs.000
Inventory 8,400
Trade receivables 10,350
Trade payables 29,957
Accrued expense – telephone 125
Prepaid expense– rent and rates 258
(5) Depreciation is to be recognized on the van at 25% of its cost. The lease on the premises is
for 50 years.
(6) Rashid estimates that his gross profit percentage is 25% on sale price, and also informs you
that he does not keep a record of the goods he took for his own use.
Required:
Prepare a statement of comprehensive income for the year ended 30 September 2015 and a
statement of financial position at that date.

Answer:
Statement of comprehensive income for the year ended 30 September 2015

% Rs.000 Rs.000

Revenue (W8) 100 142,850

Opening inventory –

Purchases (balancing figure) 115,538

Closing inventory (8,400)

Cost of sales (75) (107,138)

Gross profit 25 35,712

Expenses

Cleaning 520

Sundries 780

Depreciation

Van 1,500

Leasehold premises 3,000

Telephone (W4) 1,021

Wages (W3) 19,182

Rent and rates (W5) 1,424

Repairs (W7) 4,022 (31,449)

Net profit 34,263

© Emile Woolf International 237 The Institute of Chartered Accountants of Pakistan


Financial accounting and reporting I

Statement of financial position at 30 September 2015


Cost Depn
Rs.000 Rs.000 Rs.000
Non-current assets
Leasehold premises 150,000 3,000 147,000
Van 6,000 1,500 4,500
156,000 4,500 151,500
Current assets
Inventory 8,400
Trade receivables 10,350
Prepayment (W5) 258
Cash at bank 61,313
Cash in hand 250
80,571
Total assets 232,071

Cost Depn
Rs.000 Rs.000 Rs.000
Capital account
Capital introduced 200,000
Add Net profit 4,263
204,263
Less Drawings (W2) (2,274)
201,989
Current liabilities
Trade payables 29,957
Accrued expenses (W4) 125
30,082
Total capital and liabilities 232,071
Workings
(1) Cash

Rs.000 Rs.000

Balance b/d Nil Wages (75  52) 3,900


Total sales (Bal fig) 132,500 Cleaning (10  52) 520
Sundries (15  52) 780
Drawings (25  52) 1,300
Bank 125,750
Balance c/d 250
—————— ——————
132,500 132,500
—————— ——————

© Emile Woolf International 238 The Institute of Chartered Accountants of Pakistan


Chapter 5: Preparation of accounts from incomplete records

(2) Drawings

Rs.000 Rs.000

Cash 1,300 Balance c/d (or trf capital) 2,274


Bank 323
Total purchases (W6) 651
————— —————
2,274 2,274
————— —————

(3) Wages

Rs.000 Rs.000

Cash 3,900 P & L a/c 19,182


Bank 15,282
————— —————
19,182 19,182
————— —————

(4) Telephone

Rs.000 Rs.000

Bank 896 P & L a/c 1,021


Balance c/d 125
————— —————
1,021 1,021
————— —————

(5) Rent and rates

Rs.000 Rs.000

Bank 1,682 P & L a/c 1,424


Balance c/d 258
————— —————
1,682 1,682
————— —————

(6) Total purchases (payables)

Rs.000 Rs.000

Bank 86,232 Trading a/c 115,538


Balance c/d 29,957 Goods for own use (Bal fig) 651
—————— ——————
116,189 116,189
—————— ——————

© Emile Woolf International 239 The Institute of Chartered Accountants of Pakistan


Financial accounting and reporting I

(7) Repairs

Rs.000 Rs.000

Bank 3,637 P & L a/c 4,022


Bank 385
————— —————
4,022 4,022
————— —————

(8) Total sales (receivables)

Rs.000 Rs.000

Trading a/c (Bal fig) 142,850 Cash (W1) 132,500


Balance c/d 10,350
—————— ——————
142,850 142,850
—————— ——————

Example 15: MUDASSAR


Question:
Mudassar had retired from the army some years ago to run a grocery business in the country. On 1
October 2015 his assistant failed to report for work and it was later discovered that he had
disappeared taking the contents of the cash till with him.
An analysis of Mudassar’s bank statements for the year ended 31 December 2015 revealed the
following.
Rs.000 Rs.000
Balance b/f 280 Suppliers 13,600
Tax refund 1,000 Rent 800
Bankings 16,720 Rates 400
Insurance 200
Drawings 2,500
Bank charges 100
Balance c/f 400
18,000 18,000
A statement of affairs produced by Mudassar comprised the following.
31 December
2015 2014
Rs.000 Rs.000
Motor car (NBV) 3,200 3,600
Fixtures (NBV) 3,400 4,000
Inventory 1,200 900
Trade receivables 150 90
Rent prepaid 30 20
Cash Nil 380
Trade payable 120 110

© Emile Woolf International 240 The Institute of Chartered Accountants of Pakistan


Chapter 5: Preparation of accounts from incomplete records

A rough cash book kept by Mudassar showed the following.


Rs.000
Assistant’s wages 1,800
Sundry expenses 250
Cash purchases 300
Drawings 2,400
Cash received from customers 21,550
A footnote recorded that discounts received and discounts allowed were Rs. 200,000 and Rs.
300,000 respectively.
The insurance company agreed to admit the claim for loss of cash upon production of a full set of
accounts.
Required:
Prepare a statement of comprehensive income for the year ended 31 December 2015 and a
statement of financial position at that date.

Answer:
Statement of comprehensive income for the year ended 31 December 2015

Rs.000 Rs.000
Revenue (W4) 21,910
Cost of sales
Opening inventory 900
Purchases (W3) 14,110

15,010
Closing inventory (1,200)
(13,810)
Gross profit 8,100
Less Expenditure
Rent (800 + 20 – 30) 790
Rates 400
Insurance 200
Bank charges 100
Assistant’s wages 1,800
Discounts (net) (300 – 200) 100
Sundry expenses 250
Depreciation
Car 400
Fixtures 600 (4,640)
Net profit 3,460

© Emile Woolf International 241 The Institute of Chartered Accountants of Pakistan


Financial accounting and reporting I

Statement of financial position at 31 December 2015


Rs.000 Rs.000
Non-current assets
Motor car 3,200
Fixtures 3,400
6,600
Current assets
Inventory 1,200
Trade receivables and prepayments (150 + 30) 180
Insurance company claim (W2) 460
Bank 400 2,240
Total assets 8,840

Capital account b/f (W1) 9,160


Add: Capital introduced 1,000
10,160
Profit for the period 3,460
13,620
Less: Drawings (2,500 + 2,400) 4,900
8,720
Current liabilities
Trade payables 120
Total capital and liabilities 8,840

Workings
(1) Statement of affairs as at 31 December 2014
Rs.000

Motor car 3,600


Fixtures 4,000
Inventory 900
Receivables 90
Prepayments 20
Bank 280
Cash 380
—————
9,270
Less: Trade payables (110)
—————
9,160
—————

© Emile Woolf International 242 The Institute of Chartered Accountants of Pakistan


Chapter 5: Preparation of accounts from incomplete records

(2) Cash a/c


Rs.000 Rs.000
Balance b/f 380 Wages 1,800
Customers’ receipts 21,550 Sundry expenses 250
Purchases 300
Drawings 2,400
Bankings 16,720
Defalcation (theft) 460
————— —————
21,930 21,930
————— —————

(3) Total purchases (payables)


Rs.000 Rs.000
Cash 300 Balance b/f 110
Bank 13,600 Purchases 14,110
Discounts received 200
Balance c/d 120
————— —————
14,220 14,220
————— —————

(4) Total sales (receivables)


Rs.000 Rs.000
Balance b/f 90 Receipts 21,550
Sales 21,910 Discounts allowed 300
Balance c/f 150
————— —————
22,000 22,000
————— —————

Example 16: ASLAM


Question:
Aslam, who has been in business as a contractor since 1 January 2015 received a request from the
tax authorities for his first year’s accounts.
He had not kept proper records of his business transactions, but was able to supply the following
information.
(1) All cheques received for work done had been paid into the bank, whilst cash receipts had been
used for paying cash expenses.
(2) From bundles of receipts and a wages notebook some of the cash expenses for the year
appeared to have been as follows.
Rs.000
Wages and Social Security 3,346
Materials 1,400
Electricity 56
General expenses 14

© Emile Woolf International 243 The Institute of Chartered Accountants of Pakistan


Financial accounting and reporting I

(3) Drawings were estimated at Rs. 18,000 per week, out of which Aslam had paid the rent of
his builder’s yard of Rs. 2,000 per week. His own Social Security contributions had been
included in Wages and Social Security and totaled Rs. 65,000 for the year.
(4) On 1 April he purchased a van for Rs. 856,000. His mother lent him Rs. 400,000 for the
deposit, and the balance was payable by twelve monthly instalments of Rs. 38,000 each
commencing on 1 June. The loan from his mother had not been repaid at the end of the year.
(5) A summary of his bank account showed the following.

Rs.000 Rs.000
Balance 1 January 2015 150 Materials 4,790
Bankings 9,204 Van expenses 342
General expenses 110
Cheques drawn for cash 3,100
Cement mixer 200
Van instalments 266
Private cheques 342
Balance 31 December 2015 204
9,354 9,354

(6) On 31 December 2015 inventory (materials) amounted to Rs. 560,000, cash in hand Rs.
10,000, trade receivables Rs. 1,200,000, trade payables for materials Rs. 149,000, and
outstanding van expenses Rs. 36,000. There was no work in progress on 31 December
2015.
(7) Depreciation of Rs. 108,000 is to be recognized on the van and Rs. 50,000 on the cement
mixer.
Required:
Prepare Aslam’s statement of comprehensive income for the year ended 31 December 2015 and
a statement of financial position at that date.

Answer:
Statement of comprehensive income for the year ended 31 December 2015
Rs.000 Rs.000 Rs.000
Work done (= Revenue) (W3) 13,066
Direct expenses
Materials (W2) 5,779
Wages and Social Security (3,346 – 65) 3,281 9,060
Van expenses
Running costs (342 + 36) 378
Depreciation 108 486
Miscellaneous expenses
Electricity 56
Depreciation of cement mixer 50
Rent 104
General expenses (14 + 110) 124 334
(9,880)
Net profit for the year 3,186

© Emile Woolf International 244 The Institute of Chartered Accountants of Pakistan


Chapter 5: Preparation of accounts from incomplete records

Statement of financial position at 31 December 2015


Rs.000 Rs.000 Rs.000
Non-current assets Cost Depn
Van 856 108 748
Cement mixer 200 50 150
1,056 158 898
Current assets
Inventory 560
Trade receivables 1,200
Balance at bank 204
Cash in hand (W1) 10
1,974
Total assets 2,872
Capital account
Capital introduced 150
Add: Profit for the year 3,186
3,336
Less: Drawings (936 – 104 + 65 + 342) (1,239)
2,097
Non-current liability
Loan account – mother 400
Current liabilities
Trade payables 149
Accrued expenses 36
Van instalments (5  38) 190
375
2,872

Workings
(1) Cash a/c

Rs.000 Rs.000

Bank a/c (cash from bank) 3,100 Bank a/c (bankings) 9,204
Work done a/c (Bal fig = Wages a/c 3,281
takings) 11,866 Drawing a/c (private NIC) 65
Materials a/c 1,400
Electricity a/c 56
General expenses a/c 14
Drawings a/c (52  16) 832
Rent a/c 104
Balance c/d (cash in hand) 10
————— —————
14,966 14,966
————— —————

© Emile Woolf International 245 The Institute of Chartered Accountants of Pakistan


Financial accounting and reporting I

(2) Materials a/c

Rs.000 Rs.000
Cash a/c 1,400 Balance c/d (inventory) 560
Bank a/c 4,790 P & L a/c 5,779
Balance c/d (liability) 149
————— —————
6,339 6,339
————— —————
(3) Work done a/c

Rs.000 Rs.000
P & L a/c 13,066 Cash a/c 11,866
Balance c/d 1,200
————— —————
13,066 13,066
————— —————

Example 17: UMAR


Question:
Umar is a grocer who had not kept a full set of books. The following was a summary of his bank
statements for the year ended 31 December 2015.
Rs.000 Rs.000
Amounts credited by bank 35,170 Balance 1 January 2015 892
Payments for trade payables 30,500
Rent and rates 475
Fixtures 100
Lighting and heating 210
General expenses 800
Loan interest 120
Drawings 900
Customers’ cheques dishonored 180
Balance 31 December 2015 993
35,170 35,170
Additional information
(1) Trading receipts consisted partly of cash and partly of cheques. During the year Umar had
paid out of his cash takings, wages amounting to Rs. 2,950,000 and sundry expenditure of
Rs. 140,000. He retained Rs. 3,000 a week and maintained a balance of Rs. 20,000 in the
till for change. The balance of his takings, together with cheques amounting to Rs. 250,000,
which he had cashed out of his takings for the convenience of certain friends, was paid into
the bank.
(2) Cheques drawn payable to trade payables, but not presented at 1 January 2015, amounted
to Rs. 280,000 and at 31 December 2015 to Rs. 320,000.
(3) All dishonored cheques were re-presented and honored during the year.

© Emile Woolf International 246 The Institute of Chartered Accountants of Pakistan


Chapter 5: Preparation of accounts from incomplete records

(4) The loan interest was paid to Brough who had lent Umar Rs. 4,000,000 some years ago at a
rate of interest of 3% per annum. The interest was duly paid half-yearly on 31 March and 30
September, and the loan was still outstanding at the end of the year.
(5) Discounts allowed by suppliers amounted to Rs. 480,000 and those allowed to customers
were Rs. 520,000.

(6) 1 Jan 2015 31 Dec 2015


Rs.000 Rs.000
Inventories 4,500 5,800
(including a bad debt of Rs.200,000 to be written off) 2,800 3,200
Accrued general expenses 240 190
Rates paid in advance 40 50
Fixtures (including those purchased during year)valued at 2,800 2,550
Trade payables 1,800 2,200
Amounts due for lighting and heating 80 70
Required:
Prepare
(a) a statement of Umar’s capital at 1 January 2015
(b) a statement of comprehensive income for the year ended 31 December 2015
(c) a statement of financial position at 31 December 2015.

Answer:
(a) Capital at 1 January 2015
Assets Liabilities
Rs.000 Rs.000
Operating overdraft 1,172
Cash in till 20
Inventories 4,500
Trade receivables 2,800
Brough’s loan
Principal 4,000
Accrued interest (40000  3% 3/12) 30
Accrued general expenses 240
Rates in advance 40
Fixtures 2,800
Trade payables 1,800
Accrued light and heat 80
10,160 7,322
(7,322)
Net assets – Capital account 2,838

© Emile Woolf International 247 The Institute of Chartered Accountants of Pakistan


Financial accounting and reporting I

(b) Statement of comprehensive income for the year ended 31 December 2015
Rs.000 Rs.000 Rs.000
Revenue (W2) 39,156
Opening inventory 4,500
Purchases (W3) 31,420
35,920
Less: Closing inventory (5,800)
(30,120)
Gross profit 9,036
Administrative expenses
Rent and rates (475 + 40 – 50) 465
Light and heat (210 – 80 + 70) 200
Wages 2,950
Depreciation of fixtures (2,880 + 100 – 2,550) 350
Sundry expenses (140 + 800 – 240 + 190) 890 4,855
Financing costs
Loan interest 120
Bad debt 200
Discounts (net) (520 – 480) 40 360
(5,215)
Net profit for the year 3,821

(c) Statement of financial position at 31 December 2015


Rs.000 Rs.000
ASSETS
Non-current assets
Tangible assets at cost less depreciation
Fixtures 2,550
Current assets
Inventories 5,800
Trade receivables 3,000
Prepayment 50
Cash (993 – 320 + 20) 693 9,543
Total assets 12,093

CAPITAL AND LIABILITIES


Capital account
At 1 January 2015 (per (a)) 2,838
Add Net profit for the year (per (b)) 3,821
6,659
Less Drawings (156 + 900) (1,056)
5,603
Non-current liabilities
Loan – Brough 4,000
Current liabilities
Trade and other payables (2,200 + 190 + 70 + 30) 2,490
Total capital and liabilities 12,093

© Emile Woolf International 248 The Institute of Chartered Accountants of Pakistan


Chapter 5: Preparation of accounts from incomplete records

Workings
(1) Cash and bank a/cs

Cash Bank Cash Bank


Rs.000 Rs.000 Rs.000 Rs.000
Balance b/f 20 Balance b/f 1,172
Bank (cheques cashed) 250 Purchases 30,540
Cash (bankings) 35,170 Rent and rates 475
Sales (cash takings) 38,416 Fixtures 100
Light and heat 210
General expenses 140 800
Loan interest 120
Drawings (52  3) 156 900
Sales (dishonoured
cheques) 180
Wages 2,950
Bank
Cheques cashed 250
Bankings 35,170
Balance c/f 20 673
————— ————— ————— —————
38,686 35,170 38,686 35,170
————— ————— ————— —————

(2) Sales (or total receivables) a/c

Rs.000 Rs.000

Balance b/f 2,800 Cash a/c (takings) 38,416


Bank a/c Discounts allowed a/c 520
(dishonoured cheques) 180 Bad debts a/c 200
Trading a/c (al fig) 39,156 Balance c/f 3,000
————— —————
42,136 42,136
————— —————

(3) Purchases (or total payables) a/c

Rs.000 Rs.000

Bank a/c 30,540 Balance b/f 1,800


Discounts received a/c 480 Trading a/c (al fig) 31,420
Balance c/f 2,200
————— —————
33,220 33,220
————— —————

© Emile Woolf International 249 The Institute of Chartered Accountants of Pakistan


Financial accounting and reporting I

Example18: YASIN
Question:
Yasin received a legacy of Rs. 20,000,000 on 1 January 2015 and on that date purchased a
small retail business. The completion statement from the solicitor revealed the following.
Rs.000
Freehold shop property 10,000
Goodwill 2,000
Inventories 1,600
Trade receivables 400
Shop fixtures 2,600
Rates in advance to 31 March 2015 100
16,700
The legacy was used to discharge the amount due on completion and the balance was paid into a
newly opened business bank account.
Yasin had not kept proper records of his business transactions but was able to supply the following
information.
(1) A summary of the cash till rolls showed his shop takings for the year to be Rs. 25,505,000;
this includes all cash received from customers including those at 1 January 2015.
(2) The takings had been paid periodically into the bank after payment of the following cash
expenses.

Rs.000
Wrapping materials 525
Staff wages 3,423
Purchases for resale 165
Petrol and oil 236
(3) Personal cash drawings were estimated at Rs. 20,000 per week and goods taken for own
use at Rs. 2,000 per week.
(4) A summary of the bank statements showed the following.
Rs.000 Rs.000
Legacy – residual balance 3,300 Purchases for resale 14,863
Sale of fixtures purchased Motor expenses 728
at 1 January 2015 but not required Delivery van (cost – 1 April 1,200
(cost Rs.200,000;depreciation Nil) 130 2015)
Loan from Robin at 10% pa 2,000 General expenses 625
Cash banked 19,900 Loan interest (six months to
30 September) 100
Private cheques 1,329
Electricity 228
Rates (year to 31 March 500
2016)
Balance per statement at
31 December 2015 5,757
25,330 25,330

A cheque drawn on 28 December 2015 of Rs. 125,000 for goods purchased was presented to the
bank on 4 January 2016.

© Emile Woolf International 250 The Institute of Chartered Accountants of Pakistan


Chapter 5: Preparation of accounts from incomplete records

(5) During the year bad debts of Rs. 223,000 arose and were irrecoverable. The trade
receivables at 31 December 2015 amounted to Rs. 637,000, of which Rs. 100,000 is
doubtful and for which an allowance should be recognized should be made.
(6) At 31 December 2015 there were
Rs.000
Inventories 2,360
Store of wrapping materials 53
Trade payables – purchases 358
Electricity accrued 50
Accountancy fees accrued 100
Cash float in till 180
(7) The difference arising on the cash account was discussed with Yasin but remained
unexplained and was dealt with in an appropriate manner.
(8) Depreciation is to be recognized at the rate of 10% per annum on the fixtures and at the
rate of 20% on the van.
Required:
Prepare a statement of comprehensive income for the year ended 31 December 2015 and a
statement of financial position at that date.

Answer:
Statement of comprehensive income for the year ended 31 December 2015
Rs.000 Rs.000
Revenue (W1) 25,965
Opening inventory 1,600
Purchases (15,346 (W2) + 165 – 104) 15,407
—————
17,007
Less Closing inventory (2,360)
—————
(14,647)
—————
Gross profit 11,318
Expenses
Selling and distribution costs
Wages 3,423
Wrapping materials (525 – 53) 472
Motor expenses (728 + 236) 964
Bad debts (223 + 100) 323
Depreciation of van (1,200  20% 9/12) 180
—————
(5,362)
Administrative expenses
Rates (500 – 125 + 100) 475
General expenses 625
Electricity (228 + 50) 278
Depreciation of fixtures (2,600 – 200)  10% 240
Loss on disposal of fixtures (200 – 130) 70
Loan interest (100 + 50) 150
Accountancy costs 100
————— (1,938)
—————
Net profit for the year 4,018
—————

© Emile Woolf International 251 The Institute of Chartered Accountants of Pakistan


Financial accounting and reporting I

Statement of financial position at 31 December 2015

Cost Depn
ASSETS Rs.000 Rs.000 Rs.000
Non-current assets
Property, plant and equipment
Freehold property 10,000 – 10,000
Fixtures (2,600 – 200) 2,400 240 2,160
Delivery van 1,200 180 1,020
————— ————— —————
15,600 420 13,180
————— —————
Goodwill 2,000 – 2,000

Current assets
Inventories 2,360
Trade receivables (637 – 100) 537
Prepayments (125 + 53) 178
Cash (5,757 – 125) + 180 5,812
—————
8,887
—————
Total assets 24,067
—————

CAPITAL AND LIABILITIES Rs.000 Rs.000


Capital
Capital at 1 January 2014 20,000
Net profit for the year 4,018
—————
24,018
Drawings (1,040 + 104 + 1,329 + 36 (W3)) (2,509)
—————
21,509

Non-current liabilities
Loan 2,000
Current liabilities
Trade and other payables (358 + (50 + 50 + 100)) 558
—————
Total capital and liabilities 24,067
—————

© Emile Woolf International 252 The Institute of Chartered Accountants of Pakistan


Chapter 5: Preparation of accounts from incomplete records

Workings
(1) Trade receivables control a/c

Rs.000 Rs.000
Balance b/f 400 Cash received 25,505
Sales (βal) 25,965 Bad debt 223
Balance c/f 637
————— —————
26,365 26,365
————— —————
Balance b/f 637

(2) Trade payables control a/c

Rs.000 Rs.000
Bank 14,863 Credit purchases (Bal) 15,346
Bank (unpresented cheque) 125
Balance c/f 358
————— —————
15,346 15,346
————— —————

Balance b/f 358

(3) Cash a/c

Rs.000 Rs.000
Cash received 25,505 Wrapping materials 525
Staff wages 3,423
Purchases for resale 165
Petrol and oil 236
Drawings (20  52) 1,040
Cash banked 19,900
Balance c/f 180
Difference (drawings) (Bal) 36
————— —————
25,505 25,505
————— —————

Example 19: MUNIRA


Question:
Munira is engaged in trading of garments. She has not maintained proper accounting records.
She suspects that some of her employees are involved in some sort of misappropriation of
inventory. The list of creditors, receivables and inventories prepared by her, show the following
balances:

Balances at December 31
2015 2014
Rs. 000 Rs. 000
Trade payables 9,500 8,000
Trade receivables 3,600 2,000
Inventories at cost 8,500 12,500

© Emile Woolf International 253 The Institute of Chartered Accountants of Pakistan


Financial accounting and reporting I

The following transactions were recorded during the year ended December 31, 2015:

(Rs. 000s)
Sales to staff on cash basis 315
Discounts allowed on early payments 360
Collections banked 18,000
Paid to suppliers in cash 12,700
Trade discounts received 400
Bad debts written off 200
Additional information
(i) Normal sales are made at cost plus 20% but sales to staff are made at cost plus 5%.
(ii) About 4% of the purchases during the year were defective and had to be sold at 30% below
normal selling price.
(iii) The list of closing inventory at December 31, 2015 includes four items having a total cost
of Rs. 470,000. There was a casting error on the invoice raised by the supplier and the total
has been erroneously recorded as Rs. 740,000. The invoice is still unpaid.
(iv) Collections made in the last week of December 2015 amounting to Rs. 860,000 were
deposited in bank on January 2, 2016. Likewise, collections made in the last week of
December 2014 amounting to Rs. 500,000 were deposited in bank on January 2, 2015.
Required:
calculate the loss incurred by Munira during the year 2015 on account of misappropriations (if
any).

Answer:
(i) Purchases ledger control account
Rs. 000 Rs. 000
Cash 12,700 Bal b/f 8,000
Trade discount Received Purchases (Credit)
(not to be booked) 0 Balancing figure 13,930
Bal c/f (9,500 + 470 – 740) 9,230
21,930 21,930

(ii) Sales ledger control account


Rs. 000 Rs. 000
Balance b/f 2,000 Bank (18,000–500+860) 18,360
Credit sales
(balancing figure) 20,520 Discount allowed 360
Bad debts 200
Balance c/d 3,600
22,520 22,520

© Emile Woolf International 254 The Institute of Chartered Accountants of Pakistan


Chapter 5: Preparation of accounts from incomplete records

(iii) Cost of Sales as per record Rs. 000


Opening Inventory 12,500
Add: Purchases (W-1) 13,930
Goods available for sale 26,430
Less: Closing Inventory 8,500
Cost of sales – as per record 17,930
(iv) Cost of sales on mark-up basis
Sales
Description (Rs.000)
Total normal and old inventory sales (as per (ii) above): 20,520
Sale of defective inventories:
Cost (4% of 13,930 = 557)
Normal sale price (557 x 1.20) = 668
Sales at 70% of 668 (468)
20,052
Sales to staff 315
Cost of sales at Standard Mark-up
Staff (315 100/105) 300
Cost of defective inventories (4% of 13,930) 557
Normal sales (20,052100/120) 16,710
17,567
Less: Cost of sales as per record (see (iii) above) (17,930)
Shortage of inventory 363

Example 20: ADNAN


Question:
Adnan runs a wholesale business. On December 31, 2015 he realised that his cash and bank
balances have reduced considerably. He has requested you to investigate the situation and has
provided you the following information:
(i) Balances
2015 2014
Rs. Rs.
Cash in hand 700 14,300
Cash at bank 103,400 349,100
Sundry receivables 80,900 48,700
Inventory 27,500 15,700
Sundry creditors 130,800 116,100
Rent payable (one month) 4,500 3,500
Electricity and telephone bills payable 8,800 -

© Emile Woolf International 255 The Institute of Chartered Accountants of Pakistan


Financial accounting and reporting I

(ii) 20% of the goods were sold on cash basis at a mark-up of 22% on cost. Credit sales were
made at a profit of 20% on sales. All collections from receivables were made in cash.
(iii) Adnan paid wages, rent, electricity and telephone charges in cash out of sale proceeds.
The remaining amount of sale proceeds was deposited into bank.
(iv) The bank pass book reveals the following withdrawals:

Rupees

Creditors 1,423,800

Non-current assets (acquired on July 1, 2015) 75,000

Drawings 122,600
(v) All purchases were made on credit.
(vi) Wages amounted to Rs. 8,900 per month.
(vii) Payment on account of electricity and telephone charges amounted to Rs. 33,000.
(viii) Rent has been increased from October 2015.
(ix) The opening balance in the non-current assets account net of depreciation was Rs.
285,000. Depreciation is recorded @ 10% p.a. on declining balance method and is based
on number of months for which the assets have been in use.
Required:
(a) Prepare Adnan’s profit and loss account for the year ended December 31, 2015 and his
statements of financial position as on that date.
(b) Compute the amount of cash shortage, if any.

Answer:
Adnan: Statement of comprehensive income for the year ended December 31, 2015

Rs. Rs.
Sales (W) 1,774,815
Opening inventory 15,700
Purchases (130,800+1,423,800 – 116,100) 1,438,500
1,454,200
Closing inventory (27,500)
1,426,700
Gross profit 348,115
Less expenses:
Wages 106,800
Rent (3,500×9) + (4,500×3) 45,000
Electricity & telephone (33,0000+8,800) 41,800
Depreciation (285,000×0.1)+(75,000×0.1×6/12) 32,250
(225,850)
Net profit 122,265

© Emile Woolf International 256 The Institute of Chartered Accountants of Pakistan


Chapter 5: Preparation of accounts from incomplete records

Adnan: Statement of financial position as at 31 December

Assets 2015 2014


Rs. Rs.
(see note)
Non-current assets (285,000 + 75,000 – 32,250) 327,750
285,000
Inventory 27,500 15,700
Receivables 80,900 48,700
Bank 103,400 349,100
Cash 700 14,300
540,250 712,800

Capital and liabilities


Capital Account (Bal. Figure) 396,150 593,200
Sundry payables 130,800 116,100
Outstanding expenses:
Rent 4,500 3,500
Electricity & Telephone 8,800 -
540,250 712,800
Note: The statement of financial position for 2014 is not a part of the requirement, but
has been prepared for computing opening balance of capital.

Cash shortage Rs.


Opening Capital on December 31, 2014 593,200
Profit for the year 122,265
Less: Drawings (122,600)
Capital on December 31, 2015 396,150
Cash shortage (196,715)
Working
Computation of sales
Cost of sales (15,700 + 1,438,500 – 27,500) 1,426,700

Sales
Cost of cash sales (20% of 1,426,700) 285,340
Adjusting for mark up  1.22
Cash sales 348,115

Cost of credit sales (80% of 1,426,700) 1,141,360


Adjusting for margin 100/80
Credit sales 1,426,700
Total 1,774,815

© Emile Woolf International 257 The Institute of Chartered Accountants of Pakistan


Financial accounting and reporting I

Example 21: ASIF


Question:
Due to the death of his book-keeper, Asif failed to keep proper records for the year ended June 30,
2015. He has forwarded to you the following statements:
Statement of financial position as at June 30, 2014
Rs. Rs.
Land and building at cost 130,000
Furniture: Cost 825,000
Depreciation (485,000) 340,000
Inventory 482,500
Trade receivables: 670,000
Less: Provision (27,000) 643,000
Prepayments 53,800
Cash in hand 10,000
1,659,300

Asif-capital account 613,300


6% Loan 500,000
Trade creditors 500,100
Accrued expenses 21,700
Bank overdraft 24,200
1,659,300

Summary of the transactions in the bank book for the year ended June 30, 2015

Receipts Rs. Payments Rs.


Deposits against cash sales 624,750 Creditors 2,509,600
Receipts from receivables 3,071,000 Sundry expenses 212,500
Furniture sold on 1-Jul-14 Salaries 440,400
(purchased for Rs. 122,400 Furniture purchased on 01-Jan- 15 64,000
280,000 on 1- Jul-11)
Interest on loan up to 31-Mar-15 22,500
Total 3,818,150 Total 3,249,000
You have carried out the necessary scrutiny and ascertained the following:
(i) Asif sells the goods at a profit margin of one-third of their selling price i.e. at a profit margin
of 50% of cost of sales.
(ii) On June 30, 2015 trade receivables aggregated Rs. 600,500. These included Rs. 18,000
pertaining to goods which were sent on sale or return basis and were unsold on June 30.
(iii) Closing inventory was valued at Rs. 580,000.
(iv) Receipts from receivables include an advance of Rs. 2,500 for goods delivered in July 2015.
(v) Rs. 3,700 were recovered from a debtor which had been fully provided for on June 30, 2014.
A new customer who was introduced in 2015 and owed Rs. 4,200 was declared as bankrupt.
(vi) Sundry expenses payable on June 30, 2015 amounted to Rs. 19,000 (excluding interest on
loan) whereas prepayments amounted to Rs. 9,700.
(vii) Asif estimates that he withdrew Rs. 60,000 for his personal use and paid sundry expenses
aggregating Rs. 25,000 before depositing the proceeds from cash sales.

© Emile Woolf International 258 The Institute of Chartered Accountants of Pakistan


Chapter 5: Preparation of accounts from incomplete records

(viii) Depreciation on furniture is provided at the rate of 10% per annum on cost.
(ix) Bonus is payable to the manager at 5% of the net profit after charging such bonus.
(x) The following account balances were obtained from the memorandum records:

Rs.
Purchases 2,570,000
Discounts received 30,300
Sales returns 15,000
Required:
(a) A statement of comprehensive income of Mr. Asif for the year ended June 30, 2015; and
(b) A statement of financial position as on June 30, 2015

Answer:
Mr. Asif: Statement of comprehensive income for the year ended June 30, 2015

Rs. Rs.
Cash sales (W4) 709,750
Credit sales (W6) 2,996,000
Less: Returns (15,000)
3,690,750
Cost of goods sold:
Opening inventory 482,500
Add: Purchases 2,570,000
3,052,500
Less: Closing inventory (including inventory at cost on sale or
return basis) (W1) (592,000)
2,460,500
Gross profit 1,230,250
Discounts received 30,300
1,260,550
Less expenses
Salaries 440,400
Trade expenses
(212,500+19,000+53,800–21,700 – 9,700+25,000) 278,900
Interest on loan (6% of 500,000) 30,000
Provision for doubtful debts (4,200 – 3,700) 500
Loss on sale of furniture (W2) 73,600
Depreciation for year (W3) 57,700
(881,100)
379,450
Commission- (5/105 of 379,450) (18,069)
Net profit for the year 361,381

© Emile Woolf International 259 The Institute of Chartered Accountants of Pakistan


Financial accounting and reporting I

Statement of financial positionas at June 30, 2015

Non-current assets Rs. Rs.


Land and building at cost 130,000
Furniture
Cost: (825-280+64) 609,000
Less: Depreciation(485-84+57.7) (458,700)
150,300
280,300

Current assets Rs. Rs.


Inventory (580,000 + 12,000) 592,000
Receivables 582,500
Less: Provision (27,000 + 500) (27,500)
555,000
Prepayments 9,700
Bank balance (3,818,150-24,200-3,249,000) 544,950
Cash in hand (W5) 10,000
1,711,650
1,991,950

Capital and liabilities


Asif-capital on July 1 613,300
Add: Net profit 361,381
974,681
Less: Drawings (60,000)
914,681
Non-current liabilities
6% Loan on mortgage 500,000
Add: Accrued interest (30,000–22,500) 7,500
507,500
Current liabilities
Trade payables (W7) 530,200
Accrued expense 19,000
Advance from customer 2,500
Due to manager 18,069
569,769
1,991,950

© Emile Woolf International 260 The Institute of Chartered Accountants of Pakistan


Chapter 5: Preparation of accounts from incomplete records

Workings:
1. Closing inventory Rs.
Inventory on premises 580,000
Add: Inventory with customers on sale or return (Rs.18,000/1.2) 12,000
592,000

2. Non-current assets disposal


Cost of furniture 280,000
Less: Three years depreciation at 10% p.a. on cost: 3×28,000 84,000
Book value on July 1, 2014 196,000
Less: Sales proceeds 122,400
Loss on sale 73,600

3. Depreciation
(i) On Rs.825,000-Rs.280,000 = Rs.545,000 at 10% p.a. 54,500
(ii) On Rs.64,000 at 10% p.a. for six months 3,200
57,700

4. Cash Sales
Cost of goods sold 2,460,500
1/3 of selling price i.e. 33-1/3% of selling price = 50% of cost
Therefore, total sales would be (2,460,500×1.5) 3,690,750
Less: Credit sales – net (2,996,000 – returns 15,000) 2,981,000
Cash sales 709,750

5. Cash on hand
Opening Cash balance 10,000
Cash sales 709,750
719,750
Total of cash sales banked (624,750)
Drawings (60,000)
Sundry expenses (25,000)
Closing cash balance 10,000

6. Credit sales & receivables


Total Receivables Account
Rs. Rs.
Balance, 1st January, Receipt 3,071,000
2014 670,000
Advance receipts (Cr.) 2,500 Sales Return 15,000
Credit sales (balancing Balance, June 30, 2015 582,500
figure) 2,996,000
(Rs. 600,500-18,000)
3,668,500 3,668,500

© Emile Woolf International 261 The Institute of Chartered Accountants of Pakistan


Financial accounting and reporting I

7. Payables
Total Payables Account
Rs. Rs.
Discounts 30,300 Balance, July 1, 2014 500,100
Bank 2,509,600 Purchases 2,570,000
By balance, June 30, 2015
(bal. fig) 530,200
3,070,100 3,070,100

Example 22: MANSOOR


Question:
Mansoor deals in small electrical equipment and appliances. His Statements of financial position
for the year ended 30 June 2014 was as follows.

Assets Rupees
Fixtures 235,000
Inventories 552,000
Receivables 281,000
Property tax paid in advance 11,500
Cash in hand 35,000
Cash at bank 307,500
1,422,000

Capital and Liabilities Rupees


Capital 1,185,000
Liabilities:
Goods 220,000
Electricity charges 5,500
Accounting charges 11,500 237,000

1,422,000
On 30 June 2015, there was a fire in his shop which destroyed all his fixtures and inventories. The
following information has been gathered from the records available with him.
a) The insurance company agreed to pay Rs. 225,000 for fixtures and Rs. 630,000 for inventory
without production of accounts; the inventory on hand was however Rs. 670,000.
b) The payments made during the year were as follows:
Rupees Rupees
Personal expenses 188,000 Property tax 32,000
Sundry expenses 15,000 Rent 240,500
Accounting charges 20,500 Purchase of goods 5,061,000
Electricity 50,500 Fixtures 45,000
c) The following payments were made during the year, out of cash receipts:
(i) Assistant's salary Rs. 132,000.
(ii) Cash purchases averaging Rs. 24,000 per month.
(iii) Drawings which varied between Rs. 10,000 and Rs. 15,000 per month.

© Emile Woolf International 262 The Institute of Chartered Accountants of Pakistan


Chapter 5: Preparation of accounts from incomplete records

All other receipts were deposited into the bank. Total deposits amounted to Rs. 5,780,800 and
included scrap sale of Rs. 35,000.
d) The following balances as on 30 June 2015 were determined from the available records:

Assets and Liabilities Rupees


Receivables 494,000
Creditors for goods 212,000
Creditors for electricity charges 1,900
Accounting charges payable 1,800
Rent outstanding 15,000
Property tax paid in advance 15,000
Cash in hand 40,500
e) Included in the receivables is an amount of Rs. 14,000 which is considered uncollectible.
f) The rate of gross profit as a percentage of sale was 20%.
Required:
Prepare the statement of comprehensive income for the year ended 30 June 2015 and a statement
of financial position as on that date.

Answer:
Mansoor: Statement of comprehensive income for the year ended June 30, 2015
Rs. Rs.
Sales 125% of (552,000 + 5,341,000 - 670,000) 6,528,750

Opening inventory 552,000


Purchases (288,000+5,053,000) 5,341,000
5,893,000
Closing inventory (Destroyed in fire) (670,000)
(5,223,000)
Gross profit b/d 1,305,750
Scrap sales 35,000
1,340,750
Less: expenses
Assistant's salary 132,000
Electricity (50,500 + 1,900 - 5,500) 46,900
Rent (240,500 + 15,000) 255,500
Property tax (32,000 +11,500 -15,000) 28,500
Accounting charges (20,500 + 1,800 – 11,500) 10,800
Sundry expenses 15,000
(488,700)
Loss from fire:
Fixtures (235,000 + 45,000 – 225,000) 55,000
Inventories (670,000 – 630,000) 40,000
Provision for bad debt 14,000
(109,000)
Net profit 743,050

© Emile Woolf International 263 The Institute of Chartered Accountants of Pakistan


Financial accounting and reporting I

Mansoor: Statement of financial position as at 30 June 2015


Assets Rs.
Receivables (494,000 – 14,000) 480,000
Receivable from Insurance Co. 855,000
Property tax paid in advance 15,000
Cash in hand 40,500
Cash at bank 435,800
1,826,300

Capital and liabilities Rs. Rs.


Capital 1,185,000
Net profit 743,050
Drawings (144,450 + 188,000) (332,450)
1,595,600
Payables:
For goods 212,000
Electricity 1,900
Accounting charges 1,800
Rent 15,000
230,700
1,826,300
Workings:
Payables
Bank 5,061,000 Opening balance 220,000
Purchases
Closing balance 212,000 (Balancing figure) 5,053,000

5,273,000 5,273,000

Receivables
Opening balance 281,000 Cash (Balancing figure) 6,315,750
Sales 6,528,750 Closing balance 494,000
6,809,750 6,809,750

© Emile Woolf International 264 The Institute of Chartered Accountants of Pakistan


Chapter 5: Preparation of accounts from incomplete records

Amount in Rupees
Cash and Bank
Cash Bank Cash Bank
Opening
balance 35,000 307,500 Assistant's salary 132,000
Receivables 6,315,750 Purchases 288,000
Scrap sales 35,000 Drawings-bal figure 144,450
Cash 5,780,800 Bank 5,780,800
Drawings 188,000
Sundry expenses 15,000
Accounting charges 20,500
Electricity 50,500
Property tax 32,000
Rent 240,500
Payables 5,061,000
Fixtures 45,000
Closing balance 40,500 435,800
6,385,750 6,088,300 6,385,750 6,088,300

Example 23: DANISH


Question:
Danish does not keep proper books of account due to his lack of knowledge of double entry system
of accounting. He has supplied you the following information with respect to the year ended 31
December 2013 from the records kept in his diary:
(i) Transactions during the year:

Rupees
Cash received from customers 80,000
Discount allowed to customers 1,400
Bad debts written off 1,800
Cash paid to suppliers 63,000
Discount allowed by suppliers 1,000
Sales returns 3,000
Purchases returns 2,000
Expenses paid 6,000
Drawings 5,000
Rent paid 2,500

© Emile Woolf International 265 The Institute of Chartered Accountants of Pakistan


Financial accounting and reporting I

(ii) Opening balances as on 1 January 2013:

Assets and liabilities Rupees


Receivables 45,000
Payables 24,000
Cash 4,500
Furniture and fixtures 15,000
Inventory 25,000
Motor van 16,000

(iii) Receivables and payables as on 31 December 2013 amounted to Rs. 48,600 and Rs.
27,000respectively.
(iv) Outstanding expenses as on 31 December 2013 amounted to Rs. 1,200.
(v) Depreciation is charged on furniture and fixtures at the rate of 10% and on motor van
at20%.
(vi) Danish sells goods at cost plus 40% and follows a policy of maintaining an allowance of
5% of the outstanding receivables.
Required:
(a) Statement of comprehensive income for the year ended 31 December 2013.
(b) Statement of financial position as at 31 December 2013.

Answer:
Statement of comprehensive income for the year ended 31 December 2013
Rupees Rupees
Opening inventory 25,000 Sales (W - 3) 89,800
Purchases (W - 2) 69,000 less: Returns (3,000) 86,800
Less returns (2,000) 67,000 Closing Inventory (W - 5) 30,000
Gross profit c/d 24,800
116,800 116,800

Discount allowed 1,400 Gross profit b/d 24,800


Bad debts* 4,230 Discount received 1,000
Expenses
(6,000+1,200) 7,200
Rent 2,500
Depreciation
Furniture 1,500
Motor Van 3,200
Net profit 5,770
25,800 25,800
*1800+(48,600 × 5%)

© Emile Woolf International 266 The Institute of Chartered Accountants of Pakistan


Chapter 5: Preparation of accounts from incomplete records

Statement of financial position as at 31 December 2013


Rupees Rupees
Liabilities and Capital Assets
Capital (W - 1) 81,500 Motor Van 16,000
Less: drawings (5,000) Less: Depreciation (3,200) 12,800
Add: net profit 5,770 82,270 Furniture and fixtures 15,000
Less: Depreciation (1,500) 13,500
Closing Inventory 30,000
Payables (W-2) 27,000 Receivables (W-3) 48,600
Accrued expenses 1,200 Less: allowance (2,430) 46,170
Cash (W - 4) 8,000
110,470 110,470

W-1 Opening Capital Rupees


Assets
Motor Van 16,000
Furniture and fixture 15,000
Inventory 25,000
Receivables 45,000
Cash 4,500
105,500
Liabilities
Payables (24,000)

Opening Capital 81,500

W -2 Payables control account


Rupees Rupees
Cash paid 63,000 Balance b/d 24,000
Discount received 1,000 Purchases (bal. fig) 69,000
Purchases return 2,000
Balance c/d 27,000
93,000 93,000

W-3 Receivables control account


Rupees Rupees
Balance b/d 45,000 Cash received 80,000
Sales (bal fig) 89,800 Discount allowed 1,400
Bad debts 1,800
Sales return 3,000
Balance c/d 48,600
134,800 134,800

© Emile Woolf International 267 The Institute of Chartered Accountants of Pakistan


Financial accounting and reporting I

W-4 Cash
Rupees Rupees
Balance b/d 4,500 Payments to suppliers 63,000
Receipts from customers 80,000 Expenses paid 6,000
Drawings 5,000
Rent paid 2,500
Balance c/d 8,000
84,500 84,500

W-5 Calculation of closing inventory


Net Sales 86,800

Net purchases 67,000


Opening Inventory 25,000
92,000
Less: cost of goods sold (100/140 of net sales) 62,000
Closing Inventory 30,000

Example 24: RAHIL


Question:
Rahil runs a retail business. He appointed a cashier at a monthly salary of Rs. 13,000 on 1 April
2016. The cashier did not report for work on 1 July 2016 and it was found that he had left, taking with
him the balance in the till.
It had been Rahil's practice to deposit on each weekend the available balance in the till after retaining
a float of Rs. 5,000. He maintains record of sales on credit and a file of unpaid invoices in respect of
goods purchased by him.
The following information has been ascertained from the available records:
i. Balance Sheet as on 31 March 2016 was as follows:
Rupees Rupees
Rahil’s capital 233,000 Fixtures and fittings – WDV 161,000
Creditors for goods 159,000 Inventory 111,000
Creditors for expenses 16,000 Debtors 55,000
Cash at bank 76,000
Cash in hand 5,000
408,000 408,000
Following is a summary of the bank statement from 1 April to 30 June 2016:
Rupees Rupees
Balance on 1 April 2016 76,000 Payment to suppliers for 604,000
goods
Cheques received from 29,000 Rent & other expenses 37,000
customers
Cash deposited 627,000 Balance on 30 June 2016 91,000
732,000 732,000

© Emile Woolf International 268 The Institute of Chartered Accountants of Pakistan


Chapter 5: Preparation of accounts from incomplete records

ii. The following amounts were paid from the till:

Rs. per month


Salary to cashier 13,000
Rahil’s drawings 26,000
Petty expenses 5,000
iii. Fixtures and fittings are depreciated at 10% per annum using reducing balance method.
iv. Inventory on 1 July 2016 was Rs. 58,000.
v. Credit sales during the quarter ended 30 June 2016 amounted to Rs. 64,000 whereas the
debtors’ balances as on 30 June 2016 amounted to Rs. 66,000. However, direct confirmations
from debtors showed that receivables in fact totaled Rs. 54,000.
vi. Creditors for goods and expenses had always been paid by cheques. Unpaid invoices for goods
on 30 June 2016 totaled Rs. 181,000 and creditors for expenses amounted to Rs. 13,000.
Detailed scrutiny of records revealed that a cash receipt of Rs. 8,000 which had been received
against goods returned to a supplier had not been recorded.
vii. Rahil sells goods at a gross profit margin of 20% on sales.
Required:
(a) Prepare a statement showing calculation of the amount of defalcation.
(b) Prepare a statement of financial position as on 30 June 2016.

Answer:
(a) Statement of amount of defalcation

Rupees

Cash in hand 30/3/16 5,000

Cash sales [838,750(W-1)-64,000] 774,750

Receipts from customers as per debtor’s 24,000


account (W-3)

Less: Cash utilized

Bank lodgments (627,000)

Assistant's salary (13,000×3) (39,000)

Petty expenses (5,000×3) (15,000)

Drawings (26,000×3) (78,000)

(759,000)

Defalcation against cash sales 44,750

Difference in debtors balance 12,000

Defalcation from amount received from supplier against purchase return 8,000

Total defalcation amount 64,750

© Emile Woolf International 269 The Institute of Chartered Accountants of Pakistan


Financial accounting and reporting I

Working Notes:
W-1: Sales Rupees
Stock on 1 April 2016 111,000
Add: Purchases for 3 months (626,000 – 8000) (W-2) 618,000
729,000
Less: Stock on 30 June 2016 (58,000)
671,000
Add: Gross profit 20% on sales (671,000 × 20÷80) 167,750
Total sales 838,750

W-2: Purchases Rupees


Cash paid to creditors against goods supplied 604,000
Add: Creditors on 30 June 2016 181,000
785,000
Less: Creditors on 31 March 2016 (159,000)
Cash received for returns (8,000)
Total purchases 618,000

W-3: Cash received from credit customers Rupees


Balance on 1 April 2016 55,000
Add: Credit sales for 3 months 64,000
119,000
Less: Balances outstanding on 30 June 2016 as per books (66,000)
53,000
Less: Receipts (cheques) shown in cash book (29,000)
Amount presumably received in cash 24,000

(b) Statement of financial position of Rahil


As on 30 June 2016

Liabilities Rs. Assets Rs.


Sundry creditors 181,000 Fixtures and fittings (net) 156,975
Expenses owing 13,000 Stock in trade 58,000
Capital: Sundry debtors 54,000
Balance on 1 April 2016 233,000 Balance at bank 91,000
Add: Net profit (W-1) 10,975
Less: Drawings (78,000)
165,975
359,975 359,975

© Emile Woolf International 270 The Institute of Chartered Accountants of Pakistan


Chapter 5: Preparation of accounts from incomplete records

W-1: Net profit for 3 months Rupees


Gross profit - 20% on sales (671,000 × 20÷80) 167,750
Less:
Depreciation (161,000×10%×3÷12) 4,025
Assistant's salary and petty cash expenses 54,000
Rent & other expenses 37,000
Decrease in creditors for expenses (16,000 - 13,000) (3,000)
Loss due to defalcation 64,750
(156,775)
10,975

Example 25: S-MART GROCERY STORE


Question:
Saleem is the owner of S-Mart, a grocery store. His accountant resigned and left on 1 January 2017.
Saleem suspects that the previous accountant was involved in some sort of misappropriation. The
information available with him is as follows:
(i) Summary of bank statement:
Receipts Rupees Payments Rupees
Balance as at 1 Jan 2016 250,000 Suppliers 1,807,500
Cheques from debtors 824,000 Salaries 48,000
Cash sales 1,450,000 Rent 72,000
Sale of old vehicle on 1 Jan 2016 15,000 Utilities 36,000
Other expenses 24,750
New vehicle on 1 Mar 230,000
2016
Balance as at 31 Dec 320,750
2016
2,539,000 2,539,000

(ii) Other balances extracted from the records maintained by the previous accountant:
31-Dec-2016 31-Dec-2015
Particulars ---------- Rupees ----------
Furniture and fixtures – WDV 555,000 550,000
Equipment – WDV 64,000 80,000
Vehicle – WDV 210,000 18,500
Inventory 215,000 250,000
Debtors 340,000 260,000
Advance rent - 3,000
Cash in hand 31,510 45,000
Creditors 354,500 100,000
Salaries payable 22,000 18,000

© Emile Woolf International 271 The Institute of Chartered Accountants of Pakistan


Financial accounting and reporting I

(iii) Before depositing the receipts from cash sales in the bank, Saleem took Rs. 12,000 per month
for personal use. All other payments were made through bank and the debtors settled their
accounts through cheques.
(iv) The creditors have confirmed the balances due from them. However, review of the statement
provided by one of the creditors indicates that goods returned for cash amounting to Rs.
24,000 were not recorded in the books.
(v) Unpaid invoice for furniture purchased during the year for Rs. 45,000 is included in creditors.
(vi) The margin on cash sales and credit sales is 20% and 25% respectively. From 1 July 2016,
prices to cash customers were further reduced by 6% due to which quantity sold against cash
in the 2nd half of the year increased by 25% as compared to the first half of the year.
(vii) All the debtors confirmed their balances except an amount of Rs. 50,000. On investigation it
was found that the related goods had been issued against fake invoices.
Required:
(a) Determine the amount of suspected fraud.
(b) Prepare statement of profit or loss for the year ended 31 December 2016.

Answer:
Loss due to defalcation Rupees

Cash embezzled through purchase returns 24,000

Stock embezzled through fake debtors (500,000×0.75) 37,500

Cash defalcated from cash sales (W-1) 50,740

112,240

W-1: Cash account

Opening balance 45,000

Add: Cash sales 1,631,250

Less: Drawings (144,000)

Payment into bank (1,450,000)

Closing balance of cash (31,510)

Cash defalcated 50,740

Statement of profit or loss ------------ Rupees ------------

Total cash sales (1,687,500 -56,250) (W-2) 1,631,250

Total credit sales (W-4) 854,000

2,485,250

© Emile Woolf International 272 The Institute of Chartered Accountants of Pakistan


Chapter 5: Preparation of accounts from incomplete records

------------ Rupees ------------

Less: Cost of goods sold

Opening inventory 250,000

Purchases (W-3) 2,017,000

Less: Return outward (Alternatives are available) (24,000)

Stock misappropriated (37,500)

Less: Closing stock (215,000) (1,990,500)

Gross profit 494,750

Less: Operating expenses

Rent expenses (72,000+3,000) 75,000

Utilities 36,000

Other expenses 24,750

Loss on sale of vehicle (15,000–18,500) 3,500

Salaries expense (48,000–18,000+22,000) 52,000

Loss due to defalcation 112,240

Depreciation: Furniture (550,000+45,000–555,000) 40,000

Equipment (80,000–64,000) 16,000

Vehicle (230,000–210,000) 20,000 (379,490)

Net profit 115,260

W-2: Total cash sales


Cost Gross sales Discount (6%)
Period Ratio
----------------- Rupees -----------------
First six month 1 600,000 750,000 -
Second six months 1.25 750,000 937,500 56,250
Total (W-5)1,350,000 1,687,500 56,250

W-3: Creditors
Closing balance (354,500–45,000) 309,500
Add: Payment during the year 1,807,500
2,117,000
Less: Opening balance (100,000)
Purchases during the year 2,017,000

© Emile Woolf International 273 The Institute of Chartered Accountants of Pakistan


Financial accounting and reporting I

W-4: Debtors

Closing balance (340,000–50,000) 290,000

Add: Receipts during the year 824,000

1,114,000

Less: Opening balance (260,000)

Credit sales 854,000

Cost of goods sold credit sales (854,000×75%) 640,500

W-5: Cost of goods sold

Total Cost of goods sold (P&L) 1,990,500

Less: Cost of credit sales (W-3) (640,500)

Cost of cash sales 1,350,000

Example 26: Friday Traders


Question: Friday Traders (FT) is engaged in the business of supplying Blenders and Juicers. FT
purchases its products from Sigma Electronics. FT is presently negotiating with a bank for a long-term
loan and has been asked to provide the latest financial statements. Since FT does not maintain proper
accounting records, you are requested to prepare the financial statements from the following
information:
(i) Assets and liabilities as on 1 January 2018:

Rs. in '000

Equipment (40% depreciated) 2,490


Stock (stock value of Blenders was double of the Juicers) 3,705
Prepaid rent up to 30 April 2018 280
Trade debtors (only for Blenders) 1,410
Payable to Sigma Electronics 3,600
Salaries payable 98
Bank overdraft 740

(ii) Sales of Blenders are made on credit while Juicers are sold on cash basis.
(iii) Upto last year, FT was earning a gross profit of 30% on cost of Blenders and 35% on
sale value of Juicers. With effect from 1 January 2018:
 FT increased sales prices of both the products by 20%; and
 Sigma Electronics increased the prices of Juicers only by 40%.
(iv) 60% of the amount of purchases made during the year represents blenders.

© Emile Woolf International 274 The Institute of Chartered Accountants of Pakistan


Chapter 5: Preparation of accounts from incomplete records

(v) Summary of bank transactions during the year:


Rs. in '000
Receipts from credit customers 6,570
Payments:
Sigma Electronics 8,850
Insurance for one year starting 1 February 2018 204
Rent 826
Equipment 550
Salaries and wages 685
Total payments 11,115

(vi) Debtors amounting to Rs. 138,000 are considered as irrecoverable.


(vii) Rent of the premises was increased by 30% with effect from 1 September 2018.
(viii) Following payments were made from cash sales and remaining amounts were deposited into
the bank:
Rs. in '000
Repairs and maintenance 186
Salaries and wages 124
Drawings 477
787

(ix) Equipment is depreciated at 8% on cost.


(x) Some balances ascertained as at 31 December 2018:

Rs. in '000
Stock* – Juicers 975
– Blenders 2,597
Payable to Sigma Electronics 2,420
Salaries payable 134
*Comprises of stock purchased in 2018
Required:
(a) Prepare statement of profit or loss for the year ended 31 December 2018.
(b) Prepare statement of financial position as at 31 December 2018.
Answer:
Part (a)
Statement of profit or loss for the year ended 31 December 2018
Juicers Blenders
-------- Rs. in '000 --------
Sales
Credit sales – Blenders 4,475×156÷100(W-1) 6,981
Cash sales - Juicers Opening stock: 1,235×120÷65(W-1) 2,280
Rem.: 2,093(3,328-1,235)×120÷91(W-1) 2,760
5,040 6,981

© Emile Woolf International 275 The Institute of Chartered Accountants of Pakistan


Financial accounting and reporting I

Cost of goods sold:


Opening stock 1,235 2,470
Purchases 7,670(W-3) in 40:60 3,068 4,602
Closing stock (975) (2,597)
(3,328) (4,475)
Gross profit 1,712 2,506
Total gross profit 4,218

Operating expenses:
Insurance 204×11÷12 187
Rent (70×8)+(91×4) 924
Repair 186
Bad debts written off 138
Salary (124+685)+(134–98) 845
Depreciation - equipment (2,490÷0.6×8%)+(550×8%) 376
(2,656)
Net profit 1,562

W-1: POLICIES
Blenders Juicers
Updated with
Previous Updated Previous Updated with sales & cost
sales
Sales 130 (130×1.2) 156 100 (100×1.2)120 120
Cost 100 100 65 65 (65×1.4) 91
Profit 30 56 35 55 29

W-2: Trade debtors – gross Rs. in '000


Opening balance 1,410 Receipts 6,570
Sales 6,981 Write off 138
Closing (balancing) 1,683
8,391 8,391

W-3: Trade payables (Sigma Electronics) Rs. in '000


Bank 8,850 Opening 3,600
Closing 2,420 Purchases (Bal) 7,670
11,270 11,270

© Emile Woolf International 276 The Institute of Chartered Accountants of Pakistan


Chapter 5: Preparation of accounts from incomplete records

Statement of financial position as on 31 December 2018


Rs. in '000
Assets
Non-current assets
Equipment 2,490+550–376 2,664
Current assets:
Stock 975+2,597 3,572
Trade debtors (W-2) 1,683
Prepaid rent 280+826–924 182
Prepaid insurance 204–187 17
5,454
8,118

Equity and liabilities:


Opening capital 2,490+3,705+280+1,410–3,600–98–740 3,447
Net profit 1,562
Drawings (477)
4,532

Current liabilities
Trade payables (W-3) 2,420
Bank overdraft (W-4) 1,032
Salary payable 134
3,586
8,118

W-4 Bank Rs. in '000


Receipts from trade debtors 6,570 Opening balance 740
Amount banked 4,253 Trade payables 8,850
5,040-186-124-477 Insurance 204
Rent 826
Equipment 550
Closing balance 1,032 Salaries & wages 685
11,855 11,855
.

© Emile Woolf International 277 The Institute of Chartered Accountants of Pakistan


Financial accounting and reporting I

Example 27: Rakaposhi Traders


Question: Rakaposhi Traders (RT) was unable to retrieve complete information required to
prepare its statement of profit or loss due to a computer virus attack. In order to compute profit
for the year ended 31 December 2019, RT has gathered the following information:
(i) List of all assets and liabilities as on 1 January 2019:

Liabilities Rs. in '000 Assets Rs. in '000


Creditors 310 Furniture - net 460
Accrued rent 33 Inventories 200
Debtors 170
Cash in hand 37
Cash at bank 85
343 952

(ii) Inventories increased by 30% during the year.


(iii) Credit sales during the year amounted to Rs. 2,500,000. Collections from
debtors amounted to Rs. 2,400,000 out of which Rs. 300,000 were received in
cash. A debtor’s balance of Rs. 15,000 is irrecoverable.
(iv) Balance as per bank statement as on 31 December 2019 amounted to Rs.
90,000. However, it does not include a cheque of Rs. 40,000 deposited on 31
December 2019.
(v) Following information has been collected from the counterfoils of cheque books:

Rs. in '000
Payment to creditors 1,375
Drawings 275
Salaries 600
Cash withdrawn for office use 120

(vi) Cash in hand as at 31 December 2019 amounted to Rs. 50,000. Details of cash
sales and cash payments (expenses, payment to creditors and cash purchases)
are not available.
(vii) On 1 April 2019, the owner brought into the business a vehicle having a market
value of Rs. 360,000.
(viii) Creditors’ closing balance of Rs. 425,000 was determined from account
statements obtained from the creditors.
(ix) Rent amounting to Rs. 23,000 was outstanding as on 31 December 2019.
(x) Depreciation is charged at 10% on fixed assets.

Required:
Compute the net profit or net loss for the year ended 31 December 2019.

© Emile Woolf International 278 The Institute of Chartered Accountants of Pakistan


Chapter 5: Preparation of accounts from incomplete records

Answer:

Rakaposhi Traders
Net profit/(loss) for the year ended 31 December 2019
Rs. in '000
Closing net assets (W-1) 994
Opening net assets 952–343 (609)
Increase in net assets 385
Drawings for the year 275
Additional investment for the year (360)
Net profit for the year 300

W-1: Closing net assets Rs. in '000


Furniture – net 460×0.90 414
Vehicle – net 360–(360×0.1×9÷12) 333
Inventories 200×1.3 260
Debtors 170+2,500–2,400–15 255
Cash at bank 90+40 130
Cash in hand 50
1,442
Creditors 425
Rent accrued 23
(448)
994
.

Example 28: Gandhara Enterprises


Question: You have been appointed as accountant of Gandhara Enterprises (GE) to replace
Nasim who was terminated on suspicion of fraud. Following information has been compiled for
preparation of GE’s financial statements for the year ended 30 June 2020:

(i) Summarised bank statement:


Receipts Rs. in Payments Rs. in
'000 '000
Opening balance 600 Creditors 8,300
Cheques from debtors 7,420 Salaries 900
Cash 2,400 Repair and maintenance 450
Rent 980 Utilities 500
Office furniture 150
Drawings 640
Closing balance 460
11,400 11,400

© Emile Woolf International 279 The Institute of Chartered Accountants of Pakistan


Financial accounting and reporting I

(ii) Other balances worked out from the available records:


Particulars 30-Jun-2020 30-Jun-2019
------- Rs. in '000 -------
Fixed assets – WDV 3,400 3,460
Inventories 750 715
Goods in transit 140 -
Debtors 900 730
Unearned rent 300 450
Cash in hand 48 36
Creditors 895 690
Salaries payable 86 120
(iii) All debtors settle their accounts through cheques. All payments are made through
cheques except for average monthly petty expenses of Rs. 25,000.
(iv) Cheques of Rs. 950,000 issued to creditors in the last week of June 2020 were
presented in July 2020. Cheques from debtors amounting to Rs. 860,000
deposited on 30 June 2020 were cleared in July 2020.
(v) Goods are sold on cash and credit at cost plus 25% and 30% respectively.
(vi) Apart from misappropriating amounts from cash sales, the following matters were also
noted in respect of Nasim’s fraud:
 Physical cash count revealed that cash in hand was Rs. 20,000.
 Fixed assets having written down value of Rs. 65,000 were sold for Rs. 120,000
which was not recorded in the books.
 Goods in transit represent goods purchased in May 2020. However, in actual there
were no goods in transit.
 Goods costing Rs. 130,000 appearing in the closing inventory sheets were not
found physically.
 All the debtors confirmed their balances except for an amount of Rs. 260,000. It
was found that the related goods had been issued against fake invoices.
Required:
(a) Determine the amount of suspected fraud.
(b) Prepare GE’s statement of profit or loss for the year ended 30 June 2020.

Answer:

Rs. in '000
(a) Amount of suspected fraud:
Difference in cash balance 48–20 28
Proceeds from sale of fixed assets 65+55 120
Fake credit sales invoices 260÷1.3 200
Fake goods in transit 140
Embezzlement through inventory 750–620 130
Cash defalcated from cash sales (W-1) 763
1,381

© Emile Woolf International 280 The Institute of Chartered Accountants of Pakistan


Chapter 5: Preparation of accounts from incomplete records

W-1: Cash
Rs. in '000 Rs. in '000
Opening balance 36 Petty expenses 25×12 300
Cash sales PL 3,475 Cash banked 2,400
Cash shortage 48–20 28
Closing balance 20

Cash defalcated from cash sales (Bal.) 763


3,511 3,511

(b) Gandhara Enterprises


Statement of profit or loss for the year ended 30 June 2020
--------- Rs. in '000 ---------
Sales - Credit (W-2) 8,190

- Cash [9,080–6,300(8,190÷1.3)]×1.25 3,475


11,665
Cost of goods sold
Opening inventory 715
Purchases (W-3) 9,455
Goods in transit (140)
Good issues against fake invoices 260÷1.3 (200)
Goods physically not found (130)
Closing stock 750–130 (620)
Cost of sales (9,080)
Gross profit 2,585
Operating expenses
Salaries 86+900–120 866
Utilities 500
Repair and maintenance 450
Petty cash expenses 25×12 300
Depreciation 3,460+150–3,400 210
Loss due to defalcation (a) 1,381
Total operating expenses (3,707)
(1,122)
Rent income 980+450–300 1,130
Gain on disposal of fixed assets 120–65 55
Net profit 63

© Emile Woolf International 281 The Institute of Chartered Accountants of Pakistan


Financial accounting and reporting I

W-2: Debtors
Rs. in '000 Rs. in '000
Opening balance 730 Bank 7,420
Credit sales (Bal.) 8,190 Uncleared cheques 860
Closing balance (900–260) 640
8,920 8,920

W-3: Creditors
Rs. in '000 Rs. in '000
Bank 8,300 Opening balance 690
Unpresented cheques 950 Purchases (Bal.) 9,455
Closing balance 895
10,145 10,145

© Emile Woolf International 282 The Institute of Chartered Accountants of Pakistan


Chapter 5: Preparation of accounts from incomplete records

3 OBJECTIVE BASED QUESTIONS


01. Yusuf does not keep a full set of business records, but the following information is available for
the month of June 2019.

Rs. 000

Accounts receivable, 1 June 2019 800


Accounts receivable, 30 June 2019 550

Credit sales 6,800

Cash received from customer (credit) 6,730

Irrecoverable debt written off 40

General allowance for doubtful debts at 30 June 2019 100


Assuming no other transactions, how much discount was allowed to customers during the
month?

(a) Rs. 240,000

(b) Rs. 280,000

(c) Rs. 340,000

(d) Rs. 380,000

02. Many of the records of Ghalib have been destroyed by fire. The following information is available
for the period under review.
(i) Sales totaled Rs. 480,000
(ii) Inventory at cost was opening Rs. 36,420, closing Rs. 40,680
(iii) Trade payables were opening Rs. 29,590, closing Rs. 33,875
Gross profit for the period should represent a margin of 50%
What was the total for the period of cash paid to suppliers?

(a) Rs. 239,975

(b) Rs. 315,715

(c) Rs. 319,975

(d) Rs. 328,545

03. In the year to 31st April 2016, Abdullah’s sales were Rs. 182,000. All of his sales were made at
a mark-up of 30%. His opening inventory value was Rs. 11,800 and his closing inventory value
was Rs. 9,700.
What was the value of Abdullah’s purchases in the year to 31 April 2016?

(a) Rs. 125,300

(b) Rs. 137,900

(c) Rs. 140,000

(d) Rs. 142,100

© Emile Woolf International 283 The Institute of Chartered Accountants of Pakistan


Financial accounting and reporting I

04. The following information is relevant to the calculation of the sales figure for Arif, a sole trader
who does not keep proper accounting records:

Rs.

Opening accounts receivable 29,100

Cash received from credit customers and paid into the bank 381,600

Expenses paid out of cash received from credit customers before banking 6,800

Irrecoverable debts written off 7,200

Refunds to credit customers 2,100

Discounts allowed to credit customers 9,400

Cash sales 112,900

Closing accounts receivable 38,600

The figure which should appear in Arif’s statement of comprehensive income for sales is:

(a) Rs. 525,300

(b) Rs. 511,700

(c) Rs. 529,500

(d) Rs. 510,900

05. A sole trader who does not keep full accounting records wishes to calculate her sales revenue
for the year.
The information available is:

Rs.

1 Opening inventory 17,000

2 Closing inventory 24,000

3 Purchases 91,000

4 Standard gross profit percentage on sales revenue 40%

Which of the following is the sales figure for the year calculated from these figures?

(a) Rs. 117,600

(b) Rs. 108,000

(c) Rs. 210,000

(d) Rs. 140,000

© Emile Woolf International 284 The Institute of Chartered Accountants of Pakistan


Chapter 5: Preparation of accounts from incomplete records

06. Salman is a sole proprietor whose accounting records are incomplete. All the sales are cash
sales and during the year Rs. 50,000 was banked, including Rs. 5,000 from the sale of a business
car. He paid Rs. 12,000 wages in cash from the till and withdrew Rs. 2,000 as drawings. The
cash in the till at the beginning and end of the year was Rs. 300 and Rs. 400 respectively. There
were no other payments in the month.
What were the sales for the year?

(a) Rs. 58,900

(b) Rs. 59,100

(c) Rs. 63,900

(d) Rs. 64,100

07. There is Rs. 100,000 in the cash till at the year-end at F Ltd, but the accountant has discovered
that some cash has been stolen. At the beginning of the year there was Rs. 50,000 in the cash
till and receivables were Rs. 2,000,000. Total sales in the year were Rs. 230,000,000. Accounts
receivable at the end of the year were Rs. 3,000,000. Cheques banked from credit sales were
Rs. 160,000,000 and cash sales of Rs. 50,000,000 have been banked.
How much cash was stolen during the year?

(a) Rs. 21,050,000

(b) Rs. 18,950,000

(c) Rs. 19,050,000

(d) Rs. 50,000

08. A business operates on a gross margin of 33 ¼ %. Gross profit on a sale was Rs. 800,000 and
expenses were Rs. 680,000.
The net profit percentage is

(a) 3.75%

(b) 5%

(c) 11.25%

(d) 22.67%

09. A toyshop makes purchases of Rs. 20,248,000 and sales of Rs. 26,520,000. The proprietor’s
children take goods costing Rs. 486,000 without paying for them. Closing stock was valued at its
cost of Rs. 2,240,000 and the gross margin achieved was a constant 30% on sales.
What was the cost of the opening stock?

(a) Rs. 556,000

(b) Rs. 1,042,000

(c) Rs. 2,392,000

(d) Rs. 2,878,000

© Emile Woolf International 285 The Institute of Chartered Accountants of Pakistan


Financial accounting and reporting I

10. Which of the following calculations could produce an acceptable figure for a trader's net profit for
a period if no accounting records had been kept?

(a) Closing net assets plus drawings minus capital introduced minus opening net assets

(b) Closing net assets minus drawings plus capital introduced minus opening net assets

(c) Closing net assets minus drawings minus capital introduced minus opening net assets

(d) Closing net assets minus drawings plus capital introduced plus opening net assets

11. On 30 September 2018 part of the inventory of a company was completely destroyed by fire.
The following information is available:

Inventory at 1 September 2018 at cost Rs. 49,800,000

Purchases for September 2018 Rs. 88,600,000

Sales for September 2018 Rs. 130,000,000

Inventory at 30 September 2018 undamaged items Rs. 32,000,000

Standard gross profit percentage on sales 30%

Based on this information, what is the cost of the inventory destroyed?

(a) Rs. 17,800,000

(b) Rs. 47,400,000

(c) Rs. 15,400,000

(d) Rs. 6,400,000

12. Sarim does not keep full accounting records. His last accounts show that his capital balance was
Rs. 42,890,000. At the year end, he calculated that his assets and liabilities were:

Rs. 000

Non-current assets 41,700

Inventory 9,860

Receivables 7,695

Payables 4,194

Bank overdraft 5,537

On reviewing his calculations, you note that he did not include Rs. 258,000 of unpaid invoices for
expenses.
What is the value of Sarim’s closing capital?

(a) Rs. 49,266,000

(b) Rs. 49,544,000

(c) Rs. 60,360,000

(d) Rs. 60,876,000

© Emile Woolf International 286 The Institute of Chartered Accountants of Pakistan


Chapter 5: Preparation of accounts from incomplete records

13. During the year to 30th November 2015 Amna bought goods for resale at a cost of Rs.
75,550,000. Her inventory at 1st December 2014 was valued at Rs. 15,740,000. She did not
count her inventory at 30th November 2015, but she knows that her sales for the year to 30th
November 2015 were Rs. 91,800,000. All sales were made at a mark-up of 20%.
Based on the information above, what was the value of Amna’s inventory at 31 November 2015?

(a) Rs. 13,630,000

(b) Rs. 14,790,000

(c) Rs. 16,690,000

(d) Rs. 17,850,000

14. On 1 September 2018, Waris had inventory of Rs. 380,000. During the month, sales totalled Rs.
650,000 and purchases Rs. 480,000. On 30 September 2018 a fire destroyed some of the
inventory. The undamaged goods were valued at Rs. 220,000. The business operates with a
standard gross profit margin of 30%.
Based on this information, what is the cost of the inventory destroyed in the fire?

(a) Rs. 185,000

(b) Rs. 140,000

(c) Rs. 405,000

(d) Rs. 360,000

15. You are given the following incomplete and incorrect extract from the Statement of
comprehensive income of a company that trades at a markup of 25% on cost:

Rs. Rs.

Sales 174,258

Less: Cost of goods sold

Opening inventory 12,274

Purchases 136,527

Closing inventory X

(X)

Gross profit X

Having discovered that the sales figure should have been Rs. 174,825 and the purchase returns
of Rs. 1,084 and sales returns of Rs. 1,146 have been omitted, the closing inventory should be:

(a) Rs. 8,662

(b) Rs. 8,774

(c) Rs. 17,349

(d) Rs. 17,458

© Emile Woolf International 287 The Institute of Chartered Accountants of Pakistan


Financial accounting and reporting I

16. Profit is Rs. 1,051,000. Capital introduced is Rs. 100,000. There is an increase in net assets of
Rs. 733,000.
What are drawings?

Rs. ___________

17. The bookkeeper of Lego has disappeared. There is no cash in the till and theft is suspected. It is
known that the cash balance at the beginning the year was Rs. 240,000. Since then, total sales
have amounted to Rs. 41,250,000. Credit customers owed Rs. 2,100,000 at the beginning of the
year and owe Rs. 875,000 now. Cheques banked from credit customers have totaled Rs.
2,429,000. Expenses paid from the till receipts amount to Rs. 180,500 and cash receipts of Rs.
9,300,000 have been lodged in the bank.
What is the amount that bookkeeper stole during the period?

Rs. ___________

18. Taiwan Tyres does not keep full accounting records, but the following information is available in
respect of accounting year ended 31st December 2018.

Rs.

Cash purchases in year 3,900,000

Cash paid for goods supplied on credit 27,850,000

Payables at 1st January 2018 970,000

Payable at 31st December 2018 720,000

In the statement of comprehensive income for 2018, figure for purchases will be?

Rs. ___________

19. Deen has been trading for some time, but he neglected to maintain full accounting. He is able to
provide the following information.
He is owed Rs. 7,900 by his customers.
He has lodged Rs. 120,700 to his bank account since starting his business. This includes his
initial capital of Rs. 22,000.
All his sales are made at cost plus 30%
The value of Deen’s sale since he began trading is?

Rs. ___________

20. The diesel fuel included in the inventory at 1 November 2017 was Rs. 12,500,000 and there were
invoices wait for Rs. 1,700,000. During the year to 31 October 2018, diesel fuel bills of Rs.
85,400,000 were paid, and a delivery worth Rs. 1,300,000 had yet to be invoiced.
At 31 October 2018, the inventory of diesel fuel was valued at Rs. 9,800,000.
The diesel fuel to be charged to the Statement of comprehensive income for the year to 31
October 2018 is:

Rs. ___________

© Emile Woolf International 288 The Institute of Chartered Accountants of Pakistan


Chapter 5: Preparation of accounts from incomplete records

21. In which of the following systems of recording the financial statements reflect true and fair view
of an entity and accounting records are considered to be more accurate?
(a) Cash book system
(b) Single entry system
(c) Double entry system
(d) None of the above

22. Statement of financial position produced from incomplete accounting record is commonly known
as
(a) Statement of financial position
(b) Statement of affairs
(c) Statement of net assets
(d) Statement of financial operations

23. Which of the following businesses usually maintain incomplete accounting record of the business
activities?
(a) Large businesses
(b) Companies
(c) Partnership firms
(d) Small businesses

24. In single entry system, it is not possible to prepare,


(a) Statement of financial position
(b) Profit or loss account
(c) Trial balance from ledgers
(d) Receipt and payment account

25. The opening capital is ascertained by preparing:


(a) Cash book
(b) Creditors A/c
(c) Debtors A/c
(d) Opening statement of affairs

26. Identify the correct formula used to ascertain the closing balance of capital?
(a) Closing capital = Opening capital + Net profit – Expenses
(b) Closing capital = Opening capital + Net profit + Drawings
(c) Closing capital = Opening capital + Net profit – Drawings
(d) Closing capital = Opening capital + Revenue – Expenses

© Emile Woolf International 289 The Institute of Chartered Accountants of Pakistan


Financial accounting and reporting I

27. Net profit is calculated by:

(a) Closing capital + Drawings - Fresh capital injected – Opening capital

(b) Closing capital – Drawings + Fresh capital injected – Opening capital

(c) Closing capital + Drawings + Fresh capital injected + Opening capital

(d) None of the above

28. If opening capital = Rs.10 million and closing capital = Rs.20 million. Assuming no drawings
during the accounting period, calculated the net profit or loss for the period?

(a) Net profit = Rs.20 million

(b) Net loss = Rs.20 million

(c) Net profit = Rs.10 million

(d) Net loss = Rs.10 million

29. Which one of the following accounts is supposed to be used to get the figure of credit purchases
made during the current accounting period?

(a) Debtor account

(b) Creditor account

(c) Revenue account

(d) Expenses account

30. To obtain the amount of credit sales made during an accounting period, which account is
generally used in single entry and incomplete records?

(a) Debtor account

(b) Creditor account

(c) Revenue account

(d) Expenses account

31. If Plant (closing balance) = Rs. 8 million, Land (opening balance) = Rs. 5 million and Creditors
(opening balance) = Rs. 1 million then opening capital balance is?

(a) Rs.3 million

(b) Rs.4 million

(c) Rs.5 million

(d) Rs.8 million

© Emile Woolf International 290 The Institute of Chartered Accountants of Pakistan


Chapter 5: Preparation of accounts from incomplete records

32. Opening and closing debtors were Rs. 412,800 and Rs. 524,400 respectively. During the year
Rs. 2,684,500 was received from sales after allowing a cash discount of Rs. 17,420. Debts of
Rs. 34,840 were written off as bad during the year. Find out the credit sales during the year?

(a) Rs.2,778,680

(b) Rs.2,813,520

(c) Rs.2,848,360

(d) Rs.2,753,670

33. Opening and closing creditors were Rs. 450,000 and Rs. 700,000 respectively. During the year,
Rs. 3,400,000 was paid to suppliers. Find out the credit purchases during the year?

(a) Rs.3,150,000

(b) Rs.3,400,000

(c) Rs.3,650,000

(d) None of the above

34. Staff salary payable for the month end was Rs. 74,540 and Rs. 96,720 as its opening balance.
Salary paid during the period was Rs. 856,420. Find out the accrued salary during the period?

(a) Rs.834,240

(b) Rs.856,420

(c) Rs.861,540

(d) Rs.878,600

© Emile Woolf International 291 The Institute of Chartered Accountants of Pakistan


Financial accounting and reporting I

3 OBJECTIVE BASED ANSWERS


01. (b)
Accounts receivables
Particulars Rs. 000 Particulars Rs. 000
Bal. b/d 1,700 Bad debts 40
Sales 6,800 Cash 6,730
Discount β 280
c/d 550
7,600 7,600

02. (a)
Accounts payable
Particulars Rs. Particulars Rs.
Cash β 239,975 b/d 29, 90
c/d 33,875 Purchases 244,260
237,850 237,850

Inventory
Particulars Rs. Particulars Rs.
Bal. b/d 36,420 OS 480,000x0.5 240,000
Purchases β 244,260 c/d 40,680
280,680 280,680

03. (b)
Inventory
Particulars Rs. Particulars Rs.
Bal. b/d 11,800 COS 140,000
182,000/130x100
Purchases β 137,900 c/d 9,700
1 9,700 149,700

04. (a)
Total sales = Rs. 112,900 + Rs. 412,400 = 525,300

Accounts receivables
Particulars Rs. Particulars Rs.
Bal. b/d 29,100 Bad debts 7,200
Sales β 412,400 Cash 381,600+6,800 388,400
Refunds 2,100
Discount allowed 9,400
c/d 38,600
443,600 443,6 0

© Emile Woolf International 292 The Institute of Chartered Accountants of Pakistan


Chapter 5: Preparation of accounts from incomplete records

05. (d) Cost of sales = 17,000 + 91,000 – 24,000 = 84,000


Sales = 84,000/60 x 100 = Rs. 140,000
06. (b) Sales = 64,100 – 5,000 = 59,100
Cash a/c
Particulars Rs. Particulars Rs.
Bal. b/d 300 Bank 50,000
Cash from sales 64,100 Wages 12,000
Drawings 2,000
c/d 400
64,400 64,400

07. (b)
Accounts receivables
Particulars Rs. Particulars Rs.
Bal. b/d 2,000,000 Cash 179,000,000
Sales 180,000,000 c/d 3,000,000
+230,000,000-
50,000,000
182,000,000 182,000,000

Cash a/c
Particulars Rs. Particulars Rs.
Bal. b/d 50,000 Bank 210,000,000
160,000,000+50,000,000
Cash sales 50,000,000 Cash stolen 18,950,000

Receivables 179,000,000 c/d 100,000


229,050,000 229,050,000

08. (b) Sales = 800,000 /33.25 x 100 = 2,406,015


Net profit = 800,000 – 680,000 = 120,000
Net profit % = 120,000/2,406,015 x 100 = 5%
09. (b)
Inventory
Particulars Rs. Particulars Rs.
Bal. b/d β 1,042,000 COS 18,564,000
26,520,000x0.7
Purchases 20,248,000 Drawings 486,000
c/d 2,240,000
21,290,000 21,290,000

10. (a) Profit = Closing net assets + drawings – capital introduced - opening net assets

© Emile Woolf International 293 The Institute of Chartered Accountants of Pakistan


Financial accounting and reporting I

11. (c)
Inventory
Particulars Rs. 000 Particulars Rs. 000
Bal. b/d 49,800 COS 130,000x70% 91,000
Purchases 88,600 Destroyed β 15,400
c/d 32,000
138,400 138,400

12. (a)
Rs. 000
Non-current assets 41,700
Inventory 9,860
Receivables 7,695
Payables (4,194)
Bank overdraft (5,537)
Expense payable (258)
49,266

13. (b)
Inventory
Particulars Rs. 000 Particulars Rs. 000
Bal. b/d 15,740 COS 91,800/120x100 76,500
Purchases 75,550 c/d β 14,790

91,290 91,290

14. (a)
Inventory
Particulars Rs. Particulars Rs.
Bal. b/d 380,000 COS 650,000x70% 455,000
Purchases 480,000 Lost by fire β 185,000
c/d 220,000

15. (b)
Rs. Rs.
Sales 174,825 – 1,146 173,679
Less: Cost of goods sold
Opening inventory 12,274
Purchases 136,527 - 1,084 135,443
Closing inventory β (8,774)
Cost of sales β 138,943
Gross profit 173,679 /125 x 25 34,736

© Emile Woolf International 294 The Institute of Chartered Accountants of Pakistan


Chapter 5: Preparation of accounts from incomplete records

16. Rs. 418,000 Drawings = Opening capital + Profit + capital introduced – Closing capital
=1,051,000+100,000- 733,000 = Rs. 418,000
17. Rs.
6,515,500
Cash a/c
Particulars Rs. Particulars Rs.
Bal. b/d 240,000 Bank 9,300,000 + 11,729,000
2,429,000
Cash sales 9,300,000 Expenses 180,500
Receivables 8,885,000 Cash stolen β 6,515,500
18,425,000 18,425,000

Accounts receivables
Particulars Rs. Particulars Rs.
Bal. b/d 2,100,000 Cash β 8,885,000
Bank 24,290,000
Sales 41,250,000- 31,950,000 c/d 875,000
9,300,000
34,050,000 34,050,000

18. Rs. 31.5 Purchases = 27,600,000+3,900,000 = Rs. 31,500,000


million
Accounts payable
Particulars Rs. Particulars Rs.
Cash 27,850,000 b/d 970,000
c/d 720,000 Purchases 27,600,000
28,570,000 28,570,000

19. Rs. 106,600 Sales = Rs. 7,900+ (120,700 - 22,000) = 106,600


20. Rs. 87.7
million
Diesel Fuel
Particulars Rs. Particulars Rs.
b/d 12,500,000 b/d 1,700,000
Cash 85,400,000 PL 87,700,000
c/d 1,300,000 c/d 9,800,000
99,200,000 99,200,000

21. (c)

22. (b)

23. (d)

© Emile Woolf International 295 The Institute of Chartered Accountants of Pakistan


Financial accounting and reporting I

24. (c)

25. (d)

26. (c)

27. (a)

28. (c)

29. (b)

30. (a)

31. (b)

32. (c)
Receivables
Particulars Rs. Particulars Rs.
b/d 412,800 Cash 2,684,500
Sales 2,848,360 Discount allowed 17,420
Bad debts 34,840
c/d 524,400
3,261,160 3,261,160

33. (c)
Creditors
Particulars Rs. Particulars Rs.
Cash 3,400,000 b/d 450,000
c/d 700,000 Purchases 3,650,000
4,100,000 4,100,000

34. (a)
Salaries
Particulars Rs. Particulars Rs.
Cash 856,420 b/d 96,720
c/d 74,540 PL 834,240
930,960 930,960

© Emile Woolf International 296 The Institute of Chartered Accountants of Pakistan


Certificate in Accounting and Finance

CHAPTER
Financial accounting and reporting I

Introduction to cost of production

Contents
1 Accounting for management
2 Cost and management accounting versus financial accounting
3 Introduction to costs
4 Cost classification by type and function
5 Fixed and variable costs
6 Direct and indirect costs
7 Product costs and period costs
8 Cost accounting cycle
9 Manufacturing account
10 Objective based questions and answers

* The student must refer original handbook of IFRS.

© Emile Woolf International 297 The Institute of Chartered Accountants of Pakistan


Financial accounting and reporting I

1 ACCOUNTING FOR MANAGEMENT


Section overview

 Introduction to accounting information


 Data and information
 Qualities of good information
 Purpose of management information

1.1 Introduction to accounting information


Accounting is one of the key functions for almost any business; it may be handled by a bookkeeper
and accountant at small firms or by sizable finance departments with dozens of employees at larger
companies.
There are many definitions of accounting.
Definitions: Accounting
The systematic and comprehensive recording of financial transactions pertaining to a business and
the process of summarizing, analysing and reporting these transactions.
A systematic process of identifying, recording, measuring, classifying, verifying, summarizing,
interpreting and communicating financial information.
The process of identifying, measuring, and communicating economic information to permit
informed judgements and decisions by users of the information.

The main purposes of accounting may be summarised as follows.


 To provide a record of the financial value of business transactions, and in doing so to
establish financial controls and reduce the risks of fraud.
 To assist with the management of the financial affairs of an entity.
 To provide information - mainly information of a financial nature.
Accounting information is provided for:
 Management, so that managers have the information they need to run the company.
 Other users of information, many of them outside the entity. For example, a company
produces accounting information for its shareholders in the form of financial statements, and
financial statements are also used by tax authorities, investors, trade union representatives
and others.
Cost and management accounting is concerned with the provision of information, mainly of a
financial nature, for management.

1.2 Data and information


Data and information
The terms ‘data’ and ‘information’ are often used as if they have the same meaning. However,
there is a difference between data and information. Data is a term that refers to facts. It must be
turned into information in order for it to become useful. Information is derived from facts that have
been processed, structured and analysed.
 Data consists of unprocessed facts and statistics.
 Data is collected and processed to produce information.
 Data has no meaning until it has been processed into information.
 Information has a meaning and a purpose. It is produced from ‘data’. It is processed data
that has relevance to a particular useful purpose.

© Emile Woolf International 298 The Institute of Chartered Accountants of Pakistan


Chapter 6: Introduction to cost of production

Accounting systems are designed to capture data and process it into information.

Illustration 01: Data and information


A company engages in many different types of transactions (sales, purchases of materials,
expenses, and so on).
Each of these is processed into individual records (for example, sales are recorded on sales
invoices). This would result in thousands of individual records.
An accounting system summarises these in a meaningful manner to produce information. This is
carried out in a series of steps each of which provides information based ultimately on the original
transactions.
Sales day book – summarises the total sales made in a specified period.
The receivables control account shows the total owed to the company at any point in time.
The receivables ledger shows the total amount owed the company by individual customers at any
point in time.
The general ledger is the source of information which can be further processed into periodic reports
(financial statements).

A cost accounting system records data about the costs of operations and activities within the
entity. The sources of cost accounting data within an organisation include invoices, receipts,
inventory records and time sheets.
Many of the documents from which cost data is captured are internally-generated documents,
such as time sheets and material requisition notes.

Illustration 02:
A ship yard may employ hundreds of workers and be building and refitting several ships at any one
time.
Each worker might be required to complete job sheets which specify the length of time taken by
that worker and on which contract.
This would produce many thousands of individual records (data) which are not very useful until the
facts contained in those records are processed into information. Thus the system might produce
reports (information) to show the labour cost, by type of labour, by week for each ship.

Data is analysed and processed to produce management information, often in the form of:
 routine reports;
 specially-prepared reports;
 answers to ‘one-off’ enquiries that are input to a computer system.
Information produced from cost accounting data is management accounting information.
Management accounting systems also obtain information from other sources, including external
sources, but the cost accounting system is a major source of management accounting information.

1.3 Qualities of good information


Information is only useful to managers if it possesses certain qualities or attributes.
 Accurate
Information should be fair and free from bias. It should not have any arithmetical and
grammatical errors.
 Understandable
 Information should be understandable to the individuals who use it.
 Accounting information must be set out clearly and be properly explained.
 Purpose and relevance
 Unless information has a purpose it has no value at all and it makes no sense to
provide it.
 Information must be relevant for its purpose.

© Emile Woolf International 299 The Institute of Chartered Accountants of Pakistan


Financial accounting and reporting I

 Reliable
 Users of information must be able to rely on it for its intended purpose.
 Unreliable information is not useful.
 Information does not have to be 100% accurate to be reliable. In many cases,
information might be provided in the form of an estimate or forecast.
 Sufficiently complete
 Information should include all information necessary for its purpose.
 However, information in management reports should not be excessive, because
important information may be hidden in the unimportant information, and it will take
managers too long to read and understand.
 Timeliness
 If information is provided too late for its purpose, it has no value.
 With the widespread computerisation of accounting systems, including cost
accounting systems, it might be appropriate for up-to-date management accounting
information to be available on line and on demand whenever it is needed.
 Comparability
 In accounting it is often useful to make comparisons, such as comparisons of current
year results with previous years, or comparisons of actual results with planned results.
 To make comparisons possible, information should be prepared on the same basis,
using the same methods and the same ‘rules’.
 Communicated to the right person
 Management information should be communicated to the proper person.
 This is the person with the authority to make a decision on the basis of the information
received and who needs the information to make the decision.
 Its value must exceed its cost (Information must be cost effective)
 Management information has a value (if information has no value there is no point in
having it) but obtaining it involves a cost.
 The value of information comes from improving the quality of management decisions.
Information is worth having only if it helps to improve management decisions, and the benefits from
those decisions exceed the cost of providing the additional information.

1.4 Purpose of management information


The purpose of management accounting is to provide information for:
 planning;
 control; and
 Decision making.
Planning
Planning involves the following:
 setting the objectives for the organisation
 Making plans for achieving those objectives.
The planning process is a formal process and the end-result is a formal plan, authorised at an
appropriate level in the management hierarchy. Formal plans include long-term business plans,
budgets, sales plans, weekly production schedules, capital expenditure plans and so on.
Information is needed in order to make sensible plans – for example in order to prepare an annual
budget, it is necessary to provide information about expected sales prices, sales quantities and
costs, in the form of forecasts or estimates.

© Emile Woolf International 300 The Institute of Chartered Accountants of Pakistan


Chapter 6: Introduction to cost of production

Control
Control of the performance of an organisation is an important management task. Control involves
the following:
 monitoring actual performance, and comparing actual performance with the objective or
plan;
 taking corrective action where appropriate;
 Evaluating actual performance.
When operations appear to be getting out of control, management should be alerted so that
suitable measures can be taken to deal with the problem. Control information might be provided in
the form of routine performance reports or as special warnings or alerts when something unusual
has occurred.
Decision making
Managers might need to make ‘one-off’ decisions, outside the formal planning and control systems.
Management accounting information can be provided to help a manager decide what to do in any
situation where a decision is needed.

© Emile Woolf International 301 The Institute of Chartered Accountants of Pakistan


Financial accounting and reporting I

2 COST AND MANAGEMENT ACCOUNTING VERSUS FINANCIAL ACCOUNTING


Section overview

 Purpose and role of cost accounting, management accounting and financial accounting
 Comparison of financial accounting and cost and management accounting

2.1 Purpose and role of cost accounting, management accounting and financial
accounting
The terms cost accounting and management accounting are often used as having the same
meaning. However, there is distinction between the two:
Cost accounting
Cost accounting is concerned with identifying the cost of things. It involves the calculation and
measurement of the resources used by a business in undertaking its various activities.
Cost accounting is concerned with gathering data about the costs of products or services and the
cost of activities. There may be a formal costing system in which data about operational activities
is recorded in a ‘double entry’ system of cost accounts in a ‘cost ledger’. The cost accounting data
is captured, stored and subsequently analysed to provide management information about costs.
Cost accounting information is historical in nature, and provides information about the actual costs
of items and activities that have been incurred.
Management accounting
Management accounting is concerned with providing information to management that can be used
to help run the business.
 The purpose of management accounting is to provide detailed financial information to
management, so that they can plan and control the activities or operations for which they
are responsible.
 Management accounting information is also provided to help managers make other
decisions. In other words, management accounting provides management information to
assist with planning, control and ‘one-off’ decisions.
Management accounting includes cost accounting as one of its disciplines but is wider in scope.
Management accounting information is often prepared from an analysis of cost accounting data,
although cost estimates and revenue estimates may be obtained from sources other than the cost
accounting system.
Management accounting may be forward-looking, and used to provide information about expected
costs and profits in the future.
Financial accounting
Financial accounting is concerned with providing information about the financial performance and
cash flows of an entity in a given period and the financial position of the entity at the end of that
period.
The information is often provided to a wider range of stakeholders (those with an interest in the
business) than those who have access to management information. The most important of these
are the owners of a business who may not take part in the day to day running of the business.

2.2 Comparison of financial accounting and cost and management accounting


Financial accounting
A financial accounting system is used to record the financial transactions of the entity, such as
transactions relating to income, expenses, assets and liabilities.

© Emile Woolf International 302 The Institute of Chartered Accountants of Pakistan


Chapter 6: Introduction to cost of production

It provides a record of the assets that the company owns, and what it owes and a record of the
income that the entity has earned, and the expenditures it has incurred.
The financial accounting system provides the data that is used to prepare the financial statements
of the entity at the end of each financial year (the statement of comprehensive income, statement
of financial position, statement of cash flows, and so on).
Managers might use the information in the financial statements, but the main purpose of financial
reporting is for ‘external purposes’ rather than to provide management information. The main
purpose of the financial statements of companies is to inform the company’s shareholders (owners)
about the financial performance, cash flows and financial position of the company. They are also
used as a basis for computation of the tax that the company should pay on its profits.
Financial statements are produced at the end of the financial year. Management need information
much more regularly, throughout the year. They also need much more detailed information than is
provided by a company’s financial statements. They often need forward-looking forecasts, rather
than reports of historical performance and what has happened in the past.
There is a statutory requirement for companies to produce annual financial statements, and other
business entities need to produce financial statements for the purpose of making tax returns to the
tax authorities.
Managers might find financial statements useful, but the main users of the financial statements of
a company should be its shareholders. Other external users, such as potential investors,
employees, trade unions and banks (lenders to the business) might also use the financial
statements of a company to obtain information.
Cost and management accounting
Whereas financial statements from the financial accounting system are intended mainly for external
users of financial information, management accounting information (obtained from the cost
accounting system) is prepared specifically for internal use by management.
An entity might have a cost accounting system as well as a financial accounting system, so that it
has two separate accounting systems in operation. A cost accounting system records the costs
and revenues for individual jobs, processes, activities and products or services.
 Like the financial accounting system, a cost accounting system is based on a double entry
system of debits and credits.
 However, the accounts in a cost accounting system are different from the accounts in the
financial accounting systems. This is because the two accounting systems have different
purposes and so record financial transactions in different ways.
There is no legal requirement for a cost accounting system. Business entities choose to have a
cost accounting system, and will only do so if the perceived benefits of the system justify the cost
of operating it.
(In business entities where there is no formal cost accounting system, managers still need
management accounting information to run their business. Some management accounting
information might be extracted from the financial accounting system, but in much less detail than
a cost accounting system would provide.)
A comparison of financial and cost accounting systems of companies is summarised in the table
below:

Financial accounting system Cost accounting system

Prepared to meet a legal or regulatory Prepared to meet the needs of


requirement. management.

Used to prepare financial statements for Used to prepare information for


shareholders and other external users. (Might management (internal use only).
also provide some information for management
but this is not their primary purpose).

© Emile Woolf International 303 The Institute of Chartered Accountants of Pakistan


Financial accounting and reporting I

Financial accounting system Cost accounting system

Content usually specified by a regulatory Content specified by the management of a


framework. company.

Prepared within a time frame specified by a Prepared within a time frame specified by
legal or regulatory framework. management.

Records revenues, expenditure, assets and Records costs of activities and used to
liabilities. provide detailed information about costs,
revenues and profits for specific products,
operations and activities.

Used mainly to provide a historical record of Provides historical information, but also used
performance and financial position. extensively for forecasting (forward-looking).

© Emile Woolf International 304 The Institute of Chartered Accountants of Pakistan


Chapter 6: Introduction to cost of production

3 INTRODUCTION TO COSTS
Section overview

 Types of organisation
 Cost classification: Introduction

3.1 Types of organisation


The following classification of organisations is useful for the purpose of learning about costs:
 Manufacturing organisations: and
 Service organisations.
Manufacturing organisations
There are a great many different kinds of manufacturing organisations. They can be classified
by their type of output which in turn implies the type of costing system they might use.

Type of production Examples Costing system

Identical (similar) products in Bottles of soft drink Basic


large numbers manufacturing
Mobile phones
costing
Garments
Standard costing

Identical products in large Pharmaceuticals Process costing


amounts by passing the product (including joint
Paint
through a series of processes product and by-
Petroleum product costing)

Identical products in large Aircraft (which may vary in internal Job costing
numbers customised in some way fit and external painting)
for different customers
Own-brand foods for supermarkets

One-off products to a customer’s Ships Job costing


specification
Airport facilities
Roads
Bridges

Service organisations
Similar to the manufacturing industry there are a great many different kinds of service
organisations. For example:
 Training and education
 Healthcare
 Travel and tourism
 Financial services
 Entertainment and leisure
One of the key differences between manufacturing and service industries is the perishability of
product – manufacturing output is generally tangible and can be stored whereas output from the
service industry is generally perishable. The service is normally consumed at the time of delivery
(production). For example, a patient visiting a doctor consumes the consultation as it is given.

© Emile Woolf International 305 The Institute of Chartered Accountants of Pakistan


Financial accounting and reporting I

However, some work-in-progress (WIP) may be recorded – for example an accountant who has
spent 10 hours working on a tax advice project that will take 20 hours in total to complete. The first
10 hours would be described as WIP.
Costing systems typically used in service organisations include:
 Standard costing
 For example, the standard cost of delivering a doctor’s consultation, the standard cost
of a package holiday, the standard cost of a flight between Karachi and London
 Job costing
 For example, bespoke consultation projects in the financial services industry or the
cost of an architect designing a ship
 The professional will usually apply a standard hourly rate whilst the total number of
hours on each job varies
The need to know about costs
All organisations need to understand their costs.
An organisation needs to know:
 how much it costs to make the products that it produces, or
 how much it costs to provide its services to customers.
For an organisation that is required to make a profit, it is important to know the cost of items in
order to:
 make sure that the product or service is sold at a profit;
 measure the actual profit that has been made; and
 in the case of some companies, such as manufacturing companies, value inventory at the
end of each accounting period.
For an organisation that is not required to make a profit (a ‘not-for-profit organisation’, such as a
government department, state-owned agency or charity), it is important to know how much items
cost, in order to:
 control the entity;
 measure to what extent it is achieving its objectives; and
 Plan expenditure for the future.
Terminology

Definitions: Cost object


Cost object: Any activity for which a separate measurement of costs is needed

Examples of cost objects include:


 The cost of a product
 The cost of a service
 The cost of a department
 The cost of a project

Definitions: Cost unit


Cost unit: A unit of product or service for which costs are determined

A cost unit is the basic unit of production for which costs are being measured.
The term cost unit should not be confused with the term unit cost.

© Emile Woolf International 306 The Institute of Chartered Accountants of Pakistan


Chapter 6: Introduction to cost of production

Definitions: Unit cost


Unit cost is the cost incurred by a company to produce, store and sell one unit of a particular
product.
Unit cost includes all fixed costs and all variable costs involved in production

Cost objects and cost units should be selected so as to provide management with the cost
information they require.
Here are some examples of cost objects and cost units

Industry/activity Cost object Cost unit


Car manufacture Cars produced A car
Bakery Bread produced A batch of bread items
Steel works Steel produced Tonne of steel
Carpet manufacture Carpets produced Square metre of carpet
Retail operation Cost of items sold An item
Passenger transport Cost of transporting Cost per passenger/mile
service customers (i.e. average cost of transporting a
passenger one mile)
Road haulage Cost of transporting Cost per tonne/mile
items (i.e. average cost of carrying one tonne of
items for one mile)
University Cost of teaching Cost per student

Illustration 03:
A company manufactures tinned foods.
It has two products, tinned carrots and tinned beans. In its costing system, it has two cost objects,
carrots and beans.
Cost object Cost unit
1 Carrots Production cost per tin of carrots
2 Beans Production cost per tin of beans

Illustration 04:
A transport company has a bus depot.
The company has a cost accounting system that records and measures the cost of operating the
bus depot.
The costs of operating the depot are measured in three ways, as follows:
Cost object Cost unit
1 Buses Operating cost per bus per month
2 Bus routes Operating cost per month for each bus route
3 Bus drivers Cost of operating the depot per month, per bus driver
employed

© Emile Woolf International 307 The Institute of Chartered Accountants of Pakistan


Financial accounting and reporting I

3.2 Cost classification: Introduction


Costs can be classified in a number of ways including:
 Type of cost (material, labour, other expenses);
 Function of the cost:
 Production
 Non-production

 Selling

 Distribution

 Administration

 Finance
 Cost behaviour – i.e. how the cost varies at different levels of activity:
 costs may stay constant at different levels of activity - fixed costs; or
 costs may stay vary at different levels of activity – variable costs
 Whether the cost can be directly attributed to units of production.
 Whether a cost is recognised in this period (period cost) or is carried forward as part of the
inventory valuation (product cost).
Each of these will be explained in turn but before that note that the above classifications are not
mutually exclusive.

Illustration 05:
A car maker uses steel:
Steel is material.
Steel is a production cost (you cannot make a car without using steel).
Steel is a cost which varies with the number of cars produced.
Steel can be directly attributable to a car.
Steel is a product cost.

© Emile Woolf International 308 The Institute of Chartered Accountants of Pakistan


Chapter 6: Introduction to cost of production

4 COST CLASSIFICATION BY TYPE AND FUNCTION


Section overview

 Cost classification by type


 Cost classification by function
 Production or non-production
 Usefulness of classifying costs by function
 The importance of separating production and non-production costs
 Reporting profit

4.1 Cost classification by type


Material costs
Material costs are the costs of any material items purchased from suppliers, with the intention of
using them or consuming them in the fairly short-term future.
In a manufacturing company, material costs include the cost of the raw materials that go into
producing the manufactured output.
In an office, costs of materials consumed include the costs of stationery and replacement printer
cartridges for the office laser printers.
Labour costs
Labour costs are the remuneration costs of all employees employed and paid by the entity. This
includes the wages and salaries of part-time workers and the costs of any bonuses, pension
contributions and other items that are paid in addition to basic wages and salaries.
Other expenses
Other expenses include the costs of any items that are not material costs or labour costs. They
include the cost of services provided by external suppliers (the charges made by sub-contractors,
charges for repairs by external contractors, rental costs, telephone costs, insurance costs, costs
of energy (gas, electricity), travelling and subsistence expenses, and depreciation charges for non-
current assets.
In a cost accounting system, all these items of cost must be recorded, and there needs to be an
organised system for recording them. Cost items need to be grouped into categories of similar
costs.

4.2 Cost classification by function


A manufacturing company would classify costs according to their function: categorised as either:
 production costs (manufacturing costs); and
 non-production costs (non-manufacturing costs).
Production costs
Production costs are the costs incurred in manufacturing finished products, up to the time that the
manufacture of the goods is completed, and the goods are either transferred to the finished goods
inventory or delivered immediately to the customer.
Production costs include:
 the material cost of the raw materials and components, purchased from suppliers and used
in the production of the goods that are manufactured.
 the labour cost of all employees working for the manufacturing function, such as machine
operators, supervisors, factory supervisors and the factory manager.
 other expenses of the factory, such as rental costs for the factory building, energy costs and
the cost of depreciation of factory machinery.

© Emile Woolf International 309 The Institute of Chartered Accountants of Pakistan


Financial accounting and reporting I

Non-production costs
Non-production costs are any items of cost that are not production costs.
Non-production costs can be further classified according to their function as:
 selling costs;
 distribution costs;
 administrative costs;
 Finance costs.
Selling and distribution costs (marketing costs)
Selling and distribution costs are the costs incurred in marketing and selling goods or services to
customers, and the costs of delivering the goods to customers. The costs of after-sales services,
such as customer support services, are usually included in these costs. Sales and distribution costs
include:
 the wages and salary costs of all employees working in the selling and distribution
departments, including sales commissions for sales representatives.
 advertising costs and other marketing costs.
 operating costs for delivery vehicles (for delivering finished goods to customers), such as
fuel costs and vehicle repair costs.
 other costs, including depreciation costs for the delivery vehicles.
Administration costs
Administration costs are the costs of providing administration services for the entity. They might be
called ‘head office costs’ and usually include the costs of the human relations department and
accounting department. They should include:
 the salary costs of all the staff working in the administration departments.
 the costs of the office space used by these departments, such as office rental costs.
 other administration expenses, such as the costs of heating and lighting for the
administration offices, the depreciation costs of equipment used by the administration
departments, fees paid to the company’s solicitors for legal services, costs of office
stationery and so on.
Finance costs
Finance costs include costs that are involved in financing the organisation, for example, loan
interest or bank overdraft charges.
Finance costs might be included in general administration costs. Alternatively, finance costs might
be excluded from the cost accounting system because they are relevant to financial reporting (and
the financial accounting system) but are not relevant to the measurement of costs.
4.3 Production or non-production
Some costs might be partly production costs, partly administration costs and partly selling and
distribution costs. For example:
 The salary of the managing director, because the managing director spends time on all
aspects of the company’s operations.
 Building rental costs, when the same building is used by more than one function. For
example, administration staff and sales staff might share the same offices.
When costs are shared between two or more functions, they are divided between the functions on
a fair basis.
For example, the salary of the managing director might be divided equally manufacturing costs,
administration costs and sales and distribution costs.
Dividing shared costs on a fair basis is called apportionment of the cost.

© Emile Woolf International 310 The Institute of Chartered Accountants of Pakistan


Chapter 6: Introduction to cost of production

Illustration 06:
A company uses three categories of functional cost in its cost accounting system. These are
manufacturing costs, administration costs and sales and distribution costs.
Identify the functional cost category for each of the following costs:

1 Salary of the chief accountant

Accounting department costs are an administration cost, and the salary of the chief
accountant is treated in full as an administration costs.

2 Telephone charges

These are usually treated as administration costs, unless the charges can be traced directly
to telephones in the manufacturing department or the sales and distribution department.
When charges can be traced directly to telephones in the manufacturing department, they
should be recorded as manufacturing costs.

3 Cost of office cleaning services

These are usually treated as administration costs, unless the charges can be traced directly
to offices used by the sales and distribution staff, or the production staff.

4 Cost of warehouse staff

These are manufacturing costs when the warehouse is used to store raw materials and
components. They are sales and distribution costs when the warehouse is used to store
finished goods. If the warehouse stores raw materials and finished goods, the wages costs
should be apportioned between production costs and sales and distribution costs.

4.4 Usefulness of classifying costs by function


Separating costs into the costs for each function can provide useful information for management.
Functional costs show managers what they are expected to spend on each function (budgeted
costs) and how much they are actually spending.

Illustration 07:
Functional costs might be used in an income statement to report the profit or loss of a company
during a financial period, as follows:

Rs m Rs m
Sales revenue 600
Manufacturing cost of sales 200
Gross profit 400
Administration costs 120
Selling and distribution costs 230
350
Net profit (or net loss) 50

© Emile Woolf International 311 The Institute of Chartered Accountants of Pakistan


Financial accounting and reporting I

4.5 The importance of separating production and non-production costs


Inventory
It is important to separate production costs from non-production costs in a manufacturing business
for the purpose of valuing closing inventory which will consist of:
 finished goods that have been produced during the financial period but not yet sold (finished
goods inventory); and
 partly finished production (Work-in-Progress or WIP).
The costs of finished goods and Work-in-Progress consist of their production costs.
Total production costs during a period must therefore be divided or shared between:
 goods produced and sold in the period;
 goods produced but not yet sold (finished goods);
 work-in-progress.
Non-production costs e.g. salaries of sales persons are never included in the cost of inventory.

4.6 Reporting profit


Profit is the revenue for a financial period minus the costs for the period. The profit or loss earned
during a financial period is reported in a statement of comprehensive income (also known as an
income statement).
In most financial accounting examples, the cost of sales figure is built from purchases as adjusted
by inventory movement.
It is comprised of the cost of goods manufactured (instead of purchases) as adjusted by finished
goods inventory movement. The cost of goods made is a more complex figure than purchases. It
comprises direct materials used, direct labour and production overheads adjusted by movement in
work in progress in the year. It is often constructed in a manufacturing account. The total from this
account feeds into the statement of comprehensive income.

© Emile Woolf International 312 The Institute of Chartered Accountants of Pakistan


Chapter 6: Introduction to cost of production

5 FIXED AND VARIABLE COSTS


Section overview

 Cost behaviour
 Fixed costs
 Variable costs
 Semi-variable costs
 Stepped costs

5.1 Cost behaviour


Cost behaviour refers to the way in which costs change as the volume of activity changes. The
volume of activity may be:
 the volume of sales;
 the volume of production;
 total labour hours worked, machine hours worked;
 the number of production units inspected;
 the number of batches
 the number of journeys (for buses or trains) or deliveries, and so on.
As a general rule, total costs are expected to increase as the volume of activity rises.
Management might want information about estimated costs, or about what costs should have been.
An understanding of cost behaviour is necessary in order to:
 forecast or plan what costs ought to be; and
 compare actual costs that were incurred with what the costs should have been.
The most important classification of costs for the purpose of cost estimation is the division of costs
into fixed costs or variable costs.

5.2 Fixed costs


Fixed costs are items of cost that remain the same in total during a time period, no matter how
many units are produced, and regardless of the volume or scale of activity.
Fixed costs might be specified for a given period of time. In such cases the fixed costs for a longer
period would be scaled up.
Examples of fixed costs include:
 The rental cost of a building is Rs. 40,000 per month. The rental cost is fixed for a given
period: Rs. 40,000 per month or Rs. 480,000 per year.
 The salary costs of a worker who is paid Rs. 11,000 per month. The fixed cost is Rs. 11,000
per month or Rs. 132,000 per year.
Note that as activity levels increase the cost remains fixed. However, the cost per unit falls because
the cost is being spread over a greater number of units.

5.3 Variable costs


Variable costs are costs that increase, usually by the same amount, for each additional unit of
product that is made or each additional unit of service that is provided.
The variable cost of a cost unit is also called the marginal cost of the unit.

© Emile Woolf International 313 The Institute of Chartered Accountants of Pakistan


Financial accounting and reporting I

The variable cost per unit is often the same amount for each additional unit of output or unit of
activity.
This means that total variable costs increase in direct proportion to the total volume of output or
activity.
Examples of variable cost items.
 The cost of buying raw material is Rs.500 per litre regardless of purchase quantity. The
variable cost is Rs.500 per litre:
 the total cost of buying 1,000 litres is Rs. 500,000
 the total cost of buying 2,000 litres would be Rs. 1,000,000.
 The rate of pay for hourly-paid workers is Rs.150 per hour.
 400 hours of labour would cost Rs. 60,000; and
 500 hours would cost Rs. 75,000.
 The time needed to produce an item of product is 4 minutes and labour is paid Rs.150 per
hour.
 direct labour is a variable cost and the direct labour cost per unit produced is Rs.10
(= Rs.150 × 4/60).
 The cost of telephone calls is Rs.1 per minute.
 the cost of telephone calls lasting 6,000 minutes in total would be Rs. 6,000.
Note that as activity levels increase the cost per unit remains fixed. However, the total cost
increases as more units are being made.
Cost behaviour graphs: fixed and variable costs
Cost behaviour for items of cost (or for costs in total) can be shown graphically either showing the
total cost incurred at different activity levels or the cost per item at different activity levels.

Illustration 08: Cost behaviour graphs for fixed costs and variable costs

© Emile Woolf International 314 The Institute of Chartered Accountants of Pakistan


Chapter 6: Introduction to cost of production

5.4 Semi-variable costs


A semi-variable cost, is a cost that is partly fixed and partly variable. A cost behaviour graph
showing the total costs for an item of mixed cost is shown below.

Illustration 09: Semi variable cost

An item of cost that is a mixed cost is an item with a fixed minimum cost per period plus a variable
cost for every unit of activity or output.

Illustration 10:
A company uses a photocopier machine under a rental agreement. The photocopier rental cost is
Rs. 4,000 per month plus Rs.2 per copy produced.
The company makes 15,000 copies during a month:
Total cost is as follows:
Rs.
Fixed cost 4,000
Variable cost (15,000 Rs. 2) 30,000
34,000

Mixed costs are important in cost and management accounting. It is often assumed that the total
costs of an activity are mixed costs, consisting partly of fixed costs and partly of variable costs.
For example, it might be assumed that the total selling and distribution costs for a company each
month are mixed costs. If this assumption is used, the total mixed costs can be divided into two
separate parts, fixed costs and variable costs.
If costs can be analysed as a fixed amount of cost per period plus a variable cost per unit,
estimating what future costs should be, or what actual costs should have been, becomes fairly
simple.

Illustration 11:
The management accountant of a manufacturing company has estimated that production costs in
a factory that manufactures Product Y are fixed costs of Rs. 250,000 per month plus variable costs
of Rs.30 per unit of Product Y output.
The expected output next month is 120,000 units of Product Y.
Expected total costs are therefore:
Rs.
Variable costs (120,000 × Rs.30) 3600,000
Fixed costs 250,000
Total costs 3,850,000

© Emile Woolf International 315 The Institute of Chartered Accountants of Pakistan


Financial accounting and reporting I

5.5 Stepped cost


A stepped fixed cost is a cost which:
 Has a fixed cost behaviour pattern within a limited range of activity, and?
 Goes up or down in steps when the volume of activity rises above or falls below certain
levels.
On a cost behaviour graph, step fixed costs look like steps rising from left to right.

Illustration 12:
Total cost

Activity level

Illustration 13:
A company might pay its supervisors a salary of Rs. 20,000 each month.
When production is less than 2,000 hours each month, only one supervisor is needed:
When production is between 2,001 and 4,000 hours each month, two supervisors are needed.
When output is over 4,000 hours each month, three supervisors are needed.
The cost profile is as follows:

Activity level: Rs.


2,000 hours or less (1 Rs. 20,000) 20,000
2,001 to 4,000 (2 Rs. 20,000) 40,000
Over 4,000 (3 Rs. 20,000) 60,000
The supervision costs are fixed costs within a certain range of output, but go up or down in steps
as the output level rises above or falls below certain levels.

Illustration 14:
On the axes provided, when the vertical axis denotes cost and the horizontal axis denotes
appropriate level of activity, cost behaviour graphs of the following cost are given below:

Fixed costs Variable costs

© Emile Woolf International 316 The Institute of Chartered Accountants of Pakistan


Chapter 6: Introduction to cost of production

Semi-variable costs Annual rates bill

Direct labour cost Annual telephone bill

Direct materials cost if bulk discount is offered on all purchases once the total purchased
exceeds a certain level

Supervisory costs

© Emile Woolf International 317 The Institute of Chartered Accountants of Pakistan


Financial accounting and reporting I

Labour costs if staff is paid a fixed weekly wage for a 35-hour week and any additional
production is completed in overtime, when staff are paid time and a half.

Illustration 15:

In this cost behaviour graph below, if the following cost behaviour exists, then the total cost at
an activity level of 10,000 units are as under;

a) The cost item is a mixed cost. Up to 5,000 units of output, total fixed costs are Rs. 14,000
and the variable cost per unit is Rs. (24,000 – 14,000)/5,000 units = Rs.2 per unit.
b) At the 5,000 units of output, there is a step increase in fixed costs of Rs. 6,000 (from Rs.
24,000 total costs to Rs. 30,000 total costs). Total fixed costs therefore rise from Rs.
14,000 to Rs. 20,000. The variable cost per unit remains unchanged.
c) At the 10,000 units’ level, total costs are therefore:

Rs.
Variable costs (10,000 × Rs.2) 20,000
Fixed costs 20,000
Total costs 40,000

© Emile Woolf International 318 The Institute of Chartered Accountants of Pakistan


Chapter 6: Introduction to cost of production

6 DIRECT AND INDIRECT COSTS


Section overview

 Introduction
 Direct costs
 Indirect costs (overheads)
 Full cost
 Journal entries

6.1 Introduction
Costs may also be classified as:
 direct costs; or
 indirect costs (also known as overheads).
There are direct and indirect material costs, direct and indirect labour costs and direct and indirect
expenses.

6.2 Direct costs

Definition: Direct costs


Direct costs: Costs that can be traced in full to a cost unit.
A direct cost can be attributed in its entirety to the cost of an item that is being produced.

For example, in a manufacturing company that produces television sets, the direct cost of making
a television consists of direct materials and direct labour costs, and possibly some direct expenses.
 The direct materials cost is the cost of the raw materials and components that have gone
into making the television.
 The direct labour cost is the cost of the labour time of the employees who have been directly
involved in making the television.
Direct materials

Definition: Direct materials


Direct materials are all materials that become part of the cost unit.
Direct materials are all materials that can be attributed directly in full to a cost unit.
They are used directly in the manufacture of a product or in providing a service.

Direct materials may consist of either or both:


 raw materials, such as glass, metals and chemicals
 components purchased from an external supplier: for example, the direct materials of a car
manufacturer include components purchased from other suppliers, such as windows, wheels
and tyres.

© Emile Woolf International 319 The Institute of Chartered Accountants of Pakistan


Financial accounting and reporting I

Examples of direct materials include:

Illustration 16: Direct materials

Cost unit Direct materials

Pair of shoes Leather, glue, nails, laces

Office chair Wheels, a stand, a seat (with seat cushion), back rest, arm rests and
fabric

Restaurant meal Ingredients

Car Steel, aluminium, windows, lights, gear box, engine, wheels etc. etc.

House Bricks, wood, cement

Services might also incur some direct materials costs. For example, with catering and restaurant
services the direct materials include the major items of food (and drink).
Direct labour

Definition: Direct labour


Direct labour is labour time that can be attributed directly in full to a cost unit.

Direct labour costs are the specific costs associated with the labour time spent directly on
production of a good or service.
Labour costs are direct costs for work done by direct labour employees. Direct labour employees
are employees whose time is spent directly on producing a manufactured item or service.

Illustration 17: Direct labour employees

Cost unit Direct labour

Car Machinists working in the machining department


Assembly workers in the assembly department
Workers in the spray painting shop

House Bricklayers are direct labour employees of a house-building firm

Tonne of coal Miners

Direct labour costs also include the cost of employees who directly provide a service.

Illustration 18: Direct labour employees (service industry).

Cost unit Direct labour

Day of storage Warehouse staff

Audit (other consultancy Professional staff


product)

Teaching day Teachers (tutorial staff at a college)

© Emile Woolf International 320 The Institute of Chartered Accountants of Pakistan


Chapter 6: Introduction to cost of production

Direct expenses

Definition: Direct expenses


Direct expenses are expenses that can be attributed directly in full to a cost unit.
Direct expenses are expenses that have been incurred in full as a direct consequence of making a
unit of product, or providing a service, or running a department.

In manufacturing, direct expenses are not common for manufactured units of output, and direct
costs normally consist of just direct materials and direct labour costs.

Illustration 19: Direct expenses

Cost unit Direct expense

A house Hire of equipment (for example a cement mixer)


Payment of fees to sub-contractors.

Prime cost
The prime cost of an item is its total direct cost.

Definition: Prime cost


The prime cost of a cost unit is the sum of all of the direct costs of making that unit.

Illustration 20: Prime cost

Rs.
Direct material cost X
Direct labour cost X
Direct expenses X
Prime cost X

6.3 Indirect costs (overheads)


Definition: Indirect cost
An indirect cost (overhead cost) is any cost that is not a direct cost.
Indirect costs (overheads) cannot be attributed directly and in full to a cost unit.

Indirect costs include production overheads and non-production overheads. Each of these might
include indirect materials costs, indirect labour costs and indirect expenses costs.
Indirect material costs
Indirect materials are any materials that are used or consumed that cannot be attributed in full to
the item being costed. Indirect materials are treated as an overhead cost, and may be classified
as production overheads, administration overheads or sales and distribution overheads.
Indirect materials in production include cleaning materials and any materials used by production
departments or staff who are not engaged directly in making a product.
Indirect production materials may also include some items of materials that are inexpensive and
whose cost or value is immaterial. These may include nails, nuts and bolts, buttons and thread,
and so on. The effort of measuring a cost for these materials is not worth the value of the cost
information that would be produced; therefore, these ‘direct’ materials are often treated as indirect
materials.

© Emile Woolf International 321 The Institute of Chartered Accountants of Pakistan


Financial accounting and reporting I

Illustration 21:
In the four types of company, ways in which the fuel cost be treated as direct material cost are as
below:
1 Manufacturing company. Fuel costs are an indirect expense. Fuel used in the company’s
vehicles is unlikely to be considered a material cost at all, but would be treated as an
overhead expense.
2 Road haulage company. Since fuel is a major cost of operating a road haulage service, fuel
costs are likely to be treated as a direct material cost of operations.
3 Construction company. Fuel costs are likely to be an indirect expense, for the same reasons
that apply to a manufacturing company.
4 Motorway service station. This sells fuel to customers. In a retail operation, items sold to
customers are direct costs of sale. The cost of the fuel sold is therefore a direct material cost
(= a cost of sale).

Indirect labour costs

Indirect labour costs consist mainly of the cost of indirect labour employees. Indirect labour
employees are individuals who do not work directly on the items that are produced or the services
that are provided.
Some factory workers do not work directly in the production of cost units but are necessary so that
production takes place. In a manufacturing environment, indirect labour employees include staff in
the stores and materials handling department (for example, fork lift truck drivers), supervisors, and
repairs and maintenance engineers.
All employees in administration departments and marketing departments (sales and distribution
staff) – including management – are normally indirect employees.
Indirect expenses

Many costs incurred cannot be directly linked to cost units.


For example, the rental costs for a factory and the costs of gas and electricity consumption for a
factory cannot be attributed in full to any particular units of production. They are indirect production
costs (production overheads).
In a manufacturing company, all costs of administration are usually treated as indirect costs
(administration overheads) and all or most sales and distribution costs are also usually treated as
sales and distribution overheads.

6.4 Full cost


The full cost of a unit of product (or unit of service) is a cost that includes both direct costs and
some overheads. The full cost of a unit of product might be analysed as follows:
Illustration 22: Full cost
Rs.
Direct materials cost X
Direct labour cost X
Direct expenses X
Prime cost X
Manufacturing overhead (or production overhead) X
Full production cost X
Non-production costs
Administration overhead X
Selling and distribution overhead X
Full cost of sale X

© Emile Woolf International 322 The Institute of Chartered Accountants of Pakistan


Chapter 6: Introduction to cost of production

Notes:
1 Prime cost plus a share of production overheads is the full production cost or ‘fully absorbed
production cost’ of the cost unit.
2 In cost accounting systems, it is common practice to include production overheads in unit
costs and measure the full production cost per unit. However, administration and selling and
distribution overhead costs are not usually included in the cost of each unit. Instead, they
are treated in total as an expense for the period.

© Emile Woolf International 323 The Institute of Chartered Accountants of Pakistan


Financial accounting and reporting I

7 PRODUCT COSTS AND PERIOD COSTS


Section overview

 Product costs and period costs

7.1 Product costs and period costs


Costs are typically classified as either product costs or period costs when preparing financial
statements.

Definition: Product cost


Costs that become part of the cost of goods manufactured are called product costs. Such costs are
incurred on manufacturing process either directly as material and labour costs or indirectly as
overheads. Product costs are costs associated with goods that are produced or purchased for
resale.
Product costs are accounted for as inventory and held on the balance sheet (subject to accounting
valuation rules) until the inventory is sold. Only when the inventory is sold are product costs
expensed in the profit and loss account.

Product costs include the prime cost (direct materials + direct labour + direct expenses) plus the
production overhead.

Definition: Period cost


Period costs are not incurred on the manufacturing process and therefore these cannot be assigned
to cost of goods manufactured. These costs that are deducted as expenses during a particular
period. They do not contribute towards the value of inventory and are therefore not held on the
balance sheet. They are therefore expensed when they occur – i.e. in the period in which they
occurred.

Period costs are the non-production overheads


In summary then
 product costs are expensed when the inventory is sold
 period costs are expensed as soon as they are incurred

Illustration 23: A retailer


A retailer owns a shop, employs a shop assistant, invests in sales and advertising and acquires
goods for resale.
The cost of goods purchased for resale is product costs and accounted for as inventory. These are
only expensed when the goods are sold (which may be in a subsequent accounting period).
The sales and advertising costs and the salary of the shop assistant are period costs which are
expensed immediately in the accounting period in which they were incurred. Note that the salary
of the shop assistant would be called an administration expense.

© Emile Woolf International 324 The Institute of Chartered Accountants of Pakistan


Chapter 6: Introduction to cost of production

8 COST ACCOUNTING CYCLE


Section overview

 Introduction
 Factory ledger
 Journal entries

8.1 Introduction
Cost accounts are an expansion of the general accounts. Accounts describing manufacturing
operations are: Materials, Payroll, Factory Overhead Control, and Work in Process, Finished
Goods and Cost of Goods Sold. The flow of cost from acquisition of materials, through factory
operations to the cost of products sold is recognised and measured through these accounts. In
cost accounting, extensive use of control accounts is made such as materials accounts, factory
overhead account, Work in process account and finished goods account etc.
1. The costing process starts by using the Materials Account to record amounts for procurement
of direct material for production of the goods and its onward transfer to the production floor. In
case of textile industry, the amount includes cost of procuring the yarn, freight inwards, non-
refundable taxes etc. to produce fabric.
Following are some of the materials that fall under this category:
a) Materials which are specifically purchased acquired or produced for a particular job, order
or process.
b) Primary packing material (e.g. carton, wrapping etc.)
2. The Payroll account is then used to record direct labour (human efforts used for conversion of
materials into finished products), salary paid to production supervisor or fabric cutting cost to
a labour etc.)
3. The Factory overhead account is used for recording indirect material (e.g. cleaning supplies
of a factory), indirect labour (e.g. salary of quality assurance staff_, supplies, rent, insurance,
taxes, repairs and other factory expenses incurred on converting a raw material into finished
goods. It is indirect factory related costs that are incurred when producing a product. It is
basically the costs without which a particular product cannot be made. The Company’s cost
procedure determines the type of subsidiary record that supports the Work in Process control
account.
4. The total of cost of goods completed is transferred from the Work in Process control account
to the Finished Goods account.
5. Finally, the cost of goods sold is transferred from the Finished goods account to the Cost of
Goods Sold
6. At the end of each accounting period, Cost of Goods Sold is closed to Statement of Profit &
Loss.

Illustration 24: Cost flow

© Emile Woolf International 325 The Institute of Chartered Accountants of Pakistan


Financial accounting and reporting I

8.2 Factory ledger


A factory ledger is a group of accounts, containing the production costs of a business. These
accounts typically include the following:
 Direct material expenses
 Direct labour expenses
 Factory overhead expense
These accounts may contain the bulk of the expense transactions generated by a business, and
so are of particular interest to the cost accountant or financial analyst who is examining the
financial performance of an organizations.

8.3 Journal entries


Following journal entries are made in order to record the production and inventory cost in a
manufacturing environment.

Ref Account / Description Debit Credit

1 Raw Materials Inventory X

Cash or Accounts Payable X

Purchased raw materials inventory

2 Factory Payroll X

Wages Payable X

Record accrued wages for the period

3 Manufacturing Overhead X

Accumulated Depreciation / Accounts payable / Cash X

Record depreciation Or supplies on credit Or rent paid

4 Work in Process Inventory X

Raw Materials Inventory X

Record Direct Materials Used

5 Manufacturing Overhead X

Raw Materials Inventory X

Record INDIRECT materials used

6 Work in Process Inventory X

Factory Payroll X

Record Direct Labor

© Emile Woolf International 326 The Institute of Chartered Accountants of Pakistan


Chapter 6: Introduction to cost of production

Ref Account / Description Debit Credit

7 Manufacturing Overhead X

Factory Payroll X

Record INDIRECT labor

8 Work in Process Inventory X

Manufacturing Overhead X

Record Overhead APPLIED to production

9 Finished Goods Inventory X

Work in Process Inventory X

Record jobs or goods completed (cost of goods manufactured)

10 Cost of goods sold X

Finished Goods Inventory X

Record cost of jobs or goods completed and sold

© Emile Woolf International 327 The Institute of Chartered Accountants of Pakistan


Financial accounting and reporting I

9 MANUFACTURING ACCOUNT
Section overview
 Preparation of manufacturing account

9.1 Preparation of Manufacturing Account


A manufacturing account shows the cost of producing the goods that are sold during an accounting
period. It is split into the following sections:
 Prime cost - Direct costs of physically making the products (e.g. raw materials)
 Overhead cost - Other indirect costs associated with production but not in a direct manner
The cost of manufacturing the products will be the total of the prime cost and the overhead cost
added together.
Prime cost
The prime cost covers all direct the costs involved in physically making the products. Common
examples would include:
 Direct materials
 Direct labour/wages
 Other direct costs (e.g. packaging, royalties)
Cost of raw materials consumed
Within the prime cost adjustments will have to be made for opening and closing stocks of raw
materials. There may also be carriage inwards charged on the raw materials and returns outwards
of materials sent back to their original supplier. The overall charge for materials is referred to as
cost of raw materials consumed. The overall movement is shown in the manufacturing account as
illustrated below.
 Opening inventory
 Add: Purchases
 Less: Closing inventory
 Raw material consumed
Overhead cost
This section includes all other expenses related to the production of goods in a direct manner. This
means that if the level of production increased, then these expenses may also increase but not by
the same proportion. These are sometimes known as indirect costs, factory overheads or indirect
manufacturing costs. Common examples of overhead costs would include:
 Factory rent
 Indirect labour
 Depreciation of factory plant and equipment
Depreciation of fixed assets should be included in this section only if it is depreciation on assets
included for production. For example, depreciation of machinery would appear as an overhead
cost but depreciation of office equipment would appear in the profit and loss account.

© Emile Woolf International 328 The Institute of Chartered Accountants of Pakistan


Chapter 6: Introduction to cost of production

Illustration 25: Manufacturing account


Rs. Rs.
Raw materials
Opening inventory 25,000
Purchases 150,000
175,000
Less: Closing inventory (20,000)
Raw materials consumed 155,000
Manufacturing wages 100,000
Prime cost 255,000
Overheads
Light and power 72,000
Depreciation of production machinery 40,000
Depreciation of factory 50,000
162,000
Manufacturing costs 417,000
Opening work in progress 85,000
Closing work in progress (95,000)
Cost of goods manufactured 407,000

The cost of goods made is transferred to the statement of comprehensive income.

Illustration 26: Statement of comprehensive income to show transfer of cost of goods made.
Rs. Rs.
Sales revenue 800,000
Less cost of goods sold
Opening inventory of finished goods 50,000
Cost of goods manufactured 407,000
457,000
Closing inventory of finished goods (40,000)
Cost of sales (417,000)
Gross profit 383,000
Administration costs 86,000
Selling and distribution costs 94,000
(180,000)
Net profit for the period 203,000

© Emile Woolf International 329 The Institute of Chartered Accountants of Pakistan


Financial accounting and reporting I

Example 01:
Question: The following data has been extracted from the books of Beauty Bars Ltd at 31 December
2018:
Dr Cr.
Raw materials 15,000
Work in progress 10,000
Finished goods 25,000
Purchases of raw materials 50,000
Sales 500,000
Direct Labour 20,000
Rent 22,000
Electricity 18,000
Office Salaries 30,000
Depreciation for the year:
Office 7,000
Factory 3,000
Advertisement 16,000

Additional information:
Inventory as on 31.12.2018
Raw materials 11,000
Work in progress 6,000
Finished goods 10,000
i. Rent and electricity are to be apportioned: Factory 70%, Office 30%
ii. Finished goods are to be transferred to the trading account at a profit of
25% on factory cost.
Required:
Prepare the manufacturing account of Beauty Bars Limited for the period ended 31
December 2018 and then prepare the statement of comprehensive income.

Answer:
Beauty Bars Limited
Manufacturing account
For the period ended 31 December 2018
Rs. Rs.
Raw materials
Opening inventory 15,000
Purchases 50,000
65,000
Less: Closing inventory (11,000)
Raw materials consumed 54,000
Manufacturing wages 20,000
Prime cost 74,000

Overheads
Rent 15,400
Electricity 12,600
Depreciation 3,000
Total overheads 31,000
Manufacturing costs 105,000
Opening work in progress 10,000
Closing work in progress (6,000)
Cost of goods manufactured 109,000

© Emile Woolf International 330 The Institute of Chartered Accountants of Pakistan


Chapter 6: Introduction to cost of production

Beauty Bars Limited


Statement of comprehensive income
For the period ended 31 December 2018
Rs. Rs.
Sales revenue 500,000
Less cost of goods sold
Opening inventory of finished goods 25,000
Cost of goods manufactured (109,000 x 125%) 136,250
161,250
Closing inventory of finished goods (10,000)
Cost of sales (151,250)
Gross profit 348,750
Administration costs 72,000
Selling and distribution costs 16,000
88,000
Net profit for the period 260,750

Example 02: SIGMA LTD


Question:
The managing director of Sigma Ltd is concerned about the differences between the reports
produced and records maintained by you, as management accountant, and by the company’s
financial accountant.
Required:
Explain the differences between:
(i) the profit statements produced, and
(ii) the accounting records maintained by the two of you.

Answer:
(i) Difference between profit statements produced.
The differences in the statements reflect the different uses of the two sets of accounts, as
detailed below.
 Financial accounts analyse costs by function (e.g. production, selling, finance, etc.) and
comply with generally accepted accounting practice (e.g. IASs) and relevant legislation, as
they are used externally by shareholders and creditors.
 Management accounts analyse costs by nature (e.g. fixed, variable, semi-variable, etc.), as
they are used internally for decision-making.
 Management accounts profit statements may be prepared either on absorption or on a
marginal costing basis, the latter giving better information for short-term decision-making,
as fixed costs are treated as period costs and charged to the profit and loss account when
incurred. Management accounts record costs through cost centres (departments) and cost
units (products) to give responsibility for control of costs to individuals. A standard costing
system may be used in the business to analyse variances effectively, and management
accounts profit statements prepared, say, on a departmental basis, may include notional
intra-departmental charges (e.g. for rent).

© Emile Woolf International 331 The Institute of Chartered Accountants of Pakistan


Financial accounting and reporting I

 Financial accounts in the main give a financial record of past transactions but are very
limited in their use for control as they do not separate fixed and variable costs. They must
be prepared on an absorption basis, where fixed production overheads are treated as
product costs and charged to the income and expenditure account when units are sold, in
line with the accruals basis in IAS 1 (and The Framework) and inventory measurement
principle in IAS 2. The financial accounts profit will include non-cost items, such as finance
costs and profits or losses on disposal of assets.

(ii) Difference between accounting records maintained


 The accounting records for financial accounts are summarised records accounting for costs
as they accrue.
 The accounting records for management accounts are far more detailed in that they break
down costs by centre and unit for control purposes.
 The cost accounts produced by the management accountant may be integrated within the
financial ledgers where the basis of valuation used in each is the same. However, the cost
accounts may well be kept separately from the financial accounts in a larger organisation
where different people can be responsible for the different areas.
 Where cost accounts are kept separate, control accounts are maintained in both sets of
books to ensure that the double entry is maintained in each system.

Example 03: MANAGEMENT INFORMATION FUNCTIONS


Question: Outline the three main functions of management for which information must be
provided.
Answer:
The three main functions of management for which information must be provided are as under:
The role of the management accountant within an organisation is to provide information for
management in order that they may manage effectively. Although the information that the
management accountant must provide will be specific to the organisation and the industry in which
it operates, there are three main functions for which information is required – planning, decision-
making and control.
The planning activities of an individual manager will depend on the objectives of the organisation
as a whole. It will be assumed that normally these objectives will include the achievement of at
least a target level of profit. Planning must take place to ensure that those products are sold which
give the highest contribution towards profit. Therefore, information regarding the revenue and costs
for each product under consideration will be needed in order that relative profitability may be
evaluated.
Once all the necessary information is available, the decision-making process can take place. The
chosen production plan must be expressed in financial terms as well as in terms of units of product.
At the end of the accounting period under consideration, actual production and sales figures must
be compared with the results expected in the original plan. This is necessary for management to
control the business properly. Where there are differences between actual and planned
performance, investigation may be required so that, if necessary, corrective action may be taken.

© Emile Woolf International 332 The Institute of Chartered Accountants of Pakistan


Chapter 6: Introduction to cost of production

Example 04: JOHN PIRELLI


Question:
John Pirelli has been running a small printing business for the past six months; his accounting
records are limited to an analysed cash book, cheque book stubs and a file of invoices. Both he and
his accountant are happy with this for the preparation of annual accounts for the Inland Revenue
and the bank, but John Pirelli now wants more information for controlling the business.
When talking to his accountant about setting up a suitable costing system, John Pirelli was clear
about the difference between management and financial accounts. However, he became very
confused over different categories of cost and has asked you for some clarification.
Required:
Explain the distinction between:
(i) direct and indirect costs
(ii) fixed and variable costs
(iii) production and non-production costs
(iv) committed and discretionary costs.

Answer:
(i) Difference between direct and indirect costs
Direct cost is also called prime cost; indirect costs are often referred to as overheads.
Direct costs are those specifically attributable to units of output (clients’ jobs); these would
include printers’ time, paper costs, and plate-making costs.
Indirect costs are those not capable of such close matching, such as rent and rates, insurance,
depreciation of machinery.

(ii) Difference between fixed and variable costs


Fixed costs are those independent of the level of output (the amount of printing work done).
Into this category would come rent and rates, advertising, audit fee, electricity for lighting and
heating.
Variable costs increase as output increases, such as paper costs, electricity costs for powering
printing presses and the cost of ink or plates.
A third category of cost is “semi-variable”, such as electricity (with a fixed and variable element).
These three can best be described graphically.

(iii) Difference between production and non-production costs


The category of “production costs” is important to the extent that such types of cost can be
incorporated in the valuation of any stocks of finished work at the end of an accounting period
which in turn is needed for profit determination. Examples of these costs would be as follows.
Production Paper, all print-room costs
Non-production Your secretary’s salary, advertising, delivery van, running expenses

© Emile Woolf International 333 The Institute of Chartered Accountants of Pakistan


Financial accounting and reporting I

(iv) Difference between Committed and discretionary costs.


Committed costs are those essential for the running of the business: paper, depreciation of
presses, assistant printer’s wages, rent of printing room.
Discretionary costs are incurred at the whim of management: machine maintenance contract
charges, cost of Christmas party, advertising costs.
Note
Whatever you call them, it is necessary to recover all these costs from fees charged to customers.
To do that you need to know how much is their amount. Hence the need for your costing system.

Example 05: CLASSIFICATION OF COSTS


Question:
(a) Explain the terms fixed, variable and semi-variable costs.
(b) Classify the following expenses under the headings in (a):
(i) Telephone charges
(ii) Factory insurance
(iii) Legal expenses
(iv) Social security (%)
(v) Rent of premises
(vi) Light and heat
(vii) Direct materials
(viii) Lift operator’s wages
(ix) Machine servicing and repairs
(x) Foreman’s salary
(xi) Contract cleaning services
(xii) Casual labour

Answer:
(a) The concept of fixed, variable and semi-variable costs can be explained as under.
Fixed, variable and semi-variable costs
A fixed cost item is one for which the expenditure will not be affected by changes in the level
of activity. In general, fixed costs are incurred in providing the facilities or conditions to
undertake production. Therefore, they are more usually incurred in relation to time periods
than to activity. This is not to say that such costs are a constant amount even for short periods
of time. Clearly, price changes will vary the amount, as in the instance of paying local rates
based on rateable value where the annual charges are in line with inflation. Finally, fixed cost
per unit of product varies inversely with output.
 A variable cost item is one for which the total expenditure will tend to vary more or less
directly with output or activity. Nevertheless, the variable cost expenditure may also vary as
the result of other influences, such as inflation or competition or even changes in supply. It
is possible that any change in the number of units purchased could be more than offset by
an opposite price changing. Even so, the total expenditure at the new price will vary directly
with output. Variable cost per unit of product tends to be more or less constant at each
different level of output, other things being equal.
 A semi-variable cost is one for which the total expenditure tends to vary directly with the
volume of output/activity, but proportionately less than the change in output/activity.
Generally, such cost items are composites with a variable element and a fixed element. A
good example is telephone charges in which the rental is a fixed charge and payable
irrespective of activity levels. The variable element comprises the charge for calls made and
tends to be related to business activity. The cost per unit of product will reflect both.

© Emile Woolf International 334 The Institute of Chartered Accountants of Pakistan


Chapter 6: Introduction to cost of production

(b) Classification of the following type of cost into fixed, variable and semi-variable cost is as
under:
Fixed Variable Semi-variable
Telephone charges 
Factory insurance 
Legal expenses 
Social security (%) 
Rent of premises 
Light and heat 
Direct materials 
Lift operator’s wages 
Machine servicing and repairs 
Foreman’s salary 
Contract cleaning services 
Casual labour 

Tutorial note: Each classification is open to debate. Hard and fast rules cannot be laid down; precise
classification would depend upon the particular circumstances of the firm.

Example 06: CAR MAINTENANCE


Question: The following particulars/projections pertain to a well-maintained medium-sized car:

Rupees
Cost of car 1,200,000
Salvage value after 100,000 kilo meters (km) 300,000
Maintenance cost:
– Service after every 5,000 km 6,000

– Replacement of spares/parts (per 2,000 km) 4,000


Vehicle tax per annum (20% adjustable against income tax payable by the 7,500
owner)

Insurance per annum 36,000


Cost of petrol per liter 75
Cost of tyres replacement after 25,000 km 20,000

On an average, the car consumes one litre for every 15 km.


Required:
(a) For three different levels of use i.e. 10,000, 20,000 and 30,000 km per annum, prepare a
schedule showing:
 Variable, fixed and total costs
 Variable, fixed and total costs per km
(b) In respect of each type of cost, give appropriate justification for treating it as a variable or a
fixed cost.

© Emile Woolf International 335 The Institute of Chartered Accountants of Pakistan


Financial accounting and reporting I

Answer:
a) The variable, fixed and total cost per kilometre and in total at activity levels of 10,000, 20,000
and 30,000 km per annum are as under:
CAR MAINTENANCE

-------------- Kilometers --------------


A 10,000 20,000 30,000
Variable costs: -------------- Amount in Rs. --------------
Maintenance - service after every 5000 km 12,000 24,000 36,000
Spares 20,000 40,000 60,000
Petrol 50,000 100,000 150,000
Provision for replacement of tyres
(20,000÷25,000×A) 8,000 16,000 24,000
Depreciation
[(1,200,000 - 300,000)/100,000×A] 90,000 180,000 270,000
B 180,000 360,000 540,000
Fixed costs:
Vehicle tax 6,000 6,000 6,000
Insurance 36,000 36,000 36,000
C 42,000 42,000 42,000
Total cost 222,000 402,000 582,000

Variable cost per km(B÷A) 18.00 18.00 18.00


Fixed cost per km(C÷A) 4.20 2.10 1.40
Total cost per km 22.20 20.10 19.40

(b) In respect of each type of cost, qualification for treatment are given below:

(i) Spares and petrol:


Spares and petrol are variable costs being dependent on the usage of car.

(ii) Depreciation:
The depreciation in this case is variable because it is being charged on the basis of actual
running/usage and not on the basis of time.

(iii) Maintenance:
Service is to be done after each 5,000 kiss and is therefore a variable cost.

(iv) Vehicle tax:


Vehicle tax is payable per year irrespective of actual running and is therefore a fixed cost.

(v) Insurance:
Insurance is payable per year irrespective of usage and is therefore a fixed cost.

(vi) Tyres:
If the cost of tyres is accrued on the basis of usage, it would be a variable cost.

© Emile Woolf International 336 The Institute of Chartered Accountants of Pakistan


Chapter 6: Introduction to cost of production

Example 07:
Question: Describe briefly three major differences b/w:
(i) financial accounting
(ii) cost and management accounting??
Answer:
The three major differences b/w:
i. Financial Accounting
1. It concerns with financial information
2. Primarily deals with financial reporting for external use
3. Primarily deals with monetary values
ii. Cost and management accounting
1. It concerns with both financial as well as cost information
2. Primarily deals with cost information for internal use
3. Deals with both monetary and non-monetary information

Example 08: AB
Question: The following balances were extracted from AB's accounts as at 31 March 2007.
Rs. in "000"
Sales 3,200
Purchase of raw material 450
Purchase returns 18
Carriage inward 10
Direct labor 400
Direct overhead 60
Rent 40
Electricity 30
Insurance 55
Factory supervision salaries 65
Office salaries 70
Indirect factory wages 13
Factory cleaning 50
Office cleaning 50
Stocks at 1 April 2006:
- Raw material 110
- Work in progress 55
- Finished goods 80
Factory machinery cost 640
Provision for depreciation on factory machinery 280
Additional information at 31 March 2007:

Rent Prepaid 5
Electricity accrued 15
Insurance prepaid 10

© Emile Woolf International 337 The Institute of Chartered Accountants of Pakistan


Financial accounting and reporting I

Rs. in "000"
Stocks
Raw material 140
Work in progress 75
Finished goods 170

- Depreciation on factory machinery is to be provided at 25% per annum reducing balance.


- Rent electricity and insurance are apportioned on the basis of 80% to factory and 20% to
office.
- Finished goods are transferred to the trading account factory cost plus one third.
Required:
Prepare AB's manufacturing account for the year ended 31 March 2007.

Answer:
M/S AB
Cost of Goods Manufactured
For the period ended 31 March 2007
Rs. In "000" Rs. In "000"
Stock of raw material at 1 April 2006 110
Add: Purchases 450
Carriage inwards 10
460
less returns 18 442
552
less Stock of raw material at 31 March 2007 (140)
412
Direct Labor 400
Direct overheads 60
Prime Cost 872

Rs. In "000" Rs. In "000"


Factory Overhead
Rent 28
Electricity 36
Insurance 36
Supervisory Salaries 65
Indirect wages 13
Cleaning 50
Provision for depreciation on machinery 90 318
1,190
Work in Progress at 1 April 2006 55
less Work in progress at 31 March 2007 75 (20)
Cost of Goods Manufactured 1,170

© Emile Woolf International 338 The Institute of Chartered Accountants of Pakistan


Chapter 6: Introduction to cost of production

Example 09: BABLU LIMITED


Question:
Following is the list of balances of Bablu limited:

Rupees
Opening stock-Finished goods 50,000
Purchases of Raw Material 2,450,000
Custom duty 122,500
Sales tax paid on purchases of RM 392,000
Carriage paid on RM purchases 49,000
Demurrage and octroi 73,500
Opening stock-Raw Material 421,500
Opening Work-in-process 121,100
Labour wages paid 1,685,500
Supervisory salaries 250,000
Indirect labour wages 450,000
Office staff salaries 500,000
Sales team salaries 125,000
Postage and telegram 12,500
Fuel expenses 400,000
Electricity 560,000
Rent rates and taxes 345,000
Property taxes 14,570
finished goods purchases 985,000
Carriage paid on finished goods purchases 19,700
Carriage out 27,000
Other income 24,500
Interest expense 148,500
Interest income 16,500
Distribution expense 200,000
Depreciation 650,000
Repair and maintenance 257,850
Canteen expenses 140,000
Sales tax received on sales 2,640,000
Purchase return –Raw Material 100,000
Purchase return –Finished Goods 85,000
Purchase discount –Raw Material 40,000
Sales returns 200,000
Sales discount 24,500
Bad debts 21,450
Sales 16,500,000

© Emile Woolf International 339 The Institute of Chartered Accountants of Pakistan


Financial accounting and reporting I

Closing stock-Raw Material 350,000


Closing stock –Finished Goods 421,000
Closing-Work-in-process 100,000
- Depreciation-70%factory, 30% office
- Electricity -65%factory, 35% office
- Fuel -85%factory, 15% office
- Repairs-90%factory, 10% office
- Canteen -80%factory, 20% office
- Opening provision for doubtful debts Rs. 5,000 and closing balance Rs. 40,000
- Wages paid in advance Rs. 20,000
- Wages accrued and unpaid Rs. 40,000
- Sales Commission 5% of sales
- Directors remuneration -5% of net profit
Required:
Calculate the cost of goods manufactured and prepare statement of comprehensive income for
Bablu Limited fusing the above information.

Answer:
M/S Bablu Limited
Cost of Goods Manufactured
Particulars Rupees Rupees
Raw Material
Opening stock 421,500
Purchases 2,450,000
Demurrage and Octroi 73,500
Custom duty 122,500
Purchase return (100,000)
Carriage in 49,000
Closing stock (350,000)
Raw Material Consumed 2,666,500
Labour wages 1,705,500
Prime cost 4,372,000
Factory Overheads
Supervisor salary 250,000
Indirect labour wages 450,000
Fuel Expense 340,000
Electricity 364,000
Rent rate 345,000
Depreciation 455,000
Repairs & maintenance 232,065
Canteen expense 112,000
2,548,065
Manufacturing cost during the year 6,920,065

Opening Work-in-process 121,100


Closing Work-in-process (100,000)
Cost of Goods Manufactured 6,941,165

© Emile Woolf International 340 The Institute of Chartered Accountants of Pakistan


Chapter 6: Introduction to cost of production

M/S Bablu
Trading, Profit and loss account

Sales 16,500,000
Sales return (200,000) 16,300,000
Cost of Goods Sold
Opening stock 50,000
Purchases 985,000
Purchase return (85,000)
Carriage in 19,700
Cost of Goods Manufactured 6,941,165
Closing stock (421,000) 7,489,865
Gross profit 8,810,135
Operating expenses
Office staff salaries (500,000)
Sales team salaries (125,000)
Postage and telegram (12,500)
Carriage outward (27,000)
Interest expense (148,500)
Sales discount (24,500)
Bad debts expense (56,450)
Depreciation expense (195,000)
Electricity expense (196,000)
Fuel expense (60,000)
Distribution expense (200,000)
Repair and maintenance (25,785)
Canteen expenses (28,000)
Sales commission (957,000)
Property tax (14,570)
Directors remuneration (316,042)

Other Income
Purchase discount 40,000
Other income 24,500
Interest income 16,500
Net profit 6,004,789

© Emile Woolf International 341 The Institute of Chartered Accountants of Pakistan


Financial accounting and reporting I

Example 10: ABC Limited


Question: Following is the list of balances of ABC limited for the period ended 31 December 2015.
Rupees
Opening stock-Finished Goods 50,000
Purchases of Raw Material 2,450,000
Trade discount (30% on Direct Raw Material purchases,20% on Indirect Raw
Material purchases and 50% on Finished Goods purchases) 80,000
Custom duty (80% relates to Direct Raw Material and 20% relates to Finished
Goods purchases) 122,500
Import duty (40% is refundable) (70% relates to Raw Material and 30% relates
to Finished Goods purchases) 40,000
Sales tax paid on purchases of Raw Material (20% is non-refundable) 392,000
LC Charges (30% relates to Raw Material and 70% relates to Finished Goods
purchases) 90,000
Advance tax paid (30% relates to Raw Material purchases, 20% relates to
Indirect Raw Material purchases and remaining relates to Finished Goods
purchases) 30,000
Carriage paid (50% relates to Direct Raw Material purchases,20% relates to
Indirect Raw Material purchases and 30% relates to Finished Goods Purchases ) 49,000
Purchase of Indirect Raw Material 40,000
Octroi charges (50% relates to Direct Raw Material and remaining relates to
Indirect Raw Material) 73,500
Demurrage charges (60% relates to Direct Raw Material and remaining relates
to Indirect Raw Material) 20,000
Ab. Demurrage charges (90% relates to Direct Raw Material and remaining
relates to Indirect Raw Material) 5,000
Penalty to driver carrying Raw Material to factory 3,000
Opening stock-Direct Raw Material 421,500
Opening Work-in-process 121,100
Labor wages paid (60% relates to *manufacturing and 40%relates to admin) 2,185,500
Sales team salaries 125,000
Postage and telegram 12,500
Fuel expenses 400,000
Electricity 560,000
Rent rates and taxes 345,000
Property taxes 14,570
Finished goods purchases 985,000
Carriage out 27,000
Other income 24,500
Interest paid 148,500
interest income 16,500
Distribution expense 200,000
Depreciation 650,000
Repair and maintenance 257,850
Canteen expenses 140,000
Sales tax received on sales 2,640,000
Purchase return –Direct Raw Material 100,000
Purchase return –Finished Goods 85,000
sales returns 200,000
sales discount 24,500
Bad debts 21,450
sales 16,500,000
settlement discount received (30% on Direct Raw Material purchases,20% on
Indirect Raw Material purchases and 50% on Finished Goods purchases) 500,000

© Emile Woolf International 342 The Institute of Chartered Accountants of Pakistan


Chapter 6: Introduction to cost of production

Other Information
Closing stock- Raw Material 350,000
Closing stock –Finished Goods 421,000
Closing-Work-in-process 100,000
Closing stock-Indirect Raw Material 20,000
- Depreciation-70%factory, 30% office
- Accrued electricity is 40,000(Electricity-65%factory, 35% office)
- Fuel exp. Relating to next period is Rs. 20,000(Fuel-85%factory, 15% office)
- Repairs-90%factory, 10% office
- Cantene-80%factory, 20% office
- Opening provision for doubtful debts Rs. 5,000 and closing balance Rs. 40,000
- Wages paid in advance Rs. 30,000 Wages accrued and unpaid Rs. 70,000
- Sales Commission 5% of sales
- Directors remuneration-5% of net profit
- *Labour wages relating to manufacturing includes 80% cost of labour which are directly
involved in production
- Above Interest paid includes interest for next two months from the RD. Annual interest is
charged out
Required:
Prepare the Cost of Goods manufactured for the year ended 31 December 2015 & Statement of
comprehensive Income 31 December 2015.

Answer:
Cost of Goods Manufactured
For the period ended 31, Dec 2015

Rs. Rs.

Opening - Raw Material 421,500


Purchases 2,426,000
Custom Duty 98,000
Import Duty 16,800
Sales tax paid - Non-refundable 78,400
L.C Charged 27,000
Carriage Paid 24,500
Octroi Charges 36,750
Demurrage Charges 12,000
Purchase Return (100,000) 2,619,450

Closing - DRM (350,000)


DRM - Consumed 2,690,950
Direct Labour 1,068,240
Prime Cost 3,759,190
F.O.H

© Emile Woolf International 343 The Institute of Chartered Accountants of Pakistan


Financial accounting and reporting I

Rs. Rs.
Indirect Raw Material:
Opening Stock
Purchases 40,000
Trade Discount (16,000)
Carriage Paid 9,800
Octroi Charges 36,750
Demurrage Charge 8,000
Raw c/s -RN (20,000)
Indirect Raw Material 58,550
Indirect Labour 267,060
Fuel Expense 323,000
Electricity 390,000
Rent Rates & taxes 345,000
Depreciation 455,000
R&M 232,065
Canteen Shop 112,000
2,182,675
Manufacturing Cost 5,941,865
W.I.P (Opening) 1,211,000
W.I.P (Closing) 100,000
Cost of Goods Manufactured 5,962,965
ABC Ltd.
Statement of Comprehensive Income
For the period ended 31, Dec 2015
Rs. Rs.
Sales 16,500,000
Sales Return (200,000)
Net Sales 16,300,000

Opening Stock – Finished Goods 50,000


Purchases - Finished Goods (w1) 945,000
Custom Duty 24,500
Import Duty 7,200
L.C Charges 63,000
Carriage 14,700
Purchases Return - Finished Goods (85,000)
Cost of Goods Manufactured 5,962,965
Closing stock –Finished Goods (421,000)
Cost of Goods Sold (6,561,365)
Gross Profit 9,738,635

© Emile Woolf International 344 The Institute of Chartered Accountants of Pakistan


Chapter 6: Introduction to cost of production

Rs. Rs. Rs.


Operating Expenses
Abnormal Damages 5,000
Penalty on Driver 3,000
Salaries Wages 890,200
Sales team Salaries 125,000
Postage & telegram 12,500
Fuel Expense 57,000
Electricity 210,000
Property Taxes 145,570
Carriages 27,000
Interest Expenses 127,285
Distribution Expenses 200,000
Depreciation 195,000
Repair and maintenance 25,785
Canteen Expenditure 28,000
Sales Discount 245,000
Bad Debts 56,450
Sales Commission 957,000
3,309,790
Other Income
Other Income 24,500
Interest Income 16,500
Settlement Discount 500,000
541,000

Net Profit before Director remuneration 6,969,845


Directors remuneration (331,897)
Net Profit after Director's Remuneration 6,637,948
Working1 Direct Raw Indirect Raw
Material Material Finished Goods
Purchase Price 2,450,000 40,000 985,000
0 0 1
Trade Discount (24,000) (16,000) (40,000)
Purchase Price 2,426,000 24,000 945,000
Working 2
Sales 16,500,000
Sales tax 2,640,000
19,140,000
Commission at 5% 957,000

© Emile Woolf International 345 The Institute of Chartered Accountants of Pakistan


Financial accounting and reporting I

Example 11: Tuesday Manufacturers Limited


Question: Tuesday Manufacturers Limited produces a single product. The following costs were
incurred in the month of June 2019:
Rs. in '000
Direct labour 2,075
Depreciation on plant and machinery 380
Distribution costs 589
Factory manager’s salary 247
Indirect labour 848
Indirect material consumed 345
Raw material purchases 3,845
Selling costs 1,248
Other production overheads 580
Other administration overheads 388

Following other information is available:


(i) On 1 June 2019, stock of finished goods consisted of 1,350 units valued at Rs. 1,640 per
unit while stock of raw materials was valued at Rs. 1,490,000.
(ii) 5,200 units of finished goods were produced during June 2019.
(iii) There was no work-in-progress at the end of the month whereas work in progress at 1 June
2019 was valued at Rs. 208,000.
(iv) Stock of raw materials on 30 June 2019 was valued at Rs. 970,000.
(v) 1,500 units of finished goods were available in stock as on 30 June 2019.
(vi) Cost of finished goods is determined using FIFO method.

Required:
Compute cost of goods sold for the month of June 2019.
Answer:
Tuesday Manufacturers Limited
Cost of goods sold for the month of June 2019 Rs. in ‘000
Opening finished goods (1,350×1,640) 2,214
Cost of goods manufactured (W1) 9,048
Closing finished goods [1,500×(9,048,000÷5,200)] (2,610)
Cost of goods sold 8,652

W1: Cost of goods manufactured Rs. in ‘000


Opening raw material 1,490
Raw material purchases 3,845
Closing raw material (970)
Raw material consumed 4,365
Direct labour 2,075
Prime cost 6,440
Overheads:
Depreciation on plant and machinery 380
Factory manager’s salary 247
Indirect labour 848
Indirect material consumed 345
Other production overheads 580
2,400
Total manufacturing cost 8,840
Opening work in progress 208
Closing work in progress -
Cost of goods manufactured 9,048
.

© Emile Woolf International 346 The Institute of Chartered Accountants of Pakistan


Chapter 6: Introduction to cost of production

Example 12: Katas Industries Limited


Question: Following information pertain to Katas Industries Limited for the year ended 30
June 2020:
(i) Purchase of raw material:
Rs. in '000
Purchase price 96,100
Discount on bulk purchases 3,290
Early settlement discounts 1,580
(ii) Cost incurred at various locations:
Warehouse
Factory Head Sales Raw Finished
Description
office office material goods
------------------------------ Rs. in '000 ------------------------------
Salaries & wages 9,200* 2,000 3,800 860 640
Depreciation 3,500 1,250 750 150 120
Rent 3,640 - 2,360 380 160
Utilities 2,780 940 1,230 450 235
*75% of factory salaries & wages vary with the level of production
(iii) Breakup of inventories:
1 July 2019 30 June 2020
--------- Rs. in '000 ---------
Raw material 6,800 8,500
Work in progress 1,980 1,600
Finished goods 8,960 12,000
(iv) Due to a machine break down, raw material costing Rs. 1,560,000 was lost during
the production process.
Required:
Prepare statement of cost of goods manufactured for the year ended 30 June 2020. (Also
show total prime cost)
Answer:
Katas Industries Limited
Statement of cost of goods manufactured for the year ended 30 June 2020
Rs. in '000
Raw materials consumed (W-1) 89,550
Salaries and wages 9,200×0.75 6,900
Prime cost 96,450
Manufacturing overheads:
Salaries and wages 9,200×0.25+860 3,160
Depreciation 3,500 + 150 3,650
Rent 3,640 + 380 4,020
Utilities 2,780 + 450 3,230
Manufacturing overheads 14,060
Total manufacturing costs 110,510
Work in progress - opening 1,980
Work in progress - closing (1,600)
Cost of goods manufactured 110,890
W-1: Materials consumed
Opening stock 6,800
Purchases 96,100
Discount on bulk purchase (3,290)
Abnormal production losses (1,560)
Closing stock (8,500)
89,550

© Emile Woolf International 347 The Institute of Chartered Accountants of Pakistan


Financial accounting and reporting I

10 OBJECTIVE BASED QUESTIONS


01. Prime cost means
(a) Sum of direct material and factory overhead cost
(b) Sum of direct material and selling cost
(c) Sum of indirect material and direct labour
(d) Sum of direct material and direct labour cost

02. When a cost manager wants to know the cost of “something” then this “something” is known as
(a) Cost center
(b) Cost object
(c) Cost unit
(d) Cost behavior

03. Which of the following is an example of financing cost


(a) Selling cost
(b) Administrative cost
(c) Loan interest cost
(d) Research and development cost

04. Which of the following statement is incorrect?


(a) Variable cost per unit remains fixed but varies in total
(b) Fixed cost per unit varies but remains fixed in total
(c) Selling cost is included in the cost of product
(d) Direct labour is a component of both prime cost and conversion cost

05. Which of the following is correct for conversion cost?


(a) Cost of direct material + cost of direct labour
(b) Cost of direct material + cost of production
(c) Prime cost + production cost
(d) Cost of direct labour + production overhead

06. Among the following given costs, which can be best described as semi variable cost?
(a) Direct material cost
(b) Electricity charges
(c) Basic pay of direct labour
(d) Plant’s supervisor’s salary

© Emile Woolf International 348 The Institute of Chartered Accountants of Pakistan


Chapter 6: Introduction to cost of production

07. Direct costs are the costs that


(a) Are under the control of directors of the company
(b) Are directly charged to department
(c) Are directly under the control of a manager
(d) Can be directly identified with a product of service

08. Which chart shows the unit cost behavior of straight-line depreciation costs?

(a) Chart A
(b) Chart B
(c) Chart C
(d) Chart D

09. A particular cost is fixed in total for a period. What is the effect on the cost per unit of a reduction
in activity of 50%?
(a) Cost per unit increases by 50%
(b) Cost per unit reduces by 50%
(c) Cost per unit increases by 100%
(d) Cost per unit is unchanged.

10. A manufacturing company has four types of cost (identified as T1, T2, T3 and T4). The total cost
for each type at two different production levels is:
Total cost for180 units
Cost type Total cost for125 units Rs.
Rs.
T1 1,000 1,260
T2 1,750 2,520
T3 2,475 2,826
T4 3,225 4,644
Which two cost types would be classified as being semi-variable?

© Emile Woolf International 349 The Institute of Chartered Accountants of Pakistan


Financial accounting and reporting I

(a) T1 and T3
(b) T1 and T4
(c) T2 and T3
(d) T2 and T4

11. Costs which are included in the costs of inventories are called
(a) Period costs

(b) Selling costs

(c) Product costs

(d) Distribution costs

12. A restaurant has the following costs in a period


1. wages of the kitchen staff
2. depreciation of kitchen equipment
3. costs of ingredients for meals
4. rent paid for the restaurant building
What are indirect costs for an individual meal?

(a) 1, 2, 3 and 4

(b) 1, 2 and 3 only

(c) 1 and 2 only

(d) 2 and 4 only

13. A business pays a salesman a basic salary, plus commission based on how much he sells.
Which type of cost are the salesman’s total earnings?
(a) Fixed

(b) Semi-variable

(c) Stepped

(d) Variable

14. Which cost is treated as variable cost of a motor transport company?


(a) Advertising

(b) Driver insurance

(c) Fuel

(d) Vehicle license

© Emile Woolf International 350 The Institute of Chartered Accountants of Pakistan


Chapter 6: Introduction to cost of production

15. Which item is a direct cost?


(a) Carriage inwards on production materials
(b) Cleaning materials for the factory
(c) Factory rent
(d) Wages of the factory manager

16. A company currently produces 6,000 units of its single product each period, incurring total
variable costs of Rs. 60,000 and fixed costs of Rs. 42,000. Production will increase to 8,000 units
per period if the company expands capacity resulting in changes both to the variable costs per
unit and to the total fixed costs. For production of 8,000 units per period total variable costs would
be Rs. 76,000 and fixed costs Rs. 50,000.
What is the reduction in total cost per unit comparing the costs for 8,000 units per period with the
unit costs currently being incurred?

Rs. ___________

17. Faran Limited is manufacturer of hand bags. Following figures have been provided for the month
of April 2019;

April 1 April 30
Rs. Rs.

Raw material inventory 39,000 66,000

Raw materials purchased 400,000

Direct labour cost 300,000

What is the amount of raw materials consumed for the month of April 2019?
Rs. ___________

18. Faran Limited is manufacturer of hand bags. Following figures have been provided for the month
of April 2019;

Rs.

Raw materials consumed 373,000

Direct labour cost 300,000

Indirect labour 40,000

What is the figure for prime cost for the April 2019?
Rs. ___________

© Emile Woolf International 351 The Institute of Chartered Accountants of Pakistan


Financial accounting and reporting I

19. Faran Limited is manufacturer of hand bags. Following figures have been provided for the month
of April 2019;
Rs.
Raw materials purchased 400,000
Direct labour cost 300,000
Advertising expense 150,000
Selling and administrative salaries 140,000
Rent for factory 120,000
Depreciation - Sales equipment 110,000
Depreciation - Factory equipment 70,000
Indirect labour cost 40,000
Factory utilities 20,000
Factory insurance 10,000

Only 60% of the utilities expenses and 70% of the insurance expense apply to factory operations,
the remaining amount should be charged to selling and administrative expenses.
What is the amount of factory overheads for the month?
Rs. ___________

20. Neelam & Co. has provided following data for the month of January 2019;
(i) Material purchased for Rs. 500,000.
(ii) Opening inventory of raw material is Rs. 80,000 and closing inventory is Rs. 120,000.
(iii) Gross payroll for the month was Rs. 2,000,000. The distribution of payroll was 50%
direct labour, 20% indirect labour, 20% administrative staff salaries, and 10% sales
staff salaries.
What is the amount of prime cost incurred for the month of January 2019?
Rs. ___________

21. Which is not a fixed cost?


(a) monthly rent of Rs.100,000 contractually specified in a one-year lease
(b) an insurance premium of Rs.50,000 per year, paid last month
(c) an audit fee of Rs.500,000 per year:
(d) a post-paid mobile bill

22. Which of the following types of cost are assumed to stay the same per unit, irrespective of the
volume of output?
(a) Overheads
(b) Fixed
(c) Variable
(d) Relevant

© Emile Woolf International 352 The Institute of Chartered Accountants of Pakistan


Chapter 6: Introduction to cost of production

23. A semi variable cost would:


(a) be zero when output is zero and would decrease in direct proportion to output
(b) be more than zero if no products were made and would then increase in direct proportion
to output
(c) be a fixed amount when output was zero and would not increase in direct proportion to
output
(d) be zero when output is zero and would increase in direct proportion to output

24. An example of a semi-variable cost would be:


(a) the wages paid to operatives on basic pay topped-up by a production bonus scheme
(b) the costs of raw material to be used for production
(c) the depreciation of intangible fixed assets
(d) the salaries of departmental supervisors

25. A manufacturing company has four types of cost (identified as A, B, C and D). The total cost of
each type at two different production levels is:
Total cost for 100 units Total cost for 150 units
Cost type
----------------------- Rupees -----------------------
A 1,500 2,250
B 1,800 2,400
C 2,000 3,000
D 3,000 4,200

Which two types of cost would be classified as semi-variable?


(a) A and C
(b) A and D
(c) B and C
(d) B and D

26. A company pays to its salesman a minimum salary plus commission based on sales. Salesman’s
total remuneration is the example of:
(a) fixed cost
(b) semi-variable cost
(c) stepped cost
(d) variable cost

© Emile Woolf International 353 The Institute of Chartered Accountants of Pakistan


Financial accounting and reporting I

10 OBJECTIVE BASED ANSWERS


01. (d) Sum of direct material and direct labour cost

02. (b) Cost object

03. (c) Loan interest cost

04. (c) Selling cost is NOT included in the cost of product.

05. (d) Cost of direct labour + production overhead

06. (b) Electricity charges

07. (d) Can be directly identified with a product of service

08. (d) Chart D. The depreciation expense is fixed and it would decrease per unit when
level of activity increases.

09. (c) Cost per unit increases by 100%

10. (a) T1 and T3

11. (c) Product costs

12. (d) 2 and 4 only

13. (b) Semi-variable

14. (c) Fuel

15. (a) Carriage inwards on production materials

16. Rs. 1.25


Rs.

Cost per unit for 6,000 units (60,000+42,000) / 6,000 17

Cost per units for 6,000 units (76,000 + 50,000)/8,000 (15.75)

Reduction in cost per unit 1.25

17. Rs.
373,000 Rs.

Raw materials - opening 39,000

Raw material purchases 400,000

Raw material available for consumption 439,000

Raw materials - closing (66,000)

Raw material consumed 373,000

© Emile Woolf International 354 The Institute of Chartered Accountants of Pakistan


Chapter 6: Introduction to cost of production

18. Rs.
673,000 Rs.

Raw materials consumed 373,000

Direct labour 300,000

Prime cost 673,000

19. Rs.
249,000 Rs.

Indirect labour cost 40,000

Rent for factory 120,000

Depreciation - Factory equipment 70,000

Factory utilities - 60% 12,000

Factory insurance – 70% 7,000

249,000

20. Rs.
1,460,000 Rs.

Raw materials - opening 80,000

Raw material purchases 500,000

Raw material available for consumption 580,000

Raw materials - closing (120,000)

Raw material consumed 460,000

Direct labour – Rs. 2,000,000 x 50% 1,000,000

Prime cost 1,460,000

21. (d)

22. (c)

23. (b)

24. (a)

25. (d) B and D

26. (b) Semi-variable cost

© Emile Woolf International 355 The Institute of Chartered Accountants of Pakistan


Financial accounting and reporting I

© Emile Woolf International 356 The Institute of Chartered Accountants of Pakistan


7
Certificate in Accounting and Finance

CHAPTER
Financial accounting and reporting I

IAS 16: Property, plant


and equipment

Contents
1 Revaluation
2 Disclosure requirements of IAS 16
3 Objective based questions and answers

* The student must refer original handbook of IFRS.

© Emile Woolf International 357 The Institute of Chartered Accountants of Pakistan


Financial accounting and reporting I

1 REVALUATION
Section overview

◼ Revaluation and the entity’s accounting policy


◼ Accounting for revaluation
◼ Changing the carrying amount of an asset
◼ Depreciation of a revalued asset
◼ Realisation of the revaluation surplus
◼ The frequency of revaluations

1.1 Revaluation and the entity’s accounting policy


Property, plant and equipment is recognised at cost when it is first acquired.
IAS 16 allows a business to choose one of two measurement models as its accounting policy for
property, plant and equipment after initial recognition. However, the same measurement model
should be applied to all assets in the same class (e.g. if an entity has three different factories at
different geographical locations then it cannot discretionally revalue two factories and applies cost
model on one of its factory.
The two measurement models for property, plant and equipment after acquisition are:
❑ cost model (i.e. cost less accumulated depreciation); and
❑ revaluation model (i.e. revalued amount less accumulated depreciation since the most
recent revaluation).
For example, a company’s policy might be to value all its motor vehicles at cost, but to apply the
revaluation model to all its land and buildings.
Revaluation model – Issues
The following accounting issues have to be addressed when using the revaluation model:

Issue

1 What happens to the other side of the entry when the carrying amount of an asset is
changed as a result of a revaluation adjustment?
An asset value may increase or decrease.
What happens in each case?

2 How the carrying amount of the asset being revalued is changed? The carrying amount is
located in two accounts (cost and accumulated depreciation) and it is the net amount that
must be changed so how is this done?

3 How often should the revaluation take place?

1.2 Accounting for revaluation


When a non-current asset is revalued, its ‘carrying amount’ in the statement of financial position is
adjusted from carrying amount to its fair value (normally current market value) at the date of the
revaluation.
Asset carried at cost revalued upwards
An increase in value is credited to other comprehensive income and accumulated in equity under
the heading of revaluation surplus.

© Emile Woolf International 358 The Institute of Chartered Accountants of Pakistan


Chapter 7: IAS 16: Property, plant and equipment

Illustration 01: Upward revaluation


Land was purchased for Rs.100 million on the first day of the 2019 accounting period.
The business applies the IAS 16 revaluation model to the measurement of land after initial
recognition.
The land was revalued to Rs.130 million at the end of the first year of ownership.

Other comprehensive Statement of


Land
income comprehensive income
------------------ Rs. in million ------------------

At start 100 − −
Adjustment 30 30 Cr The surplus is
taken to other
comprehensive
31/12/19 130 30 Cr income

Double entry:

Debit Credit
Land 30 m
Revaluation surplus 30 m

Extract from the statement of financial position as at 31/12/19

ASSETS
Non-current assets
Property, plant and equipment 130 m
EQUITY AND LIABILITIES
Revaluation surplus 30 m

Asset carried at cost revalued downwards


A decrease in value is debited as an expense to the statement of comprehensive income.

Illustration 02: Downward revaluation


Land was purchased for Rs.100 million on the first day of the 2019 accounting period.
The business applies the IAS 16 revaluation model to the measurement of land after initial
recognition.
The land was revalued to Rs.90 million at the end of the first year of ownership.

Other comprehensive Statement of


Land
income comprehensive income
------------------- Rs. in million ------------------

At start 100 − −
Adjustment (10) 10Dr
31/12/19 90

© Emile Woolf International 359 The Institute of Chartered Accountants of Pakistan


Financial accounting and reporting I

Double entry:

Debit Credit

Statement of comprehensive income 10 m

Land 10 m

Asset carried at a revaluation deficit is revalued upwards


An asset might be carried at an amount lower than its original cost as a result of being revalued
downwards. If the asset is later revalued upwards, the revaluation increase is recognised in the
statement of comprehensive income to the extent of the previously recognised expense. The
amount that will be transferred to statement of comprehensive income is equal to the net expense
(expense already charged in SOCI – decrease in depreciation due to revaluation loss). That part
of any increase above the previously recognised expense is recognised in the usual way, directly
in other comprehensive income.
Asset carried at a revaluation surplus revalued downwards
An asset might be carried at an amount higher than its original cost as a result of being revalued
upwards. If the asset is later revalued downwards, the revaluation decrease is recognised in other
comprehensive income to the extent of the previously recognised surplus. That part of any
decrease above the previously recognised surplus is recognised in the statement of
comprehensive income.

Illustration 03: Downward revaluation


A business purchased a plot of land on the first day of the 2015 accounting period.
The business applies the IAS 16 revaluation model to the measurement of land after initial
recognition. The business has a policy of revaluing land annually.
The initial amount recognised and the year-end values are shown below:

Rs.
Measurement on initial recognition 100
Valuation as at:
31 December 2015 130
31 December 2016 110
31 December 2017 95
31 December 2018 116

The double entries are as follows

As at 31 December 2015 Debit Credit

Land (130 – 100) 30

Other comprehensive income 30

As at 31 December 2016 Debit Credit

Other comprehensive income 20

Land (110 – 130) 20

© Emile Woolf International 360 The Institute of Chartered Accountants of Pakistan


Chapter 7: IAS 16: Property, plant and equipment

The fall in value reverses a previously recognised surplus. It is recognised in OCI to the
extent that it is covered by the surplus.
As at 31 December 2017 Debit Credit
Other comprehensive income 10
Statement of comprehensive income 5
Land (95 – 110) 15
The fall in value in part reverses a previously recognised surplus. It is recognised in OCI to
the extent that it is covered by the surplus. This reduces the revaluation surplus to zero.
Any amount not covered by the surplus is recognised as an expense in the statement of
comprehensive income.

As at 31 December 2018 Debit Credit


Land (116 – 95) 21
Statement of comprehensive income 5
Other comprehensive income 16
A rise in value that reverses a previously recognised expense is recognised in the statement
of comprehensive income to the extent that it reverses the expense. Any amount above this
is recognised in other comprehensive income.
Overview
Other Statement of
Land comprehensive comprehensive
income income

At start 100 − −
Double entry 30 30 Cr
31/12/15 130

b/f 130

Adjustment (20) 20Dr −


31/12/16 110

b/f 110
Adjustment (15) 10Dr 5Dr
31/12/17 95

b/f 95
Adjustment 21 16Cr 5Cr
31/12/18 116

© Emile Woolf International 361 The Institute of Chartered Accountants of Pakistan


Financial accounting and reporting I

1.3 Changing the carrying amount of a revalued asset


In the previous illustration land was revalued. Land is not depreciated so the carrying amount of
land is represented in a single account. This made it easy to change:
The carrying amount of depreciable assets is the net of balances on two separate accounts. The
double entry to revalue the asset must take this into account.
A simple approach (and one that accords with IAS 16) is as follows:
❑ Step 1: Transfer the accumulated depreciation to the asset account. The result of this is that
the balance on the asset account is now the carrying amount of the asset and the
accumulated depreciation account in respect of this asset is zero.
❑ Step 2: Change the balance on the asset account to the revalued amount.

Illustration 04:
A building owned by a company is carried at Rs. 8,900,000 (Cost of Rs.9m less accumulated
depreciation of Rs. 100,000. The company’s policy is to apply the revaluation model to all its land
and buildings.
A current valuation of this building is now Rs.9.6 million.

Step 1 Rs. (000) Rs. (000)


Accumulated depreciation 100
Asset 100
Step 2
Asset (Rs.9.6m – Rs.8.9m) 700
Other comprehensive income 700
Alternatively this could be done with a single journal
Asset (Rs.9.6m – Rs.9m) 600
Accumulated depreciation 100
Other comprehensive income 700

Illustration 05:
An office building was purchased four years ago for Rs.3 million.
The building has been depreciated by Rs. 100,000.
It is now re-valued to Rs.4 million.
Based on above Information the analysis and calculation are as under

Building account
Rs. Rs.
Opening balance b/f 3,000,000 Accumulated depreciation 100,000
Revaluation account 1,100,000 Closing balance c/f 4,000,000

4,100,000 4,100,000

Opening balance b/f 4,000,000

© Emile Woolf International 362 The Institute of Chartered Accountants of Pakistan


Chapter 7: IAS 16: Property, plant and equipment

Accumulated depreciation of building account

Rs. Rs.

Building account 100,000 Opening balance b/f 100,000

Revaluation surplus

Rs. Rs.

Revaluation account 1,100,000

1.4 Depreciation of a revalued asset


After a non-current asset has been revalued, depreciation charges are based on the new valuation
over the new useful life (if changed).

Illustration 06:
An asset was purchased three years ago, at the beginning of Year 1, for Rs. 100,000.
Its expected useful life was six years and its expected residual value was Rs. 10,000.
It has now been revalued to Rs. 120,000. Its remaining useful life is now estimated to be three
years and its estimated residual value is now Rs. 15,000.
The straight-line method of depreciation is used.
Based on above Information the analysis and calculation are as under

(a) Annual depreciation originally (for Years 1 – 3)


= Rs.(100,000 – 10,000)/6 years = Rs.15,000.

Rs.
Cost 100,000
Less: Accumulated depreciation at the time of revaluation (= 3 years x
Rs.15,000) (45,000)
Carrying amount at the time of the revaluation 55,000

Re-valued amount of the asset 120,000


Transfer to the revaluation surplus 65,000

(b) Revised annual depreciation = Rs. (120,000 – 15,000)/3 years = Rs. 35,000.
(c) The annual depreciation charge in Year 4 will therefore be Rs. 35,000.

Rs.

Re-valued amount 120,000


Less: depreciation charge in Year 4 (35,000)
Carrying amount at the end of Year 4 85,000

© Emile Woolf International 363 The Institute of Chartered Accountants of Pakistan


Financial accounting and reporting I

1.5 Realisation of the revaluation surplus


All assets eventually disappear from the statement of financial position either by becoming fully
depreciated or because the company disposes them.
If nothing were done this would mean that there was a revaluation surplus on the face of the
statement of financial position that related to an asset that was no longer owned. IAS 16 allows
(but does not require) the transfer of a revaluation surplus to retained earnings when the asset to
which it relates is derecognised (realised).
This might happen over several years as the asset is depreciated or at a point in time when the
asset is disposed of.
Revalued assets being depreciated
Revaluation of an asset causes an increase in the annual depreciation charge. The difference is
known as excess depreciation:
Excess depreciation is the difference between:
❑ the depreciation charge on the re-valued amount of the asset, and
❑ depreciation that would have been charged on historical cost.
Each year a business might make a transfer from the revaluation surplus to the retained profits
equal to the amount of the excess depreciation.

Illustration 07:

Debit Credit

Revaluation surplus X

Retained earnings X

Revalued assets being sold


When a revalued asset is sold the business might transfer the balance on the revaluation surplus
in respect of the asset into retained earnings. The journal entry would be the same as above.

Illustration 08:
An asset was purchased two years ago at the beginning of Year 1 for Rs. 600,000. It had an
expected life of 10 years and nil residual value.
Annual depreciation is Rs. 60,000 (= Rs. 600,000/10 years) in the first two years.
At the end of Year 2 the carrying value of the asset -Rs. 480,000.
After two years it is revalued to Rs.640,000.
Double entry: Revaluation
Debit Credit
Asset (Rs.640,000 – Rs.600,000) 40,000
Accumulated depreciation 120,000
Other comprehensive income 160,000
Each year the business is allowed to make a transfer between the revaluation surplus and retained
profits:
Double entry: Transfer
Debit Credit
Revaluation surplus (160,000/8) 20,000
Retained profits 20,000

© Emile Woolf International 364 The Institute of Chartered Accountants of Pakistan


Chapter 7: IAS 16: Property, plant and equipment

1.6 The frequency of revaluations


When the revaluation model is applied to the measurement of property, plant and equipment, the
frequency of revaluations should depend on the volatility in the value of the assets concerned.
When the value of assets is subject to significant changes (high volatility), annual revaluations may
be necessary.

Illustration 09:
Depreciation calculation for revalued asset
ABC Ltd has a Factory that is revalued to Rs. 250,000 in the fourth year of the acquisition of the
Factory. Original cost of the building as Rs. 150,000 with estimated useful life of 10 years. The
company depreciates the factory on straight-line basis.
1. The Revaluation surplus amount is calculated as under:

Revaluation surplus amount


To calculate this, we need to know the carrying amount of asset at the time of revaluation
which is cost less accumulated depreciation of five years. Depreciation for four years is:
= 150,000 / 10 = 15,000 x 4 = Rs. 60,000
Carrying amount is therefore Rs. 90,000 (150,000 – 60,000)
As the factory was revalued to Rs. 250,000, therefore, the revaluation surplus amount is
Rs. 100,000 (250,000 – 150,000)

2. The Depreciation charge for the period is calculated as under:

Depreciation charge for the period


Divide the revalued amount over the remaining useful life to get depreciation charge for
the year:
= 250,000 / 6 = Rs. 41,667

3. The Excess depreciation to be transferred to retained earnings is calculated as under:

Excess depreciation
Depreciation on revalued amount = Rs.41,667
Depreciation on original cost = Rs.15,000
The difference is Rs. 26,667 (41,667 – 15,000) and this amount will be transferred from
revaluation surplus to retained earnings account if entity chose to do so. The journal entry
will be:

(Dr) Revaluation surplus a/c = Rs.26,667

(Cr) Retained earnings a/c = Rs.26,667

© Emile Woolf International 365 The Institute of Chartered Accountants of Pakistan


Financial accounting and reporting I

Illustration 10:
An asset was purchased four years ago at the beginning of Year 1 for Rs. 1,000,000. It had an
expected life of 10 years and nil residual value.
Annual depreciation is Rs. 100,000 (Rs. 1,000,000/10 years) in the first four years.
At the end of Year 4 the carrying value of the asset - Rs. 600,000.
After two years it is re-valued to Rs. 1,200,000.
The amount to be posted in the revaluation surplus is as under:
Double entry: Revaluation
Debit Credit
Asset (Rs. 1,200,000 – Rs. 1,000,000) 200
Accumulated depreciation 400
Revaluation surplus 600

Each year the business is allowed to make a transfer between the revaluation surplus and retained
profits:
Double entry: Transfer
Debit Credit
Revaluation surplus (600/6) 100
Retained profits 100

© Emile Woolf International 366 The Institute of Chartered Accountants of Pakistan


Chapter 7: IAS 16: Property, plant and equipment

2 DISCLOSURE REQUIREMENTS OF IAS 16


Section overview

◼ Disclosure requirements of IAS 16

2.1 Disclosure requirements of IAS 16


IAS 16 Property, plant and equipment requires the following disclosures in the notes to the
financial statements, for each major class of property, plant and equipment.
❑ The measurement bases used (cost or revaluation model) for determining the gross
carrying amount
❑ The depreciation methods used
❑ The useful lives or depreciation rates used
❑ Gross carrying amounts and the accumulated depreciation at the beginning and at the end
of the period
❑ A reconciliation between the opening and closing values for gross carrying amounts and
accumulated depreciation, showing:
• additions during the year
• disposals during the year
• depreciation charge for the year
• increase or decrease in asset resulting from revaluation and impairment losses
The following is an illustration of how a simple table for tangible non-current assets may be
presented in a note to the financial statements.
An entity must also disclose:
❑ the existence and amounts of restrictions on title, and property, plant and equipment pledged
as security for liabilities;
❑ the amount of expenditures recognised in the carrying amount of an item of property, plant
and equipment in the course of its construction;
❑ the amount of contractual commitments for the acquisition of property, plant and equipment;
and
❑ if it is not disclosed separately in the statement of comprehensive income, the amount of
compensation from third parties for items of property, plant and equipment that were
impaired, lost or given up that is included in profit or loss

Illustration 11: (Method 1)

Plant and
Property Total
equipment

Cost Rs. M Rs. m Rs. m

At the start of the year 7,200 2,100 9,300

Additions 920 340 1,260

Disposals (260) (170) (430)

At the end of the year 7,860 2,270 10,130

© Emile Woolf International 367 The Institute of Chartered Accountants of Pakistan


Financial accounting and reporting I

Accumulated depreciation
At the start of the year 800 1,100 1,900
Depreciation expense 120 250 370
Accumulated depreciation on disposals (55) (130) (185)
At the end of the year 865 1,220 2,085
Carrying amount
At the start of the year 6,400 1,000 7,400
At the end of the year 6,995 1,050 8,045

Illustration 12: (Method 2)


Plant and
Property Total
equipment
Carrying value Rs. M Rs. m Rs. m
At the start of the year 6,400 1,000 7,400
Additions 920 340 1,260
Disposals(carrying value) (205) (40) (245)
Depreciation expense (120) (250) (370)
At the end of the year 6,995 1,050 8,045

Carrying amount
At the start of the year 6,400 1,000 7,400
At the end of the year 6,995 1,050 8,045

Disclosures for assets stated at revalued amounts


When items of property, plant and equipment are stated at revalued amounts the following must
be disclosed:
❑ the effective date of the revaluation;
❑ whether an independent valuer was involved;
❑ for each revalued class of property, plant and equipment, the carrying amount that would
have been recognised had the assets been carried under the cost model; and
❑ the revaluation surplus, indicating the change for the period and any restrictions on the
distribution of the balance to shareholders.
Additional disclosures encouraged by IAS 16
IAS 16 encourages disclosure of the following information as users of financial statements might
find it to be useful.
❑ the carrying amount of temporarily idle property, plant and equipment;
❑ the gross carrying amount of any fully depreciated property, plant and equipment that is
still in use;
❑ the carrying amount of property, plant and equipment retired from active use and held for
disposal; and
❑ when the cost model is used, the fair value of property, plant and equipment when this is
materially different from the carrying amount.

© Emile Woolf International 368 The Institute of Chartered Accountants of Pakistan


Chapter 7: IAS 16: Property, plant and equipment

Example 01:
Question: Following information has been extracted from the financial statements of Newton
Pharma for the year ended 30 June 2018

Rupees
Manufacturing Unit 650,000
Accumulated depreciation 170,000
Carrying amount 480,000
Revaluation surplus 20,000

The Company provides depreciation on vehicles @ 10% per annum on straight line method.
Additional information
1. On 30 June 2018, a tablet pressing machine which was acquired at a cost of Rs. 50,000, in
exchange for another machine in January 2016, was revalued. The revalued amount was
60,000.
2. The Sealing machine, imported from Japan at a cost of 100,000 in July 2016, is annually
revalued by the Company. On June 30, 2018 the machine was devalued to 60,000. The
revaluation surplus shows 20,000 balance in respect of revaluation of the machine.
Required:
Prepare the relevant ledgers of Newton Pharma for the year ended 30 June 2018.

Answer:

Manufacturing Unit Account

Rs. Rs.

Opening balance b/f 650,000 Accumulated depreciation 170,000

Revaluation surplus 10,000 Closing balance c/f 490,000

660,000 660,000

Accumulated depreciation of Manufacturing Unit account

Rs. Rs.

Manufacturing unit account 170,000 Opening balance b/f 170,000

Revaluation surplus

Rs. Rs.

Reversal of revaluation 20,000 Opening balance 20,000


surplus – Sealing machine

Closing balance c/f 10,000 Revaluation-tablet pressing 10,000


machine

30,000 30,000

© Emile Woolf International 369 The Institute of Chartered Accountants of Pakistan


Financial accounting and reporting I

Example 02: ALI LIMITED


Question: Ali Limited (AL) uses the revaluation model for subsequent measurement of its property,
plant and equipment and has a policy of revaluing its assets on an annual basis using the net
replacement value method.
The following information pertains to AL’s building:
i. The building was purchased on 01 January 2010 for Rs. 2 hundred million with expected useful
life of ten years.
ii. AL depreciates buildings on the straight line basis over their useful life.
iii. The results of revaluations carried out during the last three years by Standard Valuation Service,
an independent firm of values, are as follows:
Revaluation date Fair value Rs. in million
1 January 2011 280
1 January 2012 170
1 January 2013 180
Required:
The journal entries relating to the above transactions including revaluations for the year ended
December 31, 2010, 2011, 2012 and 2013.

Answer:

Debit Credit
Date - Property, plant and equipment
---- Rs. in million ----
January Building 200
01,2010 Account Payable 200
December Depreciation (200÷10) 20
31,2010 Accumulated depreciation 20
January Accumulated depreciation 20
01,2011 Building 20
January Building (280-(200-20)) 100
01,2011 Revaluation surplus (OCI) 100
January Depreciation (200÷9) 31
31,2011 Accumulated depreciation 31
January Revaluation surplus (OCI) (31-20) 11
31,2011 Retained earnings 11
January Accumulated depreciation 31
01,2012 Building 31
January Revaluation surplus (SOCI) (280-31)-170 79
01,2012 Building 79
January Depreciation (170÷8) 21.25
31,2012 Accumulated depreciation 21.25
January Accumulated depreciation 21.25
01,2013 Building 21.25
January Building 31.25
01,2013 Revaluation surplus (180-(170-21.25) 31.25
December Depreciation (180÷07) 25.7
31,2013 Accumulated depreciation 25.7
December Revaluation surplus (OCI) (25.7-20) 5.7
31,2013 Retained earnings 5.7

© Emile Woolf International 370 The Institute of Chartered Accountants of Pakistan


Chapter 7: IAS 16: Property, plant and equipment

Example 03: SUNDRY QUESTION 1


Question: The following are two independent scenarios:
1. A business purchased some land and buildings on 1 January 2011 for Rs.800million (land
Rs.250 million and buildings Rs.550 million). The buildings are to be depreciated over a
period of 50 years.
On 1 January 2015 the land and buildings were revalued to Rs. 1,500 million (land Rs.400
million and buildings Rs. 1,100 million). At this date the buildings were believed to have a
remaining useful life of 40 years.
Required:
What is the original depreciation charge for the buildings and the revised charge from 1
January 2015?
Answer:
The original depreciation charge for the buildings and the revised charge from 1 January
2015 are calculated as under:
Original depreciation = Rs.550 million /50 years = Rs.11 million
Revised depreciation = Rs.1,100 million /40 years = Rs.27 million
2. A business purchased land for Rs.250 million and buildings for Rs.400 million on 1 January
2011. The buildings were to be depreciated over a period of 50 years. On 1 January 2015
the land was revalued to Rs.520 million and the buildings were revalued at Rs.750 million.
Required:
What amount is to be taken to the revaluation reserve on 1 January 2015?
Answer:
Rs.m Rs.m
Land (520 - 250) 270
Buildings – cost 400
depreciation (400/50 x 4 years) (32)
Net book value 368
Revaluation 750
Transfer to revaluation reserve 382
Total revaluation surplus 652

Example 04: ROONEY


Question: Rooney has recently finished building a new item of plant for its own use. The item is a
press for use in the manufacture of industrial diamonds. Rooney commenced construction of the
st st
asset on 1 April 2013 and completed it on 1 April 2015. The cost of manufacturing the asset were
Rs. 30,800,000. The cost of the hydraulic system is 30% of the total cost of manufacture.
The press comprises two significant parts, the hydraulic system and the ‘frame’. The hydraulic system
has a three-year life and the ‘frame’ has an eight-year life. Rooney depreciates plant on a straight
line basis.
Rooney uses the IAS 16 revaluation model in accounting for diamond presses and revalue these
assets on an annual basis.
Revaluation surpluses or deficits are apportioned between the hydraulic system and the ‘frame’ on
the basis of their year-end book values before the revaluation.
Required:
Explain the IAS 16 rules on accounting for significant parts of property, plant and equipment and
show the accounting treatment of the diamond press in the financial statements for the financial
years ending:
(i) 31st March 2016 (assume that the press has a fair value of Rs. 21 million)
(ii) 31st March 2017 (assume that the press has a fair value of Rs. 19.6 million).

© Emile Woolf International 371 The Institute of Chartered Accountants of Pakistan


Financial accounting and reporting I

Answer:
The IAS 16 rule on accounting for significant parts of property, plant and equipment is as under:
Accounting Rule
IAS 16 requires that each part of an item (that has a cost that is significant in relation to the total
cost) is depreciated separately. Therefore, the cost recognised at initial recognition must be allocated
to each part accordingly.
(i) 31st March 2016

Carrying
Cost
Depreciation value
1.4.2015 31.3.2016
Rs.000 Rs.000 Rs.000
Hydraulic system 9,240 (3,080) 6,160
“Frame” 21,560 (2,695) 18,865
30,800 (5,775) 25,025
Revaluation loss (to profit and loss) (4,025)
Fair value. 21,000
The carrying value of the assets should be written down by a factor of 21,000/25025. This
gives a carrying value for the hydraulic system (in Rs.000) of 5,169 and for the ‘frame’
15,831.
The hydraulic plant should be depreciated over two more years and the ‘frame’ over 7 more
years.
Based on above analysis if the press has a fair value of rupees 19.6 million at 31 st March
2017 then the relevant calculation is as under:
(ii) 31st March 2017

Carrying Depreciation Carrying


value Charge Value
1.4.2016 31.3.2017
Rs.000 Rs.000 Rs.000
Hydraulic system 5,169 (2,585) 2,584
“Frame” 15,831 (2,262) 13,569
21,000 (4,847) 16,153
Revalued amount 19,600
Total gain 3,447

To statement of profit or loss 3,097


Other comprehensive income 350
Fair value 19,600

The total revaluation gain is 3,447. Of this total amount, 3096 reverses the loss in the
previous year net of the benefit obtained through reduced depreciation and is therefore
reported in profit and loss for the year. The remaining 350 is reported as other
comprehensive income.

© Emile Woolf International 372 The Institute of Chartered Accountants of Pakistan


Chapter 7: IAS 16: Property, plant and equipment

Working

Hydr Frame Total


Carrying amount 5,169 15,831 21,000
Depreciation (2,585) (2,262) (4,847)
Carrying amount 2,585 13,569 16,153
FV 3,136 16,464 19,600
Value increased 551 2,895 3,447

Loss recognised last year 991 3,034 4,025


Depreciation reduced by (495) (433) (929)
Loss reversal possible thru P/L 495 2,601 3,097

Gain in OCI 56 294 350

(Tutorial note: Deferred tax has been ignored in this question.)

Example 05: EHTISHAM


Question:
The following information relates to the financial statements of Ehtisham for the year to 31 March
2015.
The head office of Ehtisham was acquired on 1 April 2012 for Rs. 1 million. Ehtisham intend to occupy
the building for 25 years. On 31 March 2014 it was revalued to Rs. 1.15 million. On 31 March 2015,
a surplus of vacant commercial property in the area had led to a fall in property prices and the fair
value was now only Rs. 0.8 million.
Required:
Explain the correct accounting treatment for the above (with calculations).

Answer:
IAS 16 permits assets to be carried at cost or revaluation. Where the latter is chosen, the asset must
be stated at its fair value.
The original depreciation was Rs. 40,000 (Rs. 1,000,000/25 years) per annum.
On 31st March 2014 the asset is two years old. Its carrying value before revaluation was therefore
Rs.1million less accumulated depreciation of Rs.80,000 ( 2/25 × Rs. 1 million).

Rs.
Cost/valuation 1,000,000
Accumulated depreciation (80,000)
Net book value 920,000
In order to effect the revaluation, the cost is uplifted to fair value of Rs.1.15m, the accumulated
depreciation is eliminated, and the uplift to the net book value is credited to a revaluation surplus
account.
Debit Credit
Building 150,000
Accumulated depreciation 80,000
Revaluation surplus 230,000

© Emile Woolf International 373 The Institute of Chartered Accountants of Pakistan


Financial accounting and reporting I

The impact of the journal is as follows:

Before Adjustment After


Cost/valuation 1,000,000 150,000 1,150,000
Accumulated depreciation (80,000) 80,000 nil
Net book value 920,000 1,150,000
The asset is depreciated over its remaining useful economic life of 23 years giving a charge of Rs.
50,000 (Rs. 1,150,000/23 years) per annum in the year to 31st March 2015.
Debit Credit
Depreciation - Statement of profit or loss 50,000
Accumulated depreciation 50,000
This results in a carrying value as at 31st March 2015 of:
Rs.
Cost/valuation 1,150,000
Accumulated depreciation (50,000)
Net book value 1,100,000

Transfer from revaluation surplus to retained earnings


As a result of the revaluation, the annual depreciation has increased from Rs.40,000 to Rs.50,000.
This extra depreciation of Rs.10,000 is transferred from the revaluation reserve to accumulated
profits each year.

Debit Credit
Revaluation surplus 10,000
Accumulated profits 10,000

By the 31st March 2015, the balance remaining on the revaluation reserve will be Rs.220,000.

Rs.
Surplus recognised at 31 March 2014 230,000
Transfer to accumulated profits (10,000)
Net book value 220,000

The fall in property values at the year-end. The asset must be revalued downwards to Rs.0.8million,
a write-down of Rs.300,000.
Rs.220,000 of this is charged against the revaluation reserve relating to this asset, and the
remaining Rs.80,000 must be charged against profits.
The reduction of the carrying amount of the asset is achieved by removing the accumulated
depreciation and adjusting the asset account by the balance.

Debit Credit
Revaluation surplus 220,000
Revaluation Loss - Statement of profit or loss (Working - 1) 80,000
Asset at valuation 350,000
Accumulated depreciation 50,000

© Emile Woolf International 374 The Institute of Chartered Accountants of Pakistan


Chapter 7: IAS 16: Property, plant and equipment

The impact of the journal is as follows:

Before Adjustment After


Cost/valuation 1,150,000 350,000 800,000
Accumulated depreciation (50,000) 50,000 nil
Net book value 1,100,000 800,000

This balance is depreciated over the remaining useful life of the asset (22 years).
Working – 1

Asset Account 2012-13 2013-14 2014-15


Opening Balance 1,000,000 960,000 1,150,000
Depreciation (40,000) (40,000) (50,000)
960,000 920,000 1,100,000
Revaluation - 230,000 (300,000)
Carrying amount 960,000 1,150,000 800,000
Revaluation Surplus
Opening balance - 230,000
Transfer - (10,000)
Balance - 220,000
Revaluation 230,000 (220,000)
230,000 -
Loss in P/L (80,000)

Example 06: CARLY


Question: The following is an extract from the financial statements of Carly on 31 December 2014.
Property, plant and equipment

Land and Plant and Computer


Total
buildings machinery equipment
Rs. Rs. Rs. Rs.
Cost
On 31 December 2014 1,500,000 340,500 617,800 2,458,300
Accumulated depreciation
On 31 December 2014 600,000 125,900 505,800 1,231,700
Carrying amount
On 31 December 2014 900,000 214,600 112,000 1,226,600
Accounting policies

Depreciation
Depreciation is provided at the following rates.
On land and buildings Over 50 years on straight line basis on buildings only
On plant and equipment 25% reducing balance
On computers 33.33% per annum straight line

© Emile Woolf International 375 The Institute of Chartered Accountants of Pakistan


Financial accounting and reporting I

During 2015 the following transactions took place.


(1) On 31 December the land and buildings were revalued to Rs. 1,750,000. Of this amount, Rs.
650,000 related to the land (which had originally cost Rs. 500,000). The remaining useful life
of the buildings was assessed as 40 years.
(2) A machine which had cost Rs. 80,000 and had accumulated depreciation of Rs. 57,000 at the
start of the year was sold for Rs. 25,000 in the first week of the year.
(3) A new machine was purchased on 31 March 2015. The following costs were incurred:
Rs.
Purchase price, before discount, inclusive of reclaimable sales tax of Rs.3,000 20,000
Trade Discount 1,000
Delivery costs 500
Installation costs 750
Interest on loan taken out to finance the purchase 300
(4) On 1 January it was decided to change the method of providing depreciation on computer
equipment from the existing method to 40% reducing balance.
Required:
Produce the analysis of property, plant and equipment as it would appear in the financial statements
of Carly for the year ended 31 December 2015.
Answer:
Property, plant and equipment
Land and Plant and Computer
Total
buildings machinery equipment
Rs. Rs. Rs. Rs.
Cost/valuation
At 1 January 2015 1,500,000 340,500 617,800 2,458,300
Cancelation (620,000) (620,000)
Revaluation 870,000 - - 870,000
Additions (W2) - 17,250 - 17,150
Disposals - (80,000) - (80,000)
------------------------ ------------------------ ------------------------ ------------------------
At 31 December 2015 1,750,000 277,750 617,800 2,645,550
------------------------ ------------------------ ------------------------ ------------------------
Accumulated depreciation
At 1 January 2015 600,000 125,900 505,800 1,231,700
Charge for the year (W1) 20,000 50,775 44,800 115,575
Cancelation (620,000) - - (620,000)
Disposals - (57,000) - (57,000)
------------------------ ------------------------ ------------------------ ------------------------
At 31 December 2015 Nil 119,675 550,600 670,275
------------------------ ------------------------ ------------------------ ------------------------
Carrying amount
At 31 December 2014 900,000 214,600 112,000 1,226,600
------------------------ ------------------------ ------------------------ ------------------------
At 31 December 2015 1,750,000 158,075 67,200 1,975,275
------------------------ ------------------------ ------------------------ ------------------------

Workings
(1) Depreciation charges
Buildings = (1,500,000 – 500,000) / 50 years = 20,000.

© Emile Woolf International 376 The Institute of Chartered Accountants of Pakistan


Chapter 7: IAS 16: Property, plant and equipment

Plant and machinery:

Rs.
New machine (17,250  25% 8/12) 2,875
Existing plant (((340,500 – 80,000) – (125,900 – 57,000))  25%) 47,900
50,775
Computer equipment = 112,000  40% = Rs.44,800
(2) Cost of new machine

Rs.
Purchase price (20,000 – 3,000 – 1,000) 16,000
Delivery costs 500
Installation costs 750
17,250

Example 07: ADJUSTMENTS LIMITED


Question: Adjustments Limited has carried out a review of its non-current assets.
(a) A grinder was purchased on 1 January 2012 for Rs. 100,000. The plant had an estimated useful
life of ten years and a residual value of nil. Depreciation is charged on the straight line basis. On
1 January 2015, when the asset’s net book value is Rs. 70,000, the directors decide that it would
be more appropriate to depreciate this asset using the sum of digits’ approach. The remaining
useful life is unchanged.
(b) The company purchased a fifty year lease some years ago for Rs. 1,000,000. This was being
depreciated over its life on a straight line basis. On 1 January 2015, when the net book value is
Rs. 480,000 and twenty-four years of the lease are remaining, the asset is revalued to Rs.
1,500,000. This revised value is being incorporated into the accounts.
Required:
Explain the effects of these changes on the depreciation for the year to 31 December 2015.
Answer:
(a) The grinder was purchased in 2012 and was originally being depreciated on a straight line basis. It
has now been decided to depreciate this on the sum of digits basis.
IAS 16 requires that depreciation methods be reviewed periodically and if there is a significant change
in the expected pattern of economic benefits, the method should be changed. Depreciation
adjustments should be made in current and future periods. This change might be appropriate if, for
instance, usage of the machine is greater in the early years of an asset’s life when it is still new and
consequently it is appropriate to have a higher depreciation charge.
If the change is implemented, the unamortised cost (the net book value) of the asset should be
depreciation over the remaining useful life commencing with the period in which the change is made.
The depreciation charge for the remaining life of the asset will therefore be as follows.
Year Digits Depreciation
Rs.
2015 7 7/28  Rs.70,000 17,500
2016 6 6/28  Rs.70,000 15,000
2017 5 12,500
2018 4 10,000
2019 3 7,500
2020 2 5,000
2021 1 2,500
—— ———–
1/2  7 (7 + 1) 28 Rs. 70,000
—— ———–

© Emile Woolf International 377 The Institute of Chartered Accountants of Pakistan


Financial accounting and reporting I

Disclosure will need to be made in the accounts of the details of the change, including the effect on
the charge in the year.
The reassessment of the depreciation method is not a change in accounting policy and neither
rectification of a fundamental error so the effects of the change will not affect the previously reported
financial statements (opening retained earnings)
(b) Leasehold land
IAS 16 requires that the subsequent charge for depreciation should be based on the revalued amount.
The annual depreciation will therefore be Rs. 62,500, i.e. Rs.1,500,000 divided by the 24 years of
remaining life.
There will then be a difference between the revalued depreciation charge and the historical
depreciation charge.
The resulting excess depreciation may be dealt with by a movement in reserves, i.e. by transferring
from the revaluation reserve to retained earnings a figure equal to the depreciation charged on the
revaluation surplus each year.

Example 08: FAM


Question: FAM had the following tangible fixed assets at 31 December 2014.
Cost Depreciation NBV
Rs. 000 Rs. 000 Rs. 000
Land 500 – 500
Buildings 400 80 320
Plant and machinery 1,613 458 1,155
Fixtures and fittings 390 140 250
Assets under construction 91 – 91
——— —— ———
2,994 678 2,316
════ ════ ════

In the year ended 31 December 2015 the following transactions occur.


(1) Further costs of Rs. 53,000 are incurred on buildings being constructed by the company. A
building costing Rs. 100,000 is completed during the year.
(2) A deposit of Rs. 20,000 is paid for a new computer system which is undelivered at the year
end.
(3) Additions to plant are Rs. 154,000.
(4) Additions to fixtures, excluding the deposit on the new computer system, are Rs. 40,000.
(5) The following assets are sold.

Depreciation brought
Cost Proceeds
forward

Rs. 000 Rs. 000 Rs. 000


Plant 277 195 86
Fixtures 41 31 2

(6) Land and buildings were revalued at 1 January 2015 to Rs. 1,500,000, of which land is worth
Rs. 900,000. The revaluation was performed by Messrs Jackson & Co, Chartered Surveyors, on
the basis of existing use value on the open market.
(7) The useful economic life of the buildings is unchanged. The buildings were purchased ten years
before the revaluation.

© Emile Woolf International 378 The Institute of Chartered Accountants of Pakistan


Chapter 7: IAS 16: Property, plant and equipment

(8) Depreciation is provided on all assets in use at the yearend at the following rates. Buildings 2%
per annum straight line
Buildings 2% per annum straight line
Plant 20% per annum straight line
Fixtures 25% per annum reducing balance
Required:
Show the disclosure under IAS 16 in relation to fixed assets in the notes to the published accounts for
the year ended 31 December 2015.

Answer:
Accounting policies
(a) Property, plant and equipment is stated at historical cost less depreciation, or at valuation.
(b) Depreciation is provided on all assets, except land, and is calculated to write down the cost or
valuation over the estimated useful life of the asset.
The principal rates are as follows.
Buildings 2% pa straight line
Plant and machinery 20% pa straight line
Fixtures and fittings 25% pa reducing balance
Fixtures, fittings,
Land and Plant and
Fixed asset movements tools and Total
buildings machinery
equipment
Cost/valuation Rs.000 Rs.000 Rs.000 Rs.000
Cost at 1 January 2015 900 1,613 390 2,903
Cancelation (80)
Revaluation adjustment 680 - - 600
Additions 100 154 40 294
Disposals (277) (41) (318)
Cost at 31 December 2015 1600 1,490 389 1,979
2015 valuation 1,500

Fixtures, fittings,
Land and Plant and
Fixed asset movements tools and Total
buildings machinery
equipment
Depreciation
At 1 January 2015 80 458 140 678
Revaluation adjustment (80) – – (80)
Provisions for year (W2) 17 298 70 385
Disposals – (195) (31) (226)
At 31 December 2015 17 561 179 757

© Emile Woolf International 379 The Institute of Chartered Accountants of Pakistan


Financial accounting and reporting I

Net book value


At 31 December 2015 1,583 929 210 2,722
At 31 December 2014 820 1,155 250 2,225
Land and buildings have been revalued during the year by Messrs Jackson & Co on the basis of an
existing use value on the open market.
The corresponding historical cost information is as follows.
Land and buildings
Rs.000
Cost
Brought forward 900
Reclassification 100
———
Carried forward 1,000
———
Depreciation
Brought forward 80
Provided in year 10
———
Carried forward 90
———
Net book value 910
———
Payments on account and assets in the course of construction

Cost at 1 January 2015 91


Additions (W1) 73
Reclassifications (100)
As at 31 December 2015 64
At 31 December 2014 91

WORKINGS Rs. 000


(1) Additions to assets under construction 53
Deposit on computer 20
——
73
——
600
(2) Depreciation on buildings 40 + (100  2%) 17

2% straight line depreciation is equivalent to a 50 year life.


The buildings are ten years old at valuation and therefore
have 40 years remaining.
Depreciation on plant (1,613 + 154 – 277)  20% 298
Depreciation on fixtures (390 + 40 – 41 – 140 + 31)  25% 70

© Emile Woolf International 380 The Institute of Chartered Accountants of Pakistan


Chapter 7: IAS 16: Property, plant and equipment

Example 09: ORCHID LIMITED


Question:
The following information pertains to property, plant and equipment of Orchid Limited (OL), a listed
company:

Cost Subsequent
Date of Original Depreciation
Description Rs. In measurement
purchase useful life method
million model
Buildings 1-Jan-15 600 30 years Straight line Revaluation
Plant 1-Jan-15 475 25 years Straight line Cost
Buildings
The revalued amount of buildings as determined by Shabbir Associates, an independent valuer, on 31
December 2015 and 2017 was Rs. 700 million and Rs. 463 million respectively.
On 30 June 2017 a building having original cost of Rs. 66 million was sold to Baqir Limited for Rs. 85
million. It was last revalued at Rs. 87 million. OL incurred a cost of Rs. 2 million on disposal.
OL transfers the maximum possible amount from revaluation surplus to retained earnings on an
annual basis.
Plant
On 31 December 2016 the recoverable amount of the plant was assessed at Rs. 360 million
with no change in useful life.
During 2017, OL has decided to change the depreciation method for plant from straight line to
reducing balance. The new depreciation rate would be 10%.
Required:
Prepare following notes (along with comparative figures) to be presented in the financial statements
of OL for the year ended 31 December 2017 in accordance with the requirements of relevant IFRSs
and Companies Act, 2017:
(a) Property, plant and equipment
(b) Change in depreciation method

Answer:
Notes to the financial statement
For the year ended 31 December 2017
Property, plant and equipment:

2017 2016
Building Plant Building Plant
-------------------------------- Rs. in million ----------------------------
Gross carrying amount - 700.00 475.00 700.00 475.00
opening
Accumulated dep. & (24.14) (115.00) - (19.00)
impairment (475÷25)
Opening carrying amount 675.86 360.00 700.00 456.00
Depreciation (22.64) (36) (24.14) (19.00)
[21.14 (700-87) ÷ 29] + (360 × 10%) (700÷29) (475÷25)
[1.5 (87÷ 29 × 6 ÷12)]

© Emile Woolf International 381 The Institute of Chartered Accountants of Pakistan


Financial accounting and reporting I

Disposal (82.50) - - -
[87-{(87÷ 29)+
(87÷29×6 ÷ 12)}]
Impairment (P&L) (77)
(456–19–
360)
Revaluation
- surplus [W-1] (90.12) - - -
- P&L [W-1] (17.60) - - -
Closing carrying amount 463.00 324.00 675.86 360.00
Gross carrying amount - 463.00 475.00 700.00 475.00
closing
Accumulated dep. & - (151) (24.14) (115.00)
impairment
Closing carrying amount 463.00 324.00 675.86 360.00

Building Plant
Measurement base Revaluation model Cost model
Useful life (years)/depreciation rate % 30 10%
Depreciation method Straight line Reducing balance
The last revaluation was performed on 31 December 2017 by Shabbir Associates, an independent
firm of valuers.

2017 2016

Carrying value of building had the cost 480.6 560


model been used instead (600–66)÷30×27 (600÷30×28)

Following disposal has been made during the year.

cost/revalued Book
Name of Sale price Gain/(loss) Mode of
amount value
purchaser disposal
------------------------- Rs. in million -------------------------

Building Baqir Limited 87.00 82.50 85.00 0.5 (85–2) – 82.5) Tender
Change in estimate
In lieu of significant change in the expected pattern or consumption of the future economic benefits
embodied in the plant, company decided to change the depreciation method of plant from straight line
to reducing balance method. The new depreciation rate would be 10%.
Had the depreciation method been not changed, profit of 2017 would have been higher by Rs. 20.35
million. (360×10%-360÷23)

W-1: Revaluation of building as on 31 December 2017 Rs. in million


Carrying value of building (675.86-22.64-82.5) 570.72
Revalued amount 463.00
107.72

© Emile Woolf International 382 The Institute of Chartered Accountants of Pakistan


Chapter 7: IAS 16: Property, plant and equipment

Available surplus [120(700-580)-23.2(W-2)]×(27÷29)


OR [120-4.14(120÷29)-3.74{4.14-(23.2÷29×0.5)}-22(23.2÷29×27.5)] (90.12)
Expense (P&L) 17.60

W-2:

Revalued amount of building sold 87.0

Carrying value (66÷30×29) 63.8

23.2

Example 10: ABID LIMITED


Question: Abid Limited (AL) uses the revaluation model for subsequent measurement of its property,
plant and equipment and has a policy of revaluing its assets on an annual basis using the net
replacement value method.
The following information pertains to AL’s buildings:
i. Four buildings were acquired in same vicinity on 1 January 2012 at a cost of Rs. 300 million. The
useful life of the buildings on the date of acquisition was 20 years.
ii. AL depreciates buildings on the straight line basis over their useful life.
iii. The results of revaluations carried out during the last three years by Premier Valuation Service,
an independent firm of valuers, are as follows:

Revaluation date Fair value Rs. in million


1 January 2013 323
1 January 2014 252
1 January 2015 272

On 30 June 2015, one of the buildings was sold for Rs. 80 million.
Required:
Prepare a note on “Property, plant and equipment” (including comparative figures) for inclusion in AL’s
financial statements for the year ended 31 December 2015 in accordance with International Financial
Reporting Standards. (Ignore taxation)??

Answer:

2015 2014
Property, plant and equipment
--------- Rs. in million ---------
Gross carrying amount 252 323
Accumulated depreciation and (14) (17)
impairment losses (323÷19)
Net carrying amount 238 306
Additions - -
Revaluation (expense)/Income (P/L) 17[272- {300-(300÷20x3)}] (18)(306-252-36)
Revaluation surplus increase/(decrease) 17 (272-238-17) (36)[{323-(300-
(OCI) 15)}-(38÷19)]

© Emile Woolf International 383 The Institute of Chartered Accountants of Pakistan


Financial accounting and reporting I

Depreciation (14) (14)


[(204÷17)+(68÷17×6÷12)] (252÷18)
Disposal (66) [68 - (68÷17×6÷12)] -
192 238

Gross carrying amount 204 252


Accumulated depreciation and (12) (14)
impairment losses
Net carrying amount 192 238

Useful life 20 years 20 years


The last revaluation was performed on 1 January 2015 by M/s Premier Valuation Services, an
independent firm of valuers. Revaluations are performed annually.

2015 2014

--------- Rs. in million ---------

Carrying value had the cost model been 180 255


used instead [225–(225÷20×4)] [300–(300÷20×3)]

4.1- Details of property, plant and equipment disposed of during the year

Cost /
Accumulated Carrying Sale
Revalued
depreciation amount proceeds Mode of Particulars of
amount
disposal buyers
---------------------- Rs. in million ----------------------

Building 68 2 66 80 Not mentioned Not mentioned

Example 11: SHAHWEZ LIMITED


Question: Shahwez Limited (SL) revalued its property on 1 April 20X1 to Rs.20 million (Rs.8 million
for the land). The property originally cost Rs.10 million (Rs.2 million for the land) 10 years ago. The
original useful life of 40 years is unchanged. SL’s policy is to make a transfer to realised profits in
respect of excess depreciation.
Required:
How will the property be accounted for in the year ended 31 March 20X2?

Answer:
Statement of comprehensive income extract for the year ended 31 March 20X2
Rs. In 000
Depreciation expense 400
Other comprehensive income:
Revaluation gain 12,000

© Emile Woolf International 384 The Institute of Chartered Accountants of Pakistan


Chapter 7: IAS 16: Property, plant and equipment

Statement of financial position extract as at 31 March 20X2


Rs. In 000
Non-current assets Property 19,600
(20,000 – 400)
Equity Revaluation reserve 11,800
(12,000 – 200)

Statement of changes in equity extracts


Revaluation reserve Retained earnings
Revaluation gain 12,000
Reserves transfer (200) 200
Workings:
Gain on revaluation: Rs. In 000
Carrying value at revaluation date
(10,000 – ((10,000 – 2,000)*40 years/10 years)) 8,000
Valuation 20,000
Gain on revaluation 12,000
Double entry:
Dr Property 10,000
(20,000 – 10,000)
Dr Accumulated depreciation 2,000
((10,000 – 2,000)/40 years x 10 years)
Cr Revaluation reserve 12,000
Depreciation charge for year to 31 March 20X2:
Dr depreciation expense 400
((20,000 – 8,000)/30 years)
Cr Accumulated depreciation 400
Reserves transfer:
Historical cost depreciation charge 200
((10,000 – 2,000)/40 years)
Revaluation depreciation charge 400
Excess depreciation to be transferred 200
Dr Revaluation reserve 200
Cr Retained earnings 200

Example 12: HAMZA LIMITED


Question: Hamza Limited (HL) acquired a building on 1 April 20X1 for Rs. 100,000 at which point
it was considered to have a useful life of 40 years. At the year-end 31 March 20X6, HL decided to
revalue the building to its current value of Rs. 98,000.
Required:
How will the building be accounted for in the year ended 31 March 20X6?

© Emile Woolf International 385 The Institute of Chartered Accountants of Pakistan


Financial accounting and reporting I

Answer:
Rupees
Statement of comprehensive income extract 31 March 20X6
Depreciation charge 2,500
Other comprehensive income:
Revaluation gain 10,500
Statement of financial position extract 31 March 20X6
Building at valuation 98,000

Statement of changes in equity extract


Revaluation gain
Revaluation reserve 10,500
Notes:
Revaluation takes place at year end, therefore a full year of depreciation must first be charged.
(W1) Depreciation year ended 31 March 20X6
100,0000 / 40 years = Rs.2,500 per annum

(W2) Revaluation as at 31 March 20X6


Carrying value of non-current asset at revaluation date
(100,000 – (100,000/40 years x 5 years)) 87,500
Valuation of non-current asset 98,000
Gain on revaluation 10,500
Double entry: Debit Credit
Accumulated depreciation 12,500
Building 2,500
Revaluation reserve 10,500

Example 13: SUNDRY QUESTION 2


Question:
(a) When the asset should be derecognised from the balance sheet? What is the accounting
treatment for gain or loss on disposal for revaluation model?
(b) What is the disclosure requirement of asset carried at revalued amount???

Answer:
(a). Property, Plant and Equipment should be derecognised (removed from PPE) either;
i. on disposal (sold or exchanged etc. by cash for asset given up) or
ii. when it is withdrawn from use and no future economic benefits are expected from the
asset (in other words, it is effectively scrapped).
A gain or loss on disposal is recognised as the difference between the disposal proceeds
(gross proceeds received minus cost of making sale) and the carrying value of the asset
(using the cost or revaluation model) at the date of disposal. This net gain is included in the
income statement. The sales proceeds should not be recognised as revenue.

© Emile Woolf International 386 The Institute of Chartered Accountants of Pakistan


Chapter 7: IAS 16: Property, plant and equipment

Where assets are measured using the revaluation model, any remaining balance in the
revaluation reserve relating to the asset disposed of is transferred directly to retained
earnings. No recycling of this balance into the income statement is permitted.
(b) General disclosures
The financial statements shall disclose, for each class of property, plant and equipment:
a) the measurement bases used for determining the gross carrying amount;
b) the depreciation methods used;
c) the useful lives or the depreciation rates used;
d) the gross carrying amount and the accumulated depreciation (aggregated with
accumulated impairment losses) at the beginning and end of the period; and
e) a reconciliation of the carrying amount at the beginning and end of the period showing
increases or decreases resulting from revaluations from comparing its revalued
amount to the book value and recognize in other comprehensive income and
accumulated in equity under the heading of revaluation surplus. However, the
revaluation increase shall be recognised in profit or loss to the extent that it reverses a
revaluation decrease of the same asset previously recognised in profit or loss.
Specific disclosures
If items of property, plant and equipment are stated at revalued amounts, the following
shall be disclosed:
i. the effective date of the revaluation;
ii. whether an independent valuer was involved;
iii. for each revalued class of property, plant and equipment, the carrying amount that
would have been recognised had the assets been carried under the cost model; and
iv. the revaluation surplus, indicating the change for the period and any restrictions on
the distribution of the balance to shareholders.

Example 14:
Question: Abbas Limited (AL) is engaged in the business of manufacturing near the Karachi-
Hyderabad Motorway. Its Property, Plant and Equipment comprises of land and buildings, plant and
machinery, and equipment and fittings.

Details for the period up to 30 June 2018 are as follows:


1. The balances of the Property, Plant and Equipment as at 30 June 2018 are given
below:

Assets Gross Carrying Amount Accumulated Depreciation


(Rs. Million) (Rs. Million)
Land 12 N/A
Buildings 125 38
Plant and Machinery 500 300
Equipment 100 36
2. The relevant information pertaining these assets is given below:
Assets Depreciation Method Subsequent Measurement
Model
Land N/A Fair Value
Buildings Straight-line Cost
Plant and Machinery Units of Production Cost
Equipment Written down value Cost

© Emile Woolf International 387 The Institute of Chartered Accountants of Pakistan


Financial accounting and reporting I

3. Abbas Limited uses proportionate policy to depreciate its Property, Plant and
Equipment.
4. All of the plant and machinery pertains to factory use whereas all the equipment
pertains to office use. However, floor areas occupied by factory and office are in the
ratio 60:40 respectively.
5. The equipment was purchased on 1 July 2016. No disposals and acquisitions took
place in the period up to 30 June 2018.
6. Until 30 June 2018, 12,000 units had been produced by Abbas Limited in its factory.
The plant and machinery does not have any residual value. No additions or disposals
of plant and machinery took place till this date.
7. The buildings were acquired on 1 July 2014 with a residual value of Rs. 11 million. No
additions and disposals took place till 30 June 2018.
8. The land had actually cost Rs. 15 million on the date of its acquisition.
9. It is assumed that value of land and buildings is spread evenly across the area
occupied.
The following information pertains to the year ended 30 June 2019:
1. On 1 July 2018, land was revalued to Rs. 20 million on 1 July 2018. The value was
determined by an independent firm M/s Ashfaq& Co. Chartered Accountants.
2. This year, 5,000 units were product in the factory of AL.
3. On January 1, 2019, AL disposed 25% of its area comprising of land and buildings at
a price of Rs. 90 million. The portion of land was sold at its fair value as determined
on 1 July 2018. The legal costs of drafting transfer agreements were Rs. 0.1 million.
It is assumed that this disposal will not affect the proportion of areas occupied by
factory and office.
4. Further equipment costing Rs. 60 million was acquired on 1 November 2018.
5. In the meeting of its board of directors, it was decided to open a new factory
premises near Lahore-Islamabad motorway. An expenditure of Rs. 20 million was
spent of the construction of the factory on 1 December 2018, financed by a loan
obtained from the bank at the rate of 12% per annum. The construction had not been
completed at the end of the year.
6. Moreover, the directors also made a contract with M/s UniPower& Co. to purchase
plant and machinery worth Rs. 35 million once the construction of factory building is
completed.
Required:
(a) Prepare journal entries to record the revaluation of land and disposal of land and
buildings.
(b) Prepare the disclosure under IAS 16 in relation to Property, Plant and Equipment in the
notes to the published accounts for the year ended 30 June 2019.
Answer:
a) The journal entries to record the revaluation of land and disposal of land and buildings
are as under:

Date Particulars Dr. Cr.


Rs. 000 Rs.000
2018

July 1, Land 8,000


Revaluation Surplus 5,000
Reversal of Revaluation Loss (Other Income) 3,000

© Emile Woolf International 388 The Institute of Chartered Accountants of Pakistan


Chapter 7: IAS 16: Property, plant and equipment

2019

Jan 1, Cash (90m - 0.1m) 89,900


Accumulated Depreciation – Buildings (W1) 10,687.5
Buildings 31,250
Land 5,000
Gain on Disposal (Other Income) 64,337.5

Jan 1, Revaluation Surplus 1,250


Retained Earnings 1,250

(W1) Rs. 000


At the start of year (38 × 25%) 9,500
During the year [(125 – 11) × 25% ÷ 12 ÷ 2] 1,187.5
Accumulated Depreciation of Buildings Disposed 10,687.5

b) The disclosure under IAS 16 in relation to Property, Plant and Equipment in the notes
to the published accounts for the year ended 30 June 2019 is as under:
Abbas Limited uses the following subsequent measurement bases to value its
Property, Plant and Equipment, and methods to calculate its depreciation.

Depreciation Useful Life/Residual


Assets Subsequent Measurement
Method Value/Rate
Land N/A N/A Fair Value
Useful life of 12 years
Cost less Accumulated
Buildings Straight-line with a residual value of
Depreciation
8.8% of cost. (W1)
Plant and Units of Rs. 25,000 per unit Cost less Accumulated
Machinery production (W2) Depreciation
Written Cost less Accumulated
Equipment Rate of 20% (W3)
down value Depreciation

Furthermore, Abbas Limited uses proportionate policy to depreciate its assets.


Schedule of Movement in Property, Plant and Equipment
For the year ended 30 June 2019 (in Rs. 000)
Plant and
Land Buildings Equipment Total
Machinery
Gross Carrying Amount:
At 1 July 2018
Acquisitions 12,000 125,000 500,000 100,000 737,000
Revaluation 60,000
Disposals 8,000 - 60,000
(5,000) (31,250) - 8,000
(36,250)
At 30 June 2019 15,000 93,750 500,000 160,000 768,750
Accumulated Depreciation
At 1 July 2018
Depreciation - 38,000 300,000 36,000
Revaluation - 8,312.5(W4) 125,000 20,800(W5) 374,000

© Emile Woolf International 389 The Institute of Chartered Accountants of Pakistan


Financial accounting and reporting I

Disposals - - - 154,112.5
(10,687.5)

(10,687.5)
At 30 June 2019 - 35,625 425,000 56,800 517,425
Carrying Amount at 1 July
12,000 87,000 200,000 64,000 363,000
2018
Carrying Amount at 30 June
15,000 58,125 75,000 103,200 251,325
2019
An amount of expenditure of Rs. 20 million was incurred on the construction of a factory near Lahore-
Islamabad Motorway on 1 December 2018. This amount was capitalized as capital work-in-progress.
A further borrowing costs of Rs. 1.4 million (W6) were capitalized in respect of interest on loan
obtained from the bank to finance this project.
A contract was made with M/s UniPower & Co. to purchase plant and machinery worth Rs. 35
million once the construction of factory building is completed.
The following depreciations are either made part of inventory or expensed out in statement of
profit or loss:
Assets Inventory Expense Total
Rs.000 Rs.000 Rs.000
Buildings 4,987.5 3,325 8,312.5
Plant and Machinery 125,000 - 125,000
Equipment - 20,800 20,800
Total 129,987.5 24,125 154,112.5
Revaluation Disclosures:
i. The revaluation of land took place on 1 July 2018. The value was determined by an
independent firm M/s Ashfaq& Co. Chartered Accountants.
ii. The carrying amount of land had the revaluation not taken place:

Rs. million
At 1 July 2018 15
Disposals during the year (3.75)
At 30 June 2019 11.25
iii. Revaluation Surplus

Rs. million
At 1 July 2018 -
Revaluation of land 5
Transfer to retained earnings (1.25)
At 30 June 2019 3.75

A further reversal of revaluation loss of Rs. 3 million was reversed during the year.
125−11
(W1) × 4 = 12 𝑦𝑒𝑎𝑟𝑠
38
300,000,000
(W2) = 𝑅𝑠. 25,000 𝑝𝑒𝑟 𝑢𝑛𝑖𝑡
12,000

2 100−36
(W3) 𝑟 =1− √
100

𝑟 = 20%

© Emile Woolf International 390 The Institute of Chartered Accountants of Pakistan


Chapter 7: IAS 16: Property, plant and equipment

(W4) Depreciation during the year Rs. 000

Disposals 1,187.5
During the year [(125 – 11) × 75% ÷ 12] 7,125

Depreciation Expense 8,312.5

(W5) Depreciation during the year Rs. 000

Acquisitions (60 × 0.2 × 8/12) 8,000


Remaining Assets (64 × 0.2) 12,800

Depreciation Expense 20,800


7
(W6) 20 𝑚𝑖𝑙𝑙𝑖𝑜𝑛 × 12% × = 1.4 𝑚𝑖𝑙𝑙𝑖𝑜𝑛
12

Example 15: GAMES LIMITED


Question: Games Limited (GL) commenced a business of preparing and burning video game CDs on
1 July 2015.
The following information pertains to the year ended 31 March 2016:
1) GL purchased 30 computers on the date of commencement of business at a cost of Rs.
20,000 each, purely for the task of burning CDs. The management of GL estimates that since
the computers are subject to obsolescence, more of its benefit can derived in its early life. The
total useful life at the date of acquisition was estimated to be 4 years and residual value was
estimated to be Rs. 4,802 for each computer.GL decided to adopt historical cost model for
subsequently measurement of computers.
2) GL purchased an office building at the date of start of business worth Rs. 3 million. GL decided
to adopt fair value model due to fluctuations in property prices. 80% of the building is occupied
by computer labs, whereas 20% is used by administrative and selling departments. The useful
life is estimated to be 10 years at the date of acquisition with no residual value, and the
economic benefits are expected to be derived evenly over its useful life. At the end of the year,
the fair value of office buildings was assessed to be Rs. 3,237,500.
3) GL also purchased fittings for its administrative and selling departments, costing Rs. 120,000
on 1 July 2015. It is to be depreciated over 10 years using the straight-line method, with no
residual value.
4) GL made a contractual commitment with Al-Karim Computers to purchase 6 computers of Rs.
20,000 each to be delivered at GL’s premises on 1 May 2017.
The following information pertains to the year ended 31 March 2017:
1) The computers were delivered at the GL’s premises by Al-Karim Computers at the said date. It
was decided to use the same method and same rate to depreciate these computers. However,
no further space was utilised by the computer labs.
2) At the end of the year, the fair value of office building was assessed to be Rs. 2 million. At the
year-end GL mortgaged entire building with JS Bank to obtain a loan worth Rs. 1.75 million for
prospective investments in other divisions.
3) Fittings with a cost of Rs. 30,000 were disposed of for Rs. 22,000 on 1 January 2017. The
Suzuki Driver was paid Rs. 1,000 to transfer the fittings to customer’s premises.
The fair values of the office building were determined byan independent firm M/s Hafeez Yasir
Chartered Accountants& Co. Moreover, GL uses proportionate policy to depreciate its assets.

© Emile Woolf International 391 The Institute of Chartered Accountants of Pakistan


Financial accounting and reporting I

Required:
(a) Prepare the disposal account to record the sale of fittings on 1 January 2017.
(b) Prepare the disclosure under IAS 16 in relation to Property, Plant and Equipment in the
notes to the published accounts for the year ended 31 March 2017 (comparatives are
required).
Answer:
(a) The disposal account to record the sale of fittings on 1 January 2017 is as under:

Disposal Account - Fittings


2017 Rs. 2017 Rs.
Jan 1, Fittings – Cost Jan 1, Accumulated
30,000
Jan 1, Cash (Disposal Depreciation – 4,500
1,000 Fittings(W1)
Costs)
Jan 1, Cash 22,000
Mar Statement of Profit or
4,500
31, Loss (Loss on Disposal)
31,000 31,000
9
(W1) 30,000 × × 2 × 10% = 4,500
12

(b) The disclosures under IAS 16 in relation to Property, Plant and Equipment in the notes to
the published accounts for the year ended 31 March 2017 (comparatives are required)
are as under:
Property, plant and equipment:
Games Limited (GL) uses the following subsequent measurement bases to value its
Property, Plant and Equipment, and methods to calculate its depreciation.

Depreciation
Assets Rate Subsequent Measurement
Method
Buildings Straight-line 10% Fair Value
Written-down Cost less Accumulated
Computers 30% (W1)
value Depreciation
Cost less Accumulated
Fittings Straight-line 10%
Depreciation
Furthermore, Games Limited uses proportionate policy to depreciate its assets.
Games Limited
Schedule of Movement in Property, Plant and Equipment
For the year ended 30 March 2017 (in Rs.)

2017 2016
Buildings Computers Fittings Buildings Computers Fittings
Gross Carrying Amount:
At start of year 3,237,500 600,000 120,000 - - -
Acquisitions - 120,000 - 3,000,000 600,000 120,000
Revaluations (1,237,500) - - 237,500 - -
Disposals - - (30,000) - - -
At end of year 2,000,000 720,000 90,000 3,237,500 600,000 120,000

© Emile Woolf International 392 The Institute of Chartered Accountants of Pakistan


Chapter 7: IAS 16: Property, plant and equipment

Accumulated Depreciation:
At start of year - 135,000 9,000 - - -
Depreciation
charge for the
year(W2) 350,000 181,000 11,250 225,000 135,000 9,000
Revaluation (350,000) - - (225,000) - -
Disposals - - (4,500) - - -
At end of year - 316,000 15,750 - 135,000 9,000
Carrying Amount
at year start 3,237,500 465,000 111,000 - - -
Carrying Amount
2,000,000 404,000 74,250 3,237,500 465,000 111,000
at year end
The entire office building was mortgaged with JS Bank on 31 March 2017, to obtain a
loan worth Rs. 1.75 million for prospective investments in other divisions.
No contractual commitments were made during the year ended 31 March 2017 to
purchase Property, Plant and Equipment.
A contract was made with Al-Karim Computers during the year ended 31 March 2016 to
purchase 6 computers of Rs. 20,000 each to be delivered on 1 May 2017.
The following depreciations are either made part of inventory or expensed out in
statement of profit or loss:
Assets 2017 2016
Inventory Expense Total Inventory Expense Total
Rs. Rs. Rs. Rs. Rs. Rs.
Buildings 280,000 70,000 350,000 180,000 45,000 225,000
Computers 181,000 - 181,000 135,000 - 135,000
Fittings - 11,250 11,250 - 9,000 9,000
Total 461,000 81,250 542,250 315,000 54,000 369,000
Revaluation Disclosures:
The revaluations of office buildings took place on 31 March 2017 and 31 March 2016
respectively. The fair values of the office building were determined by an independent
firm M/s Hafeez Yasir Chartered Accountants & Co.

The carrying amount of buildings had the revaluation not taken place:
2017 2016
Rs. Rs.
Cost:
At start of year 3,000,000 -
Acquisitions - 3,000,000
Disposals - -
At end of year 3,000,000 3,000,000
Accumulated Depreciation:
At start of year 225,000 -
Depreciation charge for the year 300,000 225,000
Disposals - -
At end of year 525,000 225,000
Carrying Amount at year start 2,775,000 -
Carrying Amount at year end 2,475,000 2,775,000

© Emile Woolf International 393 The Institute of Chartered Accountants of Pakistan


Financial accounting and reporting I

Revaluation Surplus

At start of year 462,500 -


Revaluation of Buildings (412,500) 462,500
Transfer to Retained Earnings (350,000 - 300,000) (50,000) -
At end of year - 462,500
Furthermore, a revaluation loss of Rs. 475,000 was recognized at 31 March 2017.

4 4,802
(W1) 𝑟 = 1 − √
20,000

𝑟 = 30%
(W2) 2017 2016
3,000,000
9
Buildings 3,237,500 ÷ 9.25 = 350,000 ÷ 10 ×
12
= 225,000
Rs.
Acquisitions (60,000 × 10% ×
5,500 3,000,000 × 30%
11/12)
Computers 9
Remaining [(720,000 - 135,000) × = 135,000
175,500 12
× 30%]
Total 181,000
Rs.
Disposals (30,000 × 10% × 9
2,250 120,000 ÷ 10 ×
Fittings 9/12) 12
Remaining (90,000 × 10%) 9,000 = 9,000
Total 181,000

Example 16: INDIGO LIMITED


Question: Indigo Limited (IL) purchased a building on 1 April 2015 for Rs.10 million at which point
it was considered to have a useful life of 40 years. At the year ended 31 March 2020, IL decided to
revalue the building to its market value of Rs.9.8 million.
Required:
How the above building shall be accounted for in the books of IL for the year ended 31 March 2020?

Answer:
Statement of comprehensive income extract for the year ended 31 March 2020
Profit or loss account:
Depreciation charge = Rs.0.25 million (W1)
Other comprehensive income:
Revaluation gain = Rs.1.05 million (W2)
Statement of financial position extract as at 31 March 2020
Building at valuation = Rs.9.8 million
Statement of changes in equity extract for the year ended 31 March 2020
Revaluation reserve
Revaluation gain = Rs.1.05 million (W2)

© Emile Woolf International 394 The Institute of Chartered Accountants of Pakistan


Chapter 7: IAS 16: Property, plant and equipment

Workings:
Revaluation takes place at year end, therefore a full year of depreciation must first be charged.
(W1) Depreciation for the year ended 31 March 2020
Rs.10 million / 40 years = Rs.0.25 million
(W2) Revaluation
Carrying value of building at revaluation date (10 m – (10 m/40 years x 5 years)) = Rs.8.75 m
Revaluation of building = Rs.9.8 m
Gain on revaluation = Rs.1.05 m

Example 17:
Question: At 1 April 2019, Glacier Limited (GL) carried its office at its original cost of Rs.20 million
and depreciation of Rs.4 million (based on straight line method to its original life of 50 years). GL
decided to revalue the office on 1 October 2019 to its current value of Rs.22 million. The useful life
remaining was reassessed at the time of valuation and is considered to be 40 years at this date. It
is the company’s policy to charge depreciation proportionally.
Required:
How the office will be accounted for in the financial statements of GL for the year ended 31 March
2020?

Answer:
Statement of comprehensive income extract for the year ended 31 March 2020
Depreciation charge (200,000 (W1) + 275,000 (W3) = Rs. 475,000
Other comprehensive income:
Revaluation gain (W2) = Rs.6.2 million
Statement of financial position extract as at 31 March 2020
Revaluation (office) = Rs. 22 million
Subsequent depreciation (W3) = (Rs. 275,000)
Carrying value = Rs. 21,725,000
Statement of changes in equity extract for the year 31 March 2020
Revaluation reserve (gain) = Rs. 6,200,000 (W2)
Workings:
Revaluation takes place part way through the year and therefore depreciation must first be charged
for the period 1 April 2019 – 30 September 2019, then the revaluation occurs so the depreciation
needs to be charged for the period 1 October 2019 – 31 March 2020 on current market value.
(W1) Depreciation
1 April – 30 September 2019 = Rs.20 million x 6/12 / 50 years = Rs. 200,000
(W2) Revaluation
The carrying value of the asset at 1 October 2019 can now be found and revalued.
Carrying value of office at revaluation date (20,000,000 – (4,000,000 + 200,000)) = Rs. 15,800,000
Revaluation of office = Rs.22 million
Gain on revaluation = Rs. 6,200,000
(W3) Depreciation
1 October 2019 – 31 March 2020 => Rs. 22,000,000 x 6/12 / 40 years = Rs. 275,000

© Emile Woolf International 395 The Institute of Chartered Accountants of Pakistan


Financial accounting and reporting I

3 OBJECTIVE BASED QUESTIONS


01. An entity purchased a property 15 years ago at a cost of Rs. 100,000 and have been depreciating
it at a rate of 2% per annum, on the straight-line basis. The entity has had the property
professionally revalued at Rs. 500,000.
What is the revaluation surplus that will be recorded in the financial statements in respect of this
property?

(a) Rs. 400,000

(b) Rs. 500,000

(c) Rs. 530,000

(d) Rs. 430,000

02. An entity owns two buildings, A and B, which are currently recorded in the books at carrying
amounts of Rs. 170,000 and Rs. 330,000 respectively. Both buildings have recently been valued
as follows:
Building A Rs. 400,000
Building B Rs. 250,000
The entity currently has a balance on the revaluation surplus of Rs. 50,000 which arose when
building A was revalued several years ago. Building B has not previously been revalued.
What double entry will need to be made to record the revaluations of buildings A and B?

(a) Dr Non-current assets Rs. 150,000


Dr Statement of profit or loss Rs. 80,000
Cr Other comprehensive income (revaluation surplus) Rs. 230,000
(b) Dr Non-current assets Rs. 150,000
Dr Statement of profit or loss Rs. 30,000
Cr Other comprehensive income (revaluation surplus) Rs. 180,000
(c) Dr Non-current assets Rs. 150,000
Cr Other comprehensive income (revaluation surplus) Rs. 150,000
(d) Dr Non-current assets Rs. 150,000
Dr Statement of profit or loss Rs. 50,000
Cr Other comprehensive income (revaluation surplus) Rs. 200,000

03. An entity purchased property for Rs. 6 million on 1 July 2013. The land element of the purchase
was Rs. 1 million. The expected life of the building was 50 years and its residual value nil. On 30
June 2015 the property was revalued to Rs. 7 million, of which the land element was Rs. 1.24
million and the buildings Rs. 5.76 million. On 30 June 2017, the property was sold for Rs. 6.8
million.
What is the gain on disposal of the property that would be reported in the statement of profit or
loss for the year to 30 June 2017?

(a) Gain Rs. 40,000

(b) Loss Rs. 200,000

(c) Gain Rs. 1,000,000

(d) Gain Rs. 1,240,000

© Emile Woolf International 396 The Institute of Chartered Accountants of Pakistan


Chapter 7: IAS 16: Property, plant and equipment

04. Which of the following statements are correct?


1. If the revaluation model is used for property, plant and equipment, revaluations must
subsequently be made with sufficient regularity to ensure that the carrying amount does
not differ materially from the fair value at each reporting date.
2. When an item of property, plant and equipment is revalued, there is no requirement that
the entire class of assets to which the item belongs must be revalued.

(a) Only statement 1 is correct

(b) Only statement 2 is correct

(c) Both statements are correct

(d) None of the statement is correct

05. The following trial balance extract relates to a property which is owned by Maira Limited as at 1
April 2014.

Dr Cr
Rs. 000 Rs. 000
Property at cost (20 year original life) 12,000
Accumulated depreciation as at 1 April 2014 3,600
On 1 October 2014, following a sustained increase in property prices, Maira Limited revalued its
property to Rs. 10.8 million.
What will be the depreciation charge in Maira Limited’s statement of comprehensive income for
the year ended 31 March 2015?

(a) Rs. 540,000

(b) Rs. 570,000

(c) Rs. 700,000

(d) Rs. 800,000

06. A company purchased a building on 1 April 2007 for Rs. 10,000,000. The asset had a useful
economic life at that date of 40 years. On 1 April 2009 the company revalued the building to its
current fair value of Rs. 12,000,000.
What is the double entry to record the revaluation?

(a) Dr. Building 1,500,000


Dr. Accumulated depreciation 500,000
Cr. Other comprehensive income 2,000,000

(b) Dr. Building 2,000,000


Dr. Accumulated depreciation 500,000
Cr. Profit or loss 2,500,000

(c) Dr. Building 2,000,000


Dr. Accumulated depreciation 500,000
Cr. Other comprehensive income 2,500,000

© Emile Woolf International 397 The Institute of Chartered Accountants of Pakistan


Financial accounting and reporting I

(d) Dr. Building 1,500,000


Dr. Accumulated depreciation 500,000
Cr. Profit or loss 2,000,000

07. The carrying value of property at the end of the year amounted to Rs. 108 million. On this date the
property was revalued and was deemed to have a fair value of Rs. 95 million. The balance on the
revaluation reserve relating to the original gain of the property was Rs. 10 million.
What is the double entry to record the revaluation?

(a) Dr. Profit or loss 3 million


Dr. Other comprehensive income 10 million
Cr. Property 13 million

(b) Dr. Profit or loss 10 million


Dr. Other comprehensive income 3 million
Cr. Property 13 million

(c) Dr. Profit or loss 13 million


Dr. Other comprehensive income 3 million
Cr. Property 16 million

(d) Dr. Profit or loss 13 million


Cr. Property 13 million

08. A company revalued its property on 1 April 2009 to Rs. 20m (Rs. 8m for the land). The property
originally cost Rs. 10m (Rs. 2m for the land) 10 years ago. The original useful economic life of 40
years is unchanged. The company’s policy is to make a transfer to realized profits in respect of
excess depreciation.
At which amount the property be presented at as at 31 March 2010?

(a) Rs. 20 million

(b) Rs. 19.6 million

(c) Rs. 12 million

(d) Rs. 11.6 million

09. A company revalued its property on 1 April 2009 to Rs. 20m (Rs. 8m for the land). The property
originally cost Rs. 10m (Rs. 2m for the land) 10 years ago. The original useful economic life of 40
years is unchanged. The company’s policy is to make a transfer to realized profits in respect of
excess depreciation.
What is amount of balance in revaluation surplus account as at 31 March 2010?

(a) Rs. 12 million

(b) Rs. 10 million

(c) Rs. 9.8 million

(d) Rs. 11.8 million

© Emile Woolf International 398 The Institute of Chartered Accountants of Pakistan


Chapter 7: IAS 16: Property, plant and equipment

10. Which of the following is an optional disclosure requirement of IAS 16?

(a) Measurement bases for determining gross carrying amount

(b) Depreciation method

(c) Useful lives or depreciation rates

(d) The carrying amount of temporarily idle PPE

11. Following information is available for equipment account of a business on 1st January 2018:
Opening balance of equipment, a/c (Revalued amount) Rs. 7,500,000
Surplus on revaluation of equipment a/c Rs. 2,000,000
At start of year company sold equipment for Rs. 90,000,000.
Company has a policy of charging 20% depreciation on straight line basis.
What will be treatment of revaluation surplus at disposal of asset?

(a) Dr Surplus on revaluation Rs. 2,000,000


Cr Retained earnings Rs. 2,000,000

(b) Dr Retained earnings Rs. 2,000,000,


Cr Surplus on revaluation Rs. 2,000,000

(c) Dr Surplus on revaluation Rs. 3,500,000


Cr Retained earnings Rs. 3,500,000

(d) Dr Surplus on revaluation Rs. 2,0000,000


Cr Equipment account Rs. 2,000,000

12. A non–current asset costing Rs. 216,000 and carrying value Rs. 145,000 is revalued to Rs.
291,000.
How should revaluation be recorded?

(a) Dr Asset a/c Rs. 75,000,


Cr Surplus on revaluation Rs. 75,000

(b) Dr Asset a/c Rs. 75,000,


Dr Accumulated Depreciation Rs. 71,000,
Cr Surplus on revaluation Rs. 146,000

(c) Dr Surplus on revaluation Rs. 146,000,


Cr Asset a/c Rs. 75,000,
Cr Accumulated Depreciation Rs. 71,000

(d) Dr Accumulated depreciation Rs. 146,000,


Cr Surplus on revaluation Rs. 146,000

13. When items of property, plant and equipment are stated at revalued amounts the following must
be disclosed:
(i) the effective date of the revaluation
(ii) whether an independent valuer was involved

© Emile Woolf International 399 The Institute of Chartered Accountants of Pakistan


Financial accounting and reporting I

(iii) the methods and significant assumptions applied in estimating the items’ fair values
(iv) the extent to which the items’ fair values were determined directly by reference to
observable prices in an active market or recent market transactions on arm’s length
terms or were estimated using other valuation techniques
(v) for each revalued class of property, plant and equipment, the carrying amount that
would have been recognised had the assets been carried under the cost model;
(vi) the revaluation surplus, indicating the change for the period and any restrictions on the
distribution of the balance to shareholders.

(a) (i), (ii) and (vi) only

(b) (i), (ii), (v) and (vi) only

(c) (i), (ii), (iii) and (iv) only

(d) (i) to (vi) all

14. IAS 16 encourages disclosure of the following information as users of financial statements might
find it to be useful.
(i) the carrying amount of temporarily idle property, plant and equipment
(ii) the gross carrying amount of any fully depreciated property, plant and equipment that is
still in use
(iii) the carrying amount of property, plant and equipment retired from active use and held
for disposal
(iv) when the cost model is used, the fair value of property, plant and equipment when this
is materially different from the carrying amount

(a) (i), (ii) and (iii) only

(b) (i), (ii) and (iv) only

(c) (i), (iii) and (iv) only

(d) (i) to (iv) all

15. Which of the following statements is correct?

(a) An entity may present PPE at gross carrying amount or net carrying amount under IAS 16

(b) Either useful lives or depreciation rates are to be disclosed, both are not required.

(c) Under revaluation model, PPE are revalued at end of each year

(d) If an entity chooses revaluation model, it must apply revaluation model to all of its PPE.

16. Waqas Limited purchased a machine for Rs. 30,000 on 1 January 2015 and assigned it a useful
life of 12 years. On 31 March 2017 it was revalued to Rs. 32,000 with no change in useful life.
What will be depreciation charge in relation to this machine in the financial statements for the year
ending 31 December 2017?

Rs. ___________

© Emile Woolf International 400 The Institute of Chartered Accountants of Pakistan


Chapter 7: IAS 16: Property, plant and equipment

17. A business purchased building costing Rs. 7,500,000 on 1 January 2018.


The policy of business is to charge straight line depreciation over its useful life of 20 years.
On 31 December 2020, building was revalued to Rs. 7,650,000.
What is the amount of incremental depreciation to be transferred to retained earnings at year
ending 31 December 2021?

Rs. ___________

18. A business purchased an asset on 1 January 2016 costing Rs. 5,000,000 having a useful life of
10 years with nil residual value. On 1 January 2018 balance of accumulated depreciation was Rs.
1,000,000. Asset is revalued to Rs. 4,500,000 on 1 January 2018 (start of the year).
Business has a policy to charge straight line depreciation.
What is the depreciation charge for the year ended 31 December 2018?

Rs. ___________

19. A business purchased an asset on 1 January 2016 costing Rs. 5,000,000 having a useful life of
10 years with nil residual value. On 1 January 2018 balance of accumulated depreciation was Rs.
1,000,000. Asset is revalued to Rs. 4,500,000 on 1 January 2018 (start of the year).
Business has a policy to charge straight line depreciation.
What is the amount of revaluation surplus at the date of revaluation?

Rs. ___________

20. A business purchased an asset on 1 January 2016 costing Rs. 5,000,000 having a useful life of
10 years with nil residual value. On 1 January 2018 balance of accumulated depreciation was Rs.
1,000,000. Asset is revalued to Rs. 4,500,000 on 1 January 2018 (start of the year).
Business has a policy to charge straight line depreciation.
What is the amount of incremental depreciation for the year ended 31 December 2018?

Rs. ___________

21. A revaluation gain is credited into?

(a) Revaluation reserve

(b) Capital reserve

(c) Profit and loss

(d) Any of the above

22. After initial recognition, an entity has a choice to choose cost and?

(a) Realizable model

(b) Replacement model

(c) Revaluation model

(d) Carrying value model

© Emile Woolf International 401 The Institute of Chartered Accountants of Pakistan


Financial accounting and reporting I

23. When an item of property, plant and equipment is revalued, what should be revalued?

(a) A selection of assets decided by management

(b) The whole class of assets to which it belongs

(c) The individual asset

(d) A selection of assets picked at random

24. If an asset increases in value, the increase is noted as?

(a) An increase in net profit in the SOCI

(b) An increase in retained earnings in SOFP

(c) An increase in revaluation surplus in the SOFP and other comprehensive income in the SOCI

(d) An increase in “other profit” in SOCI

25. Which of the following is not a valid reason for reporting non-current assets at revaluation amount
rather than cost?

(a) To prevent long life assets from being reported at out of date historical costs

(b) To keep owners of the business better informed of their equity in the business.

(c) To report performance correctly by matching earnings with the proper costs of assets used.

(d) To avoid having to pay higher taxes

26. An entity has a policy of revaluing its PPE. An asset cost Rs.5m on 1 January 2020 and has a
useful life of five years and is depreciated on a straight-line basis to a zero residual value. The
value of the asset at 31 December 2020 was Rs.3.8m. The fall in value will be accounted for as
follows?

(a) Depreciation Rs.1m and fall in value of Rs.200,000 both to the reserves

(b) Depreciation Rs.1m to the income statement and fall in value of Rs.200,000 ignored until
there is a revaluation surplus

(c) Depreciation Rs.1m to income statement and fall in value of Rs.200,000 to the reserves

(d) Depreciation Rs.1m and fall in value of Rs.200,000 both to the income statement

27. During the financial year, Akmal Ltd had the following increases in reserves:
i. Rs. 5 million from a revaluation of freehold premises
ii. Rs.10 million in share premium
iii. Rs.25 million from trading profit retained
Which of these are increases in capital reserves?

(a) i only

(b) ii only

(c) i. and ii. Only

(d) iii. only

© Emile Woolf International 402 The Institute of Chartered Accountants of Pakistan


Chapter 7: IAS 16: Property, plant and equipment

28. The following gains may legally be withdrawn from the company by shareholders:
i. gains that arise from the upward revaluation of non-current assets
ii. gains that arise from the sale of non-current assets
What is the validity of each statement?

(a) Both i. and ii are true

(b) i. is true and ii. is false

(c) Both i. and ii are false

(d) ii. is true and i. is false

29. The financial statements of Saadi Limited for the most recent year indicated the following:
i. a bonus issue of shares
ii. a transfer of profit retained to retained earnings
iii. an increase in the revaluation reserve due non-current assets
iv. a rights issue of shares
Which of the above involved a movement of cash?

(a) i. and ii

(b) ii. and iii.

(c) iii only

(d) iv only

30. An apartment is revalued upwards by Rs. 1 million. It was acquired 5 years ago for Rs. 5 million.
Its useful life remains same as 10 years.
What is the revised depreciation charge for the year after revaluation?

(a) Rs. 500,000

(b) Rs. 600,000

(c) Rs. 700,000

(d) Rs. 800,000

31. A building is revalued upwards by Rs. 2 million. It was acquired five years ago for Rs.10 million.
Its useful life remains same as 20 years. What is the incremental depreciation charge for the year?

(a) Rs.100,000

(b) Rs.133,333

(c) Rs.166,667

(d) Rs.200,000

© Emile Woolf International 403 The Institute of Chartered Accountants of Pakistan


Financial accounting and reporting I

32. An IT equipment being carried at revaluation model has revaluation reserve balance of Rs. 50,000.
During the year, it reduces its value due to technological obsolescence. It has Rs. 70,000 decrease
in value. What would be the impact of this revaluation decrease?

(a) The decrease of Rs.50,000 is debited to revaluation reserve and Rs.20,000 to profit or loss
for the year

(b) The decrease of Rs.50,000 is debited to profit and loss account and Rs.20,000 to revaluation
reserve for the year

(c) The whole decrease is debited to revaluation reserve

(d) The whole decrease is debited to profit or loss for the year

33. The correct accounting treatment of initial operating losses incurred during the commercial
production due to under-utilization of the plant would be to:

(a) capitalise as a directly attributable cost

(b) defer and charge to profit or loss account when profit is earned from the plant

(c) charge directly to retained earnings since these are not considered to be normal operating
losses

(d) charge to profit or loss account

34. Which of the following is NOT considered as an item of property, plant and equipment?

(a) A standby generator expected to be used for seven years

(b) A plot of land held for resale

(c) A bus for pick and drop of staff members

(d) A generator for rental to others

© Emile Woolf International 404 The Institute of Chartered Accountants of Pakistan


Chapter 7: IAS 16: Property, plant and equipment

3 OBJECTIVE BASED ANSWERS


01. (d) Rs.
Current value 500,000
Carrying amount
(100,000 – (100,000 × 2% × 15 yrs)) (70,000)
Revaluation gain 430,000
02. (a)
Building A Building B
Current value 400,000 250,000
Carrying amount (170,000) (330,000)
Revaluation gain/(loss) 230,000 (80,000)
The gain on Building A will be credited to other comprehensive income and
the revaluation surplus.
The loss on Building B will be debited to the statement of profit or loss
expenses because we do not have a balance on the revaluation surplus in
respect of building B to offset the loss.
We make an overall debit to non-current assets of Rs. 230,000 – Rs.
80,000 = Rs. 150,000

03. (a) Land Buildings Total


Rs. Rs. m Rs. m
Cost 1 July 2013 1.00 5.00 6.00
Building depreciation
Rs. 5 million/50 years x 2 years (0.2) (0.2)
Carrying amount 30 June 2015 1.00 4.80 5.80
Revaluation gain 0.24 1.96 1.20
Revalued amount 1.24 5.76 7.00
Building depreciation
Rs. 5.76m/48 years x 2 years (0.24) (0.24)
Carrying amount 30 June 2017 1.24 5.52 6.76
Disposal proceeds 6.80
Gain on disposal 0.04
The gain on disposal is Rs. 40,000. The Rs. 1.2 million balance on the
revaluation reserve is transferred from the revaluation reserve to another
reserve account (probably retained earnings) but is not reported through
the statement of profit or loss for the year.

04. (a) IAS 16 (para 31) states that when the revaluation model is used,
revaluations should be made with sufficient regularity to ensure that the
carrying value of the assets remains close to fair value. IAS 16 also states
(para 36) that, if one item in a class of assets is revalued, all the assets in
that class must be revalued.

05. (c) Six months’ depreciation to the date of the revaluation will be Rs. 300,000
(12,000/20 years × 6/12). Six months’ depreciation from the date of
revaluation to 31 March 2015 would be Rs. 400,000 (10,800/13.5 years
remaining life × 6/12). Total depreciation is Rs. 700,000.

© Emile Woolf International 405 The Institute of Chartered Accountants of Pakistan


Financial accounting and reporting I

06. (c) Building a/c


Particulars Rs. Particulars Rs.
b/d 10,000,000 Acc. dep 500,000
Surplus 2,500,000 c/d 12,000,000
12,500,000 12,500,000

Building net debited by Rs. 2,000,000 (2,500,000 – 500,000)

07. (a) Total loss Rs. 13 million, Rs. 10 will be charged to revaluation surplus and
remaining to profit or loss.

08. (b) Depreciation (20 – 8) / 30 years = Rs. 0.4 million


Carrying amount Rs. 20 million less 0.4 million = Rs. 19.6 million

09. (c) Depreciation Now (20 – 8) / 30 years = Rs. 0.4 million


Depreciation Cost (10 – 2) / 40 years = Rs. 0.2 million
Revaluation surplus
= Land Rs. 6 million + Building Rs. 4 million - incremental depreciation 0.2
million = Rs. 9.8 million

10. (d)

11. (a) On disposal of a revalued asset, the full balance of surplus on revaluation
is transferred to retained earnings.

12. (b) Accumulated depreciation = Rs. 216,000-Rs. 145,000=Rs. 71,000


Surplus = Rs. 291,000-Rs. 145,000=Rs. 146,000
Net amount debited to asset = Rs. 146,000-Rs. 71,000=Rs. 75,000

13. (d)

14. (d)

15. (b)

16. Rs. 3,087 The machine has been owned for 2 years 3 months, so the remaining
useful life at 31 March 2017 was 9 years 9 months.
Prior to revaluation it was being depreciated at Rs. 2,500 pa (30,000/12),
so the charge for the first three months of 2017 was Rs. 625.
The machine will now be depreciated over the remaining 9 years 9 months
= 117 months. So the charge for the remaining 9 months of 2017 is Rs.
2,462 ((32,000 / 117) × 9).
So total depreciation for the year ended 31.12.17 is (625 + 2,462) = Rs.
3,087

17. Rs. 75,000 Incremental depreciation = depreciation on revalued amount – depreciation


at cost
Dep. before revaluation = Rs. 7,500,000 / 20 years = Rs. 375,000
Dep. after revaluation = Rs. 7,650,000 / 17 years = Rs. 450,000
Incremental depreciation = Rs. 75,000

18. Rs. 562,500 Depreciation = Rs. 4,500,000/8= Rs. 562,500

19. Rs. 500,000 Revaluation surplus = Rs. 4,000,000 – 4,500,000= Rs. 500,000

© Emile Woolf International 406 The Institute of Chartered Accountants of Pakistan


Chapter 7: IAS 16: Property, plant and equipment

20. Rs. 62,500 Incremental depreciation = Dep on revalued amount – Dep on cost
= (4,500,000/8)– (5,000,000/10)
=Rs. 562,500 – 500,000 = 62,500
Alternatively, Rs. 500,000 surplus / 8 years = Rs. 62,500

21. (a)

22. (c)

23. (b)

24. (c)

25. (d)

26. (d)

27. (c)

28. (d)

29. (d)

30. (c)

31. (b)

32. (a)

33. (d) Charge to profit or loss account

34. (b) A plot of land held for resale

© Emile Woolf International 407 The Institute of Chartered Accountants of Pakistan


Financial accounting and reporting I

© Emile Woolf International 408 The Institute of Chartered Accountants of Pakistan


Certificate in accounting and Finance

8
Financial accounting and reporting I

CHAPTER
Non-current assets: sundry standards

Contents
1 IAS 20: Accounting for government grants and disclosure of
government assistance
2 IAS 23: Borrowing costs
3 IAS 40: Investment property
4 Objective based questions and answers

* The student must refer original handbook of IFRS.

© Emile Woolf International 409 The Institute of Chartered Accountants of Pakistan


Financial accounting and reporting I

1 IAS 20: ACCOUNTING FOR GOVERNMENT GRANTS AND DISCLOSURE OF


GOVERNMENT ASSISTANCE
Section overview

◼ Introduction and definitions


◼ Accounting treatment of government grants
◼ Repayment of government grants
◼ Government assistance
◼ Disclosure requirements

1.1 Introduction and definitions


In many countries the government provides financial assistance to industry. The most common
form of such assistance is a grant of cash from local or national government.

Definitions

Government assistance is action by government designed to provide an economic benefit specific


to an entity or range of entities qualifying under certain criteria. Government assistance does not
include benefits provided only indirectly through action affecting general trading conditions, such
as the provision of infrastructure in development areas or the imposition of trading constraints on
competitors.

Government grants are assistance by government in the form of transfers of resources to an entity
in return for past or future compliance with certain conditions relating to the operating activities of
the entity. They exclude those forms of government assistance which cannot reasonably have a
value placed upon them and transactions with government which cannot be distinguished from the
normal trading transactions of the entity.

Forgivable Loan is treated as a government grant when there is reasonable assurance that the
entity will meet the terms for forgiveness of the loan.

Low interest loans are loans which the government provides at lower interest rate as compare to
market interest rate.

IAS 20 Accounting for Government Grants and Disclosure of Government Assistance identifies two
types of government grants:
❑ grants related to assets, or
❑ Grants related to income.

Definitions

Grants related to assets are government grants whose primary condition is that an entity qualifying
for them should purchase, construct or otherwise acquire long-term assets. Subsidiary conditions
may also be attached restricting the type or location of the assets or the periods during which they
are to be acquired or held.

Grants related to income are government grants other than those related to assets.

Government grants are sometimes called by other names such as subsidies, subventions, or
premiums.

© Emile Woolf International 410 The Institute of Chartered Accountants of Pakistan


Chapter 8: Non-current assets: sundry standards

1.2 Accounting treatment of government grants


IAS 20 states that grants should not be recognized until there is reasonable assurance that:
❑ the entity will comply with any conditions attaching to the grant, and
❑ The grant will be received.
Once these recognition criteria are met, the grants should be recognized in profit or loss over the
periods necessary to match them with their related costs.
Neither type of grant should be credited directly to shareholders’ interests in the statement of
financial position. They must be reported on a systematic basis through the statement of profit or
loss (profit or loss).
Grants related to income
For grants related to income, IAS 20 states that an ‘income approach’ should be used, and the
grant should be taken to income over the periods necessary to match the grant with the costs that
the grant is intended to compensate.
IAS 20 allows two methods of doing this:
❑ Method 1. Include the grant for the period as ‘other income’ for inclusion in profit or loss for
the period
❑ Method 2. Deduct the grant for the period from the related expense.

Illustration 01: Grant related to income


A company receives a cash grant of Rs. 30,000 on 31 December Year 0.
The grant is towards the cost of training young apprentices, and the training programme is
expected to last for 18 months from 1 January Year 1.
Actual costs of the training were Rs. 50,000 in Year 1 and Rs. 25,000 in Year 2.
The grant would be accounted for as follows:
At 31 December Year 0 the grant would be recognized as a liability and presented in the
statement of financial position split between current and non-current amounts. Rs. 20,000 (12
months/18 months Rs. 30,000) is current and would be recognized in profit for Year 1. The
balance is non-current.
At the end of year 1 there would be a current balance of Rs. 10,000 (being the non-current
balance at the end of Year 0 reclassified as current) in the statement of financial position. This
would be recognized in profit in Year 2.
Extracts from the financial statements are as follows:
Statement of financial position (extracts)
31 31 31
December December December
Year 0 Year 1 Year 2
Current liabilities
Deferred income 20,000 10,000 −
Non-current liabilities
Deferred income 10,000 − −

Statement of profit or loss (extracts)


Method 1
Training costs (50,000) (25,000)
Government grant received 20,000 10,000

© Emile Woolf International 411 The Institute of Chartered Accountants of Pakistan


Financial accounting and reporting I

31 31 31
December December December
Year 0 Year 1 Year 2
Method 2
Training costs (50,000 – 20,000) 30,000
Training costs (25,000 – 10,000) 15,000

Grants related to assets


For grants related to assets, IAS 20 allows two methods of doing this:
❑ Method 1. Deduct the grant from the cost of the related asset. The asset is included in the
statement of financial position at cost minus the grant. Depreciate the net amount over the
useful life of the asset.
❑ Method 2. Treat the grant as deferred income and recognize it as income on a systematic
basis over the useful life of the asset.
Both methods achieve the same effective result.

Illustration 02: Grant related to an asset


A company receives a government grant of Rs. 400,000 towards the cost of an asset with a cost of
Rs. 1,000,000.
The asset has an estimated useful life of 10 years and no residual value.
The amounts could be reflected in the financial statements prepared at the end of Year 1 in
accordance with IAS 20 in the following ways:
Method 1:
Statement of financial position (extract)
Property, plant and equipment Rs.
Cost (1,000,000 – 400,000) 600,000
Accumulated depreciation (60,000)
Carrying amount 540,000
Included in statement of profit or loss (extract)
Depreciation charge (Rs. 600,000/10 years) 60,000
Method 2:
Statement of financial position (extract)
Property, plant and equipment Rs.
Cost 1,000,000
Accumulated depreciation (100,000)
Carrying amount 900,000
Current liabilities
Deferred income 40,000
Non-current liabilities
Deferred income 320,000
At the end of year 1 there would be Rs. 360,000 of the grant left to recognise in profit in
the future at Rs. 40,000 per annum. Rs. 40,000 would be recognised in the next year and
is therefore current. The balance is non-current

© Emile Woolf International 412 The Institute of Chartered Accountants of Pakistan


Chapter 8: Non-current assets: sundry standards

Included in statement of profit or loss (extract) Rs.


Expense: Depreciation charge (Rs. 1,000,000/10 years) (100,000)
Income: Government grant (Rs. 400,000/10 years) 40,000

Illustration 03:
On January Year 1 Entity O purchased a non-current asset with a cost of Rs. 500,000 and received
a grant of Rs. 100,000 in relation to that asset.
The asset is being depreciated on a straight-line basis over five years.
The grant would be reflected in the financial statements at the end of the first year under both
methods of accounting for the grant allowed by IAS 20 are as under:
The amounts could be reflected in the financial statements prepared at the end of Year 1 in
accordance with IAS 20 in the following ways:

Method 1:
Statement of financial position
Property, plant and equipment Rs.
Cost (500,000 – 100,000) 400,000
Accumulated depreciation (80,000)
Carrying amount 320,000
Included in statement of profit or loss
Depreciation charge (Rs.400,000/5 years) 80,000

Method 2:
Statement of financial position
Property, plant and equipment Rs.
Cost 500,000
Accumulated depreciation (100,000)
Carrying amount 400,000
Current liabilities
Deferred income 20,000
Non-current liabilities
Deferred income 60,000
At the end of year 1 there would be Rs. 80,000 of the grant left to recognise in profit in
the future at Rs. 20,000 per annum. Rs. 20,000 would be recognised in the next year
and is therefore current. The balance is non-current.
Included in statement of profit or loss Rs.
Expense: Depreciation charge (Rs. 500,000/5 years) (100,000)

Income: Government grant (Rs. 100,000/5 years) 20,000

© Emile Woolf International 413 The Institute of Chartered Accountants of Pakistan


Financial accounting and reporting I

Grants as compensation for expenses or losses


A government grant may be awarded for the purpose of giving immediate financial support to an
entity rather than as an incentive to undertake specific expenditures.
The circumstances under which the grant is made may warrant immediate recognition of the grant
in profit or loss in the period in which the entity qualifies to receive it with disclosure to ensure that
its effect is clearly understood.
A government grant may become receivable by an entity as compensation for expenses or losses
incurred in a previous period. Such a grant is recognized in profit or loss of the period in which it
becomes receivable, with disclosure to ensure that its effect is clearly understood.

1.3 Repayment of government grants


A government grant might become repayable by the entity (e.g. when the entity fails to meet the
underlying conditions for the grant).
When a government grant becomes repayable it is accounted for as a change in accounting
estimate (IAS 8: Accounting policies, changes in accounting estimates and errors).
Repayment of a grant related to income
Repayment of a grant related to income is applied in the first instance against any unamortized
deferred credit recognized in respect of the grant. If the repayment exceeds any such deferred
credit any excess is recognized immediately in profit or loss.

Illustration 04: Repayment of grant related to income


On 1 January Year 1 X Limited received a cash grant of Rs. 500,000 towards the cost of
employing an environmental impact analyst on a new project for a 5 year period.
The grant is repayable in full if the project is not completed.
The analyst was employed and the project commenced from the 1 January Year 1.
On 1 January Year 3 the project was abandoned and the grant became repayable in full.
The grant and its subsequent repayment is accounted for as follows:

Year 1 Debit Credit


Cash 500,000
Deferred income 500,000
Statement of profit or loss
(reduction of employment cost) 100,000
Deferred income 100,000
Year 2
Statement of profit or loss
(reduction of employment cost) 100,000
Deferred income 100,000
The double entry to reflect the repayment is as follows
Deferred income 300,000
Statement of profit or loss 200,000
Cash 500,000

Repayment of a grant related to an asset


Accounting for a repayment of a grant related to an asset depends on how the grant was accounted
for originally.

© Emile Woolf International 414 The Institute of Chartered Accountants of Pakistan


Chapter 8: Non-current assets: sundry standards

If the grant was accounted for as reduction of the carrying amount of the related asset, its
repayment is recognized by increasing the carrying amount of the asset.
If the grant was accounted for as deferred income, its repayment is recognized by reducing the
deferred income balance by the amount repayable.
The cumulative additional depreciation that would have been recognized in profit or loss to date in
the absence of the grant must be recognized immediately in profit or loss.
Also note that the circumstances giving rise to repayment of the grant might indicate the possible
impairment of the new carrying amount of the asset.

1.4 Government assistance


The definition of government grants (see earlier) excludes:
❑ certain forms of government assistance which cannot reasonably have a value placed upon
them (e.g. free advice and the provision of guarantees); and
❑ Transactions with government which cannot be distinguished from the normal trading
transactions of the entity (e.g. a government procurement policy that is responsible for a
portion of the entity’s sales).
There is no specified accounting treatment for either of these. However, the significance of the
benefit may require disclosure of the nature, extent and duration of the assistance in order to
prevent the financial statements being misleading.
Forgivable Loans
A forgivable loan is treated as a government grant when there is reasonable assurance that the
entity will meet the terms for forgiveness of the loan. Until then, such a loan is treated as a liability
and recognised at fair value under IFRS 9 (Financial Instruments).

Illustration 05:
ABC Pharmaceutical Company received cash from government for a research and development
project of a children vaccine.
Scenario 1
As per the terms of the loan, the cash received from the government shall be waived off if the
entity is able to develop the vaccine within 3 years and sell it free of cost for 5 years.
If the entity takes more time than three years in the development or sells the vaccine for a price
before 5 years, it will be liable to repay the loan and the loan will not be considered a forgivable
loan.
Scenario 2
As per the terms of the loan, the cash received from the government is repayable in cash only
if the entity decides to commercialize the results of the research phase of the project. If the entity
decides not to commercialize the results of the research phase, the cash received is not
repayable in cash, but instead the entity must transfer to the government the rights to the
research.
In this scenario, cash received from the government does not meet the definition of a forgivable
loan in IAS 20. This is because, in this arrangement, the government does not undertake to
waive repayment of the loan, but rather to require settlement in cash or by transfer of the rights
to the research. the cash receipt described in the submission gives rise to a financial liability to
be dealt with under IFRS 9.

Loans with Below-Market Rate of Interest


❑ The benefit of a government loan at a below market rate of interest is treated as a
government grant. The loan shall be recognized and measured as per IFRS 9.

© Emile Woolf International 415 The Institute of Chartered Accountants of Pakistan


Financial accounting and reporting I

❑ The benefit of below market rate of interest shall be measured as the difference between the
cash receipt under a government loan and the fair value of the liability the benefit will be
accounted for IAS 20.
❑ The entity shall consider the conditions and obligations to be met when identifying the costs
which the benefit of the loan is intended to compensate,
Non-monetary government grants
❑ A government grant may take the form of a transfer of a non-monetary asset, such as land
or other resources, for the use of the entity. In such case, the fair value of the non-monetary
asset should be assessed and to account for both grant and asset at that fair value.
❑ Alternatively, both asset and grant may be recorded at a nominal amount.

1.5 Disclosure requirements


IAS 20 requires the following disclosures in the notes to the financial statements:
❑ the accounting policy adopted for government grants, including the method of presentation
in the financial statements
❑ The nature and extent of government grants recognized in the financial statements and an
indication of other forms of government assistance from which the entity has directly
benefitted.
❑ Unfulfilled conditions and other contingencies attaching to government assistance (if this
assistance has been recognized in the financial statements).
Government assistance may be significant so that disclosure of the nature, extent and duration of
the assistance is necessary in order that the financial statements may not be misleading.

Example 01: Accounting treatment


Question: Discuss how the following should be dealt with in the financial statements of relevant
entities according to IAS 20 Accounting for Government Grants and Disclosure of Government
Assistance:
(a) The government makes a grant to an entity which is planning to develop teaching software for
children with learning difficulties. The purpose of the grant is to help the entity to meet its
general financing requirement in the initial phase. There are no further conditions attached to
the grant.
(b) A manufacturing entity sets up a plant in an area of high unemployment. A government grant
of Rs. 4 million is received with a condition that the grant is repayable in full if the number of its
employees fell below 100 at any time during the next four years. It is highly probable that the
entity will comply with the condition attached to the grant.
(c) Free technical advice has been provided by the government’s export promotion department to
help an exporter to market his new technology in North America.
Answer:
Part (a)
The grant has been provided for the purpose of giving immediate financial support to the entity with
no further conditions, so this grant should be immediately recognised in profit or loss in full in the
period in which the entity qualifies to receive it (when it is receivable) with disclosure to ensure that
its effect is clearly understood.
Part (b)
Since there is reasonable assurance that conditions attaching to the grant will be met, the grant is
recognised in statement of profit or loss over the four year period in which the entity incurs the costs
of employing 100 people. Amount taken to statement of profit or loss may be either be presented
as other income or shown as deduction from the related expense. The remaining amount of grant
will be presented as deferred income under liabilities in the balance sheet.

© Emile Woolf International 416 The Institute of Chartered Accountants of Pakistan


Chapter 8: Non-current assets: sundry standards

Part (c)
Free technical advice is government assistance that cannot reasonably have a value placed upon it
and therefore should not be recognised. However, an indication of such assistance should be
disclosed in financial statements.

Example 02: KATIE


Question: During the year ended 30 June Year 2, Katie received three grants, the details of which
are set out below.
(1) On 1 September, a grant of Rs. 40,000 from local government. This grant was in respect of
training costs of Rs. 70,000 which Katie had incurred.
(2) On 1 November Katie bought a machine for Rs. 350,000. A grant of Rs. 100,000 was received
from central government in respect of this purchase. The machine, which has a residual value
of Rs. 50,000, is depreciated on a straight-line basis over its useful life of five years.
(3) On 1 June a grant of Rs. 100,000 from local government. This grant was in respect of
relocation costs that Katie had incurred moving part of its business from outside the local
area. The grant is repayable in full unless Katie recruits ten employees locally by the end of
Year 2. Katie is finding it difficult to recruit as the local skill base does not match the needs
of this part of the business.
Required:
Show how the above transactions should be reflected in the financial statements of Katie for the
year ended 30 June Year 2. Where any accounting standards allow a choice you should show all
possible options.
Answer:

The above transactions should be reflected in the financial statements of Katie for the year ended
30 June Year 2 based on IAS 20 as under:
Option 1 – Net grants off related expenditure
Statement of financial position as at 30 June Year 2 (extracts)

Rs.
Non-current assets
Property, plant and equipment 223,333
Current liabilities
Other current liabilities 100,000

Notes to the financial statements for the year ended 30 June Year 2 (extracts)

Property, plant and equipment Rs.


Cost (350,000 – 100,000) 250,000
Accumulated depreciation ((250,000 – 50,000) ÷ 5  8/12) (26,667)
Carrying amount 223,333

Included in statement of profit or loss for the year ended 30 June Year 2

Rs.
Depreciation charge 26,667
Training costs (70,000 – 40,000) 30,000

© Emile Woolf International 417 The Institute of Chartered Accountants of Pakistan


Financial accounting and reporting I

Option 2 – Show grants separately from related expenditure


Statement of financial position as at 30 June Year 2 (extracts)

Rs.
Non-current assets
Property, plant and equipment 310,000

Current liabilities
Other current liabilities 186,667

Notes to the financial statements for the year ended 30 June Year 2 (extracts)

Rs.
Property, plant and equipment
Cost 350,000
Accumulated depreciation ((350,000 – 50,000) ÷ 5  8/12) (40,000)
Carrying amount 310,000
Other current liabilities
Deferred income relating to government grants 86,667
(100,000 - (100,000 ÷ 5  8/12))
Government grant repayable 100,000
186,667

Included in statement of profit or loss for the year ended 30 June Year 2

Rs.

Depreciation charge 40,000

Training costs 70,000

Government grant received (40,000)

Release of deferred government grant (13,333)

Tutorial note
The Rs. 100,000 grant in (3) has conditions attached to it. In such a situation, IAS 20 states that
grants should not be recognised until there is reasonable assurance that the entity will comply with
any conditions attaching to the grant. Since Katie is struggling to recruit, and there is only one month
left for recruitment to meet these conditions, then it does not seem that there is ‘reasonable
assurance’. Hence the grant should not be recognised as such, but should be held in current
liabilities, pending repayment.

© Emile Woolf International 418 The Institute of Chartered Accountants of Pakistan


Chapter 8: Non-current assets: sundry standards

2 IAS 23: BORROWING COSTS


Section overview

◼ Introduction
◼ Borrowing costs eligible for capitalisation
◼ Period of capitalisation
◼ Disclosures

2.1 Introduction
A company might incur significant interest costs if it has to raise a loan to finance the purchase or
construction of an asset. IAS 23: Borrowing costs defines borrowing costs and sets guidance on
the circumstances under which they are to be capitalised as part of the cost of qualifying assets.

Definition: Borrowing costs


Borrowing costs are interest and other costs that an entity incurs in connection with the borrowing
of funds.

Definition: Qualifying asset


A qualifying asset is an asset that necessarily takes a substantial period of time to get ready for
its intended use or sale.

Any of the following may be qualifying assets depending on circumstances:


❑ inventories;
❑ items of property, plant and equipment;
❑ Intangible assets.
The following are not qualifying assets:
❑ inventories that are manufactured, or otherwise produced, over a short period of time, are
not qualifying assets
❑ Assets that is ready for their intended use or sale when acquired.
❑ Qualifying assets are usually self-constructed non-current assets.

2.2 Borrowing costs eligible for capitalization


Borrowing costs that are directly attributable to the acquisition, construction or production of a
qualifying asset must be capitalised as part of the cost of that asset. All other borrowing costs are
recognized as an expense in the period in which they are incurred.
Borrowing costs that are directly attributable to the acquisition, construction or production of a
qualifying asset are those that would have been avoided if the expenditure on the qualifying asset
had not been made.
This includes the costs associated with specific loans taken to fund the production or purchase of
an asset and general borrowings. General borrowings are included because if an asset was not
being constructed it stands to reason that there would have been a lower need for cash.

© Emile Woolf International 419 The Institute of Chartered Accountants of Pakistan


Financial accounting and reporting I

Funds specifically borrowed to obtain a qualifying asset


When a specific loan is taken in order to obtain a qualifying asset the borrowing costs eligible for
capitalization are the actual borrowing costs incurred on that borrowing during the period less any
investment income on the temporary investment of those borrowings.

Illustration 06: Specific borrowings


On 1 January 2016 Okara Engineering issued a bond to raise Rs. 25,000,000 to fund a capital
project which will take three years to complete. Amounts not yet needed for the project are invested
on a temporary basis. During the year to 31 December 2016, Okara Engineering spent Rs.
9,000,000 on the project.
The cost of servicing the bond was Rs. 1,250,000 during this period and the company was able to
earn Rs. 780,000 through the temporary reinvestment of the amount borrowed.
The amounts recognized as capital work in progress in the period was:

Rs.
Costs incurred (labour, material, overhead etc.) 9,000,000
Interest capitalised:
Actual interest cost 1,250,000
Less: return on temporary investment (780,000)
470,000
Additions to capital work in progress 9,470,000

General funds used for the purpose of obtaining a qualifying asset.


When general borrowings are used the amount of borrowing costs eligible for capitalisation is
obtained by applying a capitalisation rate to the expenditures on that asset.
The capitalisation rate is the weighted average of the borrowing costs applicable to the borrowings
that are outstanding during the period except for borrowings made specifically for the purpose of
obtaining a qualifying asset.
The amount of borrowing costs capitalised cannot exceed the amount of borrowing costs it incurred
during a period.

Illustration 07: General borrowings: Capitalisation rate


Sahiwal Construction has three sources of borrowing:

Average loan in the year Interest expense incurred in


(Rs.) the year (Rs.)
7 year loan 8,000,000 800,000
10 year loan 10,000,000 900,000
Bank overdraft 5,000,000 900,000
The 7 year loan has been specifically raised to fund the building of a qualifying asset.
A suitable capitalisation rate for other projects is found as follows:
General borrowings Average loan in the year Interest expense incurred in
(Rs.) the year (Rs.)
10 year loan 10,000,000 900,000
Bank overdraft 5,000,000 900,000
15,000,000 1,800,000
Capitalisation rate = 1,800,000/15,000,000 100 = 12%

© Emile Woolf International 420 The Institute of Chartered Accountants of Pakistan


Chapter 8: Non-current assets: sundry standards

Alternatively:
Rate on 10 year loan = 900,000/10,000,000 100 = 9%
Rate on bank overdraft = 900,000/5,000,000 100 = 18%
Weighted average: 9% 10,000,000/15,000,000 + 18% 5,000,000/15,000,000
6% + 6% = 12%
The capitalisation rate is applied from the time expenditure on the asset is incurred.

Illustration 08: General borrowings: Capitalisation rate


Continuing the illustration above, Sahiwal Construction has incurred the following expenditure on
a project funded from general borrowings for year ended 31 December 2016.

Date incurred: Amount (Rs.)


31st March 1,000,000
31st July 1,200,000
30th October 800,000

The amount capitalised in respect of capital work in progress during 2016 is as follows:

Rs.
31st March − Expenditure 1,000,000
Interest (1,000,000  12% 9/12) 90,000

31st July − Expenditure 1,200,000


Interest (1,200,000  12% 5/12) 60,000

30th October − Expenditure 800,000


Interest (800,000  12% 2/12) 16,000
3,166,000

2.3 Period of capitalisation


Commencement of capitalisation
Capitalisation of borrowing costs should start only when:
❑ expenditures for the asset are being incurred; and
❑ borrowing costs are being incurred, and
❑ Activities necessary to prepare the asset have started.
Suspension of capitalisation
Capitalisation of borrowing costs should be suspended if development of the asset is suspended
for an extended period of time.
Cessation of capitalisation
Capitalisation of borrowing costs should cease when the asset is substantially complete. The costs
that have already been capitalised remain as a part of the asset’s cost, but no additional borrowing
costs may be capitalised.

© Emile Woolf International 421 The Institute of Chartered Accountants of Pakistan


Financial accounting and reporting I

Illustration 09: Commencement and suspension of capitalisation


Company A borrowed Rs. 9,000 @ 15% per annum to fund a project on 1st Jan 2016.
The following expenditures were made on the project during the year ending 31 December 2016
Date: 1st March 2016: Rs. 2,500
Date: 1st Oct 2016: Rs. 4,200
Date: 1st Dec 2016: Rs. 2,300
Surplus funds were invested @10% whenever available. The project commenced on 1st March
2016. Work on the project was suspended during the month of August and resumed in September.
Construction was completed on 31 December 2016.
The relevant calculation of borrowing costs to be capitalised is as under:
Borrowing costs to be capitalised Rs.
Borrowing costs incurred 9,000 x 15% x 9/12 1,012.5
Less: Temporary investment income 6,500 x 10% x 6/12 (325)
2,300 x 10% x 2/12 (38.3)
649.2

The project commenced on 1st March resulting in a period of 10 months up to the year end.
However, interest cannot be capitalised during the period of suspension. Therefore, interest is
capitalised only for 9 months.

Borrowing costs to be charged as expense Rs.


Borrowing costs incurred 9,000 x 15% x 3/12 337.5
Less: Temp. Inv. income 9,000 x 10% x 2/12 (before commencement) (150)
6,500 x 10% x 1/12 (suspension) (54.17)
133.33
.

2.4 Disclosures
IAS 23 requires disclosure of the following:
❑ the amount of borrowing costs capitalised during the period; and
❑ The capitalisation rate used to determine the amount of borrowing costs eligible for
capitalisation.

Example 03: SHAYAN LIMITED


Question: Shayan Limited (SL) started the construction of its new factory on 1 January 2018 with
a loan of Rs. 50,000,000 borrowed at an interest rate of 8% per annum.
The loan was used on the factory as follows:

Date of Payment Rs. in million


Jan 1, 2018 25
May 1, 2018 15
Oct 1, 2018 10

The construction of the asset was completed on 31 December 2018. However, during the
accounting period SL invested the surplus funds at an interest rate of 3%.
Required:
How much the amount of borrowing cost eligible for capitalization at 31.12.2018?

© Emile Woolf International 422 The Institute of Chartered Accountants of Pakistan


Chapter 8: Non-current assets: sundry standards

Answer:
Borrowing costs to be capitalised Rs.
Borrowing costs incurred Rs. 50m x 8% 4,000,000
Less: Temporary investment income Rs. 25m x 3% x 4/12 (250,000)
Rs. 10m x 3% x 5/12 (125,000)
3,625,000
.

Example 04: SARA LIMITED


Question: On January 1, 2018 Sara Limited (SL) started the construction of an asset. To meet the
financing requirements, borrowing was made from three different banks at the start of the year
as follows:
Banks Amount (Rs.) Interest Rate per annum
A 70,000 10%
B 60,000 8%
C 50,000 12%

The funds were used on the assets as follows:


Date of Payment Rs.
Jan 1, 2018 30,000
May 1, 2018 20,000
Oct 1, 2018 15,000

The construction of asset was completed on 31 December 2018.


Required:
Calculate the general weighted average borrowing rate and eligible borrowing cost.
Answer:
Borrowing costs eligible for capitalisation Rs.
30,000 x 9.89% x 12/12 2,967
20,000 x 9.89% x 8/12 1,318
15,000 x 9.89% x 3/12 371
Borrowing costs to be capitalised 4,656

(70,000 x 10%) + (60,000 x 8%) + (50,000 x 12%)


Capitalisation rate = X 100 = 9.89%
70,000 + 60,000 + 50,000

Example 05: LOONEY


Question: Looney has recently finished building a new item of plant for its own use. The item is a
press for use in the manufacture of industrial diamonds. Looney commenced construction of the
asset on 1st April 2013 and completed it on 1st April 2015.
1st January 2013, Looney took out a loan to finance the construction of the asset. Interest is
charged on the loan at the rate of 5% per annum. The annual interest must be paid in four equal
installments at the end of each quarter. Looney capitalises interest on manufactured assets in
accordance with the rules in IAS 23 Borrowing costs.
The costs (excluding finance costs) of manufacturing the asset were Rs. 28 million.

© Emile Woolf International 423 The Institute of Chartered Accountants of Pakistan


Financial accounting and reporting I

Required:
State the IAS 23 rules on the capitalisation of borrowing costs, calculate the cost of the asset on
initial recognition and explain the amount of borrowing cost capitalised.
Answer:
IAS 23 should be applied in accounting for borrowing costs. Borrowing costs are recognised as an
expense in the period in which they are incurred unless they are capitalised in accordance with IAS
23 which says that borrowing costs that are directly attributable to the acquisition, construction or
production of a qualifying asset can be capitalised as part of the cost of that asset.
❑ A qualifying asset is an asset that necessarily takes a substantial period of time to get ready
for its intended use or sale.
❑ Borrowing costs that are directly attributable to acquisition, construction or production are
taken to mean those borrowing costs that would have been avoided if the expenditure on the
qualifying asset had not been made.

When an enterprise borrows specifically for the purpose of funding an asset, the identification of
the borrowing costs presents no problem as the amount capitalised is the actual borrowing costs
net of any income earned on the temporary investment of those borrowings.
If funds are borrowed, generally, the amount of borrowing costs eligible for capitalisation is
determined by applying a capitalisation rate to the expenditures on that asset calculated as the
weighted average of the borrowing costs applicable to general borrowings.
IAS 23 also contains rules on commencement of capitalisation, suspension of capitalisation and
cessation of capitalisation.

Amount capitalised Rs.000


Cost of manufacture 28,000
Interest capitalised (Rs.28m × 5% × 2 years) 2,800
30,800

Example 06: GOOGLY INDUSTRIES LIMITED


Question: On 1 January 20X6 Googly Industries Limited (GIL) borrowed Rs.15 million to finance
the production of two assets, both of which were expected to take a year to build. Work started
during 20X6. The loan facility was drawn down and incurred on 1 January 20X6, and was utilised
as follows, with the remaining funds invested temporarily.

Asset A Asset B
----------- Rs. in million ---------
1 January 20X6 2.5 5
1 July 20X6 2.5 5

The loan rate was 9% and GIL can invest surplus funds at 7%.
Required:
Calculate the borrowing costs which may be capitalised for each of the assets and consequently
the cost of each asset as at 31 December 20X6.

© Emile Woolf International 424 The Institute of Chartered Accountants of Pakistan


Chapter 8: Non-current assets: sundry standards

Answer:
The borrowing cost to be capitalized & cost of assets are as under:
Cost of asset A Rs.
Expenditure 5,000,000

Borrowing costs incurred 5m x 9% x 12/12 450,000


Less: Temporary investment income 2.5m x 7% x 6/12 (87,500)
362,500
5,362,000

Cost of asset B Rs.


Expenditure 10,000,000

Borrowing costs incurred 10m x 9% x 12/12 900,000


Less: Temporary investment income 5m x 7% x 6/12 (175,000)
725,000
10,725,000
.

Example 07: KHAN LIMITED


Question: Khan Limited (KL) has the following loan arrangements as at 1 January 2020:

Rs. in million
7% Debentures 55
8% Loan notes 110
12% Line of credit 85
10% Running finance arrangement 150
On the 1 January 2020, KL commenced the construction of a new factory. The construction of the
factory will cost Rs.100 million and the company funded the construction with the existing
borrowings. The factory was completed on 31 August 2020 but was not available for use until 31
January 2021 as a result of minor modification. During the construction period, active work was
interrupted and the building construction was stopped for two months as a result of adverse weather
conditions.
Required:
Calculate the borrowing cost to be capitalised and the cost of the building to be recognised upon
initial recognition.

Answer:
The borrowing cost to be capitalized & cost of assets are as under:
Rs.
Expenditure 100,000,000
Borrowing costs to be capitalised 100,000,000 x 9.46% x 6/12 4,730,000
104,730,000

January to August are 8 months – two months suspension = 6 months

(55 x 7%) + (110 x 8%) +(85 x 12%) + (150 x 10%)


Capitalisation rate = X 100 = 9.46%
55+110+85+150

© Emile Woolf International 425 The Institute of Chartered Accountants of Pakistan


Financial accounting and reporting I

Example 08: SPIN INDUSTRIES LIMITED


Question: On September 1, 2015, Spin Industries Limited (SIL) started construction of its new
office building and completed it on May 31, 2016. The payments made to the contractor were as
follows:

Date of Payment Rs. in million

September 1, 2015 10

December 1, 2015 15

February 1, 2016 12

June 1, 2016 9

In addition to the above payments, SIL paid a fee of Rs. 8 million on September 1, 2015 for
obtaining a permit allowing the construction of the building.
The project was financed through the following sources:
(i) On August 1, 2015 a medium term loan of Rs. 25 million was obtained specifically for the
construction of the building. The loan carried mark up of 12% per annum payable semi-
annually. A commitment fee @ 0.5% of the amount of loan was charged by the bank.
Surplus funds were invested in savings account @ 8% per annum. On February 1, 2016 SIL
paid the six monthly interest plus Rs. 5 million towards the principal.
(ii) Existing running finance facilities of SIL
❑ Running finance facility of Rs. 28 million from Bank A carrying mark up of 13%
payable annually. The average outstanding balance during the period of construction
was Rs. 25 million.
❑ Running finance facility of Rs. 25 million from Bank B. The mark up accrued during
the period of construction was Rs. 3 million and the average running finance balance
during that period was Rs. 20 million.
Required:
Calculate the amount of borrowing costs to be capitalised on June 30, 2016 in accordance with the
requirements of International Accounting Standards. (Borrowing cost calculations should be based
on number of months).

Answer:
The detailed calculation is as under:

Borrowing costs to be capitalized 2016

Rs.

Commitment fee [Rs.25m x 0.5%] 125,000

Interest on specific borrowings [25m x 12% x 5/12] 1,250,000

[20m x 12% x 4/12] 800,000

Temporary Investment income [Working below] (137,500)

Interest on general borrowings [218,156 + 1,416,338] 1,634,494

3,671,994

© Emile Woolf International 426 The Institute of Chartered Accountants of Pakistan


Chapter 8: Non-current assets: sundry standards

Date Particulars Rs. m Working


01-09-15 Specific loan 25,000,000
(125,000)
Permit (8,000,000)
1st payment (10,000,000)
6,8750,000 x 8% x 3/12 = Rs. 137,500
(temporary investment income)
01-12-15 2nd payment (15,000,000)
(8,125,000) x 16.11% x 2/12 = Rs. 218,156
(Interest on general borrowings)
01-02-16 Loan interest (1,250,000)
Loan Principal (5,000,000)
3rd payment (12,000,000)
(26,375,000) x 16.11% x 4/12 = Rs. 1,416,338
(interest on general borrowings)
31-05-16 Completion

(25m x 13%) + (3m x 12/9)


Capitalisation rate = X 100 = 16.11%
25m + 20m
.

Example 09: Monday Limited


Question: The following information pertains to Monday Limited (ML):
(i) The balances of property, plant and equipment as on 1 January 2018:
Cost/Revalued Accumulated
Assets amount depreciation
----------- Rs. in million -----------
Office building 240 36
Equipment 190 60
Revaluation surplus related to the office building as at 1 January 2018 amounted to Rs.
8.5 million.
(ii) On 1 September 2018, a new equipment was acquired by making payment of Rs. 70 million
to the supplier. An old equipment was also given in exchange to the supplier. The fair values
of the old and new equipment were assessed at Rs. 21 million and Rs. 93 million
respectively. The old equipment had been acquired at a cost of Rs. 40 million on 1 July
2016. Cost incurred on installing the new equipment amounted to Rs. 5 million.
(iii) On 1 January 2018, ML commenced construction of a manufacturing plant. The whole
process of assembling and installation was completed on 31 October 2018. However, the
work was stopped from 16 to 31 August 2018 due to unexpected rains.
The total cost of Rs. 660 million incurred on the plant was paid as under:

Description Payment date Rs. in million


1st payment 1 February 2018 140
2nd payment 1 April 2018 214
3rd payment 1 September 2018 146
4th payment 1 December 2018 160

© Emile Woolf International 427 The Institute of Chartered Accountants of Pakistan


Financial accounting and reporting I

The plant was financed through a bank loan of Rs. 500 million obtained on 1 March 2018.
The loan carries a mark-up of 18% payable annually. The surplus funds available from the
loan were invested in a saving account and earned Rs. 17 million during capitalization
period.
(iv) On 31 December 2018, the revalued amount of office building was assessed at Rs. 178
million by Precise Valuers, an independent valuation firm. Value in use of the office building
as at 31 December 2018 was estimated at Rs. 186 million.
(v) Other relevant details are as follows:

Depreciation Subsequent
Assets Life/rate
method measurement

Office building Straight line 20 years* Revaluation

Equipment Reducing balance 20% Cost

Manufacturing plant Straight line 15 years Cost

* Remaining life at the date of last revaluation


ML accounts for revaluation on net replacement value method and transfers the maximum
possible amount from revaluation surplus to retained earnings on an annual basis.
Required:
In accordance with IFRSs, prepare a note on ‘Property plant and equipment’ for inclusion in ML’s
financial statements for the year ended 31 December 2018. (Comparatives figures and column for
total are not required)
Answer:
Monday Limited
Notes to the financial statement
For the year ended 31 December 2018

Property, plant and equipment: Office building Equipment Plant


------------------ Rs. in million ------------------
Gross carrying amount – opening 240.00 190.00 -
Accumulated depreciation (36.00) (60.00) -
Opening carrying amount 204.00 130.00 -

Additions 96.00 699.25


(70+21+5) (W-3)
Depreciation for the year (12.00) (30.48) (7.77)
(240÷20) (W-2) (9699.25÷15×2÷ 12)
Disposal (24.96)
(W-1)
Revaluation
- Surplus (8.00)
8.5−(8.5÷17)
- P&L (6.00)
Closing carrying amount 178.00 170.56 691.48

© Emile Woolf International 428 The Institute of Chartered Accountants of Pakistan


Chapter 8: Non-current assets: sundry standards

Gross carrying amount – closing 178.00 246.00 699.25


Accumulated depreciation - (75.44) (7.77)
Closing carrying amount 178.00 170.56 691.48

Office building Equipment Plant

Measurement base Revaluation model Cost model Cost model

Useful life (years)/depreciation rate


20 years 20% 15 years
%

Reducing
Straight line Straight line
Depreciation method balance

The last revaluation was performed on 31 December 2018 by Precise Valuers, an independent firm
of valuers.

Carrying value of building, had the cost model been used instead (178+6) 184

W-1: Disposal Rs. in million

2016 40×20%×6÷12 4.00

2017 36×20% 7.20

2018 28.8×20%×8÷12 3.84

Accumulated depreciation 15.04

Book value (40−15.04) 24.96

W-2: Depreciation for the year – equipment Rs. in million

Disposal 28.8×20%×8÷12 3.84

Others (130−28.8)×20% 20.24

Addition 96×20%×4÷12 6.40

30.48

W-3: Cost of plant Rs. in million

Payments 660.00

Borrowing cost:

Interest cost 500×18%×7.5÷12 56.25

Investment income (17.00)

699.25

© Emile Woolf International 429 The Institute of Chartered Accountants of Pakistan


Financial accounting and reporting I

3 IAS 40: INVESTMENT PROPERTY


Section overview

◼ Scope & Definitions


◼ Accounting treatment of investment property
◼ Why investment properties are treated differently from other properties
◼ Transfers and disposals of investment property
◼ Disclosure requirements

3.1 Scope & Definitions


IAS 40: Investment Property defines and sets out the rules on accounting for investment properties.

Definition: Investment property


An investment property is property (land or a building, part of a building or both) held to earn
rentals or for capital appreciation or both.
Investment property differs from other property, which is:
❑ used in the production or supply of goods, or for administrative purposes (which is covered
by IAS 16; Property, plant and equipment); or
❑ Held for sale in the ordinary course of business (which is covered by IAS 2: Inventories).
Examples of investment property
The following are examples of investment property:
❑ Land held for long-term capital appreciation rather than for short-term sale in the ordinary
course of business.
❑ Land held for a currently undetermined future use. (If an entity has not determined that it will
use the land as owner-occupied property or for short-term sale in the ordinary course of
business),
❑ Property that is being constructed or developed for future use as investment property.
Not investment property
The following are examples of items that are not investment property:
❑ property intended for sale in the ordinary course of business;
❑ owner-occupied property including (among other things),
i. property held for future use as owner-occupied property,
ii. property held for future development and subsequent use as owner-occupied property,
iii. property occupied by employees (whether or not the employees pay rent at market
rates) and
iv. owner-occupied property awaiting disposal;
Partly occupied buildings
An entity might use part of a property for the production or supply of goods or services or for
administrative purposes and hold another part of the same property to earn rentals or for capital
appreciation. In other words, part of a property might be owner occupied and part held as an
investment. The two parts are accounted for separately if they could be sold separately (or leased
out separately under a finance lease).
If this is not the case, the property is investment property only if an insignificant portion is held for
use in the production or supply of goods or services or administrative purpose.

© Emile Woolf International 430 The Institute of Chartered Accountants of Pakistan


Chapter 8: Non-current assets: sundry standards

3.2 Accounting treatment of investment property


The recognition criteria for investment property are the same as for property, plant and equipment
under IAS 16. An owned investment property should be recognized as an asset only when:
❑ it is probable that future economic benefits associated with the property will flow to the entity;
and
❑ The cost of the property can be measured reliably.

Measurement at recognition
Owned investment property should be measured initially at cost plus any directly attributable
expenditure (e.g. legal fees, property transfer taxes and other transaction costs) incurred to acquire
the property.
The cost of an investment property is not increased by:
❑ start-up costs (unless necessary to bring the property to the condition necessary for it to be
capable of operating in the manner intended by management);
❑ operating losses incurred before the investment property achieves the planned level of
occupancy; or
❑ Abnormal waste incurred in constructing or developing the property.

Measurement after recognition


After initial recognition an entity may choose as its accounting policy:
❑ the fair value model; or
❑ The cost model.
The chosen policy must be applied to all the investment property of the entity.
Once a policy has been chosen it cannot be changed unless the change will result in a more
appropriate presentation. IAS 40 states that a change from the fair value model to the cost model
is unlikely to result in a more appropriate presentation.

Fair value model for investment property

Definition: Fair value


Fair value is the price that would be received to sell an asset or paid to transfer a liability in an
orderly transaction between market participants at the measurement date.

Under the fair value model the entity should:


❑ revalue all its investment property to ‘fair value’ (open market value) at the end of each
financial year; and
❑ Recognize any resulting gain or loss in profit or loss for the period.
The property would not be depreciated.
This is different to the revaluation model of IAS 16, where gains are reported as other
comprehensive income and accumulated as a revaluation surplus.
If an entity’s policy is to measure investment properties at fair value but its fair value cannot be
measured reliably such investment property shall be measured at cost for example an investment
property under construction. However, if the entity expects the fair value of the investment property
under construction to be reliably measured when construction is complete it shall measure that
investment property under construction at cost until either its fair value becomes reliably measured
or construction is completed (whichever is earlier).

© Emile Woolf International 431 The Institute of Chartered Accountants of Pakistan


Financial accounting and reporting I

If it is not possible to arrive at a reliable fair value figure then the cost model should be adopted for
that property using the cost model in accordance with IAS 16 for owned assets or IFRS 16 for
investment property held by a lessee as a right-of-use asset. This is an exception to the rule that
all investment property must be valued under either one model or the other.

Cost model for investment property


The cost model follows the provisions of IAS 16. The property is measured at cost less
accumulated depreciation (related to the non-land element) and less impairment loss if any.

Illustration 10: Accounting for investment property


On 1 January Year 1 Entity P purchased a building for its investment potential. The building cost
Rs. 1 million with transaction costs of Rs. 10,000.
The depreciable amount of the building component of the property at this date was Rs. 300,000.
The property has a useful life of 50 years.
At the end of Year 1 the property’s fair value had risen to Rs. 1.3 million.
The amounts which would be included in the financial statements of Entity P at 31 December
Year 1, under the cost model are as follows:

Cost model
The property will be included in the statement of financial position as follows:
Rs.
Cost (1,000,000 + 10,000) 1,010,000
Accumulated depreciation (300,000 ÷ 50 years) (6,000)
Carrying amount 1,004,000
The statement of profit or loss will include depreciation of Rs. 6,000.
The amounts which would be included in the financial statements of Entity P at 31 December
Year 1, under the fair value model are as follows:

Fair value model


The property will be included in the statement of financial position at its fair value of Rs.
1,300,000.
The statement of profit or loss will include a gain of Rs. 290,000 (Rs. 1,300,000 – Rs. 1,010,000)
in respect of the fair value adjustment.

3.3 Why investment properties are treated differently from other properties
Most properties are held to be used directly or indirectly in the entity’s business. For example, a
factory, plant and equipment which is used to produce goods for sale. The property is being
consumed and it is appropriate to depreciate it over its useful life.
An investment property is held primarily because it is expected to increase in value over time
(capital appreciation) or it is held to earn rentals. It generates economic benefits for the entity
because it might earn regular stream of income in the form of rentals or might be sold at a profit.
An investment property also differs from owner-occupied properties (IAS 16) because it generates
cash flows that are largely independently of other assets held by an entity.
The most relevant information about an investment property is its fair value (the amount for which
it could be sold). Depreciation is largely irrelevant. Therefore it is appropriate to re-measure an
investment property to fair value each year and to recognize gains and losses in profit or loss for
the period.

© Emile Woolf International 432 The Institute of Chartered Accountants of Pakistan


Chapter 8: Non-current assets: sundry standards

3.4 Transfers and disposals of investment property


If a property is transferred into or out of this category it must be reclassified as an investment
property or as no longer being an investment property. A transfer of investment property can only
be made where there is a change in use (when the property meets or ceases to meet, the definition
of investment property) as illustrated below.

Circumstance for
Transfer from/to Deemed transfer value
a change in use
Commencement Transfer from investment Fair value at the date of change of use
of or development property to owner-occupied becomes the deemed cost for future
with a view to property accounting purposes
owner-occupation
End of owner- Transfer from owner- Where investment properties are measured
occupation occupied property to at fair value, revalue in accordance with IAS
investment property 16 prior to the transfer
Commencement Transfer from investment Fair value at the date of change of use
of development property to inventories becomes the deemed cost for future
with a view to sale accounting purposes
Inception of an Transfer from inventories to Fair value at the date of the transfer, and
operating lease to investment property any difference compared to previous
another party carrying amount is recognised in profit or
loss

Gain or loss on disposal


Gains or losses on disposals of investment properties are included in profit or loss in the period in
which the disposal occurs.

Illustration 11: Disposal of investment property


The investment property in the previous illustration was sold early in Year 2 for Rs. 1,550,000,
Selling costs were Rs. 50,000.
The amount that would be included in the statement of profit or loss for Year 2 in respect of this
disposal under the cost model is as follows:

Cost model Rs.


Sale value 1,550,000
Selling costs (50,000)
Net disposal proceeds 1,500,000
Minus: Carrying amount (1,004,000)
Gain on disposal 496,000
The amount that would be included in the statement of profit or loss for Year 2 in respect of this
disposal under the fair value model is as follows:
(Fair value model Rs.
Sale value 1,550,000
Selling costs (50,000)
Net disposal proceeds 1,500,000
Minus: Carrying amount (1,300,000)
Gain on disposal 200,000

© Emile Woolf International 433 The Institute of Chartered Accountants of Pakistan


Financial accounting and reporting I

3.5 Disclosure requirements


The following disclosures are required by IAS 40 in the notes to the accounts.

Disclosure requirements applicable to both the fair value model and the cost model
❑ whether the fair value model or the cost model is used
❑ the methods and assumptions applied in arriving at fair values
❑ the extent to which the fair value of investment property was based on a valuation by a
qualified, independent value with relevant, recent experience
❑ amounts recognized as income or expense in the statement of profit or loss for:
• rental income from investment property
• operating expenses in relation to investment property
❑ details of any restrictions on the ability to realize investment property or any restrictions on
the remittance of income or disposal proceeds
❑ The existence of any contractual obligation to purchase, construct or develop investment
property or for repairs, maintenance or enhancements.

Disclosure requirements applicable to the fair value model only


There must be reconciliation, in a note to the financial statements, between opening and closing
values for investment property, showing:
❑ additions during the year
❑ assets classified as held for sale in accordance with IFRS 5
❑ net gains or losses from fair value adjustments
❑ acquisitions through business combinations
This reconciliation should show separately any amounts in respect of investment properties
included at cost because their fair values cannot be estimated reliably.
For investment properties included at cost because fair values cannot be estimated reliably, the
following should also be disclosed:
❑ a description of the property
❑ an explanation as to why fair values cannot be determined reliably
❑ If possible, the range within which the property’s fair value is likely to lie.

Disclosure requirements applicable to the cost model only


❑ the depreciation methods used
❑ the useful lives or depreciation rates used
❑ gross carrying amounts and accumulated depreciation at the beginning and at the end of
the period
❑ A reconciliation between opening and closing values showing:
• additions
• depreciation
• assets classified as held for sale in accordance with IFRS 5
• acquisitions through business combinations
• impairment losses
• transfers

© Emile Woolf International 434 The Institute of Chartered Accountants of Pakistan


Chapter 8: Non-current assets: sundry standards

When the cost model is used, the fair value of investment property should also be disclosed. If the
fair value cannot be estimated reliably, the same additional disclosures should be made as under
the fair value model.

Example 10: VICTORIA


Question: Victoria owns several properties and has a year end of 31 December. Wherever possible,
Victoria carries investment properties under the fair value model.
Property 1 was acquired on 1 January Year 1. It had a cost of Rs. 1 million, comprising Rs. 500,000
for land and Rs. 500,000 for buildings. The buildings have a useful life of 40 years. Victoria uses this
property as its head office.
Property 2 was acquired many years ago for Rs. 1.5 million for its investment potential. On 31
December Year 7 it had a fair value of Rs. 2.3 million. By 31 December Year 8 its fair value had
risen to Rs. 2.7 million. This property has a useful life of 40 years.
Property 3 was acquired on 30 June Year 2 for Rs. 2 million for its investment potential. The directors
believe that the fair value of this property was Rs. 3 million on 31 December Year 7 and Rs. 3.5
million on 31 December Year 8. However, due to the specialised nature of this property, these figures
cannot be corroborated. This property has a useful life of 50 years.
Required:
For each of the above properties briefly state how it would be treated in the financial statements of
Victoria for the year ended 31 December Year 8, identifying any impact on profit or loss and produce
an analysis of for Victoria for the year ended 31 December Year 8, showing each of the above
properties separately.
Answer:
Treatment in the financial statements for the year ended 31 December Year 8
Property 1
This is used by Victoria as its head office and therefore cannot be treated as an investment property.
It will be stated at cost minus accumulated depreciation in the statement of financial position. The
depreciation for the year will be charged in the statement of profit or loss.
Property 2
This is held for its investment potential and should be treated as an investment property. It will be
carried at fair value, Victoria’s policy of choice for investment properties. It will be revalued to fair
value at each year end and any resultant gain or loss taken to the statement of profit or loss (Rs.
400,000 gain in Year 8).
Property 3
This is held for its investment potential and should be treated as an investment property. However,
since its fair value cannot be arrived at reliably it will be held at cost minus accumulated depreciation
in the statement of financial position. The depreciation for the year will be an expense in the
statement of profit or loss.
This situation provides the exception to the rule whereby all investment properties must be held
under either the fair value model, or the cost model.
Based on above analysis the property plant and equipment for the year ended 31 December year
8 is as under:
Other land and Investment Investment
buildings property held property held Total
(W1) at fair value at cost (W2)

Rs. Rs. Rs. Rs.


Cost/valuation
On 1 January Year 8 1,000,000 2,300,000 2,000,000 5,300,000
Revaluation - 400,000 - 400,000
---------------------- ---------------------- ---------------------- ----------------------
On 31 December Year 8 1,000,000 2,700,000 2,000,000 5,700,000
---------------------- ---------------------- ---------------------- ----------------------

© Emile Woolf International 435 The Institute of Chartered Accountants of Pakistan


Financial accounting and reporting I

Other land and Investment Investment


buildings property held property held Total
(W1) at fair value at cost (W2)

Rs. Rs. Rs. Rs.


Accumulated depreciation
On 1 January Year 8 87,500 - 220,000 307,500
Charge for the year (W1) 12,500 - 40,000 52,500
---------------------- ---------------------- ---------------------- ----------------------
On 31 December Year 8 100,000 - 260,000 360,000
---------------------- ---------------------- ---------------------- ----------------------
Carrying amount
On 31 December Year 7 912,500 2,300,000 1,780,000 4,992,500
---------------------- ---------------------- ---------------------- ----------------------
On 31 December Year 8 900,000 2,700,000 1,740,000 5,340,000
---------------------- ---------------------- ---------------------- ----------------------
Tutorial note
In practice, with a more complex property, plant and equipment table the investment properties
would be included within the land and buildings column with the required disclosures being given
separately in a note to the table.

Workings
(1) Depreciation on Property 1

Rs.
Brought forward (500,000 ÷ 40  7) 87,500
Year 8 (500,000 ÷ 40) 12,500

(2) Depreciation on Property 3

Rs.
Brought forward (2,000,000 ÷ 50  5.5) 220,000
Year 8 (2,000,000 ÷ 50) 40,000

Example 11: Distaghil Limited


Question: Following information pertains to non-current assets of Distaghil Limited (DL):
(i) DL purchased specialised vehicles for Rs. 370 million on 1 July 2017. The vehicles have
an estimated useful life of 10 years with residual value of Rs. 30 million.
The revalued amounts of the vehicle as at 31 December 2018 and 2019 were
determined at Rs. 302 million and Rs. 290 million respectively. There was no change in
useful life or residual value.
(ii) DL setup a manufacturing plant in a remote area at a cost of Rs. 280 million. The plant
had a useful life of 8 years. The plant was purchased on 1 January 2018 and was
available for use on 1 April 2018. The commercial production started on 1 June 2018.
On 1 July 2018, DL received a government grant of Rs. 120 million towards the cost of
the plant. The sanction letter states that if DL ceases to use the plant in the remote
area before 31 December 2021, DL would be required to repay the grant in full.

© Emile Woolf International 436 The Institute of Chartered Accountants of Pakistan


Chapter 8: Non-current assets: sundry standards

(iii) A warehouse was given on rent on 1 January 2018. Previously, the warehouse was in
use of DL.
On 1 January 2018, carrying value and remaining useful life of the warehouse was Rs.
80 million and 16 years respectively. Fair value of the warehouse on various dates are
as follows:

Rs. in million
01 January 2018 104
31 December 2018 96
31 December 2019 115
Other information:
▪ DL uses cost model for subsequent measurement of property, plant and equipment
except for specialised vehicles for which revaluation model is used.
▪ DL transfers the maximum possible amount from the revaluation surplus to retained
earnings on an annual basis.
▪ Government grant is recorded as deferred income and a part of it is transferred to income
each year.
▪ Investment property is carried at fair value model.
Required:
Prepare relevant extracts from DL’s statement of profit or loss and other comprehensive income
for the year ended 31 December 2019 and statement of financial position as on that date.
(Show comparative figures)

Answer:

Distaghil Limited
Extracts from statement of financial position as on 31 December 2019
2019 2018
---- Rs. in million ----
Non-current assets:
Property, plant and equipment:
▪ Vehicles (W-1) 290.00 302.00
▪ Plant (W-2) 218.75 253.75

Investment property 115.00 96.00

Share capital and reserves:


Revaluation surplus:
▪ Vehicles 5.00 -
▪ Warehouse[2019: 22.5–(24÷16)], 21.00 22.50
[2018: 24–(24÷16)]

Non-current liabilities:
Deferred government grant (W-2) 93.75 108.75

© Emile Woolf International 437 The Institute of Chartered Accountants of Pakistan


Financial accounting and reporting I

Extracts from statement of profit or loss and other comprehensive income


For the year ended 31 December 2019
2019 2018
------ Rs. in million ------
Profit or loss:
Depreciation:
▪ Vehicles (W-1) (32.00) (34.00)
▪ Plant (W-2) (35.00) (26.25)

Impairment / Revaluation loss – vehicles (W-1) - (17.00)


Revaluation gain - vehicles (W-1) 15.00
Grant income – Plant (W-2) 15.00 11.25
Change in fair value of investment property - warehouse
(2019: 115–96), (2018: 96–104) 19.00 (8.00)

Other comprehensive income:


Revaluation surplus:
▪ Vehicles (W-1) 5.00
▪ Warehouse (104–80) - 24.00

W-1: Vehicles Rs. in million


Purchased on 1 July 2017 370
Depreciation for 2017 (370–30)÷10×(6÷12) (17)
Book value on 31 December 2017 353
Depreciation for 2018 (370–30)÷10 (34)
Book value on 31 December 2018 before revaluation 319
Revaluation loss Balancing (17)
Revalued amount on 31 December 2018 302
Depreciation for 2019 (302–30)÷8.5 (32)
Book value on 31 December 2019 before revaluation 270
Revaluation gain to P&L 17×(7.5÷8.5) 15
Revaluation surplus Balancing 5
Revalued amount on 31 December 2019 290

W-2: Plant and government grant Plant Gov. grant


-------- Rs. in million --------
Initial balance 280.00 120.00
Depreciation for 2018 (280÷8)×(9÷12) (26.25)
Grant income for 2018 (120÷8)×(9÷12) (11.25)
Balance on 31 December 2018 253.75 108.75
Depreciation for 2019 280÷8 (35.00)
Grant income for 2019 120÷8 (15.00)
Balance on 31 December 2019 218.75 93.75
.

© Emile Woolf International 438 The Institute of Chartered Accountants of Pakistan


Chapter 8: Non-current assets: sundry standards

4 OBJECTIVE BASED QUESTIONS


01. On 1 January 2021 Aim Limited (AL) received Rs. 1,000,000 from the local government on the
condition that they employ at least 150 persons each year for the next 4 years.
Due to an economic downturn and reduced consumer demand on 1 January 2022, AL no longer
needed to employ any more staff and the conditions of the grant required full repayment.
What should be recorded in the financial statements on 1 January 2022?

(a) Reduce deferred income balance by Rs. 750,000


(b) Reduce deferred income by Rs. 750,000 and recognize a loss of Rs. 250,000
(c) Reduce deferred income by Rs. 1,000,000
(d) Reduce deferred income by Rs. 1,000,000 and recognize a gain of Rs. 250,000

02. Which of the following are acceptable methods of accounting for a government grant relating
to an asset in accordance with IAS 20 Accounting for Government Grants and Disclosure of
Government Assistance?
(i) Set up the grant as deferred income
(ii) Credit the amount received to profit or loss
(iii) Deduct the grant from the carrying amount of the asset
(iv) Add the grant to the carrying amount of the asset

(a) (i) and (ii)


(b) (ii) and (iv)
(c) (i) and (iii)
(d) (iii) and (iv)

03. On 1 January 2019, Boom Limited (BL) received Rs. 2,000,000 from the local government on
the condition that they employ at least 200 staff each year for the next 4 years. On this date, it
was virtually certain that BL would meet these requirements.
However, on 1 January 2022, due to an economic downturn and reduced consumer demand,
BL no longer needed to employ 100 staff. The conditions of the grant required half repayment.
What should be recorded in the financial statements on 1 January 2022 for repayment of grant?

(a) Debit Deferred grant by Rs. 500,000 and PL by Rs. 500,000


(b) Debit Deferred grant by Rs. 250,000 and PL by Rs. 250,000
(c) Debit Deferred grant by Rs. 1,500,000 and PL by Rs. 500,000
(d) Debit Deferred grant by Rs. 500,000 and PL by Rs. 1,500,000

04. Which TWO of the following statements about IAS 20 Accounting for Government Grants and
Disclosure of Government Assistance are true?
(a) A government grant related to the purchase of an asset must be deducted from the
carrying amount of the asset in the statement of financial position.
(b) A government grant related to the purchase of an asset should be recognised in profit
or loss over the life of the asset.

© Emile Woolf International 439 The Institute of Chartered Accountants of Pakistan


Financial accounting and reporting I

(c) Free marketing advice provided by a government department is excluded from the
definition of government grants.
(d) Any required repayment of a government grant received in an earlier reporting period is
treated as prior period adjustment.

05. Which TWO of the statements below regarding IAS 23 Borrowing Costs are correct?
(a) Borrowing costs must be capitalised if they are directly attributable to qualifying assets
(b) Borrowing costs should cease to be capitalised once the related asset is substantially
complete
(c) Borrowing costs must be capitalised if they are directly attributable to non-current assets
(d) Borrowing costs may be capitalised if they are directly attributable to qualifying assets

06. Fine Limited (FL) received a Rs.10 million loan at 7.5% on 1 April 2017. The loan was
specifically issued to finance the building of a new store.
Construction of the store commenced on 1 May 2017 and it was completed and ready for use
on 28 February 2018 but did not open for trading until 1 April 2018.
How much should be recorded as finance costs in the statement of profit or loss for the year
ended 31 March 2018?
(a) Rs. 250,000
(b) Rs. 750,000
(c) Rs. 125,000
(d) Rs. 625,000

07. Fine Limited (FL) received a Rs.10 million loan at 7.5% on 1 April 2017. The loan was
specifically issued to finance the building of a new store.
Construction of the store commenced on 1 May 2017 and it was completed and ready for use
on 28 February 2018 but did not open for trading until 1 April 2018.
How much interest should be capitalised as part of property, plant and equipment as at 31
March 2018?
(a) Rs. 250,000
(b) Rs. 750,000
(c) Rs. 125,000
(d) Rs. 625,000

08. An entity decided that not all of the funds raised were needed immediately and temporarily
invested some of the funds for one month before the construction started, earning Rs.40, 000
interest.
How should the Rs. 40,000 be accounted for in the financial statements?

(a) Net off the amount capitalised in property, plant and equipment
(b) Taken to the statement of profit or loss as investment income
(c) Taken as other comprehensive income
(d) Deducted from the outstanding loan amount in the statement of financial position

© Emile Woolf International 440 The Institute of Chartered Accountants of Pakistan


Chapter 8: Non-current assets: sundry standards

09. Shine Limited (SL) had the following bank loans outstanding during the whole of 2018:

Rs. m
9% loan repayable 2019 15
11% loan repayable 2022 24
SL began construction of a qualifying asset on 1 April 2018 and withdrew funds of Rs. 6 million
on that date to fund construction. On 1 August 2018 an additional Rs. 2 million was withdrawn
for the same purpose.
Calculate the borrowing costs which can be capitalised in respect of this project for the year
ended 31 December 2018.

(a) Rs. 545,600


(b) Rs. 472,350
(c) Rs. 750,600
(d) Rs. 350,350

10. Jazz Limited (JL) has borrowed Rs. 24 million to finance the building of a factory. Construction
is expected to take two years.
The loan was drawn down and incurred on 1 January 2019 and work began on 1 March 2019.
Rs. 10 million of the loan was not utilized until 1 July 2019 so JL was able to invest it until
needed. JL is paying 8% on the loan and can invest surplus funds at 6%.
Calculate the borrowing costs to be capitalised for the year ended 31 December 2019 in respect
of this project.

(a) Rs. 1,400,000


(b) Rs. 1,920,000
(c) Rs. 1,300,000
(d) Rs. 1,620,000

11. A company has the following loans in place throughout the year ended 31 December 2018.

Rs. m
10% bank loan 140
8% bank loan 200
On 1 July 2018 Rs. 50 million was drawn down for construction of a qualifying asset which was
completed during 2019.
What amount should be capitalised as borrowing costs at 31 December 2018 in respect of this
asset?

(a) Rs. 5.6 million


(b) Rs. 2.8 million
(c) Rs. 4.4 million
(d) Rs. 2.2 million

© Emile Woolf International 441 The Institute of Chartered Accountants of Pakistan


Financial accounting and reporting I

12. An entity purchased an investment property on 1 January 2013 for a cost of Rs. 35m. The
property had an estimated useful life of 50 years, with no residual value, and at 31 December
2015 had a fair value of Rs. 42m.
On 1 January 2016 the property was sold for net proceeds of Rs. 40m.
Calculate the profit or (loss) on disposal under both the cost and fair value (FV) model.

(a) Cost model: Rs. 7.1 m and FV model: (Rs. 2.0 m)


(b) Cost model: Rs. 2.0 m and FV model: Rs. 2.0 m
(c) Cost model: Rs. 5.0 m and FV model: (Rs. 2.0 m)
(d) Cost model: Rs. 7.1 m and FV model: Rs. 5.0 m

13. An investment property with a useful life of 10 years was purchased by Akram Limited on 1
January 2019 for Rs. 200 million. By 31 December 2019 the fair value of the property had risen
to Rs. 300 million. Akram Limited measures its investment properties under the fair value
model.
What values would go through the statement of profit or loss in the year?

(a) Gain: Rs. 100 million and Depreciation Rs. 30 million


(b) Gain: Rs. 0 and Depreciation of Rs. 30 million
(c) Gain: Rs. 100 million and Depreciation of 0
(d) Gain: Rs. 120 million and Depreciation of Rs. 20 million

14. Which of the following properties owned by an entity would be classified as an investment
property?
(a) A property that had been leased to a tenant, but which is no longer required and is now
being held for resale
(b) Land purchased for its investment potential. Planning permission has not been obtained
for building construction of any kind
(c) A new office building used as entity’s head office, purchased specifically in order to
exploit its capital gains potential
(d) A bungalow used for executive training

15. Sarfraz Limited (SL) uses fair value accounting where possible and has an office building used
by SL for administrative purposes. At 1 April 2012 it had a carrying amount of Rs. 20 million
and a remaining life of 20 years. On 1 October 2012, the property was let to a third party and
reclassified as an investment property. The property had a fair value of Rs. 23 million at 1
October 2012, and Rs. 23.4 million at 31 March 2013.
What is the correct treatment when the above property is reclassified as an investment
property?

(a) Take Rs. 3,500,000 gain to other comprehensive income


(b) Take Rs. 3,500,000 gain to the statement of profit or loss
(c) Take Rs. 4,000,000 gain to other comprehensive income
(d) Take Rs. 4,000,000 gain to the statement of profit or loss

© Emile Woolf International 442 The Institute of Chartered Accountants of Pakistan


Chapter 8: Non-current assets: sundry standards

16. A manufacturing entity receives a grant of Rs. 1,000,000 towards the purchase of a machine
on 1 January 2013. The grant will be repayable if the entity sells the asset within 4 years, which
it does not intend to do. The asset has a useful life of 5 years.
What is the deferred income liability balance at 30 June 2013?

Rs. ___________

17. A company receives a government grant of Rs. 500,000 on 1 April 2017 to facilitate purchase
on the same day of an asset which costs Rs. 750,000. The asset has a five-year useful life and
is depreciated on a 30% reducing balance basis. Company policy is to account for all grants
received as deferred income.
What amount of income will be recognized in respect of the grant in the year to 31 March 2019?

Rs. ___________

18. A manufacturing entity is entitled to a grant of Rs. 3 million for creating 50 jobs and maintaining
them for three years. Rs. 1.5m is received when the jobs are created and the remaining Rs.
1.5m is receivable after three years, provided that the 50 jobs are still in existence. The entity
creates 50 jobs at the beginning of year one and there is reasonable assurance that this level
of employment will be maintained.
What is the deferred income balance at the end of the first year?

Rs. ___________

19. An entity uses funds from its general borrowings to build a new production facility. Details of
the entity's borrowings are shown below:
Rs.10 million 6% loan
Rs.6 million 8% loan
The entity used Rs.12 million of these funds to construct the facility, which was under
construction for the entire year.
How much interest should be capitalised as part of the cost of the asset?

Rs. ___________

20. Cool Limited acquired a building with a 40-year life for its investment potential for Rs. 8 million
on 1 January 2013. At 31 December 2013, the fair value of the property was estimated at Rs.
9 million with costs to sell estimated at Rs. 200,000.
If Cool Limited uses the fair value model for investment properties, what gain should be
recorded in the statement of profit or loss for the year ended 31 December 2013?

Rs. ___________

21. If a government grant must be repaid, then it is;


(a) An error
(b) A change in accounting policy
(c) A change in accounting estimate
(d) A new transaction

© Emile Woolf International 443 The Institute of Chartered Accountants of Pakistan


Financial accounting and reporting I

22. If an entity receives a non-monetary asset as a grant, this is accounted for at the;
(a) Market value
(b) Fair value
(c) Net realizable value
(d) Present value

23. Which of the following is not covered by IAS 20 – Government Grants?


(a) Tax breaks
(b) Employment grants
(c) Subsidized loans
(d) Forgivable loans

24. Which of the following is not a correct treatment of government grants related to an asset?

(a) Deferred income


(b) Credit to income in period received
(c) Deducting the grant from the carrying amount of the asset
(d) None of the above

25. Which of the following is not a correct treatment of government grants related to income?
(a) Present as. Other income
(b) Deduct from the related expense
(c) Deduct from the cost of the asset
(d) None of the above

26. Which of the following is not considered a “borrowing cost” under IAS 23?
(a) Interest expense calculated by the effective interest method
(b) Finance charges in respect of loan
(c) Exchange differences arising from foreign currency borrowings to the extent that they
are regarded as an adjustment to interest costs
(d) Principal repayments on a loan for property, plant and equipment

27. When activities to prepare an asset for its sale or use are suspended, borrowing costs must
be?
(a) Capitalized
(b) Expensed
(c) Ignored
(d) Charged to equity

© Emile Woolf International 444 The Institute of Chartered Accountants of Pakistan


Chapter 8: Non-current assets: sundry standards

28. Which of the following is not a condition to commence capitalisation of borrowing costs?
(a) Expenditures are being incurred
(b) Borrowing costs are being incurred
(c) Repayment of borrowings has commenced
(d) Activities to produce the asset for its intended use or sale have commenced

29. Ghazi Limited (GL) is constructing an office building and is capitalising borrowing costs in
accordance with IAS 23. The office is almost complete; the only remaining work is to install
furniture. Is GL allowed to continue capitalising the borrowing costs?
(a) Yes
(b) No
(c) Don’t know
(d) None of the above

30. Which of the following is not a “qualifying asset” under IAS 23?
(a) Mass produced inventory
(b) Manufacturing plants
(c) Made to order inventory
(d) Investment property

31. Under IAS 40 – Investment Property, where should a gain or loss on disposal be recognized?
(a) Statement of Financial Position
(b) Profit and loss statement
(c) Statement of changes in equity
(d) None

32. If an entity uses part of a building for their own use, and rents the remainder. How should this
be treated?
(a) All as investment property under IAS 40 – Investment Property
(b) All under IAS 16 – Property, Plant and Equipment
(c) Account for separately under ‘IAS - 16 Property, Plant and Equipment’ and ‘IAS - 40
Investment Property’
(d) None of these

33. An investment property should initially be measured at?


(a) Cost
(b) Fair value
(c) Market value
(d) Net realizable value

© Emile Woolf International 445 The Institute of Chartered Accountants of Pakistan


Financial accounting and reporting I

34. If an entity wishes to change from a cost model to fair value model under IAS 40 – Investment
Property, when may it do so?
(a) When the board of directors approves a change
(b) When the value of the assets will improve with a revised model
(c) When a change will result in a more appropriate presentation
(d) When the market for these properties is fluctuation

35. Which two of the following properties fall under the definition of investment property and
therefore within the scope of IAS 40?
(a) Property occupied by an employee paying market rent
(b) A building owned by an entity and leased out under an operating lease
(c) Property being constructed on behalf of 3rd parties
(d) Land held for long term appreciation

36. Which of the following can NOT be a ‘qualifying asset’ under IAS 23 Borrowing Costs?
(a) Inventories
(b) Manufacturing plants
(c) Assets that are ready for their intended use when acquired
(d) Investment property

37. Afternoon Limited (AL) uses cost model for its property, plant and equipment and fair value
model for its investment property. AL has an office building which was being used for
administrative purposes. At 1 July 2018, the building had a carrying amount of Rs. 20 million.
On that date, the building was let out to a third party and therefore reclassified as an investment
property. The building had a fair value of Rs. 23 million on 1 July 2018 and Rs. 23.4 million on
30 June 2019.

What would be the increase in the profit or loss and other comprehensive income for the year
ended 30 June 2019?
Profit or loss Other comprehensive income
(a) Nil Rs. 3.4 million
(b) Rs. 0.4 million Rs. 3 million
(c) Rs. 3.4 million Nil
(d) Rs. 3 million Rs. 0.4 million

38. Which TWO of the following fall under the definition of investment property?
(a) Property occupied by an employee
(b) A building owned by an entity and leased out under an operating lease
(c) Property being constructed on behalf of third party
(d) Land held for long term appreciation

© Emile Woolf International 446 The Institute of Chartered Accountants of Pakistan


Chapter 8: Non-current assets: sundry standards

39. Capitalisation of borrowing costs should be suspended:


(a) when substantially all the activities necessary to prepare a qualifying asset for its
intended use or sale are complete
(b) during a temporary delay which is a necessary part of the process of getting an asset
ready for its intended use or sale
(c) during extended periods in which active development of a qualifying asset is interrupted
(d) all of the above

40. Under IAS 40 ‘Investment property’, which of the following disclosures is NOT required to be
made under cost model?
(a) Fair value of the property
(b) Depreciation method
(c) Reconciliation of carrying amounts at the beginning and end of a period
(d) Residual value of the property

41. Which of the following statements is correct in the context of capitalisation of borrowing costs?
(a) If funds have been arranged from various general borrowings, the amount to be
capitalised is based on the weighted average cost of borrowings
(b) Capitalisation always commences as soon as expenditure for the asset is incurred
(c) Capitalisation always continues until the asset is brought into use
(d) Capitalisation always commences as soon as borrowing costs are incurred

© Emile Woolf International 447 The Institute of Chartered Accountants of Pakistan


Financial accounting and reporting I

4 OBJECTIVE BASED ANSWERS


01. (b) This is a grant related to income and would therefore be released to the
statement of profit or loss over the 4 year life. By the end of year one, Rs.
250,000 would have been credited to the statement of profit or loss, leaving
Rs. 750,000 held in deferred income. At this point the amount is repaid,
meaning that the deferred income is removed, as well as the Rs. 250,000
income previously recorded.

02. (c) The grant can be treated as deferred income or deducted from the carrying
amount of the asset. It cannot be credited directly to profit or loss.

03. (a) Half repayment is Rs. 1,000,000 (Rs. 2,000,000 x 50%)


At the date of repayment, the balance in deferred grant would be Rs. 500,000
and additional Rs. 500,000 shall be charged to profit or loss.

04. (b, c) Item a is incorrect as the deferred income method can be used.
Item d is incorrect as any repayment is corrected in the current period, not
retrospectively.

05. (a, b) Borrowing costs must be capitalised if they are directly attributable to
qualifying assets, which are assets that take a substantial time to complete.
Capitalization should cease once substantially all the activities to prepare the
asset are complete.

06. (c) Rs. 10 million x 7.5% x 2/12 = Rs. 125,000

07. (d) Rs. 10 million x 7.5% x 10/12 = Rs. 125,000

08. (b) Temporary investment income earned during the construction period should
be netted off the amount capitalised.
However, the interest was earned prior to the period of construction. Therefore
the investment income earned should be taken to the statement of profit or
loss as investment income.

09. (a) (9% × 15𝑚) + (11% × 24𝑚)


𝐶𝑎𝑝𝑖𝑡𝑎𝑙𝑖𝑠𝑎𝑡𝑖𝑜𝑛 𝑟𝑎𝑡𝑒 = = 10.23%
15 + 24
Rs. 6m × 10.23% × 9/12 = Rs.460,350
Rs. 2m × 10.23% × 5/12 = Rs. 85,250
Total Rs. 545,600

10. (a) Rs.

March – December (Rs. 24m × 8% × 10/ 12) 1,600,000

Less investment income (Rs. 10m × 6% × 4/12) (200,000)

1,400,000

Temporary investment income before commencement would be recognized


as finance income in profit or loss.

© Emile Woolf International 448 The Institute of Chartered Accountants of Pakistan


Chapter 8: Non-current assets: sundry standards

11. (d) (10% × 140𝑚) + (8% × 200𝑚)


𝐶𝑎𝑝𝑖𝑡𝑎𝑙𝑖𝑠𝑎𝑡𝑖𝑜𝑛 𝑟𝑎𝑡𝑒 = = 8.8%
140 + 200
Rs. 50 million × 8.8% × 6/12 = Rs. 2.2 million

12. (a) Under the cost model the property will be depreciated over 50 years for 3
years up to the date of disposal. Therefore, at the disposal date the carrying
value would have been Rs. 35m – (Rs. 35m/50 × 3 years) = Rs. 32.9m and
the profit on disposal Rs. 7.1m (Rs. 40m – Rs. 32.9).
Under the fair value model the property will not be depreciated hence the loss
on disposal would be Rs. 2m (Rs. 40m – Rs. 42m).

13. (c) Under the fair value model the property will not be depreciated hence the gain
on valuation would be Rs. 100 million (Rs. 300 million – Rs. 200 million).

14. (b) Asset A would be classed as a non-current asset held for sale under IFRS 5.
Assets C and D would both be classified as property, plant and equipment
under IAS 16.

15. (a) As SL uses the fair value model for investment properties, the asset should
be revalued to fair value before being classed as an investment property. The
gain on revaluation should be taken to other comprehensive income, as the
asset is being revalued while held as property, plant and equipment.
At 1 October, the carrying amount of the asset is Rs. 19.5 million, being Rs.
20 million less 6 months’ depreciation. As the fair value at 1 October is Rs. 23
million, this leads to a Rs. 3,500,000 gain which will be recorded in other
comprehensive income.

16. Rs. 900,000 The grant should be released over the useful life, not based on the possibility
of the item being repaid. Therefore, the Rs. 1m should be released over 5
years, being a release of Rs. 200,000 a year. At 30 June 2013, 6 months
should be released, meaning Rs. 100,000 has been released (6/12 × Rs.
200,000). This leaves Rs. 900,000 in deferred income.

17. Rs. 105,000 Rs.

Grant received 1.4.17 500,000

Recognized year to 31.3.18 (500,000 × 30%) (150,000)

Balance 31.3.18 350,000

Recognized year to 31.3.19 (350,000 × 30%) 105,000

18. Rs. 500,000 The total grant income is Rs. 3m, to be recognized over a three-year period.
Annual income is therefore Rs. 1m. At the end of the first year the entity has
received Rs. 1.5m of which Rs. 1m has been recognized in the statement of
profit or loss, leaving Rs. 500,000 deferred into future periods.

19. Rs. 810,000 Rs.12m × 6.75% = Rs. 810,000


Capitalisation rate
= ((Rs.10m × 6%) + (Rs.6m × 8%))/Rs.16m = 6.75%.

© Emile Woolf International 449 The Institute of Chartered Accountants of Pakistan


Financial accounting and reporting I

20. Rs. The fair value gain of Rs. 1 million (Rs. 9m – Rs. 8m) should be taken to the
1,000,000 statement of profit or loss. Costs to sell are ignored and, since entity uses the
fair value model, no depreciation will be charged on the building.

21. (c)

22. (b)

23. (a)

24. (b)

25. (c)

26. (d)

27. (b)

28. (c)

29. (b)

30. (a)

31. (b)

32. (c)

33. (a)

34. (c)

35. (b) & (d)

36. (c) Assets that are ready for their intended use when acquired

37. (b) Profit or loss Rs. 0.4 million and Other comprehensive income Rs. 3 million

38. (b) & (d) A building owned by an entity and leased out under an operating lease & Land
held for long term appreciation

39. (c) during extended periods in which active development of a qualifying asset is
interrupted

40. (d) Residual value of the property

41. (a) If funds have been arranged from various general borrowings, the amount to
be capitalised is based on the weighted average cost of borrowings

© Emile Woolf International 450 The Institute of Chartered Accountants of Pakistan


Certificate in Accounting and Finance

9
Financial accounting and reporting I

CHAPTER
IAS 36: Impairment of assets

Contents
1 Impairment of assets

2 Objective based questions and answers

* The student must refer original handbook of IFRS.

© Emile Woolf International 451 The Institute of Chartered Accountants of Pakistan


Financial accounting and reporting I

1 IMPAIRMENT OF ASSETS

Section overview

 Objective and scope of IAS 36


 Definitions
 Stages in accounting for an impairment loss
 Indications of impairment
 Measuring recoverable amount
 Recognition & Accounting for impairment
 Summary of the approach

1.1 Objective and scope of IAS 36


An asset is said to be impaired when its recoverable amount is less than its carrying amount in the
statement of financial position. From time to time an asset may have a carrying value that is greater
than its fair value but this is not necessarily impairment as the situation might change in the future
as the reporting date approaches.
The objective of IAS 36 Impairment of assets is to ensure that assets are ‘carried’ (valued) in the
financial statements at no more than their recoverable amount.
Scope of IAS 36
IAS 36 applies to accounting for impairment of all assets except the following:
 inventories (IAS 2: Inventories);
 Assets arising from contracts with customers that are recognised in accordance with IFRS
15: Revenue from contracts with customers.
 investment property that is measured at fair value (IAS 40):
The requirements of IAS 36 also do not apply to IFRSs like IAS 12, IAS 27, IAS 28, IAS 38, IAS
41, IFRS 4, IFRS 5, IFRS 10, IFRS 11, which are out of syllabus at this level.

1.2 Definitions

Definitions
The recoverable amount of an asset is defined as the higher of its fair value minus costs of
disposal, and its value in use.
Fair value is the price that would be received to sell an asset or paid to transfer a liability in an
orderly transaction between market participants at the measurement date.
Value in use is the present value of future cash flows from using an asset, including its eventual
disposal.
Impairment loss is the amount by which the carrying amount of an asset (or a cash-generating
unit) exceeds its recoverable amount.

1.3 Stages in accounting for an impairment loss


There are various stages in accounting for an impairment loss:
Stage 1: Establish whether there is an indication of impairment.
Stage 2: If so, assess the recoverable amount.

© Emile Woolf International 452 The Institute of Chartered Accountants of Pakistan


Chapter 9: IAS 36: Impairment of assets

Stage 3: Write down the affected asset (by the amount of the impairment) to its recoverable
amount.
Each of these stages will be considered in turn.

1.4 Indications of impairment


An entity must carry out an impairment review when there is evidence or an indication that
impairment may have occurred. At the end of each reporting period, an entity should assess
whether there is any indication that impairment might have occurred. If such an indication exists,
the entity must estimate the recoverable amount of the asset, in order to establish whether
impairment has occurred and if so, the amount of the impairment.
The following are given by IAS 36 as possible indicators of impairment. These may be indicators
outside the entity itself (external indicators), such as market factors and changes in the market.
Alternatively, they may be internal indicators relating to the actual condition of the asset or the
conditions of the entity’s business operations.
When assessing whether there is an indication of impairment, IAS 36 requires that, at a minimum,
the following sources are considered:

External sources Internal sources

An unexpected decline in the asset’s market value. Evidence that the asset is damaged
or no longer of use to the entity.

Significant changes in technology, markets, economic There are plans to discontinue or


factors or laws and regulations that have an adverse restructure the operation for which
effect on the company. the asset is currently used.

An increase in interest rates, affecting the value in use of There is a reduction in the asset’s
the asset. expected remaining useful life.

The company’s net assets have a higher carrying value There is evidence that the entity’s
than the company’s market capitalisation (which expected performance is worse
suggests that the assets are over-valued in the statement than expected.
of financial position).

Internal indicators for impairment are generally refers to items under control of management while
external indicators are outside the control of management.
If there is an indication that an asset is impaired then it is tested for impairment. This involves the
calculating the recoverable amount of the item in question and comparing this to its carrying
amount.

1.5 Measuring recoverable amount


It has been explained that recoverable amount is the higher of an asset’s:
 fair value less costs of disposal; and
 its value in use.
If either of these amounts is higher than the carrying value of the asset, there has been no
impairment.
IAS 36 sets out the requirements for measuring ‘fair value less costs of disposal’ and ‘value in use’.

© Emile Woolf International 453 The Institute of Chartered Accountants of Pakistan


Financial accounting and reporting I

Measuring fair value less costs of disposal


Fair value of an asset at a particular date is normally its current market value. If no active market
exists, it may be possible to estimate the amount that the entity could obtain from the disposal.
Direct selling costs normally include legal costs, taxes and costs necessary to bring the asset into
a condition to be sold. However, redundancy and similar costs (for example, where a business is
reorganized following the disposal of an asset) are not direct selling costs.
Calculating value in use
Value in use represents the present value of the expected future cash flows from use of the asset,
discounted at a suitable discount rate or cost of capital.
The following elements should be reflected in the calculation of an asset’s value in use:
 An estimate of the future cash flows the entity expects to derive from the asset
 Expectations about possible variations in the amount or timing of those future cash flows
 The time value of money (represented by the current market risk-free rate of interest)
 The price for bearing the uncertainty inherent in the asset
 Other factors that market participants would reflect in pricing the future cash flows the entity
expects to derive from the asset.
Estimates of future cash flows should be based on reasonable and supportable assumptions that
represent management’s best estimate of the economic conditions that will exist over the
remaining useful life of the asset.
Estimates of future cash flows must include:
 cash inflows from the continuing use of the asset;
 cash outflows that will be necessarily incurred to generate the cash inflows from continuing
use of the asset; and
 net disposal proceeds at the end of the asset’s useful life.
Estimates of future cash flows must not include:
 cash inflows or outflows from financing activities; or
 income tax receipts or payments.
Also note that future cash flows are estimated for the asset in its current condition. Therefore,
any estimate of future cash flows should not include estimated future cash flows that are
expected to arise from:
 a future restructuring to which an entity is not yet committed; or
 improving or enhancing the asset’s performance.
The discount rate must be a pre-tax rate that reflects current market assessments of:
 the time value of money; and
 the risks specific to the asset for which the future cash flow estimates have not been
adjusted.
However, both the expected future cash flows and the discount rate might be adjusted to allow for
uncertainty about the future – such as the business risk associated with the asset and expectations
of possible variations in the amount or timing of expected future cash benefits from using the asset.

© Emile Woolf International 454 The Institute of Chartered Accountants of Pakistan


Chapter 9: IAS 36: Impairment of assets

Illustration 01: Measurement of recoverable amount


A company has a machine in its statement of financial position at a carrying amount of
Rs.300,000.
The machine is used to manufacture the company’s best-selling product range, but the entry of a
new competitor to the market has severely affected sales.
As a result, the company believes that the future sales of the product over the next three years will
be only Rs.150,000, Rs.100,000 and Rs.50,000. The asset will then be sold for Rs.25, 000.
An offer has been received to buy the machine immediately for Rs.240,000, but the company
would have to pay shipping costs of Rs.5, 000.The risk-free market rate of interest is 10%.
Market changes indicate that the asset may be impaired and so the recoverable amount for the
asset must be calculated.
Fair value less costs of disposal Rs.
Fair value 240,000
Costs of disposal (5,000)
235,000
Year Cash flow (Rs.000) Discount factor Present value
1 150,000 1/1.1 136,364
2 100,000 1/1.12 82,645
3 50,000 + 25,000 1/1.13 56,349
Value in use 275,358
The recoverable amount is the higher of Rs.235, 000 and Rs.275, 358, i.e. Rs.275, 358.
The asset must be valued at the lower of carrying value and recoverable amount.
The asset has a carrying value of Rs.300, 000, which is higher than the recoverable amount from
using the asset.
It must therefore be written down to the recoverable amount, and an impairment of Rs.24, 642
(Rs.300, 000 – Rs.275, 358) must be recognized.

1.6 Recognition & Accounting for impairment


The impairment loss is normally recognized immediately in profit or loss.

Illustration 02: Measurement of recoverable amount


A company has a machine in its statement of financial position at a carrying amount of
Rs.300,000.
The machine has been tested for impairment and found to have recoverable amount of Rs.275,358
meaning that the company must recognize an impairment loss of Rs.24,642.
This is accounted for as follows:

Debit Credit

Statement of profit or loss 24,642

Accumulated impairment loss 24,642

(Property, plant and equipment would be


presented net of the balance on this account
on the face of the statement of financial
position).

© Emile Woolf International 455 The Institute of Chartered Accountants of Pakistan


Financial accounting and reporting I

Illustration 03:
On 1 January Year 1 Entity Q purchased for Rs.240,000 a machine with an estimated useful life of
20 years and an estimated zero residual value.
Depreciation is on a straight-line basis.
On 1 January Year 4 an impairment review showed the machine’s recoverable amount to be
Rs.100,000 and its remaining useful life to be 10 years.
a) The carrying amount of the machine on 31 December Year 3 (immediately before the
impairment).
The carrying amount of machinery on 31st December year three immediately before the
impairment is calculated as under;

Carrying amount of the machine on 31 December Year 3 Rs.


Cost 240,000
Accumulated depreciation (3 × (240,000 ÷ 20 years)) (36,000)
Carrying amount 204,000

b) The impairment loss recognized in the year to 31 December Year 4 is calculated as under:

Impairment loss at the beginning of Year 4 of Rs.104,000 (Rs.204,000 – Rs.100,000).


This is charged to profit or loss.

c) The depreciation charge in the year to 31 December Year 4.is calculated as under:

Depreciation charge in Year 4 of Rs.10,000 (= Rs.100,000 ÷ 10). The depreciation charge


is based on the recoverable amount of the asset.

Assets having revaluation surplus


However, an impairment loss recognized in respect of an asset carried at a previously recognized
revaluation surplus is recognized in other comprehensive income to the extent that it is covered by
that surplus. Thus it is treated in the same way as a downward revaluation, reducing the revaluation
reserve balance relating to that asset.
Impairment not covered by a previously recognized surplus on the same asset is recognized in
profit or loss.

Illustration 04: Measurement of recoverable amount


A company has a machine in its statement of financial position at a carrying amount of Rs.300,
000 including a previously recognized surplus of Rs.20,000.
The machine has been tested for impairment and found to have recoverable amount of
Rs.275,358 meaning that the company must recognize an impairment loss of Rs.24,642.

This is accounted for as follows:

Debit Credit
Statement of profit or loss 4,642
Other comprehensive income 20,000
Property, plant and equipment 24,642

© Emile Woolf International 456 The Institute of Chartered Accountants of Pakistan


Chapter 9: IAS 36: Impairment of assets

Depreciation of impaired assets


After the recognition of the impairment, the future depreciation of the asset must be based on the
revised carrying amount, minus the residual value, over the remaining useful life.

Illustration 05:
On 1 January Year 1 Entity Q purchased for Rs.240, 000 a machine with an estimated useful life
of 20 years and an estimated zero residual value.
Depreciation is on a straight-line basis.
The asset had been re-valued on 1 January Year 3 to Rs.250, 000, but with no change in useful
life at that date.
On 1 January Year 4 an impairment review showed the machine’s recoverable amount to be
Rs.100, 000 and its remaining useful life to be 10 years.
a) The carrying amount of the machine on 31 December Year 2 and hence the revaluation
surplus arising on 1 January Year 3 is calculated as under:

Carrying amount on Rs.


Cost 240,000
Accumulated depreciation at 1 January Year 3 (2 years × (240,000 ÷ 20)) (24,000)
Carrying amount 216,000
Valuation at 1 January Year 3 250,000
Revaluation surplus 34,000

b) The carrying amount of the machine on 31 December Year 3 (immediately before the
impairment) is calculated as under:
When the asset is revalued on 1 January Year 3, depreciation is charged on the revalued
amount over its remaining expected useful life.
On 31 December Year 3 the machine was therefore stated at:

Rs.
Valuation at 1 January (re-valued amount) 250,000
Accumulated depreciation in Year 3 (= Rs.250,000 ÷ 18)) (13,889)
Carrying amount 236,111

c) The impairment loss recognised in the year to 31 December Year 4 is calculated as under:
On 1 January Year 4 the impairment review shows an impairment loss of Rs.136,111
(Rs.236,111 – Rs.100,000).

An impairment loss of Rs.32,111 (Rs.34,000 Rs.1,889) will be taken to other


comprehensive income (reducing the revaluation surplus for the asset to zero).

The remaining impairment loss of Rs.104,000 (Rs.136,111 Rs.34,000) is recognised in the


statement of profit or loss for Year 4.
d) The depreciation charge in the year to 31 December Year 4 is calculated as under:
Year 4 depreciation charge is Rs.10,000 (Rs.100,000 ÷ 10 years).

© Emile Woolf International 457 The Institute of Chartered Accountants of Pakistan


Financial accounting and reporting I

1.7 Summary of the approach


Impairment of an asset should be identified and accounted for as follows:
(1) At the end of each reporting period, the entity should assess whether there are any
indications that an asset may be impaired.
(2) If there are such indications, the entity should estimate the asset’s recoverable amount.
(3) When the recoverable amount is less than the carrying value of the asset, the entity should
reduce the asset’s carrying value to its recoverable amount. The amount by which the value
of the asset is written down is an impairment loss.
(4) This impairment loss is recognized as a loss for the period.
(5) However, if the impairment loss relates to an asset that has previously been re-valued
upwards, it is first offset against any remaining revaluation surplus for that asset. When this
happens it is reported as other comprehensive income for the period (a negative value) and
not charged against profit.
(6) Depreciation charges for the impaired asset in future periods should be adjusted to allocate
the asset’s revised carrying amount, minus any residual value, over its remaining useful life
(revised if necessary).

EXAMPLE 01: ABA LIMITED


Question: Aba Limited conducts its activities from two properties, a head office in the city centre
and a property in the countryside where staff training is conducted. Both properties were acquired
on 1 April 2013 and had estimated lives of 25 years with no residual value.
The company has a policy of carrying its land and buildings at current values. However, until
recently property prices had not changed for some years.
On 1 October 2015 the properties were revalued by a firm of surveyors. Details of this and the
original costs are:

Land Buildings
Rs. Rs.
Head office – cost 1 April 2013 500,000 1,200,000
– revalued 1 October 2015 700,000 1,350,000
Training premises – cost 1 April 2013 300,000 900,000
– revalued 1 October 2015 350,000 600,000

The fall in the value of the training premises is due mainly to damage done by the use of heavy
equipment during training. The surveyors have also reported that the expected life of the training
property in its current use will only be a further 10 years from the date of valuation. The estimated
life of the head office remained unaltered.
Note: Aba Limited treats its land and its buildings as separate assets. Depreciation is based on
the straight-line method from the date of purchase or subsequent revaluation.

Required:
Prepare extracts of the financial statements of Aba Limited in respect of the above properties for
the year to 31 March 2016.??

© Emile Woolf International 458 The Institute of Chartered Accountants of Pakistan


Chapter 9: IAS 36: Impairment of assets

Answer:
Aba Limited statement of profit or loss (extracts) – year to 31 March 2016
Note: workings in brackets are in Rs.000 Rs. Rs.
Depreciation: head office – 6 months to 1 October 2015
(1,200/25  6/12) 24,000
– 6 months to 31 March 2016
(1,350/22.5 (W1)  6/12) 30,000
54,000
Depreciation: training premises
– 6 months to 1 October 2016
(900/25  6/12) 18,000
– 6 months to 31 March 2016
(600/10  6/12) 30,000
48,000
Impairment loss (W2) 210,000
258,000
Statement of financial position (extracts) as at
31 March 2016 Rs. Rs.
Non-current assets
Land and buildings – head office (700 + 1,350 – 30) 2,020,000
– training premises (350 + 600 – 30) 920,000
2,940,000
Revaluation reserve
Head office land (700 – 500) 200,000
Building (1,350 – 1,080 (W1)) 270,000
Training premises land (350 – 300) 50,000
520,000
Transfer to realised profit (270/22.5 (W1)  6/12
re depreciation of buildings) (6,000)
514,000

Workings

(W1) The date of the revaluation is two and a half years after acquisition. This means the
remaining life of the head office would be 22.5 years. The carrying value of the head office
building at the date of revaluation is Rs. 1,080,000 i.e. its cost less two and a half years at
Rs. 48,000 per annum (Rs. 1,200,000 – Rs. 120,000).

(W2) Impairment loss: the carrying value of training premises at date of revaluation is Rs. 810,000
i.e. its cost less two and a half years at Rs. 36,000 per annum (Rs. 900,000 – Rs. 90,000).
It is revalued down to Rs. 600,000 giving a loss of Rs. 210,000. As the land and the buildings
are treated as separate assets the gain on the land cannot be used to offset the loss on the
buildings.

© Emile Woolf International 459 The Institute of Chartered Accountants of Pakistan


Financial accounting and reporting I

EXAMPLE 02: HUSSAIN ASSOCIATES LTD


Question: The assistant financial controller of the Hussain Associates Ltd group has identified the
matters below which she believes may indicate impairment of one or more assets:
Hussain Associates Ltd owns and operates an item of plant that cost Rs. 640,000 and had
accumulated depreciation of Rs. 400,000 at 1 October 2015. It is being depreciated at 12½% on
cost.
On 1 April 2016 (exactly half way through the year) the plant was damaged when a factory vehicle
collided into it. Due to the unavailability of replacement parts, it is not possible to repair the plant,
but it still operates, albeit at a reduced capacity. It is also expected that as a result of the damage
the remaining life of the plant from the date of the damage will be only two years.
Based on its reduced capacity, the estimated present value of the plant in use is Rs. 150,000. The
plant has a current disposal value of Rs. 20,000 (which will be nil in two years’ time), but Hussain
Associates Ltd has been offered a trade-in value of Rs. 180,000 against a replacement machine
which has a cost of Rs. 1 million (there would be no disposal costs for the replaced plant). Hussain
Associates Ltd is reluctant to replace the plant as it is worried about the long-term demand for the
product produced by the plant. The trade-in value is only available if the plant is replaced.
Required:
Prepare extracts from the statement of financial position and statement of profit or loss of Hussain
Associates Ltd in respect of the plant for the year ended 30 September 2016. Your answer should
explain how you arrived at your figures.

Answer:
 The plant had a carrying amount of Rs. 240,000 on 1 October 2015. The accident may have caused
impairment occurred on 1 April 2016. However, as per IFRS 36, the entity will do an impairment
test at the end of the reporting period, i.e, 30 September 2016.
 The depreciation on the plant from 1 October 2015 to 30 Sept 2016 would be Rs. 80,000 (640,000
x 12.5% giving a carrying amount of Rs. 160,000 at the date of impairment. An impairment test
requires the plant’s carrying amount to be compared with its recoverable amount. The recoverable
amount of the plant is the higher of its value in use of Rs. 150,000 or its fair value less costs to sell.
 If Hussain Associates Ltd trades in the plant it would receive Rs. 180,000 by way of a part exchange,
but this is conditional on buying new plant which Hussain Associates Ltd. is reluctant to do. A more
realistic amount of the fair value of the plant is its current disposal value of only Rs. 20,000.
Thus the recoverable amount would be its value in use of Rs. 150,000 giving an impairment loss of
Rs. 10,000 (Rs. 160,000 – Rs. 150,000). Thus extracts from the financial statements for the year
ended 30 September 2016 would be:

Statement of financial position


Non-current assets Rs.
Plant 150,000
Statement of profit or loss
Plant depreciation 80,000
Plant impairment loss 10,000

© Emile Woolf International 460 The Institute of Chartered Accountants of Pakistan


Chapter 9: IAS 36: Impairment of assets

EXAMPLE 03: SKY-LINE LTD (SL)


Question: Sky-Line Limited (SL) operates a 4 Star Hotel facility in Muree. The hotel was
constructed at a cost of Rs.300 million, 5 years back and it is depreciated on a straight-line basis
(total useful life of 15 years and residual value of 20%). There are indications that the property is
not performing as expected due to;

(a) opening of a competing hotel nearby,


(b) a significant drop in number of tourists to the area because of terrorism.
There is a 40% probability that the hotel will generate net cash flows of Rs.40 million per annum
and 60% probability that the cash flows would only be Rs.20 million per annum.
The property’s net operating income is Rs.30 million which is at the rate of 15%. 5% of the
proceeds from sale would be expended in closing the deal.
Required:
Calculate the impairment loss if the appropriate discount rate is 10%.

Answer:

Carrying value of asset = Rs.300 million – [5 x (300 – 60) / 15 years]


= Rs.220 million
Recoverable amount is the higher of fair value less cost to sell and value is use
Fair value less cost to sell = Net operating income / capitalization rate (since no active market) x
(1 - disposal process)
= Rs.30 m / 15% =200 million
= Rs.200 million – 200 million (5%)=190 million
Value in use = Present value of net cash flows discounted at 10% for 10 years
= (Rs.40 m x 0.4 + Rs.20 m x 0.6) x [1- (1.10)-10]/ 10%]
= Rs.28 m x 6.145
= Rs.172 m
Recoverable amount = Rs.190 million
Impairment loss = Rs.30 million

© Emile Woolf International 461 The Institute of Chartered Accountants of Pakistan


Financial accounting and reporting I

EXAMPLE 04: PREMIER LIMITED (PL)


Question: Premier Limited (PL) owns a plant which has a carrying amount of Rs.248 million as
at 1 April 2019. It is being depreciated at 12½% per annum on a reducing balance basis.
The plant is used to manufacture a specific product which has been suffering a decline in sales
due to obsolescence.
PL has estimated that the plant will be retired from use on 31 March 2023.
The estimated net cash flows from the use of the plant and their present values are:

Net cash Present


flows values

Rs. In million

Year to 31 March 2020 120 109.2

Year to 31 March 2021 80 66.4

Year to 31 March 2022 52 39

252 214.6

On 1 April 2020, PL had an alternative offer from the competitor to purchase the plant for
Rs.200 million.
Required:
??

Answer:
At 31 March 2020

Recoverable amount is the higher of value in use [PV of future net cash flows (Rs.214.6 million)
and fair value less costs of disposal (Rs.200 million)].

Carrying amount = Rs.217 million [248 m – (248 m x 12·5%)]

Impairment loss = Carrying amount – Recoverable amount

= Rs.217 million – Rs.214.6 million

= Rs.2.4 million

© Emile Woolf International 462 The Institute of Chartered Accountants of Pakistan


Chapter 9: IAS 36: Impairment of assets

EXAMPLE 05: NAVEED LIMITED


Question: Naveed Limited has an item of plant which has a carrying value of Rs.1,800,000 as at
the end of the year December 2020. It has undergone an impairment review and the following
estimates were produced:
Fair value of plant = Rs.1,400,000
Costs to sell 2% of selling price
Revenue and associated costs per annum for remaining useful life:
(assume all cash flows occur at the end of the year).
Revenue Costs
2021 Rs.960,000 Rs.240,000
2022 Rs.880,000 Rs.220,000
2023 Rs.700,000 Rs.290,000
The plant has an estimated residual value of Rs.50,000.
A discount rate of 10% is applicable to investments equivalent in risk to this plant.
Required:
Calculate the impairment loss if the appropriate discount rate is 10%.

Answer:
Cash flows 2021 2022 2023
--------------- Amount in Rs. --------------
Revenue 960,000 880,000 700,000
Costs (240,000) (220,000) (290,000)
Net Cash Inflow 720,000 660,000 410,000
Discount factor 0.909 0.826 0.751
Present Value 654,545 545,457 308,037

Residual Value (50,000 x 0.751) = 37,566


Value in use = 654,545 + 545,457 + 308,037 + 37,566 = Rs.1,545,605
Fair value less cost to sell = Rs.1,400,000 – 2% of Rs.1.4 million = Rs.1,372,000
Recoverable amount = Rs. 1,545,605
Carrying value = Rs.1,800,000
Impairment loss = Rs.254,395

Example 06: Indus Pharma Limited (IPL)


Question: On 1 July 2014, Indus Pharma Limited (IPL) received a government grant of Rs. 280
million to setup a plant in an under-developed rural area. The grant is repayable in full if the
conditions attached to the grant are not met for a period of five years from the date of
commencement of the production. At the inception, it was highly probable that IPL would comply
with the conditions for the required period.

© Emile Woolf International 463 The Institute of Chartered Accountants of Pakistan


Financial accounting and reporting I

IPL incurred total cost of Rs. 630 million on plant and it started production on1 January 2015.
Useful life of the plant was estimated at 7 years. IPL deducted government grant in arriving at the
carrying amount of the asset.
In January 2019, IPL showed its inability to comply with the conditions attached to the grant and
regulatory authority issued a notice to IPL for repayment of the grant in full. Accordingly, the grant
was repaid by IPL.
In view of repayment of the grant, IPL carried out an impairment review of the plant on 31 December
2019. Net annual cash inflows for the remaining life of the plant have been estimated at Rs. 90
million and Rs. 80 million for 2020 and 2021 respectively. These cash inflows are net of annual
interest and maintenance cost of Rs. 10 million and Rs. 6 million respectively for both years.
Applicable discount rate is 12%.
On the date of impairment review, the existing plant can be sold in the local market for Rs. 160
million. Estimated cost of disposal would be Rs. 5 million.
Required:
Prepare journal entries for the year ended 31 December 2019 in respect of the above information.
(Show all necessary workings. Narrations are not required)

Answer:
Indus Pharma Limited
General Journal
Debit Credit
Date Description
Rs. in million
Jan. 2019 Plant 280
Cash/Bank 280

Jan. 2019/ Depreciation expense/Profit or loss 280÷7×4 160


31-12-2019 Accumulated depreciation - Plant 160

31-12-2019 Depreciation expense 630÷7 90


Accumulated depreciation - Plant 90

31-12-2019 Impairment loss (W-1) 19


Accumulated impairment - Plant 19

W-1: Impairment review as on 31 December 2019 Rs. in million


Year Net inflows Discounting at 12% Present value
2020 (90+10) 100 0.8929 89
2021 (80+10) 90 0.7972 72
Value in use 161

Fair value less cost to sell 160–5 155

Recoverable amount (Higher of both) 161

WDV of the plant as on 31 December 2019 630÷7×2 180


Impairment loss 180–161 19
.

© Emile Woolf International 464 The Institute of Chartered Accountants of Pakistan


Chapter 9: IAS 36: Impairment of assets

Example 07: Harappa Industries


Question: Following information pertain to property, plant and equipment of Harappa
Industries Limited (HIL) for the year ended 30 June 2020:
(i) Balance as on 30 June 2019
Assets Cost/revalued Accumulated Revaluation Depreciation Useful
amount depreciation surplus method life/rate

----------- Rs. in '000 -----------


Land* 100,000 - - - Infinite
Buildings 70,000 14,000 16,000 Straight line 20 years
Plant 180,000 60,000 - Straight line 15 years
Vehicles 8,800 4,000 - Reducing balance 20%

*An amount of Rs. 12 million had been charged to profit or loss upon previous revaluation

(ii) On 30 June 2020, the revalued amounts of the land and buildings were assessed by
Smart Consultant at Rs. 120 million and Rs. 35 million respectively.
(iii) Setting up of a new plant was commenced on 1 July 2019 and substantially completed
on 29 February 2020. The plant was available for use on 1 April 2020 and immediately
put into use. Useful life of the plant was estimated at 10 years. Details of the cost
incurred are as under:
Description Payment date Rs. in '000
1st payment 1 August 2019 12,000

2nd payment 1 October 2019 48,000

3rd payment 29 February 2020 48,000

4th payment 31 July 2020 12,000

120,000

The cost of the plant was financed through an existing running finance facility with a
limit of Rs. 200 million carrying mark-up of 12% per annum. A government grant of Rs.
20 million related to the plant was received on 1 January 2020. The grant amount was
used for repayment of the running facility.
(iv) One of the vehicles had an engine failure on 1 January 2020 and its engine had to be
sold as scrap for Rs. 0.1 million. The vehicle had been acquired on 1 January 2018 at
a cost of Rs. 2.5 million. 40% of the cost is attributable to its engine. Though the engine
of similar capacity was available at a cost of Rs. 1.2 million, the old engine was replaced
on 1 January 2020 with a higher capacity engine at a cost of Rs. 1.8 million.
(v) HIL uses cost model for subsequent measurement of property, plant and equipment
except for land and buildings.
(vi) HIL accounts for revaluation on net replacement value method and transfers the
maximum possible amount from revaluation surplus to retained earnings on an annual
basis.
(vii) HIL deducts government grant in arriving at the carrying amount of the asset.

Required:
In accordance with IFRSs, prepare a note on ‘Property, plant and equipment’ for inclusion in
HIL’s financial statements for the year ended 30 June 2020.(Comparatives figures and column
for total are not required).

© Emile Woolf International 465 The Institute of Chartered Accountants of Pakistan


Financial accounting and reporting I

Answer:
Harappa Industries Limited
Notes to the financial statements for the year ended 30 June 2020

1 Property, plant and equipment:


Land Buildings Plant Vehicles
------------------ Rs. in '000 ------------------
Gross carrying amount - opening 100,000 70,000 180,000 8, 800
Accumulated depreciation - (14,000) (60,000) (4,000)
Opening carrying amount 100,000 56,000 120,000 4,800
(W-
Additions - - 1)102,840 1,800
Depreciation for the year - (3,500) (14,571) (1,068)
(70,000÷20) (W-2) (W-4)
Disposals - - - (W-3) (648)
Revaluation
- Surplus (Bal.) 8,000 (W-5) (15,000) - -
- P&L 12,000 (Bal.) (2,500) - -
Closing carrying amount 120,000 35,000 208,269 4,884

Gross carrying amount - closing 120,000 35,000 282,840 9,600


Accumulated depreciation - - (74,571) (4,716)
Closing carrying amount 120,000 35,000 208,269 4,884

1.1 Land Buildings Plant Vehicles


Cost Cost
Measurement base Revaluation Revaluation model model
15/10
Useful life /depreciation rate Infinite 15 Years years 20%
Straight Straight
Depreciation method - Reducing bal.
line line

1.2 The last revaluation was performed on 30 June 2020 by Smart Consultants, an independent
firm of valuers.

1.3 Had revaluations not made, the carrying value of the land and buildings as on 30 June 2020
would have been Rs. 112 million (100+12) and Rs. 37.5 million (35,000+2,500) respectively.

W-1: Cost - Plant Rs. in '000


Cost 120,000
Government grant (20,000)
Capitalisation of borrowing cost:
1 August - 1 October 2019 12,000×12%×2÷12 240
1 October - 31 December 2019 60,000×12%×3÷12 1,800
1 January - 29 February 2020 (60,000–20,000)×12%×2÷12 800
2,840
102,840

© Emile Woolf International 466 The Institute of Chartered Accountants of Pakistan


Chapter 9: IAS 36: Impairment of assets

W-2: Depreciation – Plant Rs. in '000


On opening balance 180,000÷15 12,000
On the new plant 102,840(W-1)÷10×3÷12 2,571
14,571

W-3: Written down value - Engine disposed off Rs. in '000


Cost 2,500×40% 1,000
Accumulated depreciation:
For the six months ended 30 June 2018 1,000×20%×6÷12 100
For 2018-2019 (1,000–100)×20% 180
(1,000–
For the six months ended 31 December 2019 280)×20%×6÷12 72
352
648

W-4: Depreciation – Vehicles Rs. in '000


On disposal of old engine (W-3) 72
On remaining opening balance [(8,800–1,000)–(4,000–280)]×20% 816
On addition of new engine 1,800×20%×6÷12 180
1,068

W-5: Revaluation surplus – Buildings Rs. in '000


Opening balance 16,000
Incremental depreciation 16,000÷16[(56,000÷70,000)×20] (1,000)
15,000
.

© Emile Woolf International 467 The Institute of Chartered Accountants of Pakistan


Financial accounting and reporting I

2 OBJECTIVE BASED QUESTIONS


01. If the fair value less costs to sell cannot be determined

(a) The asset is not impaired.

(b) The recoverable amount is the value-in-use.

(c) The net realizable value is used.

(d) The carrying value of the asset remains the same.

02. Which TWO of the following could be an indication that an asset may be impaired according
to IAS 36 Impairment of Assets?

(a) Decrease in market interest rates

(b) Increase in market values for the asset

(c) Damage caused to the asset

(d) Management intention to reorganise the business

03. IAS 36 Impairment of Assets contains a number of examples of internal and external events
which may indicate the impairment of an asset.
In accordance with IAS 36, which of the following would definitely NOT be an indicator of the
potential impairment of an asset (or group of assets)?

(a) An unexpected fall in the market value of one or more assets


(b) Adverse changes in the economic performance of one or more assets
(c) A significant change in the technological environment in which an asset is employed
making its software effectively obsolete
(d) The carrying amount of an entity’s net assets being below the entity’s market
capitalisation

04. A fire at the factory on 1 October 2016 damaged the machine, leaving it with a lower
operating capacity. The accountant considers that entity will need to recognise an impairment
loss in relation to this damage. The accountant has ascertained the following information at 1
October 2016:
 The carrying amount of the machine is Rs.60,750.
 An equivalent new machine would cost Rs.90,000.
 The machine could be sold in its current condition for a gross amount of Rs.45,000.
Dismantling costs would amount to Rs.2,000.
 In its current condition, the machine could operate for three more years which gives it
a value in use figure of Rs.38,685.
What is the total impairment loss associated with the above machine at 1 October 2016?

(a) Rs.nil

(b) Rs.17,750

(c) Rs.22,065

(d) Rs.15,750

© Emile Woolf International 468 The Institute of Chartered Accountants of Pakistan


Chapter 9: IAS 36: Impairment of assets

05. Which of the following is NOT an indicator of impairment?


(a) Advances in the technological environment in which an asset is employed have an
adverse impact on its future use.
(b) An increase in interest rates which increases the discount rate an entity uses.
(c) The carrying amount of an entity’s net assets is higher than the entity’s number of
shares in issue multiplied by its share price.
(d) The estimated net realisable value of inventory has been reduced due to fire damage
although this value is greater than its carrying amount.

06. Cost of disposal are

(a) Incremental costs, directly attributable to the disposal of an asset, excluding finance
costs and income tax expense

(b) Incremental costs, directly attributable to the disposal of an asset, plus finance costs,
but excluding income tax expense

(c) Incremental costs, directly attributable to the disposal of an asset, plus finance costs
and income tax expense

(d) Incremental costs, directly attributable to the disposal of an asset, plus tax expense,
but excluding finance costs

07. An asset is impaired if:

(a) Its carrying amount equals the amount to be recovered through use (or sale) of the
asset

(b) Its carrying amount exceeds the amount to be recovered through use (or sale) of the
asset

(c) The amount to be recovered through use (or sale) of the asset exceeds its carrying
amount

(d) If it has been damaged

08. Value in use is:

(a) The market value

(b) The discounted present value of future cash flows arising from use of the asset and
from its disposal.

(c) The higher of an asset’s fair value less cost to sell and its market value.

(d) The amount at which an asset is recognized in the statement of financial position.

09. IAS 36 applied to which of the following assets:


(a) Inventories.
(b) Financial assets including property plant and equipment and intangible assets
(c) Assets held for sale.
(d) Property, plant, and equipment and intangible assets

© Emile Woolf International 469 The Institute of Chartered Accountants of Pakistan


Financial accounting and reporting I

10. In accordance with IAS 36 Impairment of Assets which of the following statements are true?
1. An impairment review must be carried out annually on all intangible assets.
2. If the fair value less costs to sell of an asset exceed the carrying amount there is no
need to calculate a value in use.
3. Impairment is charged to the statement of profit or loss unless it reverses a gain that
has been recognised in equity in which case it is offset against the revaluation
surplus.
(a) All three

(b) 1 and 2 only

(c) 1 and 3 only

(d) 2 & 3 only

11. What is the recoverable amount of an asset?

(a) Its current market value less costs of disposal

(b) The lower of carrying amount and value in use

(c) The higher of fair value less costs of disposal and value in use

(d) The higher of carrying amount and market value

12. A machine has a carrying amount of Rs. 850,000 at the year end of 31 March 2019. Its market
value is Rs. 780,000 and costs of disposal are estimated at Rs. 25,000. A new machine would
cost Rs. 1,500,000. The company which owns the machine expects it to produce net cash flows
of Rs. 300,000 per annum for the next three years. The company has a cost of capital of 8%.

What is the impairment loss on the machine to be recognised in the financial statements at 31
March 2019?

(a) Rs. 76,870

(b) Rs. 95,000

(c) Rs. 1,66,700

(d) Rs. 220,000

13. IAS 36 Impairment of Assets suggests how indications of impairment might be recognised.
Which TWO of the following would be external indicators that one or more of an entity's
assets may be impaired?

(a) An unusually significant fall in the market value of one or more assets

(b) Evidence of obsolescence of one or more assets

(c) A decline in the economic performance of one or more assets

(d) An increase in market interest rates used to calculate value in use of the assets

© Emile Woolf International 470 The Institute of Chartered Accountants of Pakistan


Chapter 9: IAS 36: Impairment of assets

14. The following information relates to an item of plant.


 Its carrying amount in the statement of the financial position is Rs. 3 million.
 The company has received an offer of Rs. 2.7 million from a company in Karachi
interested in buying the plant.
 The present value of the estimated cash flows from continued use of the plant is Rs.
2.6 million.
 The estimated cost of transport the plant to Karachi is Rs. 50,000.
What is the amount of the impairment loss that should be recognised on the plant?

(a) Rs. 300,000

(b) Rs. 400,000

(c) Rs. 350,000

(d) Rs. 250,000

15. When calculating the estimates of the future cash flows, which of the following cash flows
should not be included?

(a) Cash flows from disposal.

(b) Income tax payments.

(c) Cash flows from the sale of assets produced by the asset.

(d) Cash outflows on the maintenance of the asset.

16. The following information relates to three assets held by a company:


Asset A Asset B Asset C
Rs. m Rs. m Rs. m
Carrying amount 200 100 80
Value in use 160 120 70
Fair value less cost to sell 180 130 60

What is the total impairment loss?

Rs. ___________

17. The following information relates to four assets held by the company:
A B C D
Rs.m Rs.m Rs.m Rs.m
Carrying amount 240 60 80 140
Value in use 160 140 160 40
Fair value less costs to sell 180 80 140 60

What is the total impairment loss?

Rs. ___________

© Emile Woolf International 471 The Institute of Chartered Accountants of Pakistan


Financial accounting and reporting I

18. A vehicle was involved in an accident exactly halfway through the year. The vehicle cost Rs.
10 million and had a remaining life of 10 years at the start of the year. Following the accident,
the expected present value of cash flows associated with the vehicle was Rs. 3.4 million and
the fair value less costs to sell was Rs. 6.5 million.
What is the recoverable amount of the vehicle following the accident?

Rs. ___________

19. Radium Limited (RL) acquired a non-current asset on 1 October 2019 at a cost of Rs. 100
million which had a useful life of ten years and a nil residual value. The asset had been correctly
depreciated up to 30 September 2024.
At that date the asset was damaged and an impairment review was performed. On 30
September 2024, the fair value of the asset less costs to sell was Rs. 30 million and the
expected future cash flows were Rs. 8.5 million per annum for the next five years.
The current cost of capital is 10% and a five year annuity of Rs. 1 per annum at 10% would
have a present value of Rs. 3.79.
What amount would be charged to profit or loss for the impairment of this asset for the year
ended 30 September 2024?

Rs. ___________

20. Metal Limited (ML) owns an item of plant which has a carrying amount of Rs. 248 million as at
1 April 2013. It is being depreciated at 12.5% per annum on a reducing balance basis.
The plant is used to manufacture a specific product which has been suffering a slow decline in
sales. ML has estimated that the plant will be retired from use on 31 March 2017.
The estimated net cash flows from the use of the plant and their present values are:

Net cash flows Present


values

Rs.000 Rs.000

Year to 31 March 2015 120,000 109,200

Year to 31 March 2016 80,000 66,400

Year to 31 March 2017 52,000 39,000

252,000 214,600

On 1 April 2014, Metric had an offer from a rival to purchase the plant for Rs. 200 million
At what value should the plant appear in Metric’s statement of financial position as at 31
March 2014?

Rs. ___________

21. Which of the following is covered by IAS 36 – Impairment?

(a) Non-current assets held for sale

(b) Investment property carried at cost

(c) Investment property carried at fair value

(d) Inventories

© Emile Woolf International 472 The Institute of Chartered Accountants of Pakistan


Chapter 9: IAS 36: Impairment of assets

22. Which of the following is not covered by IAS 36 – Impairment?


(a) Goodwill
(b) Investment property carried at cost
(c) Investment property carried at fair value
(d) Intangible assets

23. When should an impairment loss be recognised?


(a) Immediately
(b) Over a number of accounting periods
(c) At management’s discretion
(d) When requested by the entity’s auditors

24. Value in use is?


(a) The undiscounted present value of future cash flows expected to arise from continuing
use of asset, and from its disposal at the end of its useful life.
(b) The undiscounted future value of present cash flows expected to arise from continuing
use of asset, and from its disposal at the end of its useful life.
(c) The discounted present value of future cash flows expected to arise from continuing
use of asset, and from its disposal at the end of its useful life.
(d) The discounted present value of historical cash flows expected to arise from
continuing use of asset, and from its disposal at the end of its useful life.

25. Which of the following element is not considered while computing value in use?
(a) expectations about possible variations in the amount or timing of those future cash
flows
(b) the time value of money, represented by the current market risk-free rate of interest
(c) the price for bearing the uncertainty inherent in the asset
(d) estimated future restructuring cost

26. In measuring value in use, the discount rate used for discounting the cash flows should be
the?
(a) Pre-tax rate that reflects the market assessment of time value of money and risks
specific to the asset
(b) Pre-tax rate that reflects the market assessment of time value of money and risks
specific to the entity’s competitors
(c) Post-tax rate that reflects the entity’s assessment of time value of money and risks
specific to the asset
(d) Pre-tax rate that reflects the entity’s assessment of time value of money and risks
specific to the asset

© Emile Woolf International 473 The Institute of Chartered Accountants of Pakistan


Financial accounting and reporting I

27. When the recoverable amount of an asset is less than its carrying value in the Statement of
Financial Position, the asset is?

(a) in a revaluation deficit

(b) Flawed

(c) In negative equity

(d) Impaired

28. Which of the following is an internal indication of impairment?

(a) Decline in market value

(b) Worse economic performance than expected

(c) Increase in market interest rates

(d) Technological obsolescence

29. Which of the following is an external indication of impairment?

(a) Physical damage

(b) Worse economic performance than expected

(c) Increase in market interest rates

(d) Asset is part of a restructuring program

30. Under IAS 36, what is the recoverable amount of an asset?

(a) The lower of its cost and net realisable value

(b) The higher of fair value less costs of disposal and value in use

(c) The lower of net present value and cost

(d) The higher of net present value and cost

31. Which of the following is not permitted as a cost to sell under IAS 36?

(a) Cost to dismantle machine

(b) Auctioneers fees

(c) Standard wages for employees

(d) Transport costs for machine

© Emile Woolf International 474 The Institute of Chartered Accountants of Pakistan


Chapter 9: IAS 36: Impairment of assets

32. If the fair value less costs to sell for an asset cannot be determined, then recoverable amount
is its?

(a) Market value

(b) Fair value

(c) Value in use

(d) Replacement value

33. Which of the following is the best evidence of an asset's fair value less costs to sell?

(a) The carrying value of the asset

(b) The price in a binding sale agreement

(c) The disposal value of the asset in an arm`s length transaction

(d) An asset that is traded in an active market

34. When calculating the estimates of future cash flows which of the following cash flows should
not be included?

(a) Cash out flows on the maintenance of the asset

(b) Cash flows from disposal

(c) Cash flows from the sale of inventory produced by the asset

(d) Benefits from future restructuring

35. Under IAS 36 Impairment of Assets, if the fair value less costs to sell of an asset cannot be
determined then:

(a) the asset is not impaired

(b) the recoverable amount is the value in use

(c) the net realizable value is used

(d) the carrying value of the asset remains the same

36. Which TWO of the following would be external indicators that one or more of an entity's assets
may be impaired?

(a) An unusually significant fall in the market value of one or more assets

(b) Evidence of obsolescence of one or more assets

(c) A decline in the economic performance of one or more assets

(d) An increase in market interest rates used to calculate value in use of the assets

© Emile Woolf International 475 The Institute of Chartered Accountants of Pakistan


Financial accounting and reporting I

37. Which of the following future cash flows should NOT be included in the calculation of value in
use of an asset?

(a) Cash flows from disposal

(b) Income tax payments

(c) Cash flows from the sale of inventory produced by the asset

(d) Cash outflows on the maintenance of the asset

38. A plant has a carrying amount of Rs. 1,500,000 as at 31 December 2019. Its fair value is Rs.
900,000 and costs of disposal are estimated at Rs. 50,000. A new plant would cost Rs.
2,500,000. Cash flows from the plant for the next four years are estimated at Rs. 350,000 per
annum. Applicable discount rate is 10%.
What is the approximate impairment loss on the plant to be recognised in the financial
statements as at 31 December 2019?

(a) Rs. 650,000

(b) Rs. 390,000

(c) Rs. 1,000,000

(d) Nil

39. In measuring value in use, the discount rate used for discounting the cash flows should be the:

(a) pre-tax rate that reflects the market assessment of time value of money and risks
specific to the asset

(b) pre-tax rate that reflects the market assessment of time value of money and risks
specific to the entity

(c) post-tax rate that reflects the entity’s assessment of time value of money and risks
specific to the asset

(d) pre-tax rate that reflects the entity’s assessment of time value of money and risks
specific to the asset

© Emile Woolf International 476 The Institute of Chartered Accountants of Pakistan


Chapter 9: IAS 36: Impairment of assets

2 OBJECTIVE BASED ANSWERS


01. (b) The recoverable amount is higher of value in use and fair value less cost
to sell and in case fair value cannot be measured reliably, the recoverable
amount is value in use.

02. (c) & (d) A decrease in interest rates would reduce the discount applied to future
cash flows in calculating the value in use, therefore increasing the value
in use. An increase in market values will lead to the asset value
increasing rather than being impaired.

03. (d) The entity’s market capitalisation would not be reflected within the values
on the statement of financial position.

04. (b) Value in use of Rs.38,685 is lower than fair value less costs to sell of
Rs.43,000, so recoverable amount is Rs.43,000 and impairment is
Rs.60,750 – Rs.43,000 = Rs.17,750.

05. (d) Although the estimated net realisable value is lower than it was (due to
fire damage), the entity will still make a profit on the inventory and thus it
is not an indicator of impairment.

06. (a) Tax and finance costs are not cost of disposal.

07. (b) Asset may not be impaired even after damage. Impairment loss is excess
of carrying amount over recoverable amount.

08. (b) This is definition of value in use

09. (d) (a), (b) and (c) are excluded from scope of IAS 36 as the prudence
mechanism is already incorporated in the relevant standards of these
items.

10. (d) Item 1 is untrue. An annual impairment review is only required for
intangible assets with an indefinite life.

11. (c) The higher of fair value less costs of disposal and value in use.

12. (a)

Fair value – costs of disposal


(780,000 – 25,000) Rs. 75,500

Value in use:

300,000 × 1 / 1.08 277,780

300,000 × 1 / 1.082 257,200

300,000 × 1 / 1.083 238,150

Rs. 773,130

Recoverable amount is Rs. 773,130 and carrying amount is Rs. 850,000,


so impairment is Rs. 76,870.

© Emile Woolf International 477 The Institute of Chartered Accountants of Pakistan


Financial accounting and reporting I

13. (a & d) The other options are internal indicators of impairment.

14. (c)
Rs.

Fair value less costs of disposal (2.7m – 50,000) 2,650,000

Value in use 2,600,000

Recoverable amount is therefore: 2,650,000

Impairment loss (balancing figure) 350,000

Carrying amount 3,000,000

15. (b) Cash flows related to taxations are ignored while calculating value in use.

16. Rs. 30 million 20 + Nil + 10 = Rs. 30 million

17. Rs. 140 million 60 + Nil + Nil +80 = Rs. 140 million

18. Rs. 6.5 million The recoverable amount of an asset is the higher of its value in use
(being the present value of future cash flows) and fair value less costs to
sell. Therefore the recoverable amount is Rs. 6.5 million.

19. Rs. 17.785


million Rs. m

Cost 1 October 2019 100

Depreciation (100 /10 x 5 years) (50)

Carrying amount 50

The recoverable amount is the higher of fair value less costs to sell (Rs.
30 million) and the value in use (Rs. 8.,5 x 3.79 = Rs. 32.215).
Recoverable amount is therefore Rs. 32.215.

Rs. m

Carrying amount 50

Recoverable amount (32.215)

Impairment to statement of profit or loss 17.785

20. Rs. Is the lower of its carrying amount (Rs. 217 million) and recoverable
214,600,000 amount (Rs. 214.6 million) at 31 March 2015.
Recoverable amount is the higher of value in use (Rs. 214.6 million) and
fair value less costs to (Rs. 200 million).
Carrying amount = Rs. 217 million (248 million – (248 million × 12.5%))
Value in use is based on present values = Rs. 214.6 million

21. (b)

© Emile Woolf International 478 The Institute of Chartered Accountants of Pakistan


Chapter 9: IAS 36: Impairment of assets

22. (c)

23. (a)

24. (c)

25. (d)

26. (a)

27. (d)

28. (b)

29. (c)

30. (b)

31. (c)

32. (c)

33. (b)

34. (d)

35. (b) the recoverable amount is the value in use

36. (a) & (d) An unusually significant fall in the market value of one or more assets & An
increase in market interest rates used to calculate value in use of the
assets

37. (b) Income tax payments

38. (b) Rs. 390,000

39. (a) pre-tax rate that reflects the market assessment of time value of money
and risks specific to the asset

© Emile Woolf International 479 The Institute of Chartered Accountants of Pakistan


Financial accounting and reporting I

© Emile Woolf International 480 The Institute of Chartered Accountants of Pakistan


Certificate in Accounting and Finance

10
Financial accounting and reporting I

CHAPTER
IFRS 15: Revenue from contracts
with customers

Contents
1 IFRS 15: Revenue from contracts with customers
2 IFRS 15: The five step model
3 Other aspects of IFRS 15
4 Examinable Examples of IFRS 15
5 Objective based questions and answers

* The student must refer original handbook of IFRS.

w
© Emile Woolf International 481 The Institute of Chartered Accountants of Pakistan
Financial accounting and reporting I

1 IFRS 15: REVENUE FROM CONTRACTS WITH CUSTOMERS

Section overview

 Introduction
 Core principle and the five step model

1.1 Introduction
The IASB issued IFRS 15: Revenue from contracts with customers in May 2014.
IFRS 15 is the end product of a major joint project between the IASB and the US Financial
Accounting Standards Board and replaces IAS 18, IAS 11, IFRIC 13, IFRIC 15, IFRIC 18 and SIC
31.
IFRS 15 will have an impact on all entities that enter into contracts with customers with few
exceptions. Entities will need to reassess their revenue recognition policies and may need to revise
them. The timing and amount of revenue recognised may not change for simple contracts for a
single deliverable but will change for more complex arrangements involving more than one
deliverable.
This standard is effective for annual accounting periods beginning on or after 1 January 2017 but
earlier application is allowed.
Summary
IFRS 15:
 establishes a new control-based revenue recognition model;
 changes the basis for deciding whether revenue is recognised at a point in time or over time;
 provides new and more detailed guidance on specific topics; and
 Improves disclosures about revenue.

1.2 Core principle and the five step model


IFRS 15 is based on a core principle that requires an entity to recognise revenue:
 in a manner that depicts the transfer of goods or services to customers
 At an amount that reflects the consideration the entity expects to be entitled to in exchange
for those goods or services.
Applying this core principle involves following a five step model as follows:
 Step 1: Identify the contract(s) with the customer
 Step 2: Identify the separate performance obligations
 Step 3: Determine the transaction price
 Step 4: Allocate the transaction price
 Step 5: Recognise revenue when or as an entity satisfies performance obligations

Definitions
Revenue is income arising in the course of an entity’s ordinary activities.
A customer is a party that has contracted with an entity to obtain goods or services that are an
output of the entity’s ordinary activities.

© Emile Woolf International 482 The Institute of Chartered Accountants of Pakistan


Chapter 10: IFRS 15: Revenue from contracts with customers

2 IFRS 15: THE FIVE STEP MODEL

Section overview

 Step 1: Identify the contract(s) with a customer


 Step 2: Identify the separate performance obligations in the contract
 Step 3: Determine the transaction price
 Step 4: Allocate the transaction price to the performance obligations
 Step 5: Recognise revenue when or as an entity satisfies performance obligations

2.1 Step 1: Identify the contract(s) with a customer


The first step in IFRS 15 is to identify the contract. This may be written, oral, or implied by an
entity’s customary business practices.

Definition
A contract is an agreement between two or more parties that creates enforceable rights and
obligations.

The general IFRS 15 model applies only when or if:


 the parties have approved the contract;
 the entity can identify each party’s rights;
 the entity can identify the payment terms for the goods and services to be transferred; and
 the contract has commercial substance (i.e. the risk, timing or amount of the entity’s future
cash flows is expected to change as a result of the contract); and
 It is probable the entity will collect the consideration.
If a customer contract does not meet these criteria, revenue is recognised only when either:
 the entity’s performance is complete and substantially all of the consideration in the
arrangement has been collected and is non-refundable; or
 The contract has been terminated and the consideration received is non-refundable.

Illustration 01: Mr. Owais agreed on March 1, 2017 to sell 5 cutting machines to Axiom Enterprises.
Due to some deficiency in drafting the agreement each party’s rights cannot be identified. On March
31, 2017 Mr. Owais delivered the goods and these were accepted by Axiom Enterprises. After 10
days of delivery i.e. April 10, 2017 Axiom Enterprises made the full payment and the payment is
non-refundable.
Mr. Owais cannot identify each party’s rights so revenue recognition should be delayed until the
entity’s (Owais) performance is complete and substantially all of the consideration (cash) in the
arrangement has been collected and is non-refundable.
Therefore, Mr. Owais should record the revenue on April 10, 2017, as it is the date on which
performance is complete and non-refundable payment is received.

A contract does not exist if each party has an enforceable right to terminate a wholly
unperformed contract without compensating the other party.

Illustration 02:
A shopkeeper agreed to deliver 10 computers to Waqas Enterprises within 3 months. As per the
agreement shopkeeper can cancel the contract any time before delivering the computers. In case
of cancellation, shopkeeper is not required to pay any penalty to Waqas Enterprises.

© Emile Woolf International 483 The Institute of Chartered Accountants of Pakistan


Financial accounting and reporting I

A contract does not exist if each party (either buyer or seller) has an enforceable right to
terminate a wholly unperformed contract without compensating the other party.
As shopkeeper can cancel contract without compensating Waqas Enterprises so contract does not
exist.

Combination of contracts
An entity must combine two or more contracts entered into at or near the same time with the same
customer (or related parties of the customer) and treat them as a single contract if one or more of
the following conditions are present:
 the contracts are negotiated as a package with a single commercial objective;
 the amount of consideration to be paid in one contract depends on the price or performance
of the other contract; or
 The goods or services promised in the contracts (or some goods or services promised in the
contracts) are a single performance obligation.

Illustration 03:
Adil Ltd. enters into 2 separate agreements with customer X.
1. Agreement 1: Deliver 10,000 bricks for Rs. 100,000
2. Agreement 2: Build a boundary wall for Rs. 20,000
The two agreements should be combined and considered as a one agreement because contracts
are negotiated with a single commercial objective of building a wall. The price of two agreements
is interdependent. Adil Ltd. is probably charging high price for bricks to compensate for the
discounted price for building the wall.

2.2 Step 2: Identify the separate performance obligations in the contract


Performance obligations are normally specified in the contract but could also include promises
implied by an entity’s customary business practices, published policies or specific statements that
create a valid customer expectation that goods or services will be transferred under the contract.

Definition
A performance obligation is a promise in a contract with a customer to transfer to the customer
either:
a. a good or service (or a bundle of goods or services) that is distinct; or
b. A series of distinct goods or services that are substantially the same and that have the
same pattern of transfer to the customer.

At the inception of a contract the entity must assess the goods or services promised in a contract
with a customer and must identify as a performance obligation each promise to transfer to the
customer either:
 a good or service (or a bundle of goods or services) that is distinct; or
 a series of distinct goods or services that are substantially the same and that have the same
pattern of transfer to the customer (described by reference to promises satisfied over time,
and progress to completion assessment)

Illustration 04:
Pico Ltd. (PL) sells 10 washing machines for Rs. 20,000 each to a Retailer Co. (RC). PL also provides
the following free of cost:
• Free service and maintenance for 3 years
• 10 kg of washing powder every month for the next 18 months
• A discount voucher for a 50% discount if next purchase is made in the next 6 months.

© Emile Woolf International 484 The Institute of Chartered Accountants of Pakistan


Chapter 10: IFRS 15: Revenue from contracts with customers

There are 4 separate performance obligations as all of the goods and services are distinct because
the RC can benefit from the good and service on its own and the PC’s promise to transfer the good
or service is separately identifiable from other promises in the contract:
Following are the separate performance obligations:
1. Delivery of washing machines (point in time)
2. Service and maintenance over 3 years (over time)
3. 10 kg washing powder over the next 18 months (over time)
4. Discount voucher (point in time)

A good or service is distinct if both of the following criteria are met:


 the customer can benefit from the good or service either on its own or together with other
resources that are readily available to the customer; and
 The entity’s promise to transfer the good or service is separately identifiable from other
promises in the contract.
If a good or service is regularly sold separately, this would indicate that customers generally can
benefit from the good/service on its own or in conjunction with other available resources.
If a promised good or service is not distinct, an entity must combine that good or service with other
promised goods or services until it identifies a bundle of goods or services that is distinct. In some
cases, this would result in the entity accounting for all the goods or services promised in a contract
as a single performance obligation

Illustration 05: Promised goods and services


Goods produced by an entity for sale
Resale of goods purchased by an entity
Resale of rights to goods or services purchased by an entity
Performing a contractually agreed-upon task for a customer
Standing ready to provide goods or services
Providing a service of arranging for another party to transfer goods or services to the customer
Granting rights to goods or services to be provided in the future that a customer can resell
Constructing, manufacturing or developing an asset on behalf of a customer
Granting licences
Granting options to purchase additional goods/services

When (or as) a performance obligation is satisfied, an entity will recognise as revenue the amount
of the transaction price (excluding estimates of variable consideration that are constrained)
allocated to that performance obligation (step 5))
There are two issues to address:
 The amount of the transaction price, including any constraints (step 3))
 The allocation of that price to POs (step 4)

2.3 Step 3: Determine the transaction price

Definition
The transaction price is the amount of consideration an entity expects to be entitled to in exchange
for the goods or services promised under a contract, excluding any amounts collected on behalf of
third parties (for example, sales taxes).

© Emile Woolf International 485 The Institute of Chartered Accountants of Pakistan


Financial accounting and reporting I

An entity must consider the terms of the contract and its customary practices in determining the
transaction price.
The transaction price assumes transfers to the customer as promised in accordance with the
existing contract and that the contract will not be cancelled, renewed or modified.
The transaction price is not adjusted for effects of the customer’s credit risk, but is adjusted if the
entity (e.g. based on its customary business practices) has created a valid expectation that it will
enforce its rights for only a portion of the contract price.
An entity must consider the effects of all the following factors when determining the transaction
price:
 variable consideration;
 the constraint on variable consideration;
 time value of money;
 non-cash consideration;
 Consideration payable to the customer.

Illustration 06:
Tayyab Co. enters into a contract to build an oil rig for Rs. 100,000
If the oil rig is not completed on time, there will be a Rs. 20,000 penalty
Tayyab Co. has built similar oil rigs before and there is 90% chance that the oil rig will be completed
on time
Two possible outcomes:
Rs. 100,000 if completed on time
Rs. 80,000 if not completed on time
The “most likely amount” method better predicts the amount of consideration
Therefore, transaction price is Rs. 100,000 as there is 90% chance that the oil rig will be completed
on time.

2.4 Step 4: Allocate the transaction price to the performance obligations


The entity allocates a contract’s transaction price to each separate performance obligation within
that contract on a relative stand-alone selling price basis at contract inception.

Definition
A stand-alone selling price is the price at which an entity would sell a promised good or service
separately to a customer.

IFRS 15 suggests, but does not require, the following three methods as suitable for estimating
the stand-alone selling price:
 adjusted market assessment approach
 expected cost plus margin approach
 Residual approach.
2.5 Step 5: Recognise revenue when or as an entity satisfies performance obligations
Revenue is recognised when or as the promised goods or services are transferred to a customer.
 A transfer occurs when the customer obtains control of the good or service.

© Emile Woolf International 486 The Institute of Chartered Accountants of Pakistan


Chapter 10: IFRS 15: Revenue from contracts with customers

 A customer obtains control of an asset (good or service) when it can direct the use of and
obtain substantially all the remaining benefits from it. Control includes the ability to prevent
other entities from directing the use of and obtaining the benefits from an asset. Indicators
of control include:
 The entity has a present right to payment for the asset
 The customer has legal title
 The customer has physical possession (exceptions for bill and hold, consignment
sales and repos)
 The customer has the significant risks and rewards of ownership of the asset
 The customer has accepted the asset
 The benefits of an asset are the potential cash flows that can be obtained directly or indirectly
from the asset in many ways.
When goods or services are transferred continuously, a revenue recognition method that best
depicts the entity’s performance should be applied (updated as circumstances change).

Illustration 07:
On 1 November 2017, Shahid receives an order from a customer for 30 computers as well as 12
months of technical support for computers. Shahid delivers the computers (and transfers its legal
title) to the customer on the same day. The customer paid Rs. 25,000 upfront. The computer sells
for Rs. 20,000 and the technical support sells for Rs. 5,000.
Below are how the 5 steps would be applied to this contract:
Step 1 - Identify the contract
There is a contract between Shahid and its customer for the provision of goods (computers) and
services (technical support services)
Step 2 – Identify the separate performance obligations within a contract
There are two performance obligations (promises) within the contract:
1. The supply of a computer
2. The provision of technical support services over a year
Step 3 – Determine the transaction price
The total transaction price is Rs. 25,000 per computer.
Step 4 –Allocate the transaction price to the performance obligations in the contract
No need for any allocation as the transaction price and stand-alone price (market price) is same.
Step 5 – Recognise revenue when (or as) a performance obligation is satisfied.
Computer (Point in time)
Control over the computer has been passed to the customer so the full revenue of Rs. 20,000 for
30 computers (i.e. Rs. 600,000) should be recognized immediately.
Technical support services (Over time)
The technical support is provided over time (12 months), so revenue from this should be recognized
over time. For the year ended 31 December 2017, revenue of Rs. 25,000 (Rs.5, 000 x 30 x 2/12)
should be recognised from the provision of technical support services.

Acceptable methods include:


 Output methods: units produced, units delivered, contract milestones or surveys of work
performed; or
 Input methods: costs incurred, labour hours expended, machine hours used.

© Emile Woolf International 487 The Institute of Chartered Accountants of Pakistan


Financial accounting and reporting I

3 OTHER ASPECTS OF IFRS 15

Section overview

 Contract costs
 Presentation

3.1 Contract costs


Costs might be incurred in obtaining a contract and in fulfilling that contract.
Incremental costs of obtaining a contract
The incremental costs of obtaining a contract with a customer are recognised as an asset if the
entity expects to recover those costs.
The incremental costs of obtaining a contract are those costs that would not have been incurred if
the contract had not been obtained.
Costs to obtain a contract that would have been incurred regardless of whether the contract was
obtained are expensed as incurred (unless they can be recovered from the customer regardless
of whether the contract is obtained).

Example 01: Incremental costs of obtaining a contract


Question: X Limited wins a competitive bid to provide consulting services to a new customer.
X Limited incurred the following costs to obtain the contract:

Rs.

Commissions to sales employees for winning the contract 10,000

External legal fees for due diligence 15,000

Travel costs to deliver proposal 25,000

Total costs incurred 50,000

Required: How X Limited should recognise the above costs?


Answer:
The commission to sales employees is incremental to obtaining the contract and should be
capitalised as a contract asset.
The external legal fees and the travelling cost are not incremental to obtaining the contract because
they have been incurred regardless of whether X Limited obtained the contract or not.

An entity may recognise the incremental costs of obtaining a contract as an expense when incurred
if the amortisation period of the asset that the entity otherwise would have recognised is one year
or less.
Costs to fulfil a contract
Costs incurred in fulfilling a contract might be within the scope of another standard (for example,
IAS 2: Inventories, IAS 16: Property, Plant and Equipment or IAS 38: Intangible Assets). If this is
not the case, the costs are recognised as an asset only if they meet all of the following criteria:
 the costs relate directly to a contract or to an anticipated contract that the entity can
specifically identify;
 the costs generate or enhance resources of the entity that will be used in satisfying (or in
continuing to satisfy) performance obligations in the future; and
 The costs are expected to be recovered.

© Emile Woolf International 488 The Institute of Chartered Accountants of Pakistan


Chapter 10: IFRS 15: Revenue from contracts with customers

Costs that relate directly to a contract might include:


 direct labour and direct materials;
 allocations of costs that relate directly to the contract or to contract activities;
 costs that are explicitly chargeable to the customer under the contract; and
 Other costs that are incurred only because an entity entered into the contract (e.g. payments
to subcontractors).
The following costs must be recognised as expenses when incurred:
 general and administrative costs (unless those costs are explicitly chargeable to the
customer under the contract);
 costs of wasted materials, labour or other resources to fulfil the contract that were not
reflected in the price of the contract;
 Costs that relate to satisfied performance obligations (or partially satisfied performance
obligations) in the contract (i.e. costs that relate to past performance).
Amortisation and impairment
An asset for contract costs recognised in accordance with this standard must be amortised on a
systematic basis consistent with the transfer to the customer of the goods or services to which the
asset relates.
The amortisation must be updated to reflect a significant change in the entities expected timing of
transfer to the customer of the goods or services to which the asset relates.
An impairment loss must be recognised in profit or loss to the extent that the carrying amount of
an asset recognised exceeds:
 the remaining amount of consideration that the entity expects to receive in exchange for the
goods or services to which the asset relates; less
 The costs that relate directly to providing those goods or services and that have not been
recognised as expenses.
When the impairment conditions no longer exist or have improved a reversal of the impairment
loss is recognised. This will reinstate the asset but the increased carrying amount of the asset must
not exceed the amount that would have been determined (net of amortisation) if no impairment
loss had been recognised previously.

Example 02: Amortisation of contract costs


Question: X Limited wins a 5-year contract to provide a service to a customer. The contract contains
a single performance obligation satisfied over time. X Limited recognises revenue on a time basis.
Costs incurred by the end of year 1 and forecast future costs are as follows:

Rs.
Costs to date 10,000
Estimate of future costs 18,000
Total expected costs 28,000

Required: How X Limited should recognise the above costs?


Answer:
Costs must be recognised in the P&L on the same basis as that used to recognise revenue.
X Limited recognises revenue on a time basis, therefore 1/5 of the total expected cost should be
recognised = Rs. 5,600 per annum.

© Emile Woolf International 489 The Institute of Chartered Accountants of Pakistan


Financial accounting and reporting I

Example 03: Amortisation of contract costs


Question: X Limited wins a 5-year contract to provide a service to a customer. The contract is
renewable for subsequent one-year periods. The average customer term is seven years.
The contract contains a single performance obligation satisfied over time. X Limited recognises
revenue on a time basis.
Costs incurred by the end of year 1 and forecast future costs are as follows:

Rs.
Costs to date 10,000
Estimate of future costs 18,000
Total expected costs 28,000

Required: How X Limited should recognise the above costs?


Answer:
Costs must be recognised in the P&L on the same basis as that used to recognise revenue.
X Limited recognises revenue on a time basis. The asset relates to the services transferred to the
customer during the contract term of five years and X Limited anticipates that the contract will be
renewed for two subsequent one-year periods.
Therefore 1/7 of the total expected cost should be recognised = Rs. 4,000 per annum.

Example 04: Amortisation of contract costs


Question: X Limited wins a contract to build an asset for a customer. It is anticipated that the asset
will take 2 years to complete.
The contract contains a single performance obligation. Progress to completion is measured on an
output basis. At the end of year 1 the assets are 60% complete.
Costs incurred by the end of year 1 and forecast future costs are as follows:

Rs.
Costs to date 10,000
Estimate of future costs 18,000
Total expected costs 28,000

Required: How X Limited should recognise the above costs?


Answer:
Costs must be recognised in the P&L on the same basis as that used to recognise revenue.
Therefore 60% of the total expected cost should be recognised (Rs. 16,800) at the end of year 1.

3.2 Presentation
This section explains how contracts are presented in the statement of financial position. In order
to do this, it explains the double entries that might result from the recognition of revenue. The
double entries depend on circumstance.
An unconditional right to consideration is presented as a receivable.
The accounting treatment to record the transfer of goods for cash or for an unconditional promise
to be paid consideration is straightforward.

© Emile Woolf International 490 The Institute of Chartered Accountants of Pakistan


Chapter 10: IFRS 15: Revenue from contracts with customers

Illustration 08: Possible double entries on recognition of revenue

Debit Credit

Cash X

Receivable X

Revenue X

Example 05: Double entry – Unconditional right to consideration


Question: On 1 January 20X8, X Limited enters into a contract to transfer Products A and B to Y
Limited in exchange for Rs. 1,000. Product A is to be delivered on 28 February. Product B is to be
delivered on 31 March.
The promises to transfer Products A and B are identified as separate performance obligations.
Rs.400 is allocated to Product A and Rs.600 to Product B.

X Limited recognises revenue and recognises its unconditional right to the consideration when
control of each product transfers to Y Limited.
Required:
Pass the journal entries to record the above transactions.
Answer:
28 February: X Limited transfers Product A to Y Limited.

At 28 February Dr (Rs.) Cr (Rs.)

Receivables 400

Revenue 400

31 March: X Limited transfers Product B to Y Limited

31 March

Receivables 600

Revenue 600

In other cases, a contract is presented as a contract asset or a contract liability depending on the
relationship between the entity’s performance and the customer’s payment.
Contract assets
A supplier might transfer goods or services to a customer before the customer pays consideration
or before payment is due. In this case the contract is presented as a contract asset (excluding any
amounts presented as a receivable).
A contract asset is a supplier’s right to consideration in exchange for goods or services that it has
transferred to a customer. A contract asset is reclassified as a receivable when the supplier’s right
to consideration becomes unconditional.

© Emile Woolf International 491 The Institute of Chartered Accountants of Pakistan


Financial accounting and reporting I

Example 06: Double entry – Recognition of a contract asset


Question: On 1 January 20X8, X Limited enters into a contract to transfer Products A and B to Y
Limited in exchange for Rs. 1,000. Product A is to be delivered on 28 February. Product B is to be
delivered on 31 March.
The promises to transfer Products A and B are identified as separate performance obligations.
Rs.400 is allocated to Product A and Rs.600 to Product B.
Revenue is recognised when control of each product transfers to Y Limited. Payment for the delivery
of Product A is conditional on the delivery of Product B. (i.e. the consideration of Rs. 1,000 is due
only after X Limited has transferred both Products A and B to Y Limited). This means that X Limited
does not have a right to consideration that is unconditional (a receivable) until both Products A and
B are transferred to Y Limited.

Required:
Pass the journal entries to record the above transactions.
Answer:
28 February: X Limited transfers Product A to Y Limited
X Plc does not have an unconditional right to receive the Rs.400 so the amount is recognised as a
contract asset.

At 28 February Dr (Rs.) Cr (Rs.)

Contract asset 400

Revenue 400

31 March: X Limited transfers Product B to Y Limited


X Limited now has an unconditional right to receive the full Rs. 1,000. The Rs.400 previously
recognised as a contract asset is reclassified as a receivable and the Rs.600 for the transfer of
product B is also recognised as receivable.

31 March Dr (Rs.) Cr (Rs.)

Receivable 1,000

Contract asset 400

Revenue 600

Contract liabilities
A contract might require payment in advance or allow the supplier a right to an amount of
consideration that is unconditional (i.e. a receivable), before it transfers a good or service to the
customer.
In these cases, the supplier presents the contract as a contract liability when the payment is made
or the payment is due (whichever is earlier).
The contract liability is a supplier’s obligation to transfer goods or services to a customer for which
it has received consideration (an amount of consideration is due) from the customer.

© Emile Woolf International 492 The Institute of Chartered Accountants of Pakistan


Chapter 10: IFRS 15: Revenue from contracts with customers

Example 07: Double entry – Recognition of a contract liability


Question: On 1 January 20X8, X Limited enters into a contract to transfer Products A and B to Y
Limited in exchange for Rs. 1,000. X Limited can invoice this full amount on 31 January. Product A
is to be delivered on 28 February. Product B is to be delivered on 31 March.
The promises to transfer Products A and B are identified as separate performance obligations.
Rs.400 is allocated to Product A and Rs.600 to Product B.
Revenue is recognised when control of each product transfers to Y Limited.

Required:
Pass the journal entries to record the above transactions.
Answer:
The following accounting entries would be necessary:

At 31 January Dr (Rs.) Cr (Rs.)

Receivable 1,000

Contract liability 1,000

28 February: X Limited transfers Product A to Y Limited

At 28 February Dr (Rs.) Cr (Rs.)

Contract liability 400

Revenue 400

31 March: X Limited transfers Product B to Y Limited

31 March Dr (Rs.) Cr (Rs.)

Contract liability 600

Revenue 600

© Emile Woolf International 493 The Institute of Chartered Accountants of Pakistan


Financial accounting and reporting I

4 EXAMINABLE EXAMPLES OF IFRS 15 REVENUE FROM CONTRACTS WITH


CUSTOMERS

Section overview

 Examples of IFRS 15 as per International Financial Reporting Standards


 Other Examples

IFRS 15 is accompanied with examples that illustrate how an entity applies some of the requirements to
particular aspects of contract with a customer. Out of the said examples the following are relevant to the
learning outcome of CAF-5 Financial Accounting and Reporting I.
There are two types of examinable examples in the study text:
 Examples of IFRS 15 as per International Financial Reporting Standards
 Other Examples

4.1 Examples of IFRS 15 as per International Financial Reporting Standards

CONTRACT MODIFICATIONS
Example 5: Modification of a contract for goods
IE19 An entity promises to sell 120 products to a customer for CU12, 000 (CU100 per product).
The products are transferred to the customer over a six-month period. The entity transfers
control of each product at a point in time. After the entity has transferred control of 60
products to the customer, the contract is modified to require the delivery of an additional
30 products (a total of 150 identical products) to the customer. The additional 30 products
were not included in the initial contract.
Case A—Additional products for a price that reflects the stand-alone selling price
IE20 When the contract is modified, the price of the contract modification for the additional 30
products is an additional CU2, 850 or CU95 per product. The pricing for the additional
products reflects the stand-alone selling price of the products at the time of the contract
modification and the additional products are distinct (in accordance with paragraph 27 of
IFRS 15) from the original products.
IE21 In accordance with paragraph 20 of IFRS 15, the contract modification for the additional
30 products is, in effect, a new and separate contract for future products that does not
affect the accounting for the existing contract. The entity recognizes revenue of CU100 per
product for the 120 products in the original contract and CU95 per product for the 30
products in the new contract.
Case B—Additional products for a price that does not reflect the stand-alone selling price
IE22 During the process of negotiating the purchase of an additional 30 products, the parties
initially agree on a price of CU80 per product. However, the customer discovers that the
initial 60 products transferred to the customer contained minor defects that were unique
to those delivered products. The entity promises a partial credit of CU15 per product to
compensate the customer for the poor quality of those products. The entity and the
customer agree to incorporate the credit of CU900 (CU15 credit × 60 products) into the
price that the entity charges for the additional 30 products. Consequently, the contract
modification specifies that the price of the additional 30 products is CU1, 500 or CU50 per
product. That price comprises the agreed-upon price for the additional 30 products of CU2,
400, or CU80 per product, less the credit of CU900.

© Emile Woolf International 494 The Institute of Chartered Accountants of Pakistan


Chapter 10: IFRS 15: Revenue from contracts with customers

IE23 at the time of modification, the entity recognizes the CU900 as a reduction of the
transaction price and, therefore, as a reduction of revenue for the initial 60 products
transferred. In accounting for the sale of the additional 30 products, the entity determines
that the negotiated price of CU80 per product does not reflect the stand-alone selling price
of the additional products. Consequently, the contract modification does not meet the
conditions in paragraph 20 of IFRS 15 to be accounted for as a separate contract. Because
the remaining products to be delivered are distinct from those already transferred, the
entity applies the requirements in paragraph 21(a) of IFRS 15 and accounts for the
modification as a termination of the original contract and the creation of a new contract.
IE24 Consequently, the amount recognized as revenue for each of the remaining products is a
blended price of CU93.33 {[(CU100 × 60 products not yet transferred under the original
contract) + (CU80 × 30 products to be transferred under the contract modification)] ÷ 90
remaining products}.

Example 13—Customer simultaneously receives and consumes the benefits


IE67 An entity enters into a contract to provide monthly payroll processing services to a
customer for one year.
IE68 The promised payroll processing services are accounted for as a single performance
obligation in accordance with paragraph 22(b) of IFRS 15. The performance obligation is
satisfied over time in accordance with paragraph 35(a) of IFRS 15 because the customer
simultaneously receives and consumes the benefits of the entity’s performance in
processing each payroll transaction as and when each transaction is processed. The fact
that another entity would not need to re-perform payroll processing services for the service
that the entity has provided to date also demonstrates that the customer simultaneously
receives and consumes the benefits of the entity’s performance as the entity performs.
(The entity disregards any practical limitations on transferring the remaining performance
obligation, including setup activities that would need to be undertaken by another entity.)
The entity recognizes revenue over time by measuring its progress towards complete
satisfaction of that performance obligation in accordance with paragraphs 39–45 and
B14–B19 of IFRS 15.

Example 17—Assessing whether a performance obligation is satisfied at a point in time or over time
IE81 and entity is developing a multi-unit residential complex. A customer enters into a binding
sales contract with the entity for a specified unit that is under construction. Each unit has
a similar floor plan and is of a similar size, but other attributes of the units are different
(for example, the location of the unit within the complex).
Case A—Entity does not have an enforceable right to payment for performance completed to date
IE82 The customer pays a deposit upon entering into the contract and the deposit is refundable
only if the entity fails to complete construction of the unit in accordance with the contract.
The remainder of the contract price is payable on completion of the contract when the
customer obtains physical possession of the unit. If the customer defaults on the contract
before completion of the unit, the entity only has the right to retain the deposit.
IE83 at contract inception, the entity applies paragraph 35(c) of IFRS 15 to determine whether
its promise to construct and transfer the unit to the customer is a performance obligation
satisfied over time. The entity determines that it does not have an enforceable right to
payment for performance completed to date because, until construction of the unit is
complete, the entity only has a right to the deposit paid by the customer. Because the entity
does not have a right to payment for work completed to date, the entity’s performance
obligation is not a performance obligation satisfied over time in accordance with paragraph
35(c) of IFRS 15. Instead, the entity accounts for the sale of the unit as a performance
obligation satisfied at a point in time in accordance with paragraph 38 of IFRS 15.

© Emile Woolf International 495 The Institute of Chartered Accountants of Pakistan


Financial accounting and reporting I

Case B—Entity has an enforceable right to payment for performance completed to date
IE84 The customer pays a non-refundable deposit upon entering into the contract and will make
progress payments during construction of the unit. The contract has substantive terms that
preclude the entity from being able to direct the unit to another customer. In addition, the
customer does not have the right to terminate the contract unless the entity fails to perform
as promised. If the customer defaults on its obligations by failing to make the promised
progress payments as and when they are due, the entity would have a right to all of the
consideration promised in the contract if it completes the construction of the unit. The
courts have previously upheld similar rights that entitle developers to require the customer
to perform, subject to the entity meeting its obligations under the contract.
IE85 at contract inception, the entity applies paragraph 35(c) of IFRS 15 to determine whether
its promise to construct and transfer the unit to the customer is a performance obligation
satisfied over time. The entity determines that the asset (unit) created by the entity’s
performance does not have an alternative use to the entity because the contract precludes
the entity from transferring the specified unit to another customer. The entity does not
consider the possibility of a contract termination in assessing whether the entity is able to
direct the asset to another customer.
Example 17 (continued)
IE86 The entity also has a right to payment for performance completed to date in accordance
with paragraphs 37 and B9–B13 of IFRS 15. This is because if the customer were to default
on its obligations, the entity would have an enforceable right to all of the consideration
promised under the contract if it continues to perform as promised.
IE87 Therefore, the terms of the contract and the practices in the legal jurisdiction indicate that
there is a right to payment for performance completed to date. Consequently, the criteria
in paragraph 35(c) of IFRS 15 are met and the entity has a performance obligation that it
satisfies over time. To recognize revenue for that performance obligation satisfied over
time, the entity measures its progress towards complete satisfaction of its performance
obligation in accordance with paragraphs 39–45 and B14–B19 of IFRS 15.
IE88 in the construction of a multi-unit residential complex, the entity may have many contracts
with individual customers for the construction of individual units within the complex. The
entity would account for each contract separately. However, depending on the nature of
the construction, the entity’s performance in undertaking the initial construction works (i.e.
the foundation and the basic structure), as well as the construction of common areas, may
need to be reflected when measuring its progress towards complete satisfaction of its
performance obligations in each contract.
Case C—Entity has an enforceable right to payment for performance completed to date
IE89 The same facts as in Case B apply to Case C, except that in the event of a default by the
customer, either the entity can require the customer to perform as required under the
contract or the entity can cancel the contract in exchange for the asset under construction
and an entitlement to a penalty of a proportion of the contract price.
IE90 Notwithstanding that the entity could cancel the contract (in which case the customer’s
obligation to the entity would be limited to transferring control of the partially completed
asset to the entity and paying the penalty prescribed), the entity has a right to payment for
performance completed to date because the entity could also choose to enforce its rights
to full payment under the contract. The fact that the entity may choose to cancel the
contract in the event the customer defaults on its obligations would not affect that
assessment (see paragraph B11 of IFRS 15), provided that the entity’s rights to require the
customer to continue to perform as required under the contract (i.e. pay the promised
consideration) are enforceable.

Example 18—Measuring progress when making goods or services available


IE92 an entity, an owner and manager of health clubs, enters into a contract with a customer
for one year of access to any of its health clubs. The customer has unlimited use of the
health clubs and promises to pay CU100 per month.

© Emile Woolf International 496 The Institute of Chartered Accountants of Pakistan


Chapter 10: IFRS 15: Revenue from contracts with customers

IE93 The entity determines that its promise to the customer is to provide a service of making
the health clubs available for the customer to use as and when the customer wishes. This
is because the extent to which the customer uses the health clubs does not affect the
amount of the remaining goods and services to which the customer is entitled. The entity
concludes that the customer simultaneously receives and consumes the benefits of the
entity’s performance as it performs by making the health clubs available. Consequently,
the entity’s performance obligation is satisfied over time in accordance with paragraph
35(a) of IFRS 15.
IE94 The entity also determines that the customer benefits from the entity’s service of making
the health clubs available evenly throughout the year. (That is, the customer benefits from
having the health clubs available, regardless of whether the customer uses it or not.)
Consequently, the entity concludes that the best measure of progress towards complete
satisfaction of the performance obligation over time is a time-based measure and it
recognizes revenue on a straight-line basis throughout the year at CU100 per month.

Example 20—Penalty gives rise to variable consideration


IE102 an entity enters into a contract with a customer to build an asset for CU1 million. In
addition, the terms of the contract include a penalty of CU100, 000 if the construction is
not completed within three months of a date specified in the contract.
IE103 The entity concludes that the consideration promised in the contract includes a fixed
amount of CU900, 000 and a variable amount of CU100, 000 (arising from the penalty).
IE104 The entity estimates the variable consideration in accordance with paragraphs 50–54 of
IFRS 15 and considers the requirements in paragraphs 56–58 of IFRS 15 on constraining
estimates of variable consideration.

Example 22—Right of return


IE110 an entity enters into 100 contracts with customers. Each contract includes the sale of one
product for CU100 (100 total products × CU100 = CU10, 000 total consideration). Cash is
received when control of a product transfers. The entity’s customary business practice is to
allow a customer to return any unused product within 30 days and receive a full refund.
The entity’s cost of each product is CU60.
IE111 The entity applies the requirements in IFRS 15 to the portfolio of 100 contracts because it
reasonably expects that, in accordance with paragraph 4, the effects on the financial
statements from applying these requirements to the portfolio would not differ materially
from applying the requirements to the individual contracts within the portfolio.
IE112 Because the contract allows a customer to return the products, the consideration received
from the customer is variable. To estimate the variable consideration to which the entity
will be entitled, the entity decides to use the expected value method (see paragraph 53(a)
of IFRS 15) because it is the method that the entity expects to better predict the amount
of consideration to which it will be entitled. Using the expected value method, the entity
estimates that 97 products will not be returned.
IE113 The entity also considers the requirements in paragraphs 56–58 of IFRS 15 on constraining
estimates of variable consideration to determine whether the estimated amount of
variable consideration of CU9,700 (CU100 × 97 products not expected to be returned) can
be included in the transaction price. The entity considers the factors in paragraph 57 of
IFRS 15 and determines that although the returns are outside the entity’s influence, it has
significant experience in estimating returns for this product and customer class. In
addition, the uncertainty will be resolved within a short time frame (i.e. the 30-day return
period). Thus, the entity concludes that it is highly probable that a significant reversal in
the cumulative amount of revenue recognized (i.e. CU9, 700) will not occur as the
uncertainty is resolved (i.e. over the return period).
IE114 The entity estimates that the costs of recovering the products will be immaterial and
expects that the returned products can be resold at a profit.

© Emile Woolf International 497 The Institute of Chartered Accountants of Pakistan


Financial accounting and reporting I

IE115 Upon transfer of control of the 100 products, the entity does not recognize revenue for the
three products that it expects to be returned. Consequently, in accordance with paragraphs
55 and B21 of IFRS 15, the entity recognizes the following:
(a) revenue of CU9,700 (CU100 × 97 products not expected to be returned);
(b) a refund liability of CU300 (CU100 refund × 3 products expected to be returned);
and
(c) An asset of CU180 (CU60 × 3 products for its right to recover products from
customers on settling the refund liability).

Example 24—Volume discount incentive


IE124 an entity enters into a contract with a customer on 1 January 20X8 to sell Product A for
CU100 per unit. If the customer purchases more than 1,000 units of Product A in a calendar
year, the contract specifies that the price per unit is retrospectively reduced to CU90 per
unit. Consequently, the consideration in the contract is variable.
IE125 for the first quarter ended 31 March 20X8, the entity sells 75 units of Product A to the
customer. The entity estimates that the customer’s purchases will not exceed the 1,000-
unit threshold required for the volume discount in the calendar year.
IE126 The entity considers the requirements in paragraphs 56–58 of IFRS 15 on constraining
estimates of variable consideration, including the factors in paragraph 57 of IFRS 15. The
entity determines that it has significant experience with this product and with the
purchasing pattern of the entity. Thus, the entity concludes that it is highly probable that a
significant reversal in the cumulative amount of revenue recognized (i.e. CU100 per unit)
will not occur when the uncertainty is resolved (i.e. when the total amount of purchases is
known). Consequently, the entity recognizes revenue of CU7, 500 (75 units × CU100 per
unit) for the quarter ended 31 March 20X8.
IE127 in May 20X8, the entity’s customer acquires another company and in the second quarter
ended 30 June 20X8 the entity sells an additional 500 units of Product A to the customer.
In the light of the new fact, the entity estimates that the customer’s purchases will exceed
the 1,000-unit threshold for the calendar year and therefore it will be required to
retrospectively reduce the price per unit to CU90.
IE128 Consequently, the entity recognizes revenue of CU44, 250 for the quarter ended 30 June
20X8. That amount is calculated from CU45,000 for the sale of 500 units (500 units ×
CU90 per unit) less the change in transaction price of CU750 (75 units × CU10 price
reduction) for the reduction of revenue relating to units sold for the quarter ended 31 March
20X8 (see paragraphs 87 and 88 of IFRS 15).

Example 26—Significant financing component and right of return


IE135 and entity sells a product to a customer for CU121 that is payable 24 months after delivery.
The customer obtains control of the product at contract inception. The contract permits the
customer to return the product within 90 days. The product is new and the entity has no
relevant historical evidence of product returns or other available market evidence.
IE136 The cash selling price of the product is CU100, which represents the amount that the
customer would pay upon delivery for the same product sold under otherwise identical
terms and conditions as at contract inception. The entity’s cost of the product is CU80.
IE137 The entity does not recognize revenue when control of the product transfers to the
customer. This is because the existence of the right of return and the lack of relevant
historical evidence means that the entity cannot conclude that it is highly probable that a
significant reversal in the amount of cumulative revenue recognized will not occur in
accordance with paragraphs 56–58 of IFRS 15. Consequently, revenue is recognized after
three months when the right of return lapses.

© Emile Woolf International 498 The Institute of Chartered Accountants of Pakistan


Chapter 10: IFRS 15: Revenue from contracts with customers

IE138 The contract includes a significant financing component, in accordance with paragraphs
60–62 of IFRS 15. This is evident from the difference between the amount of promised
consideration of CU121 and the cash selling price of CU100 at the date that the goods are
transferred to the customer.
IE139 The contract includes an implicit interest rate of 10 per cent (i.e. the interest rate that over
24 months discounts the promised consideration of CU121 to the cash selling price of
CU100). The entity evaluates the rate and concludes that it is commensurate with the rate
that would be reflected in a separate financing transaction between the entity and its
customer at contract inception. The following journal entries illustrate how the entity
accounts for this contract in accordance with paragraphs B20–B27 of IFRS 15.
(a) When the product is transferred to the customer, in accordance with paragraph B21 of IFRS
15:

Asset for right to recover product to be Returned CU80(a)


Inventory CU80

(a) This example does not consider expected costs to recover the asset.
(b) During the three-month right of return period, no interest is recognized in accordance with
paragraph 65 of IFRS 15 because no contract asset or receivable has been recognized.
(c) When the right of return lapses (the product is not returned):

Receivable CU100(a)
Revenue CU100
Cost of sales CU80
Asset for product to be returned CU80

(a) The receivable recognized would be measured in accordance with IFRS 9. This example
assumes there is no material difference between the fair value of the receivable at contract
inception and the fair value of the receivable when it is recognized at the time the right of return
lapses. In addition, this example does not consider the impairment accounting for the
receivable.
IE140 Until the entity receives the cash payment from the customer, interest revenue would be
recognized in accordance with IFRS 9. In determining the effective interest rate in
accordance with IFRS 9, the entity would consider the remaining contractual term.

Example 29—Advance payment and assessment of discount rate


IE148 an entity enters into a contract with a customer to sell an asset. Control of the asset will
transfer to the customer in two years (i.e. the performance obligation will be satisfied at a
point in time). The contract includes two alternative payment options: payment of CU5, 000
in two years when the customer obtains control of the asset or payment of CU4, 000 when
the contract is signed. The customer elects to pay CU4, 000 when the contract is signed.
IE149 The entity concludes that the contract contains a significant financing component because
of the length of time between when the customer pays for the asset and when the entity
transfers the asset to the customer, as well as the prevailing interest rates in the market.
IE150 The interest rate implicit in the transaction is 11.8 per cent, which is the interest rate
necessary to make the two alternative payment options economically equivalent. However,
the entity determines that, in accordance with paragraph 64 of IFRS 15, the rate that
should be used in adjusting the promised consideration is six per cent, which is the entity’s
incremental borrowing rate.
IE151 The following journal entries illustrate how the entity would account for the significant
financing component:

© Emile Woolf International 499 The Institute of Chartered Accountants of Pakistan


Financial accounting and reporting I

(a) recognize a contract liability for the CU4,000 payment received at contract inception:
Cash CU4,000
Contract liability CU4,000
(b) during the two years from contract inception until the transfer of the asset, the entity
adjusts the promised amount of consideration (in accordance with paragraph 65 of
IFRS 15) and accretes the contract liability by recognizing interest on CU4,000 at six
per cent for two years:
Interest expense CU494(a)
Contract liability CU494
CU494 = CU4, 000 contract liability × (6 per cent interest per year for two years).
(c) recognize revenue for the transfer of the asset:
Contract liability CU4,494
Revenue CU4,494

Example 31—Entitlement to non-cash consideration


IE156 an entity enters into a contract with a customer to provide a weekly service for one year.
The contract is signed on 1 January 20X1 and work begins immediately. The entity
concludes that the service is a single performance obligation in accordance with paragraph
22(b) of IFRS 15. This is because the entity is providing a series of distinct services that are
substantially the same and have the same pattern of transfer (the services transfer to the
customer over time and use the same method to measure progress—that is, a time-based
measure of progress).
IE157 in exchange for the service, the customer promises 100 shares of its common stock per
week of service (a total of 5,200 shares for the contract). The terms in the contract require
that the shares must be paid upon the successful completion of each week of service.
IE158 The entity measures its progress towards complete satisfaction of the performance
obligation as each week of service is complete. To determine the transaction price (and
the amount of revenue to be recognized), the entity measures the fair value of 100 shares
that are received upon completion of each weekly service. The entity does not reflect any
subsequent changes in the fair value of the shares received (or receivable) in revenue.

Example 32—Consideration payable to a customer


IE160 an entity that manufactures consumer goods enter into a one-year contract to sell goods
to a customer that is a large global chain of retail stores. The customer commits to buy at
least CU15 millions of products during the year. The contract also requires the entity to
make a non-refundable payment of CU1.5 million to the customer at the inception of the
contract. The CU1.5 million payments will compensate the customer for the changes it
needs to make to its shelving to accommodate the entity’s products.
IE161 The entity considers the requirements in paragraphs 70–72 of IFRS 15 and concludes that
the payment to the customer is not in exchange for a distinct good or service that transfers
to the entity. This is because the entity does not obtain control of any rights to the
customer’s shelves. Consequently, the entity determines that, in accordance with
paragraph 70 of IFRS 15, the CU1.5 million payments is a reduction of the transaction
price.
IE162 The entity applies the requirements in paragraph 72 of IFRS 15 and concludes that the
consideration payable is accounted for as a reduction in the transaction price when the
entity recognizes revenue for the transfer of the goods. Consequently, as the entity
transfers goods to the customer, the entity reduces the transaction price for each good by
10 per cent (CU1.5 million ÷ CU15 million). Therefore, in the first month in which the entity
transfers goods to the customer, the entity recognizes revenue of CU1.8 million (CU2.0
million invoiced amount less CU0.2 million of consideration payable to the customer).
Changes in the fair value of the shares received (or receivable) in revenue.

© Emile Woolf International 500 The Institute of Chartered Accountants of Pakistan


Chapter 10: IFRS 15: Revenue from contracts with customers

Example 33—Allocation methodology


IE164 an entity enters into a contract with a customer to sell Products A, B and C in exchange for
CU100. The entity will satisfy the performance obligations for each of the products at
different points in time. The entity regularly sells Product A separately and therefore the
stand-alone selling price is directly observable. The stand-alone selling prices of Products
B and C are not directly observable.
IE165 Because the stand-alone selling prices for Products B and C are not directly observable, the
entity must estimate them. To estimate the stand-alone selling prices, the entity uses the
adjusted market assessment approach for Product B and the expected cost plus a margin
approach for Product C. In making those estimates, the entity maximizes the use of
observable inputs (in accordance with paragraph 78 of IFRS 15). The entity estimates the
stand-alone selling prices as follows:

Stand-alone
Product Method CU
selling price
Product A 50 Directly observable (see paragraph 77 of IFRS 15)
Product B 25 Adjusted market assessment approach
(see paragraph 79(a) of FRS15)
Product C 75 Expected cost plus a margin approach
(see paragraph 79(b) of IFRS 15)
Total 150
IE166 The customer receives a discount for purchasing the bundle of goods because the sum of
the stand-alone selling prices (CU150) exceeds the promised consideration (CU100). The
entity considers whether it has observable evidence about the performance obligation to
which the entire discount belongs (in accordance with paragraph 82 of IFRS 15) and
concludes that it does not. Consequently, in accordance with paragraphs 76 and 81 of IFRS
15, the discount is allocated proportionately across Products A, B and C. The discount, and
therefore the transaction price, is allocated as follows:
Product Allocated transaction price
CU
Product A 33 (CU50 ÷ CU150 × CU100)
Product B 17 (CU25 ÷ CU150 × CU100)
Product C 50 (CU75 ÷ CU150 × CU100)
-----
Total 100

Example 34—Allocating a discount


IE167 an entity regularly sells Products A, B and C individually, thereby establishing the following
stand-alone selling prices:
Product Stand-alone selling price
CU
Product A 40
Product B 55
Product C 45
-----
Total 140
IE168 in addition, the entity regularly sells Products B and C together for CU60.
Case A—Allocating a discount to one or more performance obligations

© Emile Woolf International 501 The Institute of Chartered Accountants of Pakistan


Financial accounting and reporting I

IE169 The entity enters into a contract with a customer to sell Products A, B and C in exchange
for CU100. The entity will satisfy the performance obligations for each of the products at
different points in time.
IE170 The contract includes a discount of CU40 on the overall transaction, which would be
allocated proportionately to all three performance obligations when allocating the
transaction price using the relative stand-alone selling price method (in accordance with
paragraph 81 of IFRS 15). However, because the entity regularly sells Products B and C
together for CU60 and Product A for CU40, it has evidence that the entire discount should
be allocated to the promises to transfer Products B and C in accordance with paragraph
82 of IFRS 15.
IE171 If the entity transfers control of Products B and C at the same point in time, then the entity
could, as a practical matter, account for the transfer of those products as a single
performance obligation. That is, the entity could allocate CU60 of the transaction price to
the single performance obligation and recognize revenue of CU60 when Products B and C
simultaneously transfer to the customer.
IE172 If the contract requires the entity to transfer control of Products B and C at different points
in time, then the allocated amount of CU60 is individually allocated to the promises to
transfer Product B (stand-alone selling price of CU55) and Product C (stand-alone selling
price of CU45) as follows:
Product Allocated transaction price
CU
Product B 33 (CU55 ÷ CU100 total stand-alone selling price × CU60)
Product C 27 (CU45 ÷ CU100 total stand-alone selling price × CU60)
Total 60
Case B—Residual approach is appropriate
IE173 The entity enters into a contract with a customer to sell Products A, B and C as described
in Case A. The contract also includes a promise to transfer Product D. Total consideration
in the contract is CU130. The stand-alone selling price for Product D is highly variable (see
paragraph 79(c) of IFRS 15) because the entity sells Product D to different customers for
a broad range of amounts (CU15–CU45). Consequently, the entity decides to estimate the
stand-alone selling price of Product D using the residual approach.
IE174 Before estimating the stand-alone selling price of Product D using the residual approach,
the entity determines whether any discount should be allocated to the other performance
obligations in the contract in accordance with paragraphs 82 and 83 of IFRS 15.
IE175 as in Case A, because the entity regularly sells Products B and C together for CU60 and
Product A for CU40, it has observable evidence that CU100 should be allocated to those
three products and a CU40 discount should be allocated to the promises to transfer
Products B and C in accordance with paragraph 82 of
IFRS 15 Using the residual approach, the entity estimates the stand-alone selling price of Product
D to be CU30 as follows:
Stand-alone
Product selling
Price Method
CU
Product A 40 Directly observable (see paragraph 77 of IFRS 15)
Products B
and C 60 Directly observable with discount (see paragraph 82 of IFRS 15)
Product D 30 Residual approach (see paragraph 79(c) of IFRS 15)
Total 130

© Emile Woolf International 502 The Institute of Chartered Accountants of Pakistan


Chapter 10: IFRS 15: Revenue from contracts with customers

IE176 The entity observes that the resulting CU30 allocated to Product D is within the range of its
observable selling prices (CU15–CU45). Therefore, the resulting allocation (see above
table) is consistent with the allocation objective in paragraph 73 of IFRS 15 and the
requirements in paragraph 78 of IFRS 15.
Case C—Residual approach is inappropriate
IE177 The same facts as in Case B apply to Case C except the transaction price is CU105 instead
of CU130. Consequently, the application of the residual approach would result in a stand-
alone selling price of CU5 for Product D (CU105 transaction price less CU100 allocated to
Products A, B and C). The entity concludes that CU5 would not faithfully depict the amount
of consideration to which the entity expects to be entitled in exchange for satisfying its
performance obligation to transfer Product D, because CU5 does not approximate the
stand-alone selling price of Product D, which ranges from CU15–CU45. Consequently, the
entity reviews its observable data, including sales and margin reports, to estimate the
stand-alone selling price of Product D using another suitable method. The entity allocates
the transaction price of CU105 to Products A, B, C and D using the relative stand-alone
selling prices of those products in accordance with paragraphs 73–80 of IFRS 15.

Example 36—Incremental costs of obtaining a contract


IE189 an entity, a provider of consulting services, wins a competitive bid to provide consulting
services to a new customer. The entity incurred the following costs to obtain the contract:
CU
External legal fees for due diligence 15,000
Travel costs to deliver proposal 25,000
Commissions to sales employees 10,000
Total costs incurred 50,000
IE190 in accordance with paragraph 91 of IFRS 15, the entity recognizes an asset for the CU10,
000 incremental costs of obtaining the contract arising from the commissions to sales
employees because the entity expects to recover those costs through future fees for the
consulting services. The entity also pays discretionary annual bonuses to sales supervisors
based on annual sales targets, overall profitability of the entity and individual performance
evaluations. In accordance with paragraph 91 of IFRS 15, the entity does not recognize an
asset for the bonuses paid to sales supervisors because the bonuses are not incremental
to obtaining a contract. The amounts are discretionary and are based on other factors,
including the profitability of the entity and the individuals’ performance. The bonuses are
not directly attributable to identifiable contracts.
IE191 The entity observes that the external legal fees and travel costs would have been incurred
regardless of whether the contract was obtained. Therefore, in accordance with paragraph
93 of IFRS 15, those costs are recognized as expenses when incurred, unless they are
within the scope of another Standard, in which case, the relevant provisions of that
Standard apply.

Example 38—Contract liability and receivable


Case A—Cancellable contract
IE198 On 1 January 20X9, an entity enters into a cancellable contract to transfer a product to a
customer on 31 March 20X9. The contract requires the customer to pay consideration of
CU1, 000 in advance on 31 January 20X9. The customer pays the consideration on 1 March
20X9. The entity transfers the product on 31 March 20X9. The following journal entries
illustrate how the entity accounts for the contract:
(a) The entity receives cash of CU1,000 on 1 March 20X9 (cash is received in advance of
performance):
Cash CU1,000
Contract liability CU1,000

© Emile Woolf International 503 The Institute of Chartered Accountants of Pakistan


Financial accounting and reporting I

(b) The entity satisfies the performance obligation on 31 March 20X9:


Contract liability CU1,000
Revenue CU1,000
Case B—Non-cancellable contract
IE199 The same facts as in Case A apply to Case B except that the contract is non-cancellable.
The following journal entries illustrate how the entity accounts for the contract:
(a) The amount of consideration is due on 31 January 20X9 (which is when the entity
recognizes a receivable because it has an unconditional right to consideration):
Receivable CU1,000
Contract liability CU1,000
(b) The entity receives the cash on 1 March 20X9:
Cash CU1,000
Receivable CU1,0003
(c) The entity satisfies the performance obligation on 31 March 20X9:
Contract liability CU1,000
Revenue CU1,000
IE200 If the entity issued the invoice before 31 January 20X9 (the due date of the consideration),
the entity would not present the receivable and the contract liability on a gross basis in the
statement of financial position because the entity does not yet have a right to consideration
that is unconditional.

Example 39—Contract asset recognised for the entity’s performance


IE201 On 1 January 20X8, an entity enters into a contract to transfer Products A and B to a
customer in exchange for CU1, 000. The contract requires Product A to be delivered first
and states that payment for the delivery of Product A is conditional on the delivery of
Product B. In other words, the consideration of CU1, 000 is due only after the entity has
transferred both Products A and B to the customer. Consequently, the entity does not have
a right to consideration that is unconditional (a receivable) until both Products A and B are
transferred to the customer.
IE202 The entity identifies the promises to transfer Products A and B as performance obligations
and allocates CU400 to the performance obligation to transfer Product A and CU600 to the
performance obligation to transfer Product B on the basis of their relative stand-alone
selling prices. The entity recognizes revenue for each respective performance obligation
when control of the product transfers to the customer.
IE203 The entity satisfies the performance obligation to transfer Product A:
Contract asset CU400
Revenue CU400
IE204 The entity satisfies the performance obligation to transfer Product B and to recognize the
unconditional right to consideration:
Receivable CU1,000
Contract asset CU400
Revenue CU600

Example 40—Receivable recognised for the entity’s performance


IE205 An entity enters into a contract with a customer on 1 January 20X9 to transfer products
to the customer for CU150 per product. If the customer purchases more than 1 million
products in a calendar year, the contract indicates that the price per unit is retrospectively
reduced to CU125 per product.

© Emile Woolf International 504 The Institute of Chartered Accountants of Pakistan


Chapter 10: IFRS 15: Revenue from contracts with customers

IE206 Consideration is due when control of the products transfers to the customer. Therefore,
the entity has an unconditional right to consideration (i.e. a receivable) for CU150 per
product until the retrospective price reduction applies (i.e. after 1 million products are
shipped).
IE207 In determining the transaction price, the entity concludes at contract inception that the
customer will meet the 1 million products threshold and therefore estimates that the
transaction price is CU125 per product. Consequently, upon the first shipment to the
customer of 100 products the entity recognizes the following:
Receivable CU15,000(a)
Revenue CU12,500(b)
Refund liability (contract liability) CU2,500
(a) CU150 per product × 100 products.
(b) CU125 transaction price per product × 100 products.
IE208 The refund liability (see paragraph 55 of IFRS 15) represents a refund of CU25 per
product, which is expected to be provided to the customer for the volume-based rebate
(i.e. the difference between the CU150 price stated in the contract that the entity has an
unconditional right to receive and the CU125 estimated transaction price).

Example 49—Option that provides the customer with a material right (discount voucher)
IE250 An entity enters into a contract for the sale of Product A for CU100. As part of the contract,
the entity gives the customer a 40 per cent discount voucher for any future purchases up
to CU100 in the next 30 days. The entity intends to offer a 10 per cent discount on all sales
during the next 30 days as part of a seasonal promotion. The 10 per cent discount cannot
be used in addition to the 40 per cent discount voucher.
IE251 Because all customers will receive a 10 per cent discount on purchases during the next 30
days, the only discount that provides the customer with a material right is the discount that
is incremental to that 10 per cent (i.e. the additional 30 per cent discount). The entity
accounts for the promise to provide the incremental discount as a performance obligation
in the contract for the sale of Product A.
IE252 To estimate the stand-alone selling price of the discount voucher in accordance with
paragraph B42 of IFRS 15, the entity estimates an 80 per cent likelihood that a customer
will redeem the voucher and that a customer will, on average, purchase CU50 of additional
products. Consequently, the entity’s estimated stand-alone selling price of the discount
voucher is CU12 (CU50 average purchase price of additional products × 30 per cent
incremental discount × 80 per cent likelihood of exercising the option). The stand-alone
selling prices of Product A and the discount voucher and the resulting allocation of the
CU100 transaction price are as follows:

Performance Obligation Stand-alone selling price CU


Product A 100
Discount voucher 12
Total 112

Allocated transaction price


Product A 89 (CU100 ÷ CU112 × CU100
Discount voucher 11 (CU12 ÷ CU112 × CU100)
Total 100

© Emile Woolf International 505 The Institute of Chartered Accountants of Pakistan


Financial accounting and reporting I

IE253 The entity allocates CU89 to Product A and recognizes revenue for Product A when control
transfers. The entity allocates CU11 to the discount voucher and recognizes revenue for the
voucher when the customer redeems it for goods or services or when it expires.

Example 52—Customer loyalty programme


IE267 An entity has a customer loyalty program that rewards a customer with one customer
loyalty point for every CU10 of purchases. Each point is redeemable for a CU1 discount on
any future purchases of the entity’s products. During a reporting period, customers
purchase products for CU100, 000 and earn 10,000 points that are redeemable for future
purchases. The consideration is fixed and the stand-alone selling price of the purchased
products is CU100, 000. The entity expects 9,500 points to be redeemed. The entity
estimates a stand-alone selling price of CU0.95 per point (totaling CU9, 500) on the basis
of the likelihood of redemption in accordance with paragraph B42 of IFRS 15.
IE268 The points provide a material right to customers that they would not receive without
entering into a contract. Consequently, the entity concludes that the promise to provide
points to the customer is a performance obligation. The entity allocates the transaction
price (CU100,000) to the product and the points on a relative stand-alone selling price basis
as follows:
CU
Product 91,324
[CU100,000 × (CU100,000 stand-alone selling price ÷ CU109,500)]
Points 8,676
[CU100,000 × (CU9,500 stand-alone selling price ÷ CU109,500)]
IE269 at the end of the first reporting period, 4,500 points have been redeemed and the entity
continues to expect 9,500 points to be redeemed in total. The entity recognizes revenue
for the loyalty points of CU4, 110 [(4,500 points ÷ 9,500 points) × CU8, 676] and recognizes
a contract liability of CU4, 566 (CU8, 676 – CU4, 110) for the unredeemed points at the
end of the first reporting period.
IE270 At the end of the second reporting period, 8,500 points have been redeemed cumulatively.
The entity updates its estimate of the points that will be redeemed and now expects that
9,700 points will be redeemed. The entity recognizes revenue for the loyalty points of CU3,
493 {[(8,500 total points redeemed ÷ 9,700 total points expected to be redeemed) × CU8,
676 initial allocations] – CU4, 110 recognized in the first reporting period}. The contract
liability balance is CU1, 073 (CU8, 676 initial allocation – CU7, 603 of cumulative revenue
recognized).

Example 63—Bill-and-hold arrangement


IE323 An entity enters into a contract with a customer on 1 January 20X8 for the sale of a
machine and spare parts. The manufacturing lead time for the machine and spare parts is
two years.
IE324 Upon completion of manufacturing, the entity demonstrates that the machine and spare
parts meet the agreed-upon specifications in the contract. The promises to transfer the
machine and spare parts are distinct and result in two performance obligations that each
will be satisfied at a point in time. On 31 December 20X9, the customer pays for the
machine and spare parts, but only takes physical possession of the machine. Although the
customer inspects and accepts the spare parts, the customer requests that the spare parts
be stored at the entity’s warehouse because of its close proximity to the customer’s factory.
The customer has legal title to the spare parts and the parts can be identified as belonging
to the customer. Furthermore, the entity stores the spare parts in a separate section of its
warehouse and the parts are ready for immediate shipment at the customer’s request. The
entity expects to hold the spare parts for two to four years and the entity does not have the
ability to use the spare parts or direct them to another customer.

© Emile Woolf International 506 The Institute of Chartered Accountants of Pakistan


Chapter 10: IFRS 15: Revenue from contracts with customers

IE325 The entity identifies the promise to provide custodial services as a performance obligation
because it is a service provided to the customer and it is distinct from the machine and
spare parts. Consequently, the entity accounts for three performance obligations in the
contract (the promises to provide the machine, the spare parts and the custodial services).
The transaction price is allocated to the three performance obligations and revenue is
recognized when (or as) control transfers to the customer.
IE326 Control of the machine transfers to the customer on 31 December 20X9 when the
customer takes physical possession. The entity assesses the indicators in paragraph 38 of
IFRS 15 to determine the point in time at which control of the spare parts transfers to the
customer, noting that the entity has received payment, the customer has legal title to the
spare parts and the customer has inspected and accepted the spare parts. In addition, the
entity concludes that all of the criteria in paragraph B81 of IFRS 15 are met, which is
necessary for the entity to recognize revenue in a bill-and-hold arrangement. The entity
recognizes revenue for the spare parts on 31 December 20X9 when control transfers to
the customer.
IE327 The performance obligation to provide custodial services is satisfied over time as the
services are provided. The entity considers whether the payment terms include a significant
financing component in accordance with paragraphs 60–65 of IFRS 15.

4.2 Other Examples

Example 08: Parvez Limited


Question: The following transactions took place at Parvez Limited (PL).
1) On 5 March 2017 PL sold goods to a bank for Rs.18m cash and agreed to repurchase the
goods for Rs.19m cash on 5 July 2017. The goods will be shifted to a storage facility under
bank’s control and security.
2) On 31 March PL’s car manufacturing division consigned several vehicles to independent
dealers for sale to third parties. The sales price to the dealer is PL’s list price at the date
of sale to third parties. If a vehicle is unsold after six months, the dealer has a right to
return the vehicle to PL within next fifteen days.
Required:
Discuss how the above transactions should be accounted for in the books of accounts of Parvez
Limited.
Answer:
(1) Sale and repurchase agreement
The transaction is in the nature of sale and repurchase agreement therefore the economic
phenomenon of the transaction is that of a loan for which the goods have been given as
security. Therefore, no contract of sale of goods or services is identified.
The difference between the sale price of Rs.18m and the repurchase price of Rs.19m
represents the interest on the loan for a period of four months.

To account for the transaction in accordance with its substance:


 The goods should remain in inventories of PL at the lower of cost and net realizable
value.
 No sale should be recorded.
 The amount once received from the bank should be treated as a current loan liability
of Rs.18m.
 Interest should be charged applying implicit rate to profit or loss for each reporting
period.

© Emile Woolf International 507 The Institute of Chartered Accountants of Pakistan


Financial accounting and reporting I

(2) Consignment inventories


There is a contract for sale of cars between Parvez Limited (PL) and dealer containing
confirmation of respective right and obligation, payment term, commercial substance and
probability of collection of price.
There is only one performance obligation, namely, the transfer of cars to the dealer.
As per contract, the transaction price would be list price on the date of sale to third parties
during the six-month period. Thereafter, though not specifically mentioned, after the lapse
of fifteen days the list price applicable on sixteenth day would be the transaction price of
the unsold cars not returned.
Since there is only one performance obligation, the question of allocation of transaction
price does not arise till the time of sale to third parties.
PL will recognize revenue upon satisfaction of performance obligation. Performance
obligation would be satisfied once the dealer has sold any cars to third parties during the six-
month period. Thereafter, if the dealer does not return the unsold cars within fifteen days,
the performance obligation would be considered as satisfied on sixteenth day.
On 31 March 2017 the vehicles should remain in inventories in PL books of accounts.

Example 09: Sachal Limited


Question: Sachal Limited (SL):
a) Sells standard computer software package meant for small and medium sized restaurant
management. This software package is sold:
 at price of Rs.1.5 million payables before delivery,
 with thirty days’ trial time, and
 without any maintenance support after trial time
As per practice, it takes around six months for the customers to use the package independent
of any support from SL. Practically; SL has to provide on-site support service for at least six
months to almost all customers free-of-cost. However, in case of customer’s request for support
beyond six months, SL provides services under a formal paid service contract.
b) Provides maintenance and support for the above standard software package at a price of Rs.0.3
million per annum.
c) Provides designing and development of customized software to customers. Payment is made
monthly by customers on the basis of chargeable hours of developers of SL. First year
maintenance service is provided free-of-cost. Subsequent maintenance service is provided at
the rate of 10% of the total contract price. Thereafter, for next three years’ maintenance service
is provided at 5% of the contract price per annum.
Required:
Explain the considerations to be taken into account in determining accounting for revenue by Sachal
Limited.

Answer:
General considerations for revenue recognition
International Financial Reporting Standard (IFRS 15) provides that the revenue is recognized:
i. when the performance obligation is satisfied by the entity by transferring a promised good or
service (i.e. an asset) to the customer; and
ii. The asset is transferred when the customer obtains the control of that asset.

© Emile Woolf International 508 The Institute of Chartered Accountants of Pakistan


Chapter 10: IFRS 15: Revenue from contracts with customers

Based on this principle, the following is the considerations to be taken into account in determining
accounting for revenue:
(a) Restaurant management software
There exists a contract for sale of Restaurant management software between SL and customers
containing confirmation of respective obligation and right, payment term, commercial
substance and price is collected in advance.
There are two performance obligations, namely:
 Explicit: delivery of software and
 Implicit: six-month on-site support
As per contract, the transaction price is Rs.1.5 million for both performance obligations.
Based on stand-alone selling price approach, software will be priced as Rs.1.35 million and six-
month on-site support services will be priced as Rs.0.15 million.
PL will recognize revenue from sale of software upon delivery if SL can objectively conclude that
the software meets the requirements of the customer. The term of full payment of transaction
price in advance is a reasonable evidence of clarity of specification between SL and customer.
The agreed thirty days’ trial time will be considered as a formality of the contract.
PL will recognize revenue from on-site support services over six months’ period on straight-line
basis.
(b) Maintenance support for the standard software package
Such service is provided under a written contract that contains confirmation of respective
obligation and right, payment term, commercial substance. SL will assess the collectability of
the price if not received in advance.
The performance obligation is to provide maintenance and support services. The price of the
service is Rs.0.3 million for one-year term.

Since there is only one performance obligation, the question of allocation of transaction price
does not arise.
PL will recognize revenue over one-year period on straight-line basis, as in this case input method
is appropriate. The pattern of resources consumed by SL is evenly spread over the period of
contract.
(c) Customized software
Such service is provided under a written contract that contains confirmation of respective
obligation and right, payment term, commercial substance. SL will assess the collectability of
the price.
The performance obligations are:
 Designing and development of customized software, and
 Maintenance and support services of the said software
The total price of the software and maintenance service will be determined on the basis of terms
of contract. The price will be allocated between the two performance obligations. Price of
maintenance services for the first year is included in the total contract price. The allocated price
of the first year would be 10% of the contract price, which is the stand-alone price of the said
services for the second year. For second year it would be 10% of the contract price and for next
three years the maintenance and support services will be priced at 5% of the contract price. The
price of design and development will be 90% of the contract price.
Satisfaction of performance obligation:
Revenue from design and development - PL will recognize revenue from design and development
over time, because the software at every stage is expected to be customer- specific and would
have no alternative use for SL. The monthly payment based on the basis of charged hours
confirms that SL would have an enforceable right to receive payment if the contract is
terminated before completion.

© Emile Woolf International 509 The Institute of Chartered Accountants of Pakistan


Financial accounting and reporting I

Revenue from Maintenance and support services - PL will recognize revenue over five years’
period on straight-line basis, as in this case, input method is appropriate. The pattern of resources
consumed by SL is evenly spread over the period of contract.

Example 10: GALAXY TELECOMMUNICATION


Question:
(a) Define ‘performance obligation’. List any six examples of promised goods and services as per
IFRS 15 ‘Revenue from Contracts with Customers’.
(b) On 1 October 2017, Galaxy Telecommunications (GT) entered into a contract with a bank for
supplying 20 smart phones to the bank staff with unlimited use of mobile network for one
year. The contract price per smart phone is Rs. 34,650 and the price is payable in full within
10 days from the date of contract. At the end of the contract, the phones will not be returned
to GT.
The entire amount received as per contract was credited by GT to advance from customers
account. The smart phones were delivered on 1 November 2017.
If sold separately, GT charges Rs. 18,000 for a smart phone and a monthly fee of Rs. 1,800 for
unlimited use of mobile network.
Required:
Prepare adjusting entry for the year ended 31 December 2017 in accordance with IFRS 15.

Answer:
(a) Performance obligation:
A performance obligation is a promise in a contract with a customer to transfer to the
customer either:
 a good or service (or a bundle of goods or services) that is distinct; or
 A series of distinct goods or services that are substantially the same and that have the
same pattern of transfer to the customer.
Examples of promised goods and services
(i) Goods produced by an entity for sale
(ii) Resale of goods purchased by an entity
(iii) Resale of rights to goods or services purchased by an entity
(iv) Performing a contractually agreed-upon task for a customer
(v) Standing ready to provide goods or services
(vi) Providing a service of arranging for another party to transfer goods or services to the
customer
(vii) Granting rights to goods or services to be provided in the future that a customer can
resell
(viii) Constructing, manufacturing or developing an asset on behalf of a customer
(ix) Granting licenses
(x) Granting options to purchase additional goods/services

© Emile Woolf International 510 The Institute of Chartered Accountants of Pakistan


Chapter 10: IFRS 15: Revenue from contracts with customers

(b)
Debit Credit
Adjusting entry ------ Rupees ------
Advance from customers 378,000
Revenue (Smart phones) (18,000÷*39,600×34,650)×20 315,000
(Network-usage) 63,000
(21,600÷*39,600×34,650)×{20×(2÷12)}
Full price 18,000+1,800×12=39,600

Example 11: JUPITER LIMITED


Question: Jupiter Limited (JL) entered into a two-year contract on 1 January 2017, with a customer
for the maintenance of computer network. JL has offered the following payment options:
Option 1: Immediate payment of Rs. 200,000.
Option 2: Payment of Rs. 110,000 at the end of each year.
The applicable discount rate is 6.596%.
Required:
Prepare journal entries to be recorded in the books of JL under each option over the period of
contract.

Answer:
Option 1: Lump sum payment
Debit Credit
Date Description ---------- Rupees ----------
01-01-17 Cash 200,000
Contract liability 200,000
31-12-17 Interest expense 13,193
Contract liability (200,000×6.596%) 13,193
31-12-17 Contract liability 110,000
Revenue 110,000
31-12-18 Interest expense 6,807
Contract liability 6,807
(200,000+13,192–110,000)×6.596%
31-12-18 Contract liability 110,000
Revenue 110,000

Option 2: Normal payment terms


Debit Credit
Date Description ---------- Rupees ----------
31-12-17 Cash 110,000

Revenue 110,000

31-12-18 Cash 110,000

Revenue 110,000

© Emile Woolf International 511 The Institute of Chartered Accountants of Pakistan


Financial accounting and reporting I

Example 12: PLUTO LIMITED


Question: Pluto Limited (PL) sells industrial chemicals at following standalone prices:

Products Rupees (per carton)


C-1 100,000
C-2 90,000
C-3 110,000

PL regularly sells a carton each of C-2 and C-3 together for Rs. 170,000.
Required:
Calculate the selling price to be allocated to each product, in case PL offers to sell one carton of
each product for a total price of Rs. 260,000.

Answer:
Stand- 1st discount Price after 1st 2nd discount Price
Chemical alone allocation discount allocation after 2nd
price discount
------------------------------------------- Rupees -------------------------------------------
C-1 100,000 - 100,000 3,704 96,296
(100,000×
10,000/270,000)
C-2 90,000 13,500 76,500 2,833 73,667
(90,000×30,000 (76,500×10,000/
/200,000) 270,000)
C-3 110,000 16,500 93,500 3,463 90,037
(110,000×30,000 (93,500×10,000
/200,000) /270,000)
300,000 30,000 270,000 10,000 260,000

Example 13: SINGLE PERFORMANCE OBLIGATION


Question:
i. ECL has entered into a contract with Kashif Builders for construction of a residential project,
including supply of construction material, architectural services, engineering and site
clearance. ECL and its competitors provide such services separately also.
ii. eSolutions Limited, a software developer, entered into a two-year contract with a customer to
provide software license including future software updates and post implementation support
services. The software license would remain functional even if the updates and post
implementation support services are discontinued.
Required:
(a) In view of the requirements of IFRS 15 ‘Revenue from Contracts with Customers’, discuss
whether goods and services provided in each of the above contracts represent a single performance
obligation.
(b) Define the term ‘performance obligation’ and state the criteria which should be met if goods or
services promised to a customer are to be considered as distinct.

© Emile Woolf International 512 The Institute of Chartered Accountants of Pakistan


Chapter 10: IFRS 15: Revenue from contracts with customers

Answer:
(a) Performance obligation
Performance obligation is a promise in a contract with a customer to transfer to the customer
either:
 a good or service (or a bundle of goods or services) that is distinct; or
 A series of distinct goods or services that are substantially the same and that have the
same pattern of transfer to the customer.
A good or service is distinct if both of the following criteria are met:
 the customer can benefit from the good or service either on its own or together with other
resources that are readily available to the customer; and
 the entity’s promise to transfer the good or service is separately identifiable from other
promises in the contract

(b)
i. The different services being performed under the contract are separately identifiable but
the customer cannot benefit from services separately from the other.
Based on this, ECL should account for services in the contract as a single performance
obligation.
ii. Transfer of software license, software updates and support services are distinct. However,
the software license is delivered before the other services and remains functional without
updates and technical support. Further, the customer can benefit from each of the
services either on their own or together with other services that are readily available. Thus,
the entity’s promise to transfer the good or service is separately identifiable from other
promises in the contract.
Based on the above, the contract should not be accounted for as a single performance
obligation.

Example 14: Car World


Question: Car World sells new cars on deferred payment basis whereby 40% deposit is received on
sale and the balance payment is received at the end of two years. The appropriate discount rate is
10%.
On 1 July 20X4 a car was sold to a customer for Rs. 2,000,000.
Required: Prepare necessary journal entries to record the above transaction in the books of Car
World for the years ended 30 June 2015 and 30 June 2016.
Answer:
Debit (Rs.) Credit (Rs.)
Date Particulars --------- Rupees ---------
1/7/2014 Cash (40%×2,000,000) 800,000
Accounts receivable (1,791,735-800,000) 991,735
Revenue [800,000 + 991,736(W-1)] 1,791,736
30/6/15 Accounts receivable (10% × 991,736) 99,174
Interest income 99,174
30/6/16 Accounts receivable (208,264 – 99,174) 109,091
Interest income 109,091
30/6/16 Cash 1,200,000
Account Receivables 1,200,000

© Emile Woolf International 513 The Institute of Chartered Accountants of Pakistan


Financial accounting and reporting I

W-1: Present value of future payments Rupees


Amount receivable (991,735+99,174+109,091) 1,200,000
PV=1,200,000 (1+10%)2 991,735

Example 15: Saleem Engineering (SE)


Question: Saleem Engineering (SE) is a supplier of various types of industrial machines. It also
provides services for the maintenance of these machines.
Following transactions were carried out by SE during the year ended 30 June 2016:
i. Five machines were sold on a lay away basis to one of its frequent customers. Three out of
a total of five installments had been received till the year end.
ii. A service contract for maintenance of a machine for a period of one year was signed and
SE received a non-refundable annual fee amounting to Rs. 45,000 as advance on 15 April
2016.
Required:
Discuss when it will be appropriate for SE to recognise revenue in each of the above situations.
Answer:
Part (i)
Revenue from law away sales is recognised when the goods are delivered against full
payment. However, if the SE’s historical experience (i-e. one of its frequent customers) shows
that most lay away transactions are converted into sales, then it can recognise revenue when
it receives a significant deposit, provided that the goods are on hand, identified and r eady for
delivery. Since the customer has paid significant part of installments (3 out of 5 installments), the
five industrial machines are on hand, identifiable and ready for delivery, the revenue is recognized
in full.
Part (ii)
Even though non-refundable annual fee amounting to Rs.45,000 in advance relates to the
maintenance service of machinery that the SE is required to undertake to fulfil the contract, the
advance payment does not result in the transfer of a promised maintenance service to the
customer. Instead, it is an administrative task. Therefore, the non-refundable annual fee is an
advance payment for performance obligations to be satisfied in the future and is recognised as
revenue when those future maintenance services are provided.

Example 16: RECOGNITION OF REVENUE


Question:
(i) Karim Industries Limited (KIL) has sold a machine on credit to Yawar Engineering (YE). The
machine would be used by YE if it is able to secure a contract for providing services to
AMZ & Company. KIL has agreed that the machine may be returned at 90% of the price, if
YE fail to secure the contract.
(ii) Asif Electronics (AE) is about to sell a new type of food factory. Since customer demand is
high, AE is taking advance against orders. The selling price has been fixed at Rs. 7,000 per
unit and so far 175 customers have paid the initial 25% deposit which is non-refundable.
(iii) Nazir Engineering Limited (NEL) entered into a contract for the provision of services over a
period of two years. The total contract price was Rs. 25 million and NEL had initially
expected to earn a profit of Rs. 5 million on the contract. However, the contract had not
progressed as expected. In the first year, costs of Rs. 12 million were incurred.
Management is not sure of the ultimate outcome but believes that at least the costs on
the contract would be recovered from the customer.
Required: State how revenue should be recognised in the above cases.

© Emile Woolf International 514 The Institute of Chartered Accountants of Pakistan


Chapter 10: IFRS 15: Revenue from contracts with customers

Answer:
(i) The completion of the sale transaction is uncertain because it is contingent upon
purchaser(YE) securing the contract with another company (AMZ & Company). Therefore,
KIL should not recognize any revenue since YE has not obtained the control of machine.
(ii) Revenue should be recognized when the food factory is delivered to the customer. Until
then no revenue should be recognized and the 25% deposit should be treated as a
contract liability.
(iii) If the outcome of a service transaction cannot be estimated reliably, revenue should only
be recognized to the extent that expenses incurred are recoverable from the customer.
Thus revenue to the extent of Rs. 12 million may be recognized in the first year.

Example 17: LEGAL TITLE


Question: In respect of sale of goods, give any two examples of each of the following situations:
(i) Legal title passes but the risks and rewards are retained.
(ii) Legal title does not pass but the risks and rewards are passed on to the customer.
Answer:
(i) Examples of the situations where legal title passes but risk and rewards are retained
 An entity may retain obligations for unsatisfactory performance not covered by normal
warranty provisions;
 The receipt of revenue may be contingent on derivation of revenue by the buyer for its
sale of goods.
(ii) Examples of the situations where legal title does not pass but the risks and rewards are
transferred
 A seller may retain the legal title to the goods to protect the collectability of the
amount due but transfer the significant risks and rewards of ownership.
 In retail sale, a seller may offer a refund if the customer is not satisfied.

Example 18: BRILLIANT LIMITED


Question: Brilliant Limited (BL) manufactures and sells plastic card printing machines with
laminators. A machine-specific card printing software is provided as a must part of the printing
machine. BL also sells plastic cards imported from Thailand.
BL agreed to supply the following to, Proud Learners (PL), a country-wide school network:
 15 Card printing machines – Available in ready stock
 8 Laminators – Would require 30 days to deliver
 100,000 Plastic cards – Available in ready stock
A lump sum price of Rs.9.2 million for the total contract has been agreed between BL and school
network.
Cost and list prices of the goods are:
Item Price (Rs.) Cost (Rs.)
Card printing machines 800,000 400,000
Laminators 200,000
Plastic cards 12 5

BL does not sell printing machine without laminator. However, in order to get this order x`BL went
against its policy. There is another supplier of imported card printing machine of almost similar
specification. This supplier sells the machine at Rs. 750,000.
In most recent customers’ surveys printing machine of BL has been given 7 out of 10 points as against
9 out of 10 given to competitors’ imported machine. There is no supplier of laminator in the market.

© Emile Woolf International 515 The Institute of Chartered Accountants of Pakistan


Financial accounting and reporting I

Required:
Identify the performance obligations and allocate the transaction price to the identified
performance obligations.
Answer:
Identification of performance obligations: There are three performance obligations:
1. Transfer of 15 Plastic card printing machines and its software
2. Transfer of 8 Laminators
3. Transfer of 100,000 plastic cards
Although the software is distinct from printing machine, but both are highly dependable to each
other and inter-related. In the context of this contract, these are providing a combined output to PL.
Therefore, software is not a separate performance obligation.
The total transaction price as per the contract is Rs.9.2 million.
On the basis of available information, the stand-alone prices of each item will be estimated using
the following approaches:
Plastic card printing machines and its software: In the absence of observable stand-alone price,
we may use ‘adjusted market assessment’ approach. The competitor’s machine is sold at Rs.
750,000 which is similar (not identical) to BL’s machine. As per given information, we may use
customers’ rating for adjustment of competitors’ price that worked out as follows:
RUPEES
Competitors’ price 750,000
Adjusted price of BL machine (7/9*750,000) 583,000
Total price (15*583,000) 8,745,000
Laminators: There is neither observable stand-alone price nor any comparable competitors’
product available in the market in which BL operates. In this case, we may use ‘expected cost plus
a margin approach’. The estimated stand-alone price is worked out as follows:
Rupees
Expected cost to BL 200,000
Margin estimated (800,000 - 600,000)/600,000 = 33% 66,000
266,000
Total price (8*266,000) Plastic cards: 2,128,000
Observable stand-alone price is available Total price (100,000*12) 1,200,000
Total of stand-alone prices is:
Plastic card printing machines and its software 8,745,000
Laminators 2,128,000
Plastic cards 1,200,000
Total 12,073,000
Allocation of Rs.9.2 million (transaction price) will be based on relative stand-alone prices, as the
difference of Rs.2.873 million between stand-alone price and transaction price is not specific to
any performance obligation.
Plastic card printing machines and its software 6,663,961
(9,200,000*8,745,000/12,073,000)
Laminators (9,200,000*2,128,000/12,073,000) 1,621,602
Plastic cards (9,200,000*1,200,000/12,073,000) 914, 437
Total 9,200,000

© Emile Woolf International 516 The Institute of Chartered Accountants of Pakistan


Chapter 10: IFRS 15: Revenue from contracts with customers

Example 19: Thursday Enterprises


Question: Thursday Enterprise (TE) is a supplier of product Zee and has provided you the following
information:
(a) On 1 August 2018, TE entered into a six months contract with customer Alpha for sale of
Zee for Rs. 250 per unit, under the following terms and conditions:
 if Alpha purchases more than 5,000 units during the contract period, the price per
unit would be retrospectively reduced to Rs. 215 per unit.
 TE’s unconditional right to receive consideration would be established upon:
- completion of quality control procedures by Alpha for the first order. The
procedure would take a week after receiving the goods.
- placement of order by Alpha for subsequent orders.
At the inception of the contract, TE concludes that Alpha’s purchases will not exceed the
5,000 units threshold for the discount.
Alpha placed the following orders:

Delivery date
Order date Units Payment date
(Transfer of control)

10 August 2018 3,000 28 August 2018 12 September 2018

25 December 2018 4,000 15 January 2019 10 January 2019

(b) On 1 February 2019, TE entered into a six months contract with another customer Beta for
sale of Zee for Rs. 250 per unit, under the following terms and conditions:
 if the Beta purchases more than 15,000 units during the contract period, the price
per unit would be retrospectively reduced to Rs. 215 per unit.
 TE’s unconditional right to receive consideration would be established upon
delivery of goods to Beta.
At the inception of the contract, TE concludes that Beta will meet 15,000 units threshold
for the discount.
Beta placed the following orders:

Delivery date (Transfer


Order date Units Payment date
of control)

14 February 2019 10,000 28 February 2019 20 March 2019

1 June 2019 8,000 15 July 2019 18 July 2019

Required:
In respect of the above contracts, prepare journal entries to be recorded in the books of TE for the
years ended 31 December 2018 and 2019. (Entries without date will not be awarded any marks)

© Emile Woolf International 517 The Institute of Chartered Accountants of Pakistan


Financial accounting and reporting I

Answer:
(a) General Journal
Debit Credit
Date Description
---------- Rupees ----------
28-08-18 Contract asset – Alpha (3,000×250) 750,000
Revenues 750,000

05-09-18 Receivable - Alpha 750,000


Contract asset - Alpha 750,000

12-09-18 Cash/Bank 750,000


Receivable - Alpha 750,000

25-12-18 Revenue (3,000×35) 105,000


Contract liability / Refund liability – Alpha 105,000

25-12-18 Receivable (4,000×215)–105,000 755,000


Contract liability / Refund liability – Alpha 755,000

10-01-19 Cash/Bank 755,000


Receivable - Alpha 755,000

15-01-19 Contract liability / Refund liability – Alpha 860,000


Revenue (755,000+105,000) 860,000

(b) General Journal


Debit Credit
Date Description
---------- Rupees ----------
28-02-19 Receivable – Beta (10,000×250) 2,500,000
Revenue (10,000×215) 2,150,000
Contract liability / Refund liability – Beta 350,000

20-03-19 Cash/Bank 2,500,000


Receivable – Beta 2,500,000

15-07-19 Receivable – Beta 1,370,000


Contract liability / Refund liability – Beta 350,000
Revenue 1,720,000

18-07-19 Cash/Bank 1,370,000


Receivable – Beta 1,370,000
.

© Emile Woolf International 518 The Institute of Chartered Accountants of Pakistan


Chapter 10: IFRS 15: Revenue from contracts with customers

Example 20: Trich Mir Limited


Question: Financial statements of Trich Mir Limited (TML) for the year ended 31 December
2019 are under preparation. While reviewing revenues from contract with customers,
following matters have been identified:
(i) On 1 October 2019, TML sold Machine C to Chan Limited for Rs. 25 million. As per the
contract, payment would be made after 2 years. The accountant recognised sales
revenue of Rs. 25 million upon delivery on 1 October 2019. Further, commission paid
to sales employees for winning the contract of Rs. 1.6 million was capitalised and is
being amortised over 2 years period. Applicable discount rate is 10% per annum.

(ii) TML entered into a contract to manufacture a specialised machine for Dhan Limited at
a price of Rs. 30 million. The contract meets the criteria of recognition of revenue over
time. At the year end, the machine was 60% complete and it was estimated that a
further cost of Rs. 10 million would be incurred. Cost of Rs. 15 million incurred till year
end has been included in closing inventory and receipts of Rs. 11 million have been
credited to revenues.

(iii) TML entered into a contract to sell one unit of Machine A and Machine B for a total
price of Rs. 16 million. Machine A was delivered in December 2019 to the customer
while Machine B was delivered in January 2020. The consideration of Rs. 16 million is
due only after TML transfers both the machines to the customer. TML sells machines A
and B at standalone prices of Rs. 12 million and Rs. 8 million respectively. The
accountant recognised receivable and revenue of Rs. 12 million upon delivery of
Machine A.
Required:
Prepare correcting entries for the year ended 31 December 2019 in accordance with IFRS
15 ‘Revenue from Contracts with Customers’.

Answer:
Trich Mir Limited
Correcting entries for the year ended 31 December 2019
Debit Credit
S.No. Description
---- Rs. in million ----
(i) Revenues 25–20.66{25×(1.1)–2} 4.34
Receivable 4.34

Receivable 20.66×10%×(3÷12) 0.52


Interest income 0.52

Commission expense 1.60


Amortization expense 1.6÷2×3÷12 0.20
Contract cost 1.40

(ii) Cost of goods sold 15.00


Inventories 15.00

Receivable (30×60%)–11 7.00


Construction revenues 7.00

© Emile Woolf International 519 The Institute of Chartered Accountants of Pakistan


Financial accounting and reporting I

(iii) Revenues 12–12×16÷(12+8) 2.40


Receivable 2.40

Contract asset (12–2.4) 9.60


Receivable 9.60

Example 21: Stupa Limited (SL)


Question:
(a) Stupa Limited (SL) sells electrical products at following standalone prices:

Products Rupees

E-1 30,000

E-2 30,000

E-3 50,000

Required:
Calculate transaction price to be allocated to each product under each of the following
independent situations:
(i) SL offered to sell one unit of each of the above products for Rs. 90,000. SL regularly
sells one unit each of E-2 and E-3 together for Rs. 70,000.
(ii) SL offered to sell one unit of E-1 and two units of E-3 for Rs. 104,000.
(b) On 1 October 2018, Kushan Construction Limited (KCL) entered into a contract to construct a
commercial building for a customer for Rs. 50 million and a bonus of Rs. 10 million if the
building is completed on or before 31 December 2019.
Till 30 June 2019, KCL expected that the building will be completed within time at a total cost
of Rs. 40 million. However, due to bad weather and time involved in regulatory approvals, the
building was completed on 28 February 2020 at a total cost of Rs. 42 million of which Rs. 26
million was incurred till 30 June 2019.
Required:
Compute profit to be recognized for the years ended 30 June 2019 and 2020, if:
(i) performance obligation under the contract is satisfied over time.
(ii) performance obligation under the contract is satisfied at a point in time.
(c) The nature, timing and amount of consideration promised by a customer affect the estimate
of the transaction price.
Define the term ‘transaction price’ and list down the factors that may affect determination of
the transaction price.

© Emile Woolf International 520 The Institute of Chartered Accountants of Pakistan


Chapter 10: IFRS 15: Revenue from contracts with customers

Answer:
Part (a) (i) Allocation of transaction price
Standalone Allocation of Rs. 70,000 to Transaction
Allocation of Rs. 90,000
Product price E-2 and E-3 price
-------------------------------------- Rs. --------------------------------------
E-1 30,000 30,000 (30,000×90,000÷100,000) 27,000
E-2 30,000 (30,000×70,000÷80,000) 26,250 (26,250×90,000÷100,000) 23,625
E-3 50,000 (50,000×70,000÷80,000) 43,750 (43,750×90,000÷100,000) 39,375
110,000 100,000 90,000

Part (a) (ii)


Standalone price Allocation of Rs. 104,000 Transaction price
Product
-------------------------------------- Rs. --------------------------------------
E-1 30,000 (30,000×104,000÷130,000) 24,000
E-3 (50,000×2) 100,000 (100,000×104,000÷130,000) ((40,000×2) 80,000
130,000 104,000

Part (b) (i) Computation of profit – performance obligation satisfied ‘over time’:
2019 2020
Completion % 65%
(26÷40×100) 100%
----------------- Rs. -----------------
Revenue 39.0 11.0
(50+10)×65% (50–39)
Cost (16.0)
(26.0)
(42–26)
Profit/(loss) 13.0 (5.0)

Part (b) (ii) Computation of profit – performance obligation satisfied ‘at a point in time’:
2019 2020
----------------- Rs. -----------------
Revenue - 50.0
Cost - (42.0)
Profit - 8.0

Part (c)
Transaction price:
The transaction price is the amount of consideration to which an entity expects to be entitled in
exchange for transferring promised goods or services to a customer, excluding amounts to be
collected on behalf of third parties.

Factors affecting determination of the transaction price:


(i) Variable consideration
(ii) Constraints on variable consideration
(iii) Existence of a significant financing components (time value of money)
(iv) Non-cash consideration
(v) Consideration payable to a customer

© Emile Woolf International 521 The Institute of Chartered Accountants of Pakistan


Financial accounting and reporting I

5 OBJECTIVE BASED QUESTIONS


01. Which of the following is not one of the 5 steps for recognizing revenue according to IFRS 15
Revenue from contracts with customers?
(a) Identify the contract
(b) Assess the likelihood of economic benefits
(c) Determine the contract price
(d) Allocate the transaction price to the performance obligations in the contract.

02. Whale Limited (WL) is an agent who works on behalf of Dolphin, a famous performer. WL has
just collected Rs. 100 million from a promoter in terms of ticket sales for a recent show done
by Dolphin. WL earns commission of 10% in relation to Dolphin's work.
What is the correct double entry for the receipt of the Rs? 100 million?
(a) Dr Cash Rs. 100 million
Dr Trade Receivables Rs. 10 million
Cr Trade payables Rs. 100 million
Cr Revenue Rs. 10 million
(b) Dr Cash Rs. 100 million
Dr COS Rs. 90 million
Cr Revenue Rs. 100 million
Cr Trade payables Rs. 90 million
(c) Dr COS Rs. 90 million
Dr Cash Rs. 10 million
Cr Revenue Rs. 100 million
(d) Dr Cash Rs. 100 million
Cr Revenue Rs. 10 million
Cr Trade payables Rs. 90 million

03. Coin Limited (CL) sells a specialized piece of equipment to Orbit Limited on 1st September
2017 for Rs. 4m. Due to the specialized nature of the equipment, CL has additionally agreed
to provide a support service for the next two years. The cost per annum to CL of providing this
service will be Rs. 300,000. CL usually earns a gross margin of 20% on such contracts.
What revenue should be included in the statement of profit or loss of CL for the year ended 31
December 2017?
(a) Rs. 3,343,750
(b) Rs. 3,250,000
(c) Rs. 3,375,000
(d) Rs. 4,000,000

04. River Limited (RL) has prepared its draft financial statements for the year ended 30 September
2014. It has included the following transactions in revenue at the amounts stated below.
Which of these has been correctly included in revenue according to IFRS 15 Revenue from
Contracts with Customers?

© Emile Woolf International 522 The Institute of Chartered Accountants of Pakistan


Chapter 10: IFRS 15: Revenue from contracts with customers

(a) Agency sales of Rs. 2.5 million on which RL is entitled to a commission of 10%.
(b) Sale proceeds of Rs. 20 million for motor vehicles which were no longer required by RL
(c) Sales of Rs. 15 million on 30 September 2014. The amount invoiced to and received
from the customer was Rs. 18 million, which includes Rs. 3 million for ongoing servicing
work to be done by RL over the next two years.
(d) Sales of Rs. 20 million on 1 October 2013 to an established customer who (with the
agreement of RL) will make full payment on 30 September 2015. RL has a cost of capital
of 10%.

05. Cat Limited (CL) sold and installed an item of machinery for Rs. 800,000 on 1 November 2017.
Included within the price was 2 years servicing contract which has a value of Rs. 240,000 and
a fee for installation of Rs. 50,000.
How much should be recorded in CL’s revenue in its statement of profit or loss for the year
ended 31 December 2017 in relation to the machinery sale?
(a) Rs. 530,000
(b) Rs. 680,000
(c) Rs. 560,000
(d) Rs. 580,000

06. Sales director of a company is close to selling a machine which it sells for Rs. 650,000, offering
free service, therefore selling the entire machine for Rs. 560,000 including installation. The
company never sells servicing separately.
How should this discount be applied in relation to the sale of the machinery?
(a) Machine only
(b) Machine and Installation only
(c) Machine and Service only
(d) Machine, Installation and Service

07. Cheetah Limited (CL) works as an agent for a number of smaller contractors, earning
commission of 10%. CL’s revenue includes Rs. 6 million received from clients under these
agreements with Rs. 5.4 million in cost of sales representing the amount paid to the contractors.
What adjustment needs to be made to revenue in respect of the commission sales?
(a) Reduce revenue by Rs. 6 million
(b) Reduce revenue by Rs. 5.4 million
(c) Increase revenue by Rs. 600,000
(d) No adjustment is required

08. An entity regularly sells Products A, B and C individually, thereby establishing the following
stand-alone selling prices:

Product Stand-alone selling price Rs.

Product A 40

Product B 55

Product C 45

In addition, the entity regularly sells Products B and C together for Rs. 60.

© Emile Woolf International 523 The Institute of Chartered Accountants of Pakistan


Financial accounting and reporting I

The entity enters into a contract with a customer to sell Products A, B and C in exchange for
Rs. 100.
Allocate the transaction price of Rs. 100 to Product A, B and C in accordance with IFRS 15
(a) A Rs. 40 and B Rs. 55 and C Rs. 45
(b) A Rs. 29 and B Rs. 39 and C Rs. 32
(c) A Rs. 40 and B Rs. 33 and C Rs. 27
(d) A Rs. 40 and B Rs. 27 and C Rs. 33

09. An entity enters into a contract with a customer to sell Products A, B and C in exchange for Rs.
100.
Product Stand-alone selling price
Product A 50
Product B 25
Product C 75
Total 150
Allocate the transaction price of Rs. 100 to Product A, B and C in accordance with IFRS 15
(a) A Rs. 50 and B Rs. 25 and C Rs. 75
(b) A Rs. 33 and B Rs. 17 and C Rs. 50
(c) A Rs. 33 and B Rs. 50 and C Rs. 17
(d) A Rs. 17 and B Rs. 33 and C Rs. 50

10. Which of the following items has correctly been included in Hakeem Limited (HL)’s revenue for
the year to 31 December 2011?
(a) Rs. 2 million in relation to a fee negotiated for an advertising contract for one of HL’s
clients. HL acted as an agent during the deal and is entitled to 10% commission.
(b) Rs. 500,000 relating to a sale of specialized equipment on 31 December 2011. The full
sales value was Rs. 700,000 but Rs. 200,000 relates to servicing that HL will provide
over the next 2 years, so HL has not included that in revenue this year.
(c) Rs. 800,000 relating to a sale of some surplus land owned by HL.
(d) Rs. 1 million in relation to a sale to a new customer on 31 December 2011. Control
passed to the customer on 31 December 2011. The Rs. 1 million is payable on 31
December 2013. Interest rates are 10%.

11. Hover Limited (HL) is a car retailer. On 1 April 2014, HL sold a car to a customer on the following
terms:
The selling price of the car was Rs. 25.3 million. The customer paid Rs. 12.65 million (half of
the cost) on 1 April 2014 and will pay the remaining Rs. 12.65 million on 31 March 2016 (two
years after the sale). The customer can obtain finance at 10% per annum.
What is the total amount which HL should credit to profit or loss in respect of this transaction in
the year ended 31 March 2015?
(a) Rs. 23.105 million
(b) Rs. 23.000 million
(c) Rs. 20.909 million
(d) Rs. 24.150 million

© Emile Woolf International 524 The Institute of Chartered Accountants of Pakistan


Chapter 10: IFRS 15: Revenue from contracts with customers

12. Determining the amount to be recognized in the first year of a long term contract with a
customer is an example of which step in the IFRS 15’s 5-step model?
(a) Determining the transaction price
(b) Recognizing revenue when a performance obligation is satisfied
(c) Identifying the separate performance obligations
(d) Allocating the transaction price to the performance obligations

13. X Limited wins a competitive bid to provide consulting services to a new customer. X Limited
incurred the following costs to obtain the contract:
Rs.
Commissions to sales employees for winning the contract 10,000
External legal fees for due diligence 15,000
Travel costs to deliver proposal 25,000
Total costs incurred 50,000
How to recognize the above costs?
(a) Capitalize Rs. Nil and expense Rs. 50,000
(b) Capitalize Rs. 10,000 and expense Rs. 40,000
(c) Capitalize Rs. 25,000 and expense Rs. 25,000
(d) Capitalize Rs. 50,000 and expense Rs. Nil

14. On 1 January 2019, an entity enters into a non-cancellable contract to transfer a product to a
customer on 31 March 2019. The contract requires the customer to pay consideration of Rs.
1,000 in advance on 31 January 2019 but the customer pays the consideration on 1 March
2019. The entity transfers the product on 31 March 2019.
What journal entry is required to be passed on 31 January 2019?
(a) No entry is required
(b) Debit Cash Rs. 1,000 and Credit Contract liability Rs. 1,000
(c) Debit Receivables Rs. 1,000 and Credit Contract liability Rs. 1,000
(d) Debit Receivables Rs. 1,000 and Credit Revenue Rs. 1,000

15. An entity enters into 100 contracts on 31 December 2017 with customers. Each contract
includes the sale of one product for Rs.100.
Cash is received when control of a product transfers. The entity’s customary business practice
is to allow a customer to return any unused product within 30 days and receive a full refund.
The entity’s cost of each product is Rs. 60.
Using the expected value method, the entity estimates that 97 products will not be returned.
The entity estimates that the costs of recovering the products will be immaterial and expects
that the returned products can be resold at a profit.
What should be recognized in respect of above?
(a) Revenue Rs. Nil and Contract Liability Rs. 10,000
(b) Revenue Rs. 300 and Contract Liability Rs. 9,700
(c) Revenue Rs. 9,700 and Contract Liability Rs. 300
(d) Revenue Rs. 10,000 and Contract Liability Rs. Nil

© Emile Woolf International 525 The Institute of Chartered Accountants of Pakistan


Financial accounting and reporting I

16. Mechanical Limited (ML) sells machines, and also offers installation and technical support
services. The individual selling prices of each product are shown below.
Sale price of goods Rs. 75,000
Installation Rs. 30,000
One-year service Rs. 45,000
ML sold a machine on 1 May 2011, charging a reduced price of Rs. 100,000 including
installation and one year’s service. ML only offers discounts when customers purchase a
package of products together.
According to IFRS 15 Revenue from Contracts with Customers, how much should ML record
in revenue for the year ended 31 December 2011?
Rs. ___________

17. Car Limited (CL) sold a large number of vehicles spare parts to a new customer for Rs. 10
million on 1 July 2017. The customer paid Rs. 990,000 up front and agreed to pay the remaining
balance on 1 July 2018. CL has a cost of capital of 6%.
How much should initially be recorded in revenue in respect of the sale of vehicles spare parts
in the statement of profit or loss for the year ended 31 December 2017?
Rs. __________

18. Golden Limited enters into a contract with a major chain of retail stores. The customer commits
to buy at least Rs.20m of products over the next 12 months. The terms of the contract require
Golden Limited to make a payment of Rs.1 m to compensate the customer for changes that it
will need to make to its retail stores to accommodate the products.
By the 31 December 2011, Golden Limited has transferred products with a sales value of Rs.4m
to the customer.
How much revenue should be recognized by Golden Limited in the year ended 31 December
2011?
Rs. ___________

19. Silver Limited sells a machine and one year’s free technical support for Rs. 100,000. It usually
sells the machine for Rs. 95,000 but does not sell technical support for this machine as a
standalone product. Other support services offered by Silver Limited attract a markup of 50%.
It is expected that the technical support will cost Silver Limited Rs. 20,000.
How much of the transaction price should be allocated to the technical support?
Rs. ___________

20. Jupiter Limited (JL) entered into a two-year contract on 1 January 2017, with a customer for
the maintenance of computer network. JL has offered the following payment options:
Option 1: Immediate payment of Rs. 200,000.
Option 2: Payment of Rs. 110,000 at the end of each year.
The applicable discount rate is 6.596%.
What amount of revenue should be recognized under option 2 on 31 December 2017?
Rs. ___________

© Emile Woolf International 526 The Institute of Chartered Accountants of Pakistan


Chapter 10: IFRS 15: Revenue from contracts with customers

21. Which two standards have been replaced by IFRS 15 Revenue from Contracts with
Customers?
(a) IAS 20 Government Grants and IAS 36 Impairment of Assets
(b) IAS 36 Impairment of Assets and IAS 11 Construction Contracts
(c) IAS 18 Revenue and IAS 20 Government Grants
(d) IAS 18 Revenue and IAS 11 Construction Contracts

22. The accounting principle applied by IFRS 15 when determining whether or not revenue should
be recognized in respect of a repurchase agreement is:
(a) Prudence
(b) Relevance
(c) Substance over form
(d) Verifiability

23. With regard to the definition of revenue given by IFRS 15, which of the following statements is
true?
(a) Revenue includes cash received from share issues
(b) Revenue includes cash received from borrowings
(c) Revenue may arise from either ordinary activities or extraordinary activities
(d) Revenue arises from ordinary activities only

24. Identifying contract with customer under IFRS 15, a contract with customer exist when all the
following criteria are met when;
(a) It is approved and enforceable, can identify each party rights, payment terms, and
probable to collect consideration
(b) It is approved, can identify each party rights, payment terms, has commercial substance
and probable to collect consideration
(c) It is approved and enforceable, can identify each party rights, has commercial substance
and probable to collect consideration
(d) It is approved, can identify payment terms, has commercial substance and probable to
collect consideration

25. Step 1, “identifying the contract” of IFRS 15 states that certain conditions must be satisfied
before an entity can account for a contract with a customer. Which of the following is not one
of these conditions?
(a) Each party's rights with regard to the goods or services concerned can be identified
(b) The payment terms can be identified
(c) The entity and the customer have approved the contract and are committed to perform
their contractual obligations
(d) It is certain that the entity will collect the consideration to which it is entitled

26. Step two requires the identification of the separate performance obligations in the contract. This
is often referred to as unbundling and is done at beginning of a contract. What is the key factor
in identifying a separate performance obligation?
(a) The passing of the risks and rewards to the customer
(b) The distinctiveness of the good or service
(c) The identification of the payment terms
(d) The enforceability of the contract

© Emile Woolf International 527 The Institute of Chartered Accountants of Pakistan


Financial accounting and reporting I

27. Step three requires the entity to determine the transaction price. This is the amount of
consideration that an entity expects to be entitled to in exchange for the promised goods or
services. The transaction price might include variable or contingent consideration. How does
the entity estimate the amount of the variable consideration?
(a) The expected value or the most likely amount whichever best predicts the consideration
(b) The lower of the expected value or the most likely amount
(c) The choice of the expected value or the most likely amount
(d) The higher of the expected value or the most likely amount

28. Step 4 requires the allocation of the transaction price to separate performance obligations. The
allocation is based on the relative standalone selling prices of the goods or services promised
and are made at inception of the contract. It is not adjusted to reflect subsequent changes in
the standalone selling prices of those goods or services. What is the best evidence of
standalone selling price?
(a) An estimate that maximizes the use of observable inputs
(b) The observable price of a good or service when the entity sells that good or service
separately
(c) Unadjusted market prices for similar goods or services
(d) Expected cost

29. Step 5 allows an entity to recognize revenue when (or as) each performance obligation is
satisfied. Revenue is recognized in line with the pattern of transfer. If an entity does not satisfy
its performance obligation over time, it satisfies it at a point in time and revenue will be
recognized when control is passed at that point in time. Which of the following factors may not
indicate the passing of control?
(a) The present right to payment for the asset
(b) The customer has legal title to the asset
(c) The entity has physical possession but has transferred a portion of the economic risks
(d) The entity has transferred physical possession of the asset

30. Which of the following is true regarding discounts offered on a bundle of products/services?
(a) The discount should be applied across each performance obligation in the contract
(b) The discount should be recorded within cost of sales
(c) The discount should be applied to the largest component of the contract
(d) The discount should be recorded as an administrative cost

31. An entity can only include variable consideration in the transaction price to the extent that it is
highly probable that a subsequent change in the estimated variable consideration will not result
in a significant revenue reversal. What action should the entity take if it is not appropriate to
include all of the variable consideration in the transaction price?
(a) The entity should not include any of the variable consideration
(b) The entity can use its judgment in all matters such as this
(c) The entity should assess whether it should include part of the variable consideration
subject to the revenue reversal test
(d) The entity should assess whether it should include part of the variable consideration
without the need to use the revenue reversal test

© Emile Woolf International 528 The Institute of Chartered Accountants of Pakistan


Chapter 10: IFRS 15: Revenue from contracts with customers

32. Which one of the following condition is not allow when performance condition to be satisfied
over time?
(a) the customer simultaneously receives and consumes the benefits provided by the entity’s
performance as the entity performs
(b) the entity’s performance creates or enhances an asset that the customer controls as the
asset is created or enhance
(c) they customer has paid the consideration in advance and goods / services are still to be
received
(d) the entity’s performance does not create an asset with an alternative use to the entity

33. In general, contract costs incurred in relation to a contract with a customer must be:
(a) Recognized as an expense when incurred
(b) Recognized as an asset if they relate to a performance obligation which has been
satisfied
(c) Recognized as an asset if they are not expected to be recovered
(d) Recognized as an asset if they relate to a performance obligation which has not yet been
satisfied

34. A company enters into a construction contract to build a warehouse for a customer. The agreed
price is Rs.20 million and the specified completion date is 31 October 2020. However, the
contract provides that the company should receive an incentive payment of a further Rs.2.5
million if the warehouse is completed before 30 June 2020. Similarly, the price will be reduced
by Rs. 2 million if the warehouse is not completed until after 31 December 2020.
The company estimates that there is a 15% probability that the warehouse will be completed
before 30 June 2020, an 80% probability that it will be completed by 31 October 2020 and a
5% probability that it will not be completed until after 31 December 2020.
What is the expected value of the transaction price for this contract?
(a) Rs. 20 million
(b) Rs. 20.275 million
(c) Rs.20.5 million
(d) Rs.20.75 million

© Emile Woolf International 529 The Institute of Chartered Accountants of Pakistan


Financial accounting and reporting I

5 OBJECTIVE BASED ANSWERS


01. (b) Assessing the likelihood of economic benefits is not one of the five steps.
It is one of the criteria for identifying the contract with customer.

02. (d) As an agent, WL should only record the commission of Rs. 10 million in
revenue. As the cash has been received, WL must record that in cash and
create a payable for Rs. 90 million to Dolphin.

03. (c) There are two performance obligations here. The sale of the equipment
should be recognizing at a point in time, and the revenue in relation to the
support should be recognized over time. The services element costs Rs.
300,000 a year.
As CL makes a margin of 20% a year, this would be sold for Rs. 375,000
per year (300,000 × 100/80). Therefore, the total revenue on the service
for 2 years = Rs. 375,000 × 2 = Rs. 750,000.
The revenue on the goods = Rs.4m – Rs. 750,000 = Rs. 3,250,000.
The revenue in relation to the service is released over 2 years.
By 31 December, 4 months of the service has been performed so can be
recognized in revenue (Rs. 375,000 × 4/12 = Rs. 125,000).
Therefore, the total revenue = Rs.3,250,000 + Rs.125,000 = Rs.3,375,000

04. (c) Although the invoiced amount is Rs. 180,000, Rs. 30,000 of this has not
yet been earned and must be deferred until the servicing work has been
completed. This is only correct inclusion in sales.

05. (d) The revenue in relation to the installation and the machine itself can be
recognized, with the revenue on the service recognized over time as the
service is performed. The service will be recognized over the 2-year period.
By 31 December 2017, 2 months of the service has been performed.
Therefore, Rs. 20,000 can be recognized (Rs. 240,000 × 2/24). Total
revenue is therefore Rs. 580,000, being the Rs. 800,000 less the Rs.
220,000 relating to the service which has not yet been recognized.

06. (d) Discounts should be applied evenly across the components of a sale
unless any one element is regularly sold separately at a discount. As entity
does not sell the service and installation separately, the discount must be
applied evenly to each of the three elements.

07. (b) Revenue as an agent is made by earning commission. Therefore, the


revenue on these sales should only be Rs. 600,000 (10% of Rs. 6 million).
As CL currently has Rs. 6 million in revenue, Rs. 5.4 million needs to be
removed, with Rs. 5.4 million also removed from cost of sales.

08. (c)
Product Allocated price
Product A 40 Remaining amount
Product B 33 (55/100 x Rs. 60)
Product C 27 (45/100 x Rs. 60)
Total 100

The entire discount relates to Product B and C as when Product A is added


it total stand-alone price has been added in the package price.

© Emile Woolf International 530 The Institute of Chartered Accountants of Pakistan


Chapter 10: IFRS 15: Revenue from contracts with customers

09. (b)
Product Allocated price
Product A 33 (Rs. 50 / Rs. 150 × Rs. 100)
Product B 17 (Rs. 25 / Rs. 150 × Rs. 100)
Product C 50 (Rs. 75 / Rs. 150 × Rs. 100)
Total 100

10. (b) For item (b) the sale of the goods has fulfilled a contractual obligation so
the revenue in relation to this can be recognized. The service will be
recognized over time, so the revenue should be deferred and recognized
as the obligation is fulfilled.
For item (a) HL acts as an agent, so only the commission should be
included in revenue.
For item (c) any profit or loss on disposal should be taken to the statement
of profit or loss. The proceeds should not be included within revenue.
For item (d) the Rs. 1 million should be initially discounted to present value
as there is a significant financing component within the transaction. The
revenue would initially be recognized at Rs. 826,000, with an equivalent
receivable. This receivable would then be held at amortized cost with
finance income of 10% being earned each year.

11. (d) At 31 March 2015, the deferred consideration of Rs. 12.65 million would
need to be discounted by 10% for one year to Rs. 11.5 million (effectively
deferring a finance cost of Rs. 1.15 million).
The total amount credited to profit or loss would be Rs. 24.15 million (12.65
million + 11.5 million).

12. (b) Recognizing revenue when a performance obligation is satisfied, it may


be a point in time or over time.

13. (b) The commission to sales employees is incremental to obtaining the


contract and should be capitalized as a contract asset. The external legal
fees and the travelling cost are not incremental to obtaining the contract
because they have been incurred regardless of whether X Limited obtained
the contract or not.

14. (c) The receivable is recorded when unconditional right to receive payment is
established and as entity has not performed its performance obligation yet;
a contract liability shall be recognized.

15. (c) Revenue Rs. 9,700 (97 x Rs. 100 for products expected to be not
returned) and remaining as contract liability.

16. Rs. 90,000 The discount should be allocated to each part of the bundled sale.
Applying the discount across each part gives revenue as follows:
Goods Rs. 50 (Rs. 75 × Rs. 100/Rs. 150)
Installation Rs. 20 (Rs. 30 × Rs. 100/Rs. 150)
Service Rs. 30 (Rs. 45 × Rs. 100/Rs. 150)
The revenue in relation to the goods and installation should be recognized
on 1 May 2011.
As 8 months of the service has been performed (from 1 May to 31
December 2011), then Rs. 20 should be recognized (Rs. 30 × 8/12).
This gives a total revenue for the year of 50 + 20 + 20 = Rs. 90.

© Emile Woolf International 531 The Institute of Chartered Accountants of Pakistan


Financial accounting and reporting I

17. Rs. 9,490,000 The fact that CL has given the customer a year to pay on such a large
amount suggests there is a significant financing component within the sale.
The Rs. 990,000 received can be recognized in revenue immediately. The
remaining Rs. 9.01 million must be discounted to its present value of Rs.
8.5 million. This is then unwound over the year, with the interest recognized
as finance income.
Therefore, total initial revenue = Rs. 990,000 + Rs. 8,500,000 = Rs.
9,490,000.

18. Rs. 3,800,000 The payment made to the customer is not in exchange for a distinct good
or service. Therefore, the Rs.1m paid to the customer is a reduction of the
transaction price.
The total transaction price is being reduced by 5% (Rs.1m/Rs.20m).
Therefore, Golden Limited reduces the transaction price of each good by
5% as it is transferred. By 31 December 2011, Golden Limited should have
recognized revenue of Rs.3.8m (Rs.4m × 95%).

19. Rs. 24,000 The selling price of the service would be Rs. 30,000 (Rs. 20,000 × 150%).
The total standalone selling prices of the machine and support are Rs.
125,000 (Rs. 95,000 + Rs. 30,000).
The transaction price allocated to the machine is Rs. 76,000 (Rs. 95,000 ×
100,000 / 125,000). The transaction price allocated to the technical support
is Rs.24, 000 (Rs.30, 000 × 100,000 / 125,000).

20. Rs. 110,000 No need to calculate present value under option 2 as cash is being
received exactly when performance obligation is being satisfied.

21. (d)

22. (c)

23. (d)

24. (b)

25. (d)

26. (b)

27. (a)

28. (b)

29. (c)

30. (a)

31. (c)

32. (c)

33. (d)

34. (b) (20 x 80%) + (22.5 x 15%) + (18 x 5%) = Rs. 20.275 million

© Emile Woolf International 532 The Institute of Chartered Accountants of Pakistan


Certificate in Accounting and Finance

11

CHAPTER
Financial accounting and reporting I

Interpretation
of financial statements

Contents
1 Users of Financial statements and their information needs
2 Interpretation of financial statements by ratio analysis
3 Profitability ratios
4 Working capital efficiency ratios
5 Liquidity ratios
6 Debt ratios
7 Financial Statement Analysis
8 Limitations of financial statements and ratio analysis
9 Objective based questions and answers

* The student must refer original handbook of IFRS.

© Emile Woolf International 533 The Institute of Chartered Accountants of Pakistan


Financial accounting and reporting I

1 USERS OF FINANCIAL STATEMENTS AND THEIR INFORMATION NEEDS


Section overview

 Users of the financial statements and their information needs

1.1 Users of the financial statements and their information needs


The IASB Conceptual Framework identifies several groups of people who may use financial
statements:
 Investors and potential investors
 Lenders
 Employees
 Suppliers
 Customers
 Government and government agencies
 The general public and
 The management
All these groups are interested in financial performance, financial position and cash flows. Some
users are mainly interested in performance and profitability, while others may be more interested
in liquidity and gearing or other matters.
For example:
 A private investor needs to know whether to continue to hold shares or to sell them. He or
she will tend to be most interested in profitability ratios (such as gross and net profit margin
and return on capital employed) and investor ratios (such as earnings per share, dividend
cover and price earnings ratio).
 A potential acquirer needs information about an entity’s profitability and probably also
information about whether or not the entity is managed efficiently. The acquirer’s
management is likely to focus on profit margins, return on capital employed, asset turnover
and working capital ratios.
 Employees are interested in fair wages, adequate fringe benefits and bonus linked with
productivity/profitability. Ratio analysis provides them adequate information regarding
efficiency and profitability of the entity.
 A bank that has been approached to lend money to an entity needs to know whether it will
receive interest payments when these are due and whether the money that it lends will
eventually be repaid. A bank manager will normally be most interested in cash flows and
liquidity ratios, (current ratio, acid test ratio) gearing and interest cover. A potential lender
will also be interested in predicting future performance as without sales there will be no cash.
 The management do financial forecasting, planning (of a future based on the past and
present) and controlling (budgets) as a decision maker.
An examination question might ask you to interpret an entity’s financial statements for the benefit
of specific people or groups of people. Therefore, your analysis should focus on the needs of the
users.
 What do they need to know?
 What are they interested in?
 What decision do they need to make?

© Emile Woolf International 534 The Institute of Chartered Accountants of Pakistan


Chapter 11: Interpretation of financial statements

2 INTERPRETATION OF FINANCIAL STATEMENTS WITH FINANCIAL RATIOS


Section overview

 Purpose and Importance of Financial Ratio Analysis


 Categories of Financial Ratios

Financial statements are used to make decisions. They are used by shareholders and investors,
and also by lenders, as well as by management. The financial statements contain a large number
of figures, but the figures themselves do not necessarily have much meaning to a user of the
financial statements. However, the figures can be analysed and interpreted by calculating financial
ratios.
Financial ratios can help the user of the financial statements to assess:
 the financial position of the entity, and
 its financial performance

2.1 Purpose and Importance of Financial Ratio Analysis


Financial ratio analysis helps a business in a number of ways. The importance and advantages of
financial ratios are given below:
 Ratios help in analysing the performance trends over a long period of time.
 They also help a business to compare the financial results to those of competitors.
 Ratios assist the management in decision making.
 They also point out problem and weak areas along with the strength areas.
 Ratios help to develop relationships between different financial statement items.
 Ratios have the advantage of controlling for differences in size. For example, two
businesses may be quite different in size but can be compared in terms of profitability,
liquidity, etc., by the use of ratios.

2.2 Categories of Financial Ratios


The main financial ratios can be classified as:
 Profitability Ratios Working capital ‘turnover’ ratios
 Liquidity ratios
 Debt ratios
 Investor ratios
The details of above ratios are given in next section

© Emile Woolf International 535 The Institute of Chartered Accountants of Pakistan


Financial accounting and reporting I

3 PROFITABILITY RATIO
Section overview

 Return on capital employed


 Return on shareholder capital (equity) or investment
 Return on assets
 Analysing return: profitability and asset utilisation
 Profit/sales ratio
 Cost / sales ratios
 Sales/capital employed ratio or assets turnover ratio
 Percentage annual growth in sales

3.1 Return on capital employed


Profit-making companies should try to make a profit that is large enough in relation to the amount
of money or capital invested in the business. The most important profitability ratio is probably
return on capital employed (ROCE) or return on investment (ROI).
For a single company:

Formula:

Profit before interest and taxation

ROCE = X 100%
(Share capital and reserves + long-term debt capital + preference
share capital)

The capital employed is the share capital and reserves, plus long-term debt capital such as bank
loans, bonds and loan stock.
Where possible, use the average capital employed during the year. This is usually the average of
the capital employed at the beginning of the year and end of the year.
A higher ROCE indicates more efficient use of capital. ROCE should be higher than the company’s
capital cost; otherwise it indicates that the company is not employing its capital effectively and is
not generating shareholder value.

Illustration 01:
Sting Company achieved the following results in Year 1.

1 January 31 December
Year 1 Year 1

Rs. Rs.

Share capital 200,000 200,000

Share premium 100,000 100,000

Retained earnings 500,000 600,000

Bank loans 200,000 500,000

© Emile Woolf International 536 The Institute of Chartered Accountants of Pakistan


Chapter 11: Interpretation of financial statements

Rs.

Profit before tax 210,000

Income tax expense 65,000

Profit after tax 145,000

Interest charges on bank loans were Rs.30, 000. Dividend payments to shareholders were Rs.45,
000. Sales during the year were Rs.5, 800,000.
Based on above Information the analysis and calculation are as under:
Capital employed at the beginning of the year = Rs.1, 000,000.
Capital employed at the end of the year = Rs.1, 400,000.
Average capital employed = [Rs.1, 000,000 + Rs.1, 400,000]/2 = Rs.1, 200,000.
Profit before interest and taxation = Rs.210, 000 + Rs.30, 000 = Rs.240, 000.
240,000
ROCE =  100% = 20%
1,200,000
Comment:
The 20% return on the capital employed indicates a return of Rs.20 on every 100 rupees invested
into the business. The ratio shows how efficiently the entity’s long term funds are being employed.
The investors are interested to invest in the company that has a higher ROCE than the other
available option(s). Besides the company’s return should always be higher than the cost at which
the funds were acquired. For example if a company borrows at 15% and achieves a return of only
10%, that says they are actually losing money.

ROCE is a useful measure of comparing profitability across competing entities based on the
amount of capital they use. It becomes more useful when the comparison is between capital-
intensive entities. Moreover, for a single company, the ROCE trend over the years is a significant
performance indicator. Generally speaking, the investors are more inclined to invest in the
companies that have stable and rising ROCE figures as compared to those where the ROCE is
volatile and inconsistent.

Illustration 02:
The two companies A and B operating in the similar lines of business for the year ender 31 st
December 2016. The following data is available.

Company A Company B

Rs. Rs.

Profit before Interest and Taxation 539,900 2,616,100

Capital employed 3,659,000 12,193,400

Based on above Information the analysis and calculation are as under:


Company A

539,900
ROCE =  100% = 14.8%
3,659,000
Company B

2,616,100
ROCE =  100% = 21.5%
12,193,400

© Emile Woolf International 537 The Institute of Chartered Accountants of Pakistan


Financial accounting and reporting I

Comment:
Based on the given figures, Company B appears to be utilising its capital better than Company A.
Company B can reinvest a greater portion of its profits back into the business operations to the
benefit of the shareholders. While we see that Company B’s ROCE is higher than that of Company
A yet there is not a lot to be attained from using data from one angle and at a single point of time.
Greater insight can be achieved if trends over time are analysed. As Company B though is
performing better than A yet it might have been facing a constant decline in the ROCE that may
point to a loss of competitive advantage.

3.2 Return on shareholder capital (equity) OR Investment


Return on shareholder capital (ROSC), or return on equity, measures the return on investment
that the shareholders of the company have made. This ratio normally uses the values of the
shareholders’ investment as shown in the statement of financial position (rather than market
values of the shares).

Formula:
Profit after taxation and preference dividend
ROSC = X 100%
Share capital and reserves

The average value of shareholder capital should be used if possible. This is the average of the
shareholder capital at the beginning and the end of the year.
Profit after tax is used as the most suitable measure of return for the shareholders, since this is a
measure of earnings (available for payment as dividends or for reinvestment in the business).

Illustration 03:
Using the figures in the previous illustration:
Shareholders’ capital at the beginning of the year = Rs.200, 000 + Rs.100, 000 + Rs.500, 000
= Rs.800, 000.
Shareholders’ capital at the end of the year= Rs.200,000 + Rs.100,000 + Rs.600,000
= Rs.900, 000.
Average shareholders’ capital employed = [Rs.800, 000 + Rs.900, 000]/2 = Rs.850, 000.

145,000
ROSC =  100% = 17.06%
850,000
Comment:
The ROSC measures the ability of the entity to generate profits from the investments made by its
shareholders. The figure above shows a percentage of 17.06% meaning that the entity generates
a return of Rs.17 for every 100 rupees invested by the shareholder into the business. ROSC is the
indicator of effective management of equity financing.

3.3 Return on assets


Formula:
Profit before interest and taxation
ROA = X 100%
Assets

The normal convention is to use ‘total assets’ which includes both current and non-current
assets. However, other variations are sometimes used such as non-current assets only.

© Emile Woolf International 538 The Institute of Chartered Accountants of Pakistan


Chapter 11: Interpretation of financial statements

The return on assets ratio is a profitability ratio and measures the return produced by the total
assets. It helps both, the management and the investors, to know how well the entity can convert
its investment in assets into profits. The figures of ROA depend highly on the industry and hence
can vary substantially. This suggests that when ROA has to be used as a comparative measure
then the best practice is to compare it against a company’s previous ROA figures or the ROA of a
company in the similar business line.

Illustration 04:
A company’s chartered accountant is calculating ROA for the year 2016. The year-end figures of
total non- current assets and total current assets are Rs. 882,900 and Rs. 360,000 respectively.
The net profit for the year was Rs.685,000
Based on above Information the analysis and calculation are as under:
(a) Total assets

Calculating ROA using total assets

685,000
ROA =  100%= 55%
1,242,900

(b) Non-current assets

Calculating ROA using non-current assets

685,000
ROA =  100%= 77.6%
882,900

Comment:

This means that on average every single rupee invested in business’s assets generated 55 paisa in
profit.

If taken from the perspective of non-current assets only, they contribute to the
extent of 77.6%. That says for every 100 rupees invested in the non-current assets around 78
rupees are generated in profit.

The analysis would be more useful and meaningful when compared with the entity’s own
performance over the years and against the figures of the firm(s) competing in the similar industry.

3.4 Analysing return: profitability and asset utilisation


The size of the return on capital employed, or the size of the return on shareholders’ capital,
depends on two factors:
 The profitability of the goods or services that the entity has sold
 The volume of sales that the entity has achieved with the capital and assets it has
employed: this is known as asset utilisation or asset turnover.

3.5 Profit/sales ratio


The profit/sales ratio is the ratio of the profit that has been achieved for every Rs.1 of sales.

Formula:

Profit
Profit/sales ratio = X 100%
Sales

© Emile Woolf International 539 The Institute of Chartered Accountants of Pakistan


Financial accounting and reporting I

Profit/sales ratios are commonly used by management to assess financial performance, and a
variety of different figures for profit might be used. The definition of profit can be any of the
following:
 Profit before interest and tax
 Gross profit (= Sales minus the Cost of sales)
 Net profit (= Profit after tax)
It is important to be consistent in the definition of profit, when comparing performance from one
year to the next.
So there are 3 types of profit to sales ratio:
a) Profitability/Operating profit ratio
b) Gross profit margin ratio
c) Net profit ratio
The gross profit ratio is often useful for comparisons between companies in the same industry, or
for comparison with an industry average. It is also useful to compare the net profit ratio with the
gross profit ratio. A high gross profit ratio and a low net profit ratio indicate high overhead costs
for administrative expenses and selling and distribution costs.

Illustration 05:
Using the figures in the previous illustration, profit/sales ratios can be calculated as follows:
 If profit is defined as profit before interest and tax, the profit/sales ratio =
Rs.240,000/Rs.5,800,000 = 0.0414 = 4.14%
 If profit is defined as profit after interest and tax, the profit/sales ratio =
Rs.145,000/Rs.5,800,000 = 0.025 = 2.5%
Comment:
The figure suggests that Rs.4.14 is earned on every 100 rupees of sales before interest and tax
are deducted. After the deduction this figure becomes Rs.2.5 in the given scenario.
The profit to sales ratios show the percentage of sales that is left over after the business has paid
all its expenses. The ratio helps to determine how effectively a company’s sales are converted into
net income. Again the figures have to be compared with the industry averages and over the years
for the same company to arrive at a more meaningful conclusion.

Illustration 06:
Using the given figures the gross profit ratio of a company may be computed as:
Gross Profit Rs.235,000
Net Sales Rs.910,000
235,000
GP Margin=  100% = 26%
910,000
Comment:
The rounded off figure of GP margin is 26% that implies the company may reduce the selling price
of its products up to around 26% without incurring any loss. The GP ratio is an important ratio as it
evaluates the operational performance of the entity. Gross profit is an important figure for the
business, it should be sufficient enough to cover all the expenses and provide for the profit to the
investors.
In general, a higher ratio is a better ratio. The profitability of the business can be measured by
comparing it with the competing entities in the similar industry and with the past trend for the same
company. A consistent growth over the years indicates a sustainable continuous improvement in
the business’s processes and practices.

© Emile Woolf International 540 The Institute of Chartered Accountants of Pakistan


Chapter 11: Interpretation of financial statements

3.6 Cost/ sales ratios


It is also useful to monitor the ratio of different types of cost to sales. The following ratios can be
useful to highlight an unexpected change in a period or to indicate a difference between the
company and another in a similar industry:
 Ratio of (Cost of sales/Sales) × 100%
 Ratio of (Administration costs/Sales) × 100%
 Ratio of (Selling and distribution costs/Sales) × 100%

Illustration 07:
Following figures have been extracted from the Income Statement of Alpha ltd.

Rs.

Net Sales 650,000

Cost of sales 422,500

Administration costs 26,000

Selling and distribution costs 39,000

The cost of goods sales ratio, administration costs ratio and Selling and distribution costs ratio are
calculated as:
Cost of sales ratio
422,500
=  100% = 65%
650,000
Administration costs ratio
26,000
=  100% = 4%
650,000
Selling and distribution costs ratio
39,000
=  100% = 6%
650,000
Comment:

Costs ratios represent what extent of sales is an individual expense or a group of expenses. The
lower the ratio the better is the profitability status of the organisation. Care must be taken in dealing
with the variable expenses as they vary with the change in the sales volume. This ratio doesn’t
normally change significantly with the rise or decline in the sales volume. Whereas the ratios for
fixed expenses change significantly with the increase or decrease in the sales volume.

In the given scenario the cost of sales/sales ratio states that every 65 rupees out of 100 rupees of
sales represent cost of sales. These are the direct costs that vary with the level of sales.

Looking at the other two ratios we find that in this particular period every 4 rupees out of every 100
rupees of sales were spent on the administration costs and 6 rupees were expensed on selling and
distribution costs.

These ratios help the management in controlling and estimating future expenses.

© Emile Woolf International 541 The Institute of Chartered Accountants of Pakistan


Financial accounting and reporting I

3.7 Sales/capital employed ratio or Assets turnover ratio


The sales/capital employed ratio is also called the ‘asset turnover ratio’. It measures the amount
of sales achieved during the period for each Rs.1 of investment in assets.

Formula:

Sales
Asset turnover ratio = (Share capital and reserves + long-term debt capital + preference
shares)

It is measured as ‘x times a year’.


The sales/capital employed ratio is also a ratio of sales to (assets – current liabilities). This is
because capital employed = total assets minus liabilities excluding long-term debt.

Illustration 08:
Using the figures in the previous illustration, the asset turnover ratio = Rs.5,800,000/Rs.1,200,000
= 4.83 times.
Note that:
ROCE = Profit/sales ratio × Asset turnover ratio
(Where profit is defined as profit before interest and taxation).
Using the figures in the previous illustration:

ROCE = Profit/sales × Sales/capital employed


240,000 240,000 5,800,000
= ×
1,200,000 5,800,000 1,200,000
20% = 4.14% × 4.83 times

Comment:
The Sales/Capital employed ratio measures how efficiently an organisation’s assets generate
revenues. The figure in the solution says that every single rupee of the capital employed in the
business is generating revenue of Rs.4.83. It must also be taken into account that the age of a
company’s assets can heavily impact hence result in different asset turnover ratios for similar
companies. For example a company having older assets with lower book values might have a higher
asset turnover ratio than the one with the similar revenues but newer, higher net book value assets.
A constantly declining assets turnover ratio or a lower ratio as compared to the industry averages
might indicate towards the issues related to the excess production capacity, poor inventory
management, and sloppy collection methods etc. The higher the ratio the better it is considered
yet capital investment for purchasing assets in anticipation of future growth or sale of existing
unnecessary assets for an anticipated decline in future can suddenly and may be artificially change
the company’s assets turnover ratio. Besides companies in the capital-intensive industries tend to
have a lower assets turnover ratio than the ones operating with fewer assets. Therefore for a more
meaningful analysis, the companies should be compared within the same industry.

3.8 Percentage annual growth in sales


It can be useful to measure the annual growth (or decline) in sales, measured as a
percentage of sales in the previous year.
For example, if sales in the year just ended were Rs.5,800,000 and sales in the previous
year were Rs.5,500,000, the annual growth in sales has been (Rs.300,000/Rs.5,500,000) ×
100% = 5.45%.

© Emile Woolf International 542 The Institute of Chartered Accountants of Pakistan


Chapter 11: Interpretation of financial statements

4 WORKING CAPITAL EFFICIENCY RATIOS


Section overview

 Purpose of working capital efficiency ratios


 Average time to collect (receivables days or days sales outstanding)
 Average time for holding inventory
 Average time to pay suppliers
 Cash operating cycle/working capital cycle
 Turnover (multiples) ratios

4.1 Purpose of working capital efficiency ratios


Working capital efficiency ratios measure the efficiency with which the entity has managed its
receivables, inventory and trade payables. The ratios are usually measured in terms of an average
number of days.
The working capital ratios are a useful measure of whether the entity has too much or too little
invested in working capital.
Excessive investment in working capital is indicated by a long cash cycle (a long working capital
cycle) that appears to be getting even longer. When too much is invested in working capital, the
return on capital employed and ROSC will be lower than they should be.
Under-investment in working capital is an indication of possible liquidity difficulties. When
working capital is low in comparison with the industry average, this might indicate that current
assets are being financed to an excessive extent by current liabilities, particularly trade payables
and a bank overdraft.
(The cash cycle, also called the operating cycle and the working capital cycle, is explained later).

4.2 Average time to collect (receivables days or days sales outstanding)


This ratio estimates the time that it takes on average to collect the payment from customers after
the sale has been made. It could be described as the average credit period allowed to customers
or the ‘average collection period’.

Formula:
Trade receivables
Average days to collect = X 365 days
Sales

Trade receivables should be the average value of receivables during the year. This is the average
of the receivables at the beginning of the year and the receivables at the end of the year.
However, the value for receivables at the end of the year is also commonly used.
Sales are usually taken as total sales for the year. However, if sales are analysed into credit sales
and cash sales, it is probably more appropriate to use the figure for credit sales only.
The average time to collect money from credit customers should not be too long. A long average
time to collect suggests inefficient collection of amounts due from receivables.

4.3 Average time for holding inventory


This ratio is an estimate of the average time that inventory is held before it is used or sold.

Formula:
Inventory
Inventory holding period = X 365 days
Cost of sales

© Emile Woolf International 543 The Institute of Chartered Accountants of Pakistan


Financial accounting and reporting I

In theory, inventory should be the average value of inventory during the year. This is the average
of the inventory at the beginning of the year and the inventory at the end of the year.
However, the value for inventory at the end of the year is also commonly used, particularly in
examinations.

4.4 Average time to pay suppliers


The average time to pay suppliers may be calculated as follows:

Formula:
Trade payables
Average time to pay = X 365 days
Cost of purchases

Trade payables should be the average value of trade payables during the year. This is the average
of the trade payables at the beginning of the year and the trade payables at the end of the year.
However, the value for trade payables at the end of the year is also commonly used. When the
cost of purchases is not available, the cost of sales should be used instead. This figure is obtained
from the profit and loss information in the statement of profit or loss and other comprehensive
income.

Illustration 09:
The following information is available for The Brush Company for Year 1.

1 January Year 1 31 December Year 1


Rs. Rs.
Inventory 300,000 360,000
Trade receivables 400,000 470,000
Trade payables 150,000 180,000

Sales in Year 1 totalled Rs.3, 000,000 and the cost of sales was Rs.1, 800,000
Based on above the analysis and calculation are as under:

Average inventory = [Rs.300,000 + Rs.360,000]/2 = Rs.330,000


Average trade receivables = [Rs.400,000 + Rs.470,000]/2 = Rs.435,000
Average trade payables = [Rs.150, 000 + Rs.180, 000]/2 = Rs.165, 000.
Turnover ratios
Average days to collect = [435,000/3,000,000] × 365 days = 52.9 days
Inventory turnover period = [330,000/1,800,000] × 365 days = 66.9 days
Average time to pay = [165,000/1,800,000] × 365 days = 33.5 days.
Comment:
The relationship among the three ratios given above is quite critical. The sale of inventory generates
receivables, that when collected are used to settle creditors and the cycle goes on. The firm’s ability
to get the cycle repeated continuously depends heavily on the entity’s short-term liquidity and the
cash generating ability. In the given scenario the firm is responsible to settle its payables at least
around 19 days before it receives payments from its customers. The working capital need then
would be financed through some bank operating line and that would bring a cost which in turn
would reduce the profitability of the organisation. The credit policy therefore needs to get revised
and improved and sloppy collections to be gotten rid of. Also the agreement with the supplier might
be refreshed for the purpose of extending the credit period.

© Emile Woolf International 544 The Institute of Chartered Accountants of Pakistan


Chapter 11: Interpretation of financial statements

4.5 Cash operating cycle/working capital cycle


The cash operating cycle or working capital cycle is the average time of one cycle of business
operations:
 From the time that suppliers are paid for the resources they supply
 To the time that cash is received from customers for the goods (or services) that the entity
makes (or provides) with those resources and then sells.
A cash cycle or operating cycle is measured as follows. Figures are included for the purpose of
illustration:

Days Days
Inventory turnover A 40.2
Average days to collect B 88.2
–––––
128.4
Average time to pay (C) (33.5)
––––––––– –––––
Cash cycle/operating cycle A+B–C 94.9
––––––––– –––––
The working capital ratios and the length of the cash cycle should be monitored over time. The
cycle should not be allowed to become unreasonable in length, with a risk of over-investment or
under-investment in working capital.
A positive working capital cycle balances incoming and outgoing payments to minimise net working
capital and maximise free cash flow. For example, a company that pays its suppliers in 30 days
but takes 60 days to collect its receivables has a working capital cycle of 30 days. This 30 day
cycle usually needs to be funded through a bank operating line, and the interest on this financing
is a carrying cost that reduces the company's profitability.
Growing businesses require cash, and being able to free up cash by shortening the working capital
cycle is the most inexpensive way to grow. Sophisticated buyers review closely a target's working
capital cycle because it provides them with an idea of the management's effectiveness at managing
their balance sheet and generating free cash flow.
In a manufacturing organization there are 3 types of inventory that are:
a) Raw material inventory
b) Work in process inventory
c) Finished goods inventory
So inventory turnover ratio is analysed as:
Raw material inventory holding period

Formula:

Raw material Inventory


Raw material Inventory holding period = X 365 days
Material consumed

Work in process inventory holding period

Formula:
Work in process Inventory
Work in Porcess Inventory holding period = X 365 days
Cost of goods manufactured

© Emile Woolf International 545 The Institute of Chartered Accountants of Pakistan


Financial accounting and reporting I

Finished goods inventory holding period

Formula:
Finished goods Inventory
Finished goods Inventory X 365 days
holding period = Cost of goods sold

4.6 Turnover (multiples) ratios


Inventory turnover

Formula:
Cost of sales
Inventory turnover = times
Average inventory

Receivables (debtor) turnover

Formula:
Credit sales
Receivables turnover = times
Average trade receivables

Payables (creditor) turnover

Formula:
Credit purchases times
Payables turnover =
Average trade payables

© Emile Woolf International 546 The Institute of Chartered Accountants of Pakistan


Chapter 11: Interpretation of financial statements

5 LIQUIDITY RATIOS
Section overview

 The meaning of liquidity


 Current ratio
 Quick ratio or acid test ratio

5.1 The meaning of liquidity


Liquidity means having cash or access to cash readily available to meet obligations to make
payments.
For the purpose of ratio analysis, liquidity is measured on the assumption that the only sources of
cash available are:
 Cash in hand or in the bank, plus
 Current assets that will soon be converted into cash during the normal cycle of trade.
It is also assumed that the only immediate payment obligations faced by the entity are its current
liabilities.
There are two ratios for measuring liquidity:
 Current ratio
 Quick ratio, also called the acid test ratio.
The more suitable ratio for use depends on whether inventory is considered a liquid asset that will
soon be used or sold, and converted into cash from sales.

5.2 Current ratio


The current ratio is the ratio of current assets to current liabilities.
Formula:
Current assets
Current ratio =
Current liabilities

The amounts of current assets and current liabilities in the statement of financial position at the
end of the year may be used. It is not necessary to use average values for the year.
It is sometimes suggested that there is an ‘ideal’ current ratio of 2.0 times (2:1).
However, this is not necessarily true and in some industries, much lower current ratios are
normal. It is important to assess the liquidity ratios by considering:
 Changes in the ratio over time
 The liquidity ratios of other companies in the same period
 The industry average ratios.
Liquidity should be monitored by looking at changes in the ratio over time.

Illustration 10:
The following information is available for X ltd. for Year 1. The current ratio can be calculated and
interpret as:
Current Assets Rs.1,100,000
Current Liabilities Rs.400,000
1,100,000
Current Ratio = = 2.75 times
400,000

© Emile Woolf International 547 The Institute of Chartered Accountants of Pakistan


Financial accounting and reporting I

Comment
A current ratio of 2:1 or higher is considered satisfactory for most of the entities yet the analysts
should be very careful at interpreting it. A simple calculation of current ratio might not disclose the
exact liquidity position of the company. A deeper analysis into the individual items of current assets
and liabilities would add value to the results. A higher current ratio might not indicate the ability to
pay off the entity’s current obligations efficiently as a huge portion of current assets may comprise
of needless, obsolete and/or slow moving inventory items.

5.3 Quick ratio or acid test ratio


The quick ratio or acid test ratio is the ratio of current assets excluding inventory to current
liabilities. Inventory is excluded from current assets on the assumption that it is not a very liquid
item.

Formula:
Current assets excluding inventory
Quick ratio =
Current liabilities

The amounts of current assets and current liabilities in the statement of financial position at the
end of the year may be used. It is not necessary to use average values for the year.
This ratio is a better measurement of liquidity than the current ratio when inventory turnover times
are very slow, and inventory is not a liquid asset.
It is sometimes suggested that there is an ‘ideal’ quick ratio of 1.0 times (1:1).
However, this is not necessarily true and in some industries, much lower quick ratios are normal.
As indicated earlier, it is important to assess liquidity by looking at changes in the ratio over time
and comparisons with other companies and the industry norm.

Illustration 11:
Kashif’s Clothing Store has applied for a loan to remodel the shop front. The bank has asked him
for a detailed balance sheet, so it can compute the quick ratio. Kashif's balance sheet includes the
following figures:
Cash: Rs.20,000
Accounts Receivable: Rs.10,000
Inventory: Rs.5,000
Stock Investments: Rs.2,000
Prepaid taxes: Rs.500
Current Liabilities: Rs.30,000
The bank can compute Kashif's quick ratio like this.
20,000  10,000  2,000
Quick Ratio = = 1.07 times
30,000

Comment
The Quick ratio of Kashif’s Clothing store turns out to be 1.07 times that says Kashif can pay off all
his current liabilities with liquid assets and can still have some quick assets left over. The Acid-test
ratio gives a more rigorous assessment of the company’s ability to pay off its current liabilities as
it considers only highly liquid assets. Had it been below 1 it would have represented the company
as an overly leveraged company that is struggling to; maintain or increase sales, settling its
creditors quickly, or/and collecting receivables on time.

© Emile Woolf International 548 The Institute of Chartered Accountants of Pakistan


Chapter 11: Interpretation of financial statements

6 DEBT RATIOS OR LONG TERM SOLVENCY RATIO


Section overview

 Gearing (debt to equity) ratio


 Interest cover ratio

Debt ratios are used to assess whether the total debts of the entity are within control and are not
excessive.
6.1 Gearing (debt to equity) ratio
Gearing, also called leverage, measures the total long-term debt of a company as a percentage
of either:
 The equity capital in the company, or
 The total capital of the company

Formula:
Long-term debt
Gearing = X 100%
Share capital and reserves

Alternatively:
Formula:
Long-term debt
Gearing = X 100%
Share capital and reserves + Long-term debt

It is usually appropriate to use the figures from the statement of financial position at the end of the
year. However, a gearing ratio can also be calculated from average values for the year.
When there are redeemable preference shares it is usual to include them within debt capital. This
is because redeemable preference shares behave more like a long-term loan or bond with fixed
annual interest followed by future redemption.
Irredeemable preference shares behave more like Equity (as they are never redeemed) and should
therefore be treated as equity.
A company is said to be high-geared or highly-leveraged when its debt capital exceeds its share
capital and reserves. This means that a company is high-geared when the gearing ratio is above
either 50% or 100%, depending on which method is used to calculate the ratio.
A company is said to be low-geared when the amount of its debt capital is less than its share
capital and reserves. This means that a company is low-geared when the gearing ratio is less than
either 50% or 100%, depending on which method is used to calculate the ratio.
A high level of gearing may indicate the following:
 The entity has a high level of debt, which means that it might be difficult for the entity to
borrow more when it needs to raise new capital.
 High gearing can indicate a risk that the entity will be unable to meet its payment
obligations to lenders, when these obligations are due for payment.
The gearing ratio can be used to monitor changes in the amount of debt of a company over time.
It can also be used to make comparisons with the gearing levels of other, similar companies, to
judge whether the company has too much debt, or perhaps too little, in its capital structure.
6.2 Interest Cover Ratio
Formula:
Interest cover Profit before interest and tax
= Times
ratio = Interest payable

© Emile Woolf International 549 The Institute of Chartered Accountants of Pakistan


Financial accounting and reporting I

7 FINANCIAL STATEMENTS ANALYSIS


Section overview

 Overview
 Horizontal analysis
 Vertical analysis

7.1 Overview
Financial statement analysis is the process of analysing a company's past, current and projected
performance for decision-making purposes
Financial statement analysis allows analysts to identify trends by comparing ratios across multiple
periods and statement types to allow analysts to measure liquidity, profitability, company-wide
efficiency, and cash flow.
Financial statement analysis is of the following types:
 Horizontal analysis
 Vertical analysis
 Ratio analysis (already explained in above sections)

7.2 Horizontal analysis


Horizontal analysis is used to compare historical data, such as ratios, or line items, over a number
of accounting periods.
Financial analysts and investors need to identify trends and growth patterns in the company’s
performance over a number of years, a year-end balance sheet or income statement is not enough
to evaluate whether the company is operating efficiently and profitably.
Horizontal analysis also makes it easier to compare growth rates and profitability among different
companies.
The following is the formula for horizontal analysis:
Amount in comparison year – Amount in base year x 100
Base year
The following figure is an example of how to prepare a horizontal analysis for two years.
Carnations Ltd
Profit & Loss Account
For the year ended December 31, 2018

%age change
2018 2017
from 2017 to 2018
Rs. in millions
Sales 86,320 75,200 14.79
Cost of Sales (44,618) (40,900) 9.09
Gross Profit 41,702 34,300 21.58
Distribution costs (19,597) (15,380) 27.42
Administrative expenses (2,339) (2,053) 13.93
Other operating expenses (1,322) (1,052) 25.67

© Emile Woolf International 550 The Institute of Chartered Accountants of Pakistan


Chapter 11: Interpretation of financial statements

%age change
2018 2017
from 2017 to 2018
Rs. in millions
Other income 1,488 1,000 48.80
Profit before interest 19,932 16,815 18.54
Finance cost (343) (300) 14.33
Profit before taxation 19,589 16,515 18.61

7.3 Vertical analysis


In vertical analysis each category of accounts on the balance sheet is shown as a percentage of
the total account.
Line items on an income statement can be stated as a percentage of gross sales, while line items
on a balance sheet can be stated as a percentage of total assets or liabilities. This analysis of
income statements gives the company a heads up if cost of goods sold or any other expense
appears to be too high when compared to sales and allows the management to identify the reasons
and take action to fix the problem(s).
Carnations Ltd
Statement of Financial Position
For the year ended December 31, 2018

2018 2017
------------------ Rs. in millions ------------------
Assets
Non-Current Assets
Property, plant & equipment 15,000 42.33% 12,000 32.71%
Intangibles 500 1.41% 600 1.64%
Long term investments 120 0.34% 100 0.27%
Long term loans 200 0.56% 150 0.41%
Long term deposits and
prepayments 70 0.20% 180 0.49%
15,890 44.84% 13,030 35.51%
Current Assets
Stores and spares 650 1.83% 585 1.59%
Stock in trade 6,000 16.93% 5,500 14.99%
Trade debts 2500 7.05% 1200 3.27%
Loans and advances 800 2.26% 300 0.82%
Short term deposits and 750 900
2.12%
prepayments 2.45%
Other receivables 350 0.99% 175 0.48%
Cash and bank balances 8,500 23.98% 15,000 40.88%
19,550 55.16% 23,660 64.49%
Total assets 35,440 36,690

© Emile Woolf International 551 The Institute of Chartered Accountants of Pakistan


Financial accounting and reporting I

2018 2017

------------------ Rs. in millions ------------------

Equity and liabilities

Share capital and reserves

Share capital 1,000 2.82% 1,000 2.73%


Reserves 2,950 8.32% 7,095 19.34%

Liabilities

Non-current liabilities

Staff retirement benefits 290 0.82% 295 0.80%

Current liabilities

Trade and other payables 30,000 84.65% 27,500 74.95%

Provisions 1200 3.39% 800 2.18%


Total current liabilities 31,200 88.04% 28,300 77.13%

Total liabilities 31,490 88.85% 28,595 77.94%

Total equity and liabilities 35,440 36,690

Carnations Ltd
Profit & Loss Account
For the year ended December 31, 2018

2018 2017

------------------ Rs. in millions ------------------

Sales 86,320 100% 75,200 100%

Cost of Sales (44,618) 51.69% (40,900) 54.39%

Gross Profit 41,702 48.31% 34,300 45.61%

Distribution costs (19,597) 22.70% (15,380) 20.45%

Administrative expenses (2,339) 2.71% (2,053) 2.73%

Other operating expenses (1,322) 1.53% (1,052) 1.40%

Other income 1,488 1.72% 1,000 1.33%

Profit before interest 19,932 23.09% 16,815 22.36%

Finance cost (343) 0.40% (300) 0.40%

Profit before taxation 19,589 22.69% 16,515 21.96%

© Emile Woolf International 552 The Institute of Chartered Accountants of Pakistan


Chapter 11: Interpretation of financial statements

8 LIMITATIONS OF FINANCIAL STATEMENTS AND RATIO ANALYSIS


Section overview

 Limitations of financial statements and ratio analysis

8.1 Limitations of financial statements and ratio analysis


Financial statements are time and cost producing. Ratio analysis can be used to compare
information taken from the financial statements to gain an analytical understanding of the results,
financial position and cash flows of a business. This analysis is a useful tool, especially for an
outsider such as a supplier, lender or an investor. However, there are a number of limitations of
ratio analysis which are given below:
Historical
All of the information used in ratio analysis is derived from actual historical results. This does not
mean that the same results will carry forward into the future. However, you can use ratio analysis
on pro forma information and compare it to historical results for consistency.
Historical versus current cost
The information on the income statement is stated in current costs (or close to it), whereas many
elements of the balance sheet are stated at historical cost (which could vary substantially from
current costs). This disparity can result in unusual ratio results.
Inflationary effect
If the rate of inflation has changed in any of the periods under review, this can mean that the
numbers are not comparable across periods. For example, if the inflation rate was 100% in one
year, sales would appear to have doubled over the preceding year, when in fact sales did not
change at all.
Aggregation
The information in a financial statement line item that you are using for a ratio analysis may have
been aggregated differently in the past, so that running the ratio analysis on a trend line does not
compare the same information through the entire trend period.
Accounting policies and estimates
Different companies in a similar industry may have different policies for recording the
same accounting transaction. This means that comparing the ratio results of different companies
may be like comparing apples and oranges. For example, one company might use reducing
balance method while another company uses straight-line depreciation.
Business conditions
You need to place ratio analysis in the context of the general business environment. For example,
60 days of sales outstanding for receivables might be considered poor in a period of rapidly
growing sales, but might be excellent during an economic contraction when customers are in
severe financial condition and unable to pay their bills.
Interpretation
It can be quite difficult to ascertain the reason for the results of a ratio. For example, an acid test
ratio of 2:1 might appear to be excellent, until you realize that the company just sold a large amount
of its stock to bolster its cash position. A more detailed analysis might reveal that the acid test ratio
will only temporarily be at that level, and will probably decline in the near future.
Company strategy
It can be difficult to interpret a ratio analysis comparison between two companies that are pursuing
different strategies. For example, one company may be following a low-cost strategy, and so is
willing to accept a lower gross margin in exchange for more market share. Conversely, a company
in the same industry is focusing on a high customer service strategy where its prices are higher
and gross margins are higher, but it will never attain the revenue levels of the first company.
In short, ratio analysis has a variety of limitations that can restrict its usefulness. However, as long
as you are aware of these problems and use alternative and supplemental methods to collect and
interpret information, ratio analysis is still useful.

© Emile Woolf International 553 The Institute of Chartered Accountants of Pakistan


Financial accounting and reporting I

Example 01: Wasim Pvt. Ltd


Question: Wasim Pvt. Ltd. is an importer and retailer of vegetable oils. Extracts from the financial
statements for this year and last are set out below:
Income statements for the years ended 30 September
Year 7 Year 6
Rs.000 Rs.000
Revenue 2,160 1,806
Cost of sales (1,755) (1,444)
----------------- -----------------
Gross profit 405 362
Distribution costs (130) (108)
Administrative expenses (260) (198)
----------------- -----------------
Profit before tax 15 56
Income tax expense (6) (3)
----------------- -----------------
Profit for the period 9 53
----------------- -----------------
Statements of financial position as of 30 September
Year 7 Year 6
Rs.‘000 Rs.‘000
Assets
Non-current assets
Property, plant and equipment 78 72

Current assets
Inventories 106 61
Trade receivables 316 198
Cash - 6
----------------- -----------------
422 265
----------------- -----------------
Total assets 500 337
----------------- -----------------
Equity and liabilities
Equity
Ordinaryshares 110 85
Preference shares 23 11
Share premium 15 -
Revaluation reserve 20 20
Retained earnings 78 74
----------------- -----------------
246 190
Current liabilities
Bank overdraft 49 -
Trade payables 198 142
Current tax payable 7 5
----------------- -----------------
254 147
----------------- -----------------
Total equity and liabilities 500 337
----------------- -----------------
Required: Calculate profitability ratios, liquidity ratios and working capital ratios for Wasim (private)
Limited for two years.

© Emile Woolf International 554 The Institute of Chartered Accountants of Pakistan


Chapter 11: Interpretation of financial statements

Answer:

Profitability ratios

Year 7 Year 6
Gross profit % =
Gross prof it 405 362
x 100 x 100 = 19% x 100 = 20%
Sales 2,160 1,806

Net profit % =
Net prof it 9 53
x 100  0.4%
x 100 = x 100 = 2.9%
Sales 2,160 1,806

Prof it bef oreinterest and tax


Return on capital employed =
Share capital and reserv es+ Long - term debt capital
  
15 56
x 100 = 6% x 100 = 29%
246 190

Sales
Asset turnover = x 100
Share capital and reserv es+ Long - term debt capital

2,160 1,806
= 8.8 times = 9.5 times
246 190

Liquidity Ratios
 
Current ratio =
Current assets 422 265
= 1.7 times = 1.8 times
Current liabilities 254 147
Quick ratio =
Current assets excludinginv entory 422 - 106 265 - 61
   1.2 times
  1.4 times
Current liabilities 254 147

Working capital ratios


 
Average time to collect =
Trade receiv ables 316 x 365 198 x 365
x 365  53 day s  40 day s
Sales 2,160 1,806

Average time to pay =


Trade pay ables 198 x 365 142 x 365
 x 365  =
41 day s = 36 day s
Cost of purchases 1,755 1, 444

Inventory turnover =
Inv entory 106 x 365 61 x 365
 x 365   day s
 22 = 15 day s
Cost of sales 1,755 1, 444

  

© Emile Woolf International 555 The Institute of Chartered Accountants of Pakistan


Financial accounting and reporting I

Example 02: AMIR & MO Limited


Question: The income statements and statements of financial position of two manufacturing
companies in the same sector are set out below.

Amir Mo
Rs. Rs.
Revenue 150,000 700,000
Cost of sales (60,000) (210,000)
------------------- -------------------
Gross profit 90,000 490,000
Interest payable (500) (12,000)
Distribution costs (13,000) (72,000)
Administrative expenses (15,000) (35,000)
------------------- -------------------
Profit before tax 61,500 371,000
Income tax expense (16,605) (100,170)
------------------- -------------------
Profit for the period 44,895 270,830
------------------- -------------------
Assets
Non-current assets
Property - 500,000
Plant and equipment 190,000 280,000
------------------- -------------------
190,000 780,000
Current assets
Inventories 12,000 26,250
Trade receivables 37,500 105,000
Cash at bank 500 22,000
------------------- -------------------
50,000 153,250
------------------- -------------------
Total assets 240,000 933,250
------------------- -------------------

Required:
Calculate profitability ratios, liquidity ratios and working capital ratios for Amir and Mo to make
comparison.

© Emile Woolf International 556 The Institute of Chartered Accountants of Pakistan


Chapter 11: Interpretation of financial statements

Answer:
Profitability ratios

Amir Mo
Gross profit % =
Gross prof it 90,000 490,000
x 100 x 100 = 60% x 100 = 70%
Sales 150,000 700,000

Net profit % =
Net prof it 44,895 270,830
x 100  x 100
= 30% x 100 = 39%
Sales 150,000 700,000

Prof it bef ore interest and tax


Return on capital employed =
Sharecapital and reserv es+
Long - term debt capital


Amir 61,500 + 500


x 100 = 28.5%

207, 395 +10,000

Mo 371,000 +12,000
x 100 = 47%
565,580 + 250,000

Sales
Asset turnover = x 100
Share capital and reserv es+ Long - term debt capital

Amir 150,000
= 0.7 times
207,395 + 10,000

Mo  700,000
= 0.85 times
565,580 + 250,000

Liquidity Ratios

 Amir Mo
Current ratio =
Current assets 50,000 153,250
= 2.2 times = 1.3 times
Current liabilities 22,605 117,670

Quick ratio =
Current assets excludinginv entory 50,000 - 12,000 153,250 - 26,250
 Current liabilities
 22,605  times
= 1.7
117,670
= 1.1 times

Working capital ratios

Average time to collect =  


Trade receiv ables 37,500 105,000
x 365 x 365 = 91 day s x 365 = 55 day s
Sales 150,000 700,000

Average time to pay =


Trade pay ables 22,605 117,670
x
365 day s
x 365 = 137 x 365 = 204 day s
 Cost of purchases 60,000 210,000

Inventory turnover =
Inv entory 12,000 26,250
 
x 365
60,000  s
x 365 = 73 day
210,000
x 365 = 46 day s
Cost of sales

  

© Emile Woolf International 557 The Institute of Chartered Accountants of Pakistan


Financial accounting and reporting I

Example 03: Alpha Limited and Omega Limited


Question: Alpha Limited and Omega Limited are in the same trade, but operate in different areas.
Their accounts for the year ended 31 December, 2016 are as follows:

Profit and loss account Alpha Limited Omega Limited


Rs.’000 Rs.’000 Rs.’000 Rs.’000
Sales 1,440 1,720
Less: Cost of sales 1,120 1,342
Gross profit 320 378
Less: Overheads 220 300
Profit before tax 100 78
Taxation 40 30
Dividends 20 24
60 54
Retained earnings 40 24
Statement of financial position
Share capital of Rs. 1 each 600 200
Reserves 240 104
840 304
8% Debentures - 120
840 424
Represented by:
Non-current assets at cost 660 520
Less: Depreciation 200 160
460 360
Current assets:
Inventory 280 172
Receivables 310 300
Cash 30 32
620 504
Current liabilities:
Taxation 40 30
Creditors 180 344
Bank overdraft - 42
Dividends 20 24
240 440
Net Current assets 380 64
840 424
Required:
Compute the current ratio, acid test ratio, creditors ratio and collection period for each of the
companies and carry out the comparative analysis of the companies based on the computed ratios.

© Emile Woolf International 558 The Institute of Chartered Accountants of Pakistan


Chapter 11: Interpretation of financial statements

Answer:
Liquidity Ratios

Alpha Limited Omega Limited

Current ratio=

Current assets 620,000 504,000


Current liabilities 240,000 440,000

=2.58 times =1.15 times

Quick ratios=

Current assets – inventory 620,000-280,000 504,000-172,000


Current liabilities 240,000 440,000

=1.42 times =.75 times

Working capital ratios

Average time to collect =

Trade receivables x 365 310,000 x365 300,000 x 365


Sale 1440,000 1720,000

x365= 79 days = 64 days

Average time to pay=

Trade payables x 365 180,000 x 365 344,000 x 365


Credit purchases 1120,000 1342,000

= 59 days =94days

The comments on comparative analysis of both companies based on the ratios computed above
are as under:
(i) In terms of working capital and liquidity, Alpha Limited is in a better position to honor its
obligations as they fall due because its current ratio and acid test ratio are higher than those
of Omega Limited.
(ii) Omega Limited’s payment period is better than that of Alpha Limited’s because Omega
Limited uses supplier’s funds to finance its operation.
(iii) Omega Limited’s collection period is also better than that of Alpha Limited. It extends shorter
credit period to its customers than Alpha Limited.
(iv) Omega Limited’s credit policy is better than that of Alpha Limited. This is because there is 30
days difference between its payments period and collection periods compared with Alpha
Limited that had a longer collection period than its payment period.

© Emile Woolf International 559 The Institute of Chartered Accountants of Pakistan


Financial accounting and reporting I

Example 04: Boom Limited (BL)


Question: Boom Limited (BL) is a manufacturer of sports goods. Following financial statements for
the year ended 31 December 2017 have been submitted to the Chief Executive Officer (CEO)
Statement of profit or loss

Rs. in ‘000

Revenues 21,000

Cost of sales (17,500)

Gross profit 3,500

Operating expenses (1,900)

Finance cost (450)

Profit before tax 1,150

Taxation (345)

Profit after tax 805

Statement of financial position

Rs. in ‘000

Property, plant and equipment 7,500

Current assets 1,500

9,000

Share capital 4,000

Reserves 1,000

Non-current liabilities 3,000

Current liabilities 1,000

9,000

Although performance of BL has improved from the last year, CEO wants to compare the results
with other companies operating in sports manufacturing industry. In this respect, following industry
data has been gathered:

Gross profit margin 23.5%

Net profit margin 7.7%

© Emile Woolf International 560 The Institute of Chartered Accountants of Pakistan


Chapter 11: Interpretation of financial statements

Current ratio 2.75


Gearing ratio 50:50
Return on non-current asset 32.9%
Return on capital employed 27.4%
Return on equity 31.3%

Required:
(a) Compute BL’s ratios for comparison with the industry.
(b) For each ratio, give one possible reason for variation from the industry.
Answer:

Ratios (a) BL's ratios Industry's ratios (b) Reasons for variation from industry

Gross profit 16.67% 23.50% Lower than industry


margin
 Purchase of raw material at higher
prices as compared to its competitors
 Inability to obtain economies of scale
in production as compared to its
competitors
 Higher production costs due
to inefficiencies
 Deliberately keeping selling
prices lower to gain the market share

Net profit 3.83% 7.70% Lower than industry


margin
 BL’s gross profit margin is 6.8% lower
than industry (16.6% Vs 23.5%)
whereas net profit margin is only
3.9% lower which indicates that BL’s
operating expenses as a percentage
of sales are approximately 2.9% lower
than the industry

Current ratio 1.50 2.75 Lower than industry


Since gearing ratio is lower than the
industry so BL might have:
 obtained running finances as
compared to long-term financing by
the industry availed extended credit
terms from suppliers
 Low inventory levels are maintained
by BL
 Shorter credit terms are given to
debtors

© Emile Woolf International 561 The Institute of Chartered Accountants of Pakistan


Financial accounting and reporting I

Ratios (a) BL's ratios Industry's (b) Reasons for variation from industry
ratios

Gearing ratio 37.5: 62.5 50 : 50 Lower than industry


 Difficulty in raising long-term
finance from banks due to low
profits
 Running finance or extended
credit terms from suppliers are
available for BL

Return on 21.33% 32.90% Lower than industry


non- current
 Lower profit margins
assets
 Relatively newer non-current
assets have higher carrying
value

Return 20.00% 27.40% Lower than industry


on
 Lower profit margins
capital
employed  High shareholder’s equity

Return 16.10% 31.30% Lower than industry


on
 Lower profit margins
equity
 Higher shareholder’s equity/low
gearing ratio

Example 05: Progressive Steel Limited


Question: Progressive Steel Limited (PSL) commenced business in 2015. The following
comparative data pertains to the year ended 30 June 2017:

PSL Industry
Description
2017 2016 2017

Gross profit margin 13% 13% 16%

Net profit margin 8% 7% 10%

Return on shareholders’ equity 22% 18% 25%

Current ratio 1.2 1.6 1.5

Debt to equity ratio 40:60 30:70 50:50

Cash operating cycle in days 119 135 118

Required:
For each ratio/data give possible reasons for variation from comparative and industry data.

© Emile Woolf International 562 The Institute of Chartered Accountants of Pakistan


Chapter 11: Interpretation of financial statements

Answer:
Reasons for fluctuation
Ratios Reason for fluctuation with Industry
with previous year
Gross profit In line with previous year. Lower than industry
margin No variation.  The company is in initial phase and may
have kept the selling prices lower than
the industry to gain the market share.
 The company may not have been able to
purchase raw material at prices which is
available to its competitors.
 The company may not have been able to
obtain economies of scale in its
production which may have been
obtained by its competitors.
 Possibility of higher production costs.

Net profit Higher than previous year: Lower than industry however, the difference is
margin  Tight control over mainly attributed to lower gross profit margin.
operating costs.
 Increase in other
income.
Decrease in fixed cost per
unit due to increase in
sale.
Return on Higher than previous year: Lower than industry
shareholder's  Reduction in tax  Lower gross profit and net profit
equity rates. margins.
 Reduction in  Lower leverage.
interest rates.  Higher net assets resulting in higher
 Decrease in equity equity.
might be due to buyback
of shares.
 Distribution of
profits from previous
year which resulted in
decrease in equity.
Current ratio Lower than previous year: Lower than industry
 The company
might have obtained
running finance facility
to fund it's operations
in the current year.
 Long term loan
payments might have
become due in the next
12 month, which
decreases the current
ratio.
 Decrease in
current assets due to
better inventory
management/
reduction in credit
period of debtors.

© Emile Woolf International 563 The Institute of Chartered Accountants of Pakistan


Financial accounting and reporting I

Reasons for fluctuation


Ratios Reason for fluctuation with Industry
with previous year
Debt to equity Higher than previous year Lower than industry
ratio  Decrease in  Being a new entrant the company
reserves due to may be in the phase of expansion thereby
dividend pay- out. raising debt accordingly.
 Further debt
obtained during the
period.
 Decrease in
equity might be due to
buyback of shares.
Cash operating Lower than previous year In line with industry.
cycle  Increase in current
liabilities might be due
to increase in credit
period.
 Decrease in
current assets which
might be due to greater
stock turnover or better
inventory management.
 By giving lower
credit days to debtors.
.

Example 06: Dairy Foods Limited


Question: The following information has been gathered by an analyst, in respect of Dairy Foods
Limited (DFL) which specializes in various dairy products.

Industry
Ratio 2016 2015 2014
average

Profit margin % 11% 10% 8% 10.45%

Quick ratio 1.38 1.40 1.42 1.52

Current ratio 1.84 1.67 1.59 1.73

Days purchases in payables 80 91 89 82

In the latest annual report to the shareholders, Directors of DFL have claimed that liquidity position
of the Company has improved significantly.
Required: Critically analyse and discuss whether you agree with the claim.
Answer:
While analyzing liquidity positions of DFL, it is noted that current ratio has steadily increased over
the years and is better than industry average. However, the quick ratio has steadily declined and is
even lower than industry average. This is a clear evidence that the increase in liquidity is caused by
an increase in inventory.

Further, by considering the nature of highly perishable inventories kept by a dairy food company, it
is a possibility that DFL may bear high inventory losses due to short expiry. Based on the above, I
do not agree with the claim of DFL’s directors.

© Emile Woolf International 564 The Institute of Chartered Accountants of Pakistan


Chapter 11: Interpretation of financial statements

Example 07: Comparison


Question: Extracts from latest financial statements of two companies are as follows;
Extracts from statements of financial position
A B Assets A B
Equity & Liabilities
Rs. in millions Rs. in millions
51,690 72,114 Fixed assets 34,460 48,076
Equity & reserves
- 36,057 Stock in trade 21,700 20,000
Long term loan
35,790 45,135 Trade debtors 24,470 44,030
Trade creditors
12,000 8,500 Cash and bank 18,850 49,700
Other payables
99,480 161,806 99,480 161,806

Extract from Statement of Comprehensive Income

A B

Rs. in million

Revenue 161,600 220,150

Cost of sales (135,160) (180,520)

Gross profit 26,440 39,630

Operating expenses (9,840) (13,870)

Interest expense (720) (2,313)

Profit before tax 15,880 23,447

Income tax (333) (409)

Profit after tax 15,547 23,038

Required:
Analyze the profitability, liquidity and working capital ratios of both the companies.

Answer:

Profitability ratios A B
Gross profit ratio (GP ÷ sales) 16.36% 18.00%
Profit to sales (Profit after tax ÷ sales) 9.62% 10.46%
Return on capital employed (Profit before interest and tax ÷
capital employed) 32.11% 23.81%
Return on asset employed (Profit before interest and tax ÷ 16.69% 15.92%
assets)

Company B's gross profit and net profit ratio is slightly higher as compared to Company A. The
difference is not significant and may be on account of higher level of sales resulting in lesser fixed
costs per unit.

© Emile Woolf International 565 The Institute of Chartered Accountants of Pakistan


Financial accounting and reporting I

Company A’s return on capital employed ratio and return on asset employed ratio are better than
Company B, because Company B has accumulated large balances of cash despite of availing long
term loan. Had Company B had used its cash balances to pay off the long term loan; it would have
both of these ratio better than Company A.

Liquidity Ratios A B
Current ratio (current assets ÷ current liabilities) 1.36 2.12
Quick ratio (current asset-inventory ÷ liabilities) 0.91 1.75

Company B has better current and quick ratio. However, it appears that these ratios are better than
Company A due to substantially high amount of trade debts in term of percentage of sales as sales
days. It also represents a risk that these trade debts may prove irrecoverable. Moreover, they may
be indicative of inefficient in debt collection as well.

Working capital turnover ratios A B


Stock turnover days (Stock ÷ Cost of goods sold × 365) [A] 58.60 40.44
Debtor turnover days (Debtor ÷ Revenue × 365) [B] 55.27 73.00
Creditor turnover days (Creditor ÷ Cost of goods sold × 365) [C] 96.65 91.26
Cash operating cycle [A+B–C] (days) 17.22 22.18

Stock turnover of Company B is better than that of Company A. Company B is turning over its stock
9 times whereas company A is doing it 6 times a year.
Company A is more effectively collecting its debtors than Company B. This could also be due to the
fact that Company B is following a lenient credit policy to attract more revenue. This fact is also
supported from higher stock turnover ratio of Company B.
Company A have availed better credit facility from its creditors but it may have forgone some
settlement discounts which might have resulted in lower gross profit ratio than that of Company B.
Overall cash operating cycle of Company A is better than Company B. Furthermore, Company B has
accumulated large balances of cash despite the fact that it has also availed long term loan. Excess
cash balance should have been used to pay off the long term loan to reduce the finance cost.

Example 08: Shispare Limited and Trivor Limited


Question: Following are the summarised financial statements of Shispare Limited (SL) and its
competitor Trivor Limited (TL) for the year ended 31 December 2019:
Statement of financial position

Assets SL TL Equity & SL TL


liabilities
Rs. in million Rs. in million
Fixed assets 5,400 7,800 Capital and 8,400 9,450
reserves
Current Long-term loan 1,900 4,600
assets:
Inventory 4,800 7,100 Current liabilities:
Debtors 2,700 3,200 Creditors 2,900 4,500
Cash 1,200 800 Accrued 900 350
expenses
8,700 11,100 3,800 4,850
14,100 18,900 14,100 18,900

© Emile Woolf International 566 The Institute of Chartered Accountants of Pakistan


Chapter 11: Interpretation of financial statements

Statement of profit or loss

SL TL
--- Rs. in million ---
Sales 16,700 35,400
Cost of goods sold (11,400) (27,800)
Gross profit 5,300 7,600
Operating expenses (3,500) (4,900)
Finance cost (250) (600)
Net profit 1,550 2,100

Required:
Compute relevant ratios for SL and TL to assess which company seems to:
(i) give more incentives to its customers to pay on time
(ii) avail extended credit terms from its suppliers
(iii) be more efficient in the use of capital
(iv) keep lower selling prices to gain the market share
(v) have better liquidity position
(vi) have higher ability to convert its assets into profit
(vii) control operating expenses more efficiently
(viii) have higher ability to raise bank loan in future

Answer:

Relevant ratios SL TL
Debtors collection period (i) 59.01 days 32.99 days
Debtors 2,700 3,200
= ×365 = ×365 = ×365
Sales 16,700 35,400
TL is giving more incentives to its customers to pay on time.

Creditors payment period (ii) 92.85 days 59.08 days


Creditors 2,900 4,500
= ×365 = ×365 = ×365
Purchases 11,400 27,800
SL avail extended credits terms

Return on capital (iii) 17.48% 19.22%


Profit before interest 1,550+250 2,100+600
= ×100 = ×100 = ×100
Capital employed 8,400+1,900 9,450+4,600
TL is more efficient in the use of capital

Gross profit margin (iv) 31.74% 21.47%


Gross profit 5,300 7,600
= ×100 = ×100 = ×100
Sales 16,700 35,400
TL is deliberately keeping selling prices lower to gain the market share.

© Emile Woolf International 567 The Institute of Chartered Accountants of Pakistan


Financial accounting and reporting I

Relevant ratios SL TL
Current ratio (v) 2.29 2.29
Current assets 8,700 11,100
= = =
Current liabilities 3,800 4,850
Quick ratio 1.03 0.82
Current assets-inventory 8,700 − 4,800 11,100 − 7,100
= = =
Current liabilities 3,800 4,850
SL has better liquidity position

Return on assets (vi) 12.77% 14.29%


Profit before interest 1,550+250 2,100+600
= ×100 = ×100 = ×100
Total assets 14,100 18,900
TL has higher ability to convert its assets into profit

Operating expenses %age (vii) 20.95% 13.84%


Operating expenses 3,500 4,900
= ×100 = ×100 = ×100
Sales 16,700 35,400
TL is efficiently controlling the operating expenses.

Gearing ratio (viii) 0.18 0.33


Debt 1,900 4,600
= = =
Debt + Equity 8,400+1,900 9,450+4,600
SL is going to raise a bank loan relatively easily in future
.

Example 09: Limitations of ratio analysis


Question: Ratios are computed by using numerical values from financial statements to gain
meaningful information about an entity. However, due to inherent limitations of ratio analysis,
it may not reflect the correct financial situation.
Required:
Briefly explain any four limitations of ratio analysis.
Answer:
(i) Historical
All information used in ratio analysis is derived from actual historical results. This does not
mean that the same results will carry forward into the future. However, ratio analysis can be
used on pro forma information and compare it to historical results for consistency.
(ii) Historical versus current cost
The information on the income statement is stated in current costs (or close to it), whereas
many elements of the balance sheet are stated at historical cost (which could vary
substantially from current costs). This disparity can result in unusual ratio results.
(iii) Inflationary effect
If the rate of inflation has changed in any of the periods under review, this can mean that the
numbers are not comparable across periods. For example, if the inflation rate was 100% in
one year, sales would appear to have doubled over the preceding year, when in fact sales did
not change at all.
(iv) Aggregation
The information in a financial statement line item that is used for a ratio analysis may have
been aggregated differently in the past, so that running the ratio analysis on a trend line does
not compare the same information through the entire trend period.

© Emile Woolf International 568 The Institute of Chartered Accountants of Pakistan


Chapter 11: Interpretation of financial statements

9 OBJECTIVE BASED QUESTIONS


01. The acid test also known as quick ratio should include which of the following?
(1) Inventory
(2) Accounts receivables
(3) Bank overdraft
(4) Accruals
(a) (1), (2), (3) and (4)
(b) (1), (2) and (3) only
(c) (1), (2) and (4) only
(d) (2), (3) and (4) only

02. Salik has net current liabilities in its statement of financial position. He has decided to pay off
its accounts payables using surplus cash.
What will be the effect of the above transaction on the current ratio?
(a) Decrease
(b) Increase
(c) No effect
(d) The ratio could either increase or decrease

03. Extracts from the statement of Comprehensive Income and statement of financial position of
the Huda Limited are shown below:
Rs.
Revenue from sales (all on credit) 900,000
Cost of goods sold 756,000
Purchases (all on credit) 504,000
Receivables 112,500
Trade payables 75,600
Inventory 333,000
What is the length of the working capital cycle (also known as the operating or cash cycle) to
the nearest day?
(a) 170 days
(b) 152 days
(c) 261days
(d) 60 days

04. Which of the following should be included in acid test or quick ratio?
(a) Finished goods inventory
(b) raw materials and consumables
(c) long-term loans
(d) Accounts payable

© Emile Woolf International 569 The Institute of Chartered Accountants of Pakistan


Financial accounting and reporting I

05. Given below are included in the financial statements of Haris Limited for the year ending 30
June:
2015 2016
Rs. 000 Rs. 000
Receivables for disposal of PPE 36 54
Accounts receivables 108 72
144 126
Sales for the year amounted to Rs. 630,000, of which Rs. 90,000 were cash sales.
The average receivables turnover (in times) during the year ended 30 June 2016 was?
(a) 7
(b) 6
(c) 5
(d) 4

06. Dawn Limited has provided following information:


31st December 31st December
2014 2015
Rs. 000 Rs. 000
Non -current assets 36 61.2
Inventory 18 23.4
Accounts receivable 25.2 19.8
Bank balance 5.4 3.6
Accounts payable 19.8 16.2
Sales on credit 252 216
Purchases on credit 144 108
Which TWO of the following statements are correct?
(a) Net current assets have increased between the two years
(b) The quick (acid test) ratio has improved between the two years
(c) Customers are paying more quickly in 2015 than in 2014
(d) Dawn Limited is paying its suppliers more quickly in 2015 than in 2014

07. Amir sells fish and Abid sells books. Both operate on a 50% mark-up on cost.
However, their gross profit ratios are as follows.
Amir 25%
Abid 33%
The highest gross profit ratio of the bookseller may be because?
(a) There is more wastage with fish stocks than with book stocks
(b) Amir has a substantial bank loan whereas the Abid’s business is entirely financed by
her family
(c) Amir has expensive high street premises whereas Abid has cheaper back street
premises
(d) Amir’s turnover is declining whereas that of the Abid is increasing

© Emile Woolf International 570 The Institute of Chartered Accountants of Pakistan


Chapter 11: Interpretation of financial statements

08. Maira Limited has a current ratio of 2:1.


This ratio will decrease if Maira Limited
(a) will receive cash in respect of a long-term loan
(b) will receive cash in respect of a short-term loan
(c) pays an existing trade payable
(d) writes off an existing receivable against the provision for doubtful debts

09. Haris Limited has positive working capital.


What effect will the payment of a declared (payable) dividend using cash balances have upon
the current ratio and working capital?
(a) Increase in current ratio and increase in working capital
(b) Increase in current ratio and no effect on working capital
(c) No effect on current ratio and working capital
(d) Decrease in current ratio and decrease in working capital

10. After declaring a final dividend, Kashan Limited has a current ratio of 2.0 and a quick asset
ratio of 0.8.
If the company now uses its positive cash balance to pay that final dividend, what will be the
effect upon the two ratios?
(a) Increase in current ratio and increase in quick asset ratio
(b) Increase in current ratio and decrease in quick asset ratio
(c) Decrease in current ratio and increase in quick asset ratio
(d) Decrease in current ratio and decrease in quick asset ratio

11. The draft accounts of Super Star Limited for the year ended 31 December 2018 include the
following:

Revenue Rs. 360 million

Gross profit Rs. 90 million

It was subsequently discovered that the revenue was overstated by Rs. 45 million and the
closing inventory understated by Rs. 15 million.
After correction of these errors the gross profit percentage will be?
(a) 9.5%
(b) 19.0%
(c) 23.8%
(d) 33.3%

© Emile Woolf International 571 The Institute of Chartered Accountants of Pakistan


Financial accounting and reporting I

12. The following has been extracted from the financial statements of a business.
SOCI Rs. SOFP Rs.
Profit from operations 86,400 7% debenture 117,000
Debenture interest (8,190) Ordinary share capital 171,000
Profit for the year 72,360 Share premium 13,500
Retained earnings 63,000
What was the return on capital employed (ROCE)?
(a) 19.9%
(b) 23.7%
(c) 29.2%
(d) 34.9%

13. Waris Limited buys and sells a single product. The following is an extract from its statement
of financial position at 31 December 2018.
2018 2017
Rs. Rs.
Inventory 75 60
Receivables 24 36
Sales and purchases during 2018 were Rs. 300,000 and Rs. 180,000 respectively. 20% of
sales were for cash.
Which TWO of the following are correct?
(a) Average receivables collection period is 37 days
(b) Average receivables collection period is 46 days
(c) Gross profit % is 35%
(d) Gross profit % is 45%

14. A company’s gearing ratio would rise if


(a) A decrease in long term loans is less than a decrease in shareholders’ funds
(b) A decrease in long-term loan is more than a decrease in shareholders’ funds
(c) Interest rates rose
(d) Dividends were paid

15. A business has the following trading account for the year ending 31 May 2018:
Rs. Rs.
Sales 450,000
Opening inventory 40,000
Purchases 260,500
300,500
Less: closing inventory (60,000) (240,500)
Gross profit 209,500

© Emile Woolf International 572 The Institute of Chartered Accountants of Pakistan


Chapter 11: Interpretation of financial statements

Its rate of inventory turnover for the year is?


(a) 4.9 times
(b) 5.3 times
(c) 7.5 times
(d) 9 times

16. Tara Ltd produces a single product with a margin on sales of 25%.
Total sales for the year Rs. 400,000
Receivables collection period 64 days
Average receivables Rs. 32,000
The value of inventory held during the year was constant.
The cost of credit sales was?
Rs. ___________

17. The following are extracts from the financial statements of Laiba Ltd for the year ended 31
December 2018.
Statement of financial position Statement of Comprehensive Income
Rs. 000 Rs. 000
Issued share capital 3,600 Operating profit 1,431
Reserves 1,800 Debenture interest (216)
5,400 1,215
12% debenture 2018 1,800
7,200
What is the return (%) on long-term funds?

___________%

18. The opening inventory for a business was Rs. 108,000. The closing inventory was Rs.
144,000.
Inventory turnover for the year was 10 times.
The gross margin was 30%.
What were the sales for the year?
Rs. ___________

19. Adeel Limited has trade payables (creditors) of Rs. 12,000 and a bank overdraft of Rs. 3,000.
Its current ratio is 2.5: 1 and its quick (acid test) ratio is 1.5:1.
What is the value of its inventory (stock)?
Rs. ___________

© Emile Woolf International 573 The Institute of Chartered Accountants of Pakistan


Financial accounting and reporting I

20. Extracts from statement of financial position of Turab Limited at 31 March 2019 are presented
below:
Rs. 000
Loans due in more than one year 32
5% loan notes 24
Ordinary shares - Rs. 1 each fully paid 80
6% redeemable preferred shares, Rs. 1 each fully paid 16
Retained profits 104
Revaluation reserve 40
The gearing ratio is (to one decimal place)?

___________%

21. Which ratio is considered as safe margin of solvency?


(a) Liquid ratio
(b) Quick ratio
(c) Current ratio
(d) Net profit ratio

22. Liquid ratio is also known as


i. Quick ratio
ii. Acid test ratio
iii. Working capital ratio
iv. Stock turnover ratio
(a) i. and ii.
(b) i. and iii.
(c) iii. and iii.
(d) iii. and iv.

23. Care pharmacy Ltd. has a current ratio equal to 1.6 and a quick ratio equal to 1.2. The
company has Rs.200 million in sales and its current liabilities are Rs.10 million. what is the
value of company's current assets?
(a) Rs.14 million
(b) Rs.16 million
(c) Rs.18 million
(d) Rs.20 million

© Emile Woolf International 574 The Institute of Chartered Accountants of Pakistan


Chapter 11: Interpretation of financial statements

24. Determine working capital turnover ratio if, current assets is Rs.150 million, current liabilities
is Rs.100 million and Sales during the year are Rs.500 million.
(a) 5 times
(b) 10 times
(c) 15 times
(d) 20 times

25. A debtor turnover ratio of 12 times means that:


(a) one-twelfth of debtors will turn out to be bad debts
(b) debtors are about twelve times as big as creditors
(c) in any given month, twelve debtors are expected to pay in full
(d) the average debtor takes about one month to pay.

26. What is the impact of collection of debtors on the current ratio;


(a) Increase
(b) Decrease
(c) No impact
(d) None of the above

27. Which of the following is not included in the computation of acid test ratio?
(a) Debtors
(b) Cash at bank
(c) Short-term investments
(d) Stock

28. Determine inventory turnover ratio if, opening inventory is Rs.31 million, closing inventory is
Rs.29 million, sales are Rs.320 million and gross profit margin is 25%.
(a) 6 times
(b) 8 times
(c) 11 times
(d) 16 times

29. Debt-equity ratio is a sub-part of


(a) Short-term solvency ratios
(b) Long-term solvency ratios
(c) Debtors turnover ratios
(d) None of the above

© Emile Woolf International 575 The Institute of Chartered Accountants of Pakistan


Financial accounting and reporting I

30. Night Limited has a current ratio of 1.8. This ratio will increase if Night Limited:
(a) receives cash in respect of a short term loan
(b) receives cash from an existing receivable
(c) pays an existing trade payable
(d) purchases inventory on credit

31. Night Limited has a current ratio of 1.8. This ratio will increase if Night Limited:
(a) receives cash in respect of a short term loan
(b) receives cash from an existing receivable
(c) pays an existing trade payable
(d) purchases inventory on credit

32. Which of the following companies is most likely to face cash flow problems?
(a) A loss making government organisation
(b) A company which has recently sold part of its operations so as to concentrate on its
core areas
(c) A reasonably profitable and long established company with no expansion plans
(d) A profitable retailer about to embark on ambitious expansion plans

33. A company has current ratio and quick ratio of 2.0 and 0.8 respectively. If the company uses
its positive cash balance to pay a creditor, it will:
(a) increase current ratio as well as quick ratio
(b) increase current ratio and decrease quick ratio
(c) have no effect on current ratio as well as quick ratio
(d) decrease current ratio as well as quick ratio

34. Which of the following would increase gearing ratio?


(a) Issuance of shares at premium
(b) Issuance of shares at discount
(c) Issuance of bonus shares
(d) Declaration and payment of cash dividend

© Emile Woolf International 576 The Institute of Chartered Accountants of Pakistan


Chapter 11: Interpretation of financial statements

9 OBJECTIVE BASED ANSWERS


01. (d) Acid test ratio is calculated as (current assets - inventory) / current
liabilities

02. (a) Assume initially that Salik has assets of Rs. 100,000 and liabilities of Rs.
150,000
Its current ratio prior to the transaction will be: Rs. 100,000/Rs. 150 ,000=
0.67
If it then pays its trade payables by Rs. 50,000 the current assets will be Rs.
50,000 and the liabilities Rs. 100,000.
Hence the new current ratio is 50,000/100,000 = 0.50, i.e. a decrease.

03. (b)
days
Inventory turnover: 333/756× 365 160.8
Average collection period: 112.5/900 × 365 45.6
Average time to pay: 75.6/504× 365 (54.8)
151.6

04. (d) The acid test ratio excludes all inventory balances, and is based on short-
term creditors only.

05. (b) Credit sales = 630,000 – 9,000 = Rs. 540,000


Average receivables = (72000 + 108000)/2 =90,000
Receivable turnover =540,000/ 90,000 = 6

06. (a & c)
2014 2015
Net current (18 + 25.2 + 5.4 28.8 (23.4+ 19.8+ 30.6
assets – 19.8) 3.6 – 16.2)
Quick ratio (25.2 + 5.4)/19.8 1.55 (19.8 + 1.44
3.6)/16.2
Collection (25.2 /252) × 36.5 (19.8/216) × 33.5 days
period 365 days 365
Payment (19.8/144) × 365 50.2 (16.2/108) × 54.8 days
period days 365

07. (a) Cost plus 50% is equivalent to a gross profit ratio of 33%. Amir’s gross
profit margin may be low because of wastage.
The loan interest and rental would not affect gross profit (only affects net
profit) and declining turnover would not directly affect the gross profit
percentage.

08. (b) Receiving cash for a long-term loan increases current assets with no
change in current liabilities, hence improves the ratio. Payment on an
existing creditor improves the ratio.
Writing off a receivable against a provision has no effect on current assets.

© Emile Woolf International 577 The Institute of Chartered Accountants of Pakistan


Financial accounting and reporting I

Therefore receiving cash in respect of a short-term loan must be the correct


choice.
Thus, suppose current ratio is 2:1, that means assets 100,000 and current
liabilities 50,000
Now say Maira Limited receives loan = Rs. 50,000
Current assets will be 100,000 + 50,000=150,000
Current liabilities 50,000 + 50,000 = 100,000
New Current ratio 1.5:1

09. (b) Before payment of proposed dividend


Rs.
Current assets 150,000
Current liabilities including proposed dividend Rs. 50,000
30,000
Current ratio 3:1
Working capital 100,000
After payment of proposed dividend
Rs.
Current assets 120,000
Current liabilities 20,000
Current ratio 6:1
Working capital 100,000

10. (b) Suppose that inventories are Rs. 120,000, cash plus receivables are Rs.
80,000 and creditors (including a Rs. 10,000 dividend) are Rs. 100,000.
Payment of the dividend will cause cash plus receivables to fall to Rs.
70,000 and creditors to fall to Rs. 90,000.
The current ratio will increase to 2.11 (190 ÷ 90).
The quick ratio will decrease to 0.77 (70 ÷ 90).

11. (b) Rs. m Rs. m


Turnover 360-45 315
Cost of sales 270-15 (255)
Gross profit 60
Gross profit % =60/315= 19.0%

12. (b) ROCE = 86,400/364,500= 23.7%

13. (a) & (d) Credit sales = Rs. 300,000 x 80%= Rs. 240,000
Average receivables =Rs. (24,000+ 36,000)/2 = Rs. 30,000
Receivables’ turnover =Rs. 240,000/Rs. 30,000 = 8
Collection period =365/8 = 46 days
Rs. 000 %
Sales 300 100
Cost of sales (60+180-75) (165) 55
Gross profit 135 45

© Emile Woolf International 578 The Institute of Chartered Accountants of Pakistan


Chapter 11: Interpretation of financial statements

14. (b)

15. (a) Inventory turnover = COS/ average inventory


= 240,500 /[(40,000 + 60 ,000)/2 = 4.9 times

16. Rs. 136,875 Receivables of Rs. 32,000 represent 64 days’ credit sales.
Therefore, receivables of Rs. 182,500 would represent 365 days’ credit
sales. [32,000 x 365/64]
Cost of credit sales = Rs. 182,500 x 75% = Rs. 136,875

17. 19.88% Return on long-term funds = Operating profit (before debenture interest)
/(Share capital + Reserves + Debentures)
=1,431/7,200
= 19.88%

18. Rs. Inventory turnover =CGS/ average inventory


1,800,000
CGS = average inventory x inventory turnover
= ((108,000 + 144,0000)/2) x 10
= 126,000x10 = Rs. 1,260,000
Sales = Rs. 1,260,000 / 0.7 = Rs. 1,800,000

19. Rs. 15,000 Current ratio = Current assets/ current liabilities


Current assets = Current ratio x Current liabilities
= 2.5 x (Rs. 12,000 + Rs. 3000) = Rs. 37,500
Quick ratio = (C.A – inventory)/ C.L
C.A – inventory = Quick ratio x C.L
Rs. 37,500 – inventory = 1.5 x 15,000
Rs. 37,500 – inventory = Rs. 22,500
Inventory = Rs. 15,000

20. 24.3 % Debt = 32 + 24 + 16 = 72


Equity = 80 + 104 + 40 = 224
Gearing = 72/(72 + 224) = 24.3 %

21. (c)
22. (a)

23. (b)

24. (b)
25. (d)

26. (c)

27. (d)
28. (b)

29. (b)

30. (c) Pays an existing trade payable

© Emile Woolf International 579 The Institute of Chartered Accountants of Pakistan


Financial accounting and reporting I

31. (c) Average debtors are collected in 2 months

32. (d) A profitable retailer about to embark on ambitious expansion plans


33. (b) increase current ratio and decrease quick ratio
34. (d) Declaration and payment of cash dividend

© Emile Woolf International 580 The Institute of Chartered Accountants of Pakistan


I
Certificate in Accounting and Finance
Financial accounting and reporting I

Index

received from sale of investment 78


a from new share issues
from new loans/repay loans
80
81
Control 301
Accounting for
Cost
Government grants and disclosure of
accounting 302
grant assistance 410
accounting cycle 325
Impairment 452
behaviour graphs 314
Revaluation 358
behaviour 314
management 298
classification 308
Accounting treatment of investment
and management accounting 302
Property 431
unit 306
Accumulated fund 151
structures 218
Administration costs 310
Current ratio 547
Accruals based figures 55
customer 482
Accounting equation 209
contract 482
Allowances for doubtful debts 60
cash flows
Analysing returns 539
from financing activities 80
Average time to collect 543
from investing activities 71
To pay suppliers 544
from operating activities 51
For holding inventories 543
changes in
Acid test ratio 548
inventory 61
trade and other receivables 59

c changes in
trade payables 62
Comparison of financial accounting and cost
Changing the carrying amount of an asset 362 and management 302
Cash Calculation of opening capital 211
equivalents 50
from disposal of P, P & E 77
paid for purchase of investment 78

© Emile Woolf International 581 The Institute of Chartered Accountants of Pakistan


Financial accounting and reporting I

d i
Data and information 298 IAS 1: Presentation of Financial
Depreciation of a revalued asset 363 Statements 26
Direct IAS 7: statement of cash flows 47
Cost 319 IAS 16: Property, Plant & Equipment 357
expenses 321 IFRS 15: Revenue from contracts with
labour 320 customer 481
materials 319 Five step model 482
Disposal of property, plant and equipment 77 Identifying missing balances 211
Disclosure requirements Indirect costs (overheads) 321
For IAS 16: P, P & E 367 Indirect method 53
Assets stated at revalued amounts 368 Cash flow from operating activities 53
Debt ratios 549 Investing activities 71
Donations 151
Dividends payments to equity
shareholders 81 l
e Labour costs
Lack of detail
309
64
Life membership fees 148
Equity 3 Liquidity ratios 547

f m
Finance costs 310 Management accounting 302
Financing activities 80 Manufacturing accounts 328
Fixed & variable costs 313 Material costs 309
Full cost 322 marketing costs (selling and distribution) 310
Financing of a sole proprietor or partnership 83 Measurement
Financial ratio analysis 535 after initial recognition (P, P & E) 431
Frequency of revaluations 365 Memorandum cash and bank account 217
Memorandum control accounts 214
Missing inventory figure 221
g
Gain or loss on disposal 433
n
of property, plant and equipment 71
of non current asset 54 Non-production costs 310
Nature of incomplete records 208
Non-cash items 54
Not for profit organizations 146

© Emile Woolf International 582 The Institute of Chartered Accountants of Pakistan


Index

o s
Opening capital (calculation of) 211 Selling and distribution (marketing) costs 310
Operating cash flows 53 Semi-variable cost 315
Overheads (Indirect costs) 321 Special funds 155
Operating activities 50 Stepped cost 316

p Statement of
Financial position 154
Cash flows 50
Percentage annual growth in sales 542 Subscription account 148
Performance obligations 484 Surplus from
Period costs 324 Running an operation 151
Running an event 152
Planning 300
Sales/capital employed ratio 542
Prime cost 321
Profit before taxation 54
Profit/sales ratio
Product costs
539
324 t
Production and non-production costs 312
Profit and cash flow 49 Transaction price 485
Purpose of management information 300 Turnover ratio 542

q u
Qualities of good information 299 Unit costs 306
Quick ratio or acid test ratio 548 Usefulness of classifying costs by function 311

r v
Revaluation model 358 Variable cost 313
Revaluation of property,
plant and equipment 358
Revenue
Realisation of the revaluation surplus
482
364
w
Revalued assets
being sold 364 Working capital
being depreciated 364 adjustments 58
Reporting profit 312 efficiency ratios 543
Return on cycle 545
capital employed 536 Write-off of subscription 149
shareholder capital (equity) 538
assets 538

© Emile Woolf International 583 The Institute of Chartered Accountants of Pakistan


Financial accounting and reporting I

© Emile Woolf International 584 The Institute of Chartered Accountants of Pakistan

You might also like